Sunteți pe pagina 1din 247

Credits

This book is all credited to Dr Jansen Koh who had shared his wisdom
and presentation skills with worldwide medical & master students in
preparation for MRCP exams.
Dr Jansen Koh obtained his MBBS from the National University of
Singapore in 2001. He is a consultant at the department of Respiratory &
Critical Care Medicine, Changi General Hospital. He recently completed a
fellowship in Clinical Simulation and Medical Education at the Learning
Institute, SickKids Hospital, Toronto, Canada.
He is currently the Associate Program Director of the Internal Medicine
Residency Program at Singhealth. He also serves as Chair of the
Education Technology Workgroup at Changi General Hospital, a group
responsible for identifying, selecting and utilizing technology to enhance
medical education.
His specific clinical interest in Respiratory and Critical Care is point of
care ultrasonography, having obtained his certification from the
American College of Chest Physicians. In addition, he has developed
several educational initiatives in simulation medical education at Changi
General Hospital. He is passionate about medical education for students
and doctors and was awarded the 2012 NUS-YLL Dean’s Award for
Teaching Excellence as well as the 2012 Changi General Hospital Best
Teacher Award. His research interest lies in advancing our understanding
and effective use of simulation for clinical teaching and process
improvement in critical care.
Table of Contents
TOPIC PAGE NO.

SECTION I LONG CASES

1) CARDIOVASCULAR SYSTEM

- MITRAL REGURGITATION 1
- MITRAL VALVE PROLAPSE 6
- MITRAL STENOSIS 9
- AORTIC REGURGITATION 12
- AORTIC STENOSIS 14
- MIXED MITRAL VALVE DISEASE 17
- MIXED AORTIC VALVE DISEASE 18
- PROSTHETIC HEART VALVES 19
- VENTRICULAR SEPTAL DEFECT 21
- ATRIAL SEPTAL DEFECT 24
- HYPERTROPHIC OBSTRUCTIVE CARDIOMYOPATHY 27
- APPROACH TO CENTRAL CYANOSIS & CLUBBING 29
- DETROCARDIA 31

2) RESPIRATORY SYSTEM

- BRONCHIECTASIS 32
- INTERSTITIAL LUNG DISEASE 36
- CHRONIC OBSTRUCTIVE PULMONARY DISEASE 39
- PLEURAL EFFUSION 42
- LUNG COLLAPSE 45
- CONSOLIDATION 48
- LOBECTOMY/ PNEUMONECTOMY 52
- APPROACH TO LATERAL THORACOTOMY SCAR 53
- LUNG TRANSPLANT 54

3) GASTROINTESTINAL SYSTEM

- CHRONIC LIVER DISEASE 56


- HEPATOMEGALY 62
- SPLENOMEGALY 63
- HEPATOSPLENOMEGALY 65
- ASCITES 67
- UNILATERAL ENLARGED KIDNEY 70
- BILATERAL ENLARGED KIDNEY 71
- TRANSPLANTED KIDNEY 75

! I!
4) NEUROLOGICAL SYSTEM

ASSESSMENT OF HIGHER CORTICAL FUNCTION 78


CRANIAL NERVES 80
- ISOLATED THIRD CRANIAL NERVE PALSY 83
- ISOLATED SIXTH CRANIAL NERVE PALSY 86
- SEVENTH CRANIAL NERVE PALSY 89
- MYASTHENIA GRAVIS 93
APPROACH TO EYE EXAMINATION 96
- GAZE PALSIES 97
- APPROACH TO UNILATERAL PTOSIS 99
- APPROACH TO BILATERAL PTOSIS 101
UPPER LIMBS OVERVIEW 102
- RADIAL NERVE PALSY 104
- MEDIAN NERVE PALSY 106
- ULNAR NERVE PALSY 109
- WASTED HANDS 111
- PERIPHERAL NEUROPATHY 113
- SYRINGOMYELIA 116
- DYSTROPHIA MYOTONICA 118
- CEREBELLAR DISEASE 121
- CHOREA 125
- PARKINSONISM 127
LOWER LIMB OVERVIEW 131
- FLACCID PARAPARESIS 133
- SPASTIC PARAPARESIS 138
- PERIPHERAL NEUROPATHY 144
- PROXIMAL MYOPATHY 146
- BROWN-SEQUARD SYNDROME 148
- FOOTDROP 149
- HEMIPARESIS/ HEMIPLEGIA 150
- GAIT ASSESSMENT 153
- PARKISONISM 154

SECTION II SHORT CASES

5) EYE DISEASE
- DIABETIC RETINOPATHY 158
- HYPERTENSIVE RETINOPATHY 162
- OPTIC ATROPHY 164
- PAPILLOEDEMA 165
- CENTRAL AND BRANCH RETINAL VEIN OCCLUSION 168
- CENTRAL RETINAL ARTERY OCCLUSION 170
- RETINITIS PIGMENTOSA 172
- VISUAL FIELD DEFECTS 173
- OTHER EYE CONDITIONS 176

! II!
6) NEUROLOGICAL SYSTEM
- CHARCOT’S JOINT 178
- DYSTROPHIA MYOTONICA 180
- MEDIAN NERVE PALSY 183
- RADIAL NERVE PALSY 186
- ULNAR NERVE PALSY 188
- WASTED HANDS 190

7) RHEUMATOLOGY
- RHEUMATOID ARTHRITIS 193
- GOUTY HANDS 195
- PSORIATIC ARTHRITIS 198
- OSTEOARTHRITIS 201
- SCLERODERMA 202
- ANKYLOSING SPONDYLITIS 205
- MARFAN’S SYNDROME 207
- MISCELLANEOUS RHEUMATOLOGICAL HAND CONDITIONS 211

8) ENDOCRINAL SYSTEM
- ACROMEGALY 213
- CUSHING’S SYNDROME 216
- GOITRE 219
- PAGET’S DISEASE 225
- MISCELLANEOUS ENDOCRINAL DISORDERS 228

9) DERMATOLOGICAL SYSTEM
- PSORIASIS 230
- LICHEN PLANUS 232
- NEUROFIBROMATOSIS 234
- PURPURA 237
- DERMATOMYOSITIS 239

! III!
Mitral Regurgitation

Presentation

Sir, this gentleman has mitral regurgitation that is moderately severe in nature.

There is a pansystolic murmur heard best at the apex which radiates to the axilla. (If it radiates to the carotids
– posterior mitral leaflet rupture) This is a grade 3/6 murmur and is not associated with a systolic thrill. The
first heart sound is soft and there is presence of a third heart sound(S3). I did not detect any mid-dastolic
murmur. The apex is thrusting and displaced, located at the 6th IC at the anterior axillary line.

This is complicated by pulmonary hypertension as evidenced by a palpable and loud pulmonary component
of the second heart sound associated with a left parasternal heave. There are no clinical signs of heart failure.

On the peripheral examination, patient is in atrial fibrillation with an irregularly irregular pulse which is rate
controlled at 80 beats per min. There is no bruising to suggest overanticoagulation. There are also no stigmata
of infective endocarditis.

To complete the examination, I would like to take the patient’s blood pressure, as well as temperature chart
for any fever. (Mention abdominal examination, urine dipstick and fundoscopy if clinically suggestive of IE)

In summary, this gentleman has mitral regurgitation that is moderately severe in nature, with complication
atrial fibrillation and pulmonary hypertension. There are no complications of heart failure or infective
endocarditis. My differential diagnoses include IHD, MVP and Rh heart disease. (If thoracotomy scar, think
of mitral valvotomy for MS)

Questions

How do you grade the severity of mitral regurgitation clinically?


•! Mild – No Pulm hypt
•! Moderate – Pulmonary hypertension
•! Severe – LVF, S3

What are the causes of mitral regurgitation?


!! Common causes are MVP, IHD, Rh heart disease and endocarditis
!! Left ventricular dilatation, cardiomyopathy, Marfan’s, Rheumatoid, AS
!! Acute causes: MI, IE, Trauma, Surgery, spontaneous rupture
!! Anterior leaflet: radiates to axilla and back
!! Posterior leaflet: radiates to carotids
!! Mitral valvotomy if a thoracotomy scar seen
!! If elderly and mild to moderate, typically due to annular calcification

What are the differential diagnoses for a pansystolic murmur?


!! MR
!! TR
!! VSD

What congenital conditions can be associated with MR?


!! Corrected TGA
!! Partial AV canal
!! Ostium primum atrial defect (cleft mitral valve)

1
What causes a third heart sound?
!! Rapid filling of the left ventricle from the large volume of blood from the left atrium occurring in early
diastole

Why is the pulse jerky?


!! Pulse is sharp and abbreviated due to lack of sustained forward stroke volume with a reduced systolic
ejection time because of regurgitant leak into the left atrium

How do you differentiate an MDM from severe MR vs MS?


!! MS has an opening snap
!! Severe MR associated with S3
!! MS murmur is longer
!! MS has loud S1

How do you differentiate between MR and TR murmur?


Mitral Regurgitation Triscupid Regurgitation
PSM heard best at Apex PSM heard best at the LLSE
Radiates towards the axilla Radiates towards the right of sternum
Louder on expiration Louder on inspiration
Displaced apex beat Apex beat not displaced
Jerky pulse character Normal pulse character
Normal JVP unless complication Giant V wave with pulsatile liver

How do you differentiate between an MR murmur and that of a VSD?


MR VSD
Loudest at the apex Loudest at the LLSE
High pitched Harsh, low pitched
Soft S1 Normal S1

What are the types of dynamic manoeuvres that you are aware of and what are their uses?
!! Respiration
!! Murmurs on the right side louder on inspiration due to increased venous return and blood flow to the
right side of heart
!! Converse is true
!! Valsalva manoeuvre (decrease preload)
!! 3 phases
!! Phase 1 – beginning of maneuver
!! Rise in intrathoracic pressure and a transient increase in LV output
!! Phase 2 – Straining phase
!! Systemic return falls
!! Reduced filling of the right and left heart chambers
!! SV and BP drops while HR increases
!! Most murmurs become softer and shorter except
!! MVP – Systolic click and murmur begins earlier (LV size is smaller), ie longer and louder
!! HOCM – murmur is louder as LV volume is reduced
!! Phase 3
!! Release of maneuvre
!! Right heart murmurs becomes louder followed by left heart murmurs
!! Squatting (increases venous return and systemic arterial resistance)
!! Most murmurs are louder
!! MVP – click occurs later and murmur is shorter because LV size increased
!! HOCM – LV size increased which reduced the obstruction to outflow and systolic murmur is softer
2
!! Standing
!! Most murmurs are softer except
!! MVP – louder and longer and HOCM - louder
!! Isometric exercises (increases afterload)
!! AS – Softer murmur as there is reduction of pressure gradient across the valve
!! MVP – click occurs later and murmur is shorter because LV size increased
!! HOCM – LV size increased which reduced the obstruction to outflow and systolic murmur is softer
!! MR/AR/VSD louder
!! Amyl Nitrite inhalation
!! Initial relative hypotension
!! MR/AR/VSD decrease
!! AS increases because of increased stroke volume
!! Later tachycardia phase
!! MS and right murmurs increase
!! Can use to differentiate Austin Flint from MS

What are the signs of severity for MR?


!! Presence of S3
!! Short MDM
!! Apex thrusting and displaced
!! Pulmonary hypertension
!! CCF

What is the pathophysiology of MR?


!! MR leads to LV overload
!! Compensatory LV dilatation
!! Eventually, decompensate resulting in heart failure and increased risk of sudden death
!! Also, regurgitation into the LA leads to enlargement of LA with AF and elevated pulmonary pressures

Should all murmurs be investigated?


•! All should be Ix except 1. mid-systolic, grade 2 or < murmurs with no associated findings or symptoms
2. continuous murmurs of venous hum or mammary soufflé of pregnancy

How would you investigate this patient?


!! ECG
!! LA enlargement – P mitrale (II – P >0.12s, Limb; Bifid P waves in limb leads with inter-peak > 0.04s,
terminal P negativity in V1)
!! LVH – Sokolow & Lyon Criteria (S in V1 and R in V5 or 6 >35mm)
!! AF
!! Pulmonary hypertension
!! CXR
!! CCF – pulmonary congestion, enlarged heart
!! Left atrial enlargement
!! Pulmonary artery enlargement
!! Echocardiogram
!! Dx of MR
!! Severity – EF <60% and LV end-systolic diameter >45mm
!! Cause
!! Complications eg IE
!! Cardiac catheterisation
!! Not indicated in most patients but useful if there is discrepancy between echocardiographic and
clinical findings
!! Useful to stenosis, regurgitation and intracardiac shunting

3
How would you manage this patient?
!! Education
!! Medical therapy
!! Antibiotic prophylaxis
!! Treatment of underlying cause eg IHD, dilated CMP (Rx of CCF and afterload reduction)
!! Treatment of complications eg AF, IE, CCF
!! Surgical
!! Indications
!! Symptomatic or
!! EF<60% or
!! LV end-systolic dimension >45mm
!! Types of surgery
!! Mitral valve repair if technical feasible is best
!! Mitral valve replacement if technically not feasible provided EF >30%
!! Controversial
!! Varied causes for MR
!! If due to IHD or dilated CMP, then Sx is controversial
!! If due to MVP, timing of surgery
!! Indicated if symptomatic, AF, pulmonary hypt, EF<60% or ESD>45
!! If asymptomatic, risk stratify according to regurgitant orifice(doppler)
!! <20mm2
!! 20-39mm2
!! >40mm2 (this affects Px and closer follow up necessary)

How do you diagnose infective endocarditis?


!! Duke’s criteria
!! 2 Major, 1 Major 3 Minor or 5 minor
!! Major
!! Persistently positive blood c/s with typical organism
!! Persistently positive blood c/s
!! 2 or more positive c/s > 12h apart
!! 3 or more positive c/s each 1 hr apart
!! if 4 or more taken, >70% positive
!! Typical organism
!! Strep viridans, Strep bovis, enterococci, Staph aureus
!! HACEK: Haemophilus, Actinobacillus, Cardiobacterium, Eikenella, Kingella
!! Endocardial involvement
!! Positive echocardiogram: vegetations, abscesses, valve perforation, dehiscence
!! New valvular regurgitation
!! Minor
!! Predisposing heart condition
!! Fever
!! Vascular phenomena: arterial emboli, septic pulmonary emboli, mycotic aneurysm, ICH,
Janeway lesion
!! Immunologic phenomena: GN, Osler’s nodes, Roth spots, Rh factor
!! Positive blood c/s not satisfying major criteria
!! Positive echocardiogram not satisfying major criteria

What are the types of endocarditis?


!! Native valve endocarditis
!! Strep, enterococci, Staph
!! Rh, Cong HD, MVP with murmur and degenerative valvular disease
!! Prosthetic
!! Early(<60 days): Staph aureus or S. epidermis
!! Late(>60 days): Similar to native endocarditis
4
!! Fungal :IVDA and ICU
!! IVDA: TV involvement, AV also; Staph aureus, MRSA, fungi, Strep, GNB

How would you investigate?


•! Bloods
•! Blood C/S (as above)
•! FBC (NCNC anaemia, raised TW with left shift), ESR, CRP
•! 2D echo
•! CXR, ECG

How do you treat infective endocarditis?


!! General measures
!! Eg oxygen, treat fever
!! Antibiotics
!! IV CP 12-18MU/d 4H for 4 weeks
!! Can also add IV gentamicin 1mg/kg 8H for first 2 weeks
!! If allergic, use vancomycin 30mg/kg/d in 2 divided doses for 4 weeks
!! HACEK organism: IV ceftriaxone
!! MSSA: IV cefazolin or nafcillin or cloxacillin
!! Surgery
!! Heart failure
!! Failure of medical therapy
!! Presence of fever and inflammatory syndrome after 1 week of appropriate and adequate
antibiotics
!! Presence of mobile vegetation >10mm with 1 major embolism 1 week A/B
!! Presence of mobile vegetation >15mm with 1 week of A/B
!! Valvular complication eg valvular abscess, valvular obstruction, rupture into the pericardium, septal
formation, fistula
!! Fungal endocarditis
!! Prosthetic valves esp if unstable or early(<60 days) or caused by S aureus

When and how should you prophylax against IE? (3 steps: Risk stratify, Type of Procedure and Type of
antibiotics)
!! Risk stratify
•! Highest risk: prosthetic valves, both bioprosthetic and mechanical; previous IE; congenital cyanotic
heart disease; or surgically produced systemic/pulmonary shunts.
•! Moderate risk: (1) all other congenital cardiac conditions, except isolated secundum atrial septal
defects and surgical repairs of an atrial septal defect or patent ductus arteriosus or ventricular septal
defect more than 6 months ago; (2) acquired valvular dysfunction (eg, rheumatic heart disease,
calcific aortic stenosis); or (3) hypertrophic cardiomyopathy and mitral valve prolapse with valvular
regurgitation and/or thickened leaflets. Thickening of the anterior leaflets of the mitral valve
correlates with significant mitral insufficiency, especially in men older than 45 years.
•! Low risk: mitral valve prolapse without significant regurgitation or thickened leaflets on
echocardiography, implanted cardiac PMs, implanted defibrillators, implanted coronary stents, or
"innocent" murmurs. An important caveat is that in elderly individuals, an innocent murmur may not
be hemodynamically significant but may signify the presence of a calcified leaflet that is susceptible
to infection during a transient bacteremia.
•! Procedures that require antibiotic prophylaxis in high-to-moderate risk patients are as follows:
•! Invasive manipulation of the respiratory tract (eg, tonsillectomies, rigid bronchoscopy)
•! Gastrointestinal surgery, biliary tract surgery, sclerotherapy of esophageal varices, dilatation
of esophageal strictures, and endoscopic retrograde cholangiopancreatography in the presence
of biliary obstruction
•! Prostate surgery, cystoscopy, and urethral dilatation

5
•! Generally, hysterectomies, vaginal delivery, cesarean delivery, urethral catheterizations, dilation and
curettage, therapeutic abortions, sterilization procedures, insertion or removal of intrauterine devices,
cardiac catheterizations, angioplasties, or endoscopies with or without biopsies do not require
prophylaxis.
•! Adult antimicrobial IE preventive regimens for dental, oral, respiratory tract, or esophageal
procedures recommended by the American Heart Association to prevent streptococcal IE from oral-
dental sources are as follows:
•! Administer amoxicillin at 2 g orally 1 hour before the procedure or ampicillin at 2 g IM or IV 30
minutes before the procedure.
•! If the individual is allergic to penicillin, clindamycin at 600 mg, cephalexin at 2 g, or
azithromycin at 500 mg orally 1 hour before the procedure are alternatives.
•! If the individual is allergic to penicillin and is unable to take oral medication, clindamycin at 600
mg IV or cefazolin at 1 g IM or IV should be given 30 minutes before the procedure.
•! Adult IE prophylactic regimens for individuals undergoing lower gastrointestinal tract surgery or
instrumentation of genitourinary tract procedures are for preventing enterococcal endocarditis. They
are as follows:
•! High Risk regimens recommended by the American Heart Association are ampicillin at 2 g IM
or IV plus gentamicin at 1.5 mg/kg (not to exceed 120 mg) within 30 minutes of the procedure,
followed by ampicillin at 1 g IM, IV, or orally 6 hours later.
•! High-risk individuals who are allergic to penicillins should receive vancomycin at 1 g IV over
1-2 hours plus gentamicin at 1.5 mg/kg IV or IM (not to exceed 120 mg) within 30 minutes of
starting the procedure.
•! For moderate-risk patients, amoxicillin at 2 g orally 1 hour before the procedure or ampicillin
at 2 g IM or IV within 30 minutes of starting the procedure is recommended.
•! The alternative for patients who are allergic to penicillin who are at moderate risk is vancomycin
at 1 g IV over 1-2 hours, completed 30 minutes before the procedure.

What is the prognosis?


!! Depends on underlying cause
!! If IHD or dilated CMP, Px dependent on the underlying disease
!! If due to MVP
!! Asypmtomatic regurgitation is a serious disease with a 5-yr death rate of 22-33% form cardiac
adverse events

Mitral Valve Prolapse (Floppy MV, Barlow’s syndrome, Click-murmur syndrome)

Presentation

Sir, this patient has got a MR that is severe and secondary to a mitral valve prolapse.

I say this because there is a presence of a mid-systolic click associated with a late systolic crescendo-
decrescendo murmur heard best at the apex.

This murmur radiates towards the axilla and is a grade…..(present as for MR)

I would like to complete my examination by asking the patient to perform the valsalva manoeuvre as well as
to stand to accentuate the murmur; take BP and temperature chart as well as a neurological examination for
signs of stroke.

6
Questions

What causes a mid-systolic click?


o! Inability of the papillary muscles or the chordae tendinea to tether the mitral valves in the late stages
of systole
o! The prolapsing of the valve leaflet into the LA and sudden tensing of the mitral valve apparatus causes
the mid-systolic click

What are the differential diagnoses for a systolic murmur?


o! AS
o! PS
o! MVP
o! HOCM

What are the causes and associations of a MVP?


o! Myxomatous degeneration of the mitral valve tissue
o! Associated with
o! Heart conditions
!! ASD (Secundum type)
!! Cardiomyopathy
!! Myocarditis
o! Systemic conditions
!! Marfan’s syndrome
!! Ehlers danlos syndrome
!! Osteogenesis Imperfecta
!! Polycystic Kidney disease
!! SLE

What manoeuvres can accentuate the findings of an MVP and why?


o! Valsalva manoeuvre and standing
o! Decrease preload
o! Reduction in the cardiac volume
o! Further imparing the papillary muscles or chordae tendinea from maintaining tension on the leaflets
and preventing the leaflets from prolapsing into the LA
o! Hence the systolic click occurs earlier with a longer duration of the systolic murmur

How do patients with MVP present?


o! Asymptomattic
o! Symptomatic
o! Palpitations
o! Anxiety
o! Atypical chest pain
o! Light-headedness
o! Complications of MR
o! CCF – fatigue and dyspnea
o! IE
o! Arrythmias
o! Embolic phenemenon
o! Sudden death

How would you Ix?


o! Echocardiogram
o! Confirm Dx
7
o! Complications of MR

How would you manage?


o! Education and reassurance
o! Medical
o! Antibiotic prophylaxis
!! Only if associated with MR
!! Otherwise not necessary if just MVP
o! Symptomatic
!! Palpitations Rx with beta blockers (benign ventricular ectopy)
o! Rx underlying cause or associations
o! Rx complications such as MR with CCF, IE or AF or TIA
o! Surgical
o! As for M

8
Mitral Stenosis

Presentation

Sir, this patient has mitral stenosis which is severe in nature with complications of infective endocarditis and
congestive cardiac failure.

There is presence of a mid diastolic murmur heard best at the apex and is accentuated in the left lateral
position. It is a grade 3/6 murmur and is not associated with any diastolic thrill. It is severe as it is associated
with an early opening snap and a long mid diastolic murmur. The first heart sound is also loud. The apex
beat is tapping in nature and is not displaced, located just medial to the mid-clavicular line in the 5th
intercostal space.

There is associated pulmonary hypertension with a palpable pulmonary component of the second heart sound
with left parasternal heave. There is a loud pulmonary component of the second heart with a presence of a
functional TR as evidence of PSM at the LLSE that is louder with inspiration associated with a giant V wave
which is elevated at 5cm. There is no associated Graham Steel murmur (PR).
This is associated with congestive cardiac failure with bilateral basal crepitations and bilateral pedal edema.

Examination of the peripheries reveals evidence of stigmata of infective endocarditis. Patient is clubbed with
Janeway lesions on the palms and Osler’s nodes noted on the pulp of the fingers. There are also splinter
haemorrhages with presence of conjunctival pallor. There is presence of a peripherally inserted central
catheter suggesting use of long term antibiotics. The patient is on IV cloxacillin suggesting that the infective
organism is MSSA. There is complication of atrial fibrillation. The HR is irregularly irregular with a rate of
84 bpm. There is a characteristic pulsus parvus pulse. There are no bruises to suggest overanticoagulation.

I did not notice any mitral facies. The patient’s voice is also not hoarse which may suggest Ortner’s
syndrome.

(mention the lateral thoracotomy scar with possible mitral valvotomy as intervention)

I would like to complete the examination by looking at the patient temperature chart as well as taking the
patient’s BP.

In summary, this patient has MS that is severe in nature with complications of atrial fibrillation, pulmonary
hypertension congestive cardiac failure and infective endocarditis secondary to MSSA. The possible causes
for MS include rheumatic heart disease or congenital parachute valves.

Questions

How do you grade the severity of MS clinically?


•! Mild – no PHT
•! Moderate – PHT
•! Severe - CCF

What are your differentials for a mid-diastolic murmur?


!! MS
!! Atrial myxoma
!! Ball-valve thrombosis
!! Flow across the TV in ASD
!! MR with increased flow through the mitral valve during diastole
!! Austin Flint murmur (in severe AR)

9
What are the causes of mitral stenosis?
!! Common
!! Rheumatic heart disease
!! Congenital parachute valve
!! Rare
!! Calcification of mitral annulus and leaflets
!! CTDs: SLE, RA
!! Carcinoid (malignant)

What causes a tapping apex beat?


!! An accentuated first heart sound

What causes an opening snap?


!! Opening of a stenosed mitral valve and indicates that leaflets are pliable

Why is the first heart sound loud?


!! The mitral valve is held open during diastole by the transmitral gradient. The valve is suddenly slammed
shut during ventricular contraction

What causes presystolic accentuation of the murmur?


!! Occurs in sinus rhythm only during atrial systole which increases flow from the LA to the LV through
the stenotic valve

How do patients present?


!! Asymptomatic
!! Women, may have h/o RH heart disease
!! Symptoms ppt especially during pregnancy or development of AF
!! Usually left-sided heart failure: exertional dyspnea, PND, orthopnea
!! Less frequently with right sided heart failure, hemoptysis and hoarseness of voice, although this
symptoms are more specific

How do you diagnose rheumatic fever?


!! Duckett-Jones criteria
!! 2 major or 1 major + 2 minor
!! Major
!! Carditis, Sydenham’s chorea, SC nodules, erythema marginatum, arthritis
!! Minor
!! Past H/o RHD, fever, arthralgia, prolonged PR, ESR, CRP
!! + a h/o streptococcal infection (ASOT raised, recent scarlet fever, +GpA
Strep throat c/s or kits for GpA Strep with high specificity but low sensitivity.

How do you manage Rheumatic fever?


•! Prevention
o! Primary prevention: Rx with IM Benzathine Pen G 1 dose or 10 days of Pen V
o! Secondary prevention: all patient with history of Rh fever should receive prophylaxis; IM Pen
G once/month or PO Pen V daily bd

What are the signs of severity for MS?


!! Early opening snap
!! Long MDM
!! Pulmonary hypertension
!! CCF
!! Pulsus parvus

10
What is Ortner’s syndrome?
!! Hoarseness of voice from compression of the recurrent laryngeal nerve from an enlarged left atrium

What is Lutembacher’s syndrome?


!! Association of MS with ASD

How would you investigate?


!! ECG
!! P mitrale
!! P pulmonale, RVH, RAD,
!! AF
!! CXR
!! Calcified mitral valve
!! Enlarged LA (double silhoutte sign, straightening of the left heart border, filling of the pulmonary
bay by enlarged LA, horizontalization of the left bronchus)
!! Prominent pulmonary trunk
!! Pulmonary congestion (Upper lobe diversion, Kerly B lines)
!! Echocardiogram
!! Dx
!! Assess severity (Valve area calculation and transmitral gradient)
!! Look for complications (eg IE)
!! Assessment suitabilty for balloon valvotomy

What is the normal cross-sectional area of the valve?


!! 4-6 cm2

What is a significantly stenosed mitral valve?


!! <1 cm2 and >10 mmHg gradient across the valve

How would you manage?


!! Education
!! Medical treatment
!! Antibiotic prophylaxis
!! Treat complications
!! AF – anticoagulation and rate control
!! CCF – symptomatic treatment vs improvement of mortality
!! Surgery
!! Indications
!! Symptomatic (limit activity) with significant stenosis (<1 cm2 and >10mmHg)
!! Pulmonary hypertension
!! Hemoptysis
!! Recurrent thromboembolic events despite adequate anticoagulation
!! Type
!! Closed valvotomy
!! Closed mitral valvotomy
!! Balloon valvuloplasty (procedure of choice) but need to satisfy
!! Good mobility of valves
!! Minimal calcification
!! No or mild MR
!! Minimal subvalvular disease
!! Open valvotomy
!! Mitral replacement

In which trimester does pregnancy results in symptomatic MS?


!! Second trimester due to increase in blood volume
11
Aortic Regurgitation

Examination

Proceed as per cardiovascular examination


On detecting AR, to examine eyes for Argyll-Robertson pupil and auscultate the femorals for pistols shots
(Traube sign) and Duroziez sign.

Presentation

Sir, this patient has aortic regurgitation that is severe. My findings are:
Presence of a high-pitched early diastolic murmur heard best at the left lower sternal edge and is loudest at
end expiration with the patient sitting forwards. It is a grade 4/6 murmur and is associated with a diastolic
thrill. It is severe as the murmur is of a long duration associated a soft second heart sound with a S3 present.
There is also an Austin Flint murmur with presence of a mid diastolic murmur heard at the apex not associated
with an opening snap.
The apex beat is displaced at the 6th IC anterior axillary line and is thrusting in nature.
This is associated with evidence of CHF with bibasal crepitations, raised JVP at 4 cm with a prominent V
wave as well as bilateral peal edema.

Peripheral examination showed no evidence of IE. The pulse is bounding and collapsing in nature at a rate
of 90 bpm in SR. There is no RR or RF delay to suggest coarctation of the aorta. In addition, quinke’s sign
was negative.
There was no conjunctival pallor but Corrigan’s sign and brachial dance were present. Muller’s sign,
Duroziez and Traube’s signs were not detected.

In terms of etiology, there is no evidence of symmetrical deforming polyarthropathy to suggest RA and


patient does not have a Marfanoid habitus or a high arched palate. There is no Argyll-Robertson pupil to
suggest lewitic disease. To complete my examination, I would like to take patient’s BP looking for wide
pulse pressure and severe hypertension. I would also want to look at the patient’s temperature chart.

In summary, this patient has got AR that is severe with complication of CCF. Possible causes for this patient’s
AR are Rh heart disease, infective endocarditis or congenital bicuspid valve.

Questions

What are the signs of severity of AR?


!! S3
!! Austin Flint murmur (functional mdm at the apex due to regurgitant jet striking the anterior leaflet of the
MV, therefore obstructing flow from the LA into the LV)
!! Soft S2
!! Duration of the decrescendo murmur and loudness of murmur (cf with AS)
!! Apex beat displaced and thrusting
!! CCF
!! Wide pulse pressure
!! Hill’s sign

What are the characteristic signs of AR?


!! Collapsing pulse
!! Brachial dance
!! Quinke’s sign (visible capillary pulsation in the nail bed)
!! Corrigan’s sign (Visible Carotid pulsation in the neck)
!! De Musset’s sign (head nodding in time with the heart beat
!! Muller’s sign (pulsation of the uvula)
!! Traube’s sign(pistol shots) and Duroziez sign(to and fro murmur on sl compression of the femoral artery)
12
What are the causes of a collapsing pulse?
!! AR
!! PDA
!! An aortopulmonary window
!! A ruptured aneurysm of the aortic sinus
!! Active Paget’s
!! High fever
!! Severe Anaemia
!! Pregnancy

What would you expect to find on taking this patient’s blood pressure?
!! Wide pulse pressure
!! Severe hypertension (with functional AR)
!! UL and LL discrepancy with systolic in LL>UL = Hill’s sign

How do you differentiate an Austin Flint murmur from mitral stenosis?


!! Opening snap
!! Loud S1
!! Tapping apex beat, which is not displaced

What are the causes of AR?


!! Valvular
!! Rh, IE and congenital biscupid valve (associated with CoA)
!! Aortic root dilatation
!! Syphilis, RA, AS, Marfan, severe hypertension
!! Acute causes
!! IE, trauma, Aortic dissection, rupture of sinus of valsalva

How would you investigate?


!! ECG – LVH with diastolic overload pattern – deep but narrow Q, isoelectric ST, and tall T waves in left
praecordial leads
!! CXR – valvular calcification, cardiomegaly, pulmonary congestion, widened aorta
!! 2D echo
!! Confirm Dx
!! Assess cause
!! Severity
!! Complications

How would you manage this patient?


!! Education
!! Medical
!! Antibiotic prophylaxis
!! Treat underlying cause
!! Treat complications such as CCF, IE
!! Vasodilators – ACE and CCB
!! Surgical
!! Indications
!! Symptomatic – CCF, angina and severe AR
!! LV ESD >55mm
!! Aortic root >55mm
!! Reduction of EF >5% on exercise
!! Types of surgery

13
Aortic Stenosis

Presentation

Sir, this patient has Aortic stenosis that is severe in nature. My findings include:

Presence of an ejection systolic murmur heard best at the aortic area and radiates to the carotids. It is a grade
4/6 systolic murmur a/w with a systolic thrill. It is severe as there is an early ejection click a/w a long systolic
murmur with delayed peaking of the murmur. I could not detect an S4 and the second heart sound is soft.
There was also no paradoxical splitting of the second heart sound.
The apex beat is heaving in nature and is displaced, located at the 6th IC space at the just lateral to the mid-
clavicular line.
This is associated with signs of congestive cardiac failure as evidenced by presence of bibasal crepitations,
raised JVP at 3 cm with prominent V wave and bilateral pedal edema but she does not require supplemental
oxygen.

Peripheral examination does not reveal any stigmata of IE. The pulse is regular at 84bpm and is
anacrotic/pulsus parvus et tardus in nature. There are no features suggestive of haemolytic anaemia with no
conjunctival pallor and patient is not jaundice.

I would like to complete my examination by taking the patient’s blood pressure to look for a narrow pulse
pressure as well as his temperature chart. I would also like to enquire on patient’s symptoms of angina,
syncope and dyspnea as these are important prognostic markers.

In summary, this patient has got aortic stenosis that is severe in nature with complication of congestive
cardiac failure. There is no evidence of infective endocarditis or haemolytic anaemia. The most likely causes
include Rh heart disease, calcified biscupid aortic valve or degenerative calcified aortic valves.

Questions

What are the differential diagnoses for an ejection systolic murmur?


•! AS
•! PS
•! HOCM
•! MVP/MR
•! Coarctation

How do you differentiate between them?


•! AS and PS – expiration and inspiration
•! AS and HOCM – Valsalva, squatting
•! AS and MVP – location and clicks
•! AS and Coarctation – differential pulse

What are the types of pulses associated with aortic stenosis?


•! Pulsus parvus et tardus – means low volume pulse with delayed upstroke due to a reduction in systolic
pressure and a gradual decline in diastolic pressure
•! Anacrotic pulse – small volume pulse with a notch on the upstroke

What does a normal pulse volume in AS mean?


•! The travsvalvular gradient is <50 mmHg

What does a palpable systolic thrill implies?


14
•! It means that the transvalvular gradient is > 40mmHg

What does the second heart sound indicate about the aortic stenosis?
•! Soft second heart sound means poorly mobile and stenotic valve
•! Reversed splitting means mechanical or electrical prolongation of ventricular systole; S2 is normally
created by the closure of the aortic valve followed by the pulmonary valve, if the closure of the aortic
valve is delayed enough, it may close after the pulmonary, creating an abnormal paradoxical splitting
of S2.
•! Single second heart sound implies fibrosis and fusion of the leaflets
•! Normal second heart sound implies insignificant stenosis

What is Gallavardin phenomenon?


•! Systolic murmur may radiate towards the apex, which may be confused with a MR murmur

How can haemolytic anaemia result from aortic stenosis?


•! MAHA from severely calcified aortic valve

What are the causes of aortic stenosis?


•! Rheumatic heart disease (<60)
•! Degenerative calcification (>75)
•! Calcified biscupid (60-75, males)

What are the severity markers?


•! Early ejection click
•! Long Systolic murmur
•! Late peaking of the murmur
•! 4th heart sound
•! Paradoxical splitting of S2
•! Heaving apex beat which is displaced
•! Systolic thrill
•! Pulsus parvus et tardus
•! Narrow pulse pressure
•! Symptoms (ASD) Px
Angina 5 years
Syncope 3 years
Dyspnea (Most impt) 2 years

How do you differentiate AS from aortic sclerosis?


•! No severity signs as above
•! ESM which is localised to aortic area with a normal S2 in elderly person

How do patients present?


•! Asymptomatic and incidental finding
•! Angina
o! Increase oxygen requirement for hypertrophied LV with hypoperfusion of the subendocardial
myocardium
•! Syncope
o! Cardiac arrythmias
o! Peripheral vasodilatation eg post exercise without concomitant increase in CO
o! Transient elctromechanical dissociation
•! Dyspnea
o! Implies LV dysfunction and heart failure
How would you investigate?
15
•! ECG – LVH with strain, 1st degree heart block, LBBB
•! CXR – Calcified aortic valve, cardiomegaly, pulmonary congestion
•! 2D echo
o! Dx
o! Severity
!! LVH, EF
!!
Severity Area Transvalvular gradient
Mild >1.5 <25
Moderate 1-1.5 25-50
Severe <1 50-80
Critical <0.7 >80
o! Complications eg IE

How would you manage?


•! Education
•! Medical
o! Antibiotic prophylaxis
o! Rx complications such as arrythmias and CCF (caution with antihypt to avoid reducing
preload)
o! Statins may have a role in reducing calcification of the aortic valve
•! Surgical treatment
o! Indications
!! Symptomatic and severe
!! Asymptomatic but has
•! Area<0.6
•! LV systolic dysfunction
•! Hypotension on exercise
•! VT
•! LVH>15mm
!! Moderate AS but going for Sx for CABG, MVR or aortic root surgery
o! Options
!! Valve replacement (Sx of choice)
!! Valvuloplasty (for moribund patients)

What are your thoughts on a young person with AS murmur but a normal aortic valve?
•! Supravalvular stenosis
o! Can be isolated or associated with Williams syndrome
o! It is an inherited disorder, autosomal dominant, Ch 7
o! Features of elfin facies, hypertension and mental retardation with other cardiac lesions such
as PS
•! Subvalvular stenosis

What abdominal condition is associated with AS?


•! Angiodysplasia of the colon (PR bleed)

What is pulse pressure?


•! Difference between systolic and diastolic pressure
•! Normal – 40mmHg
•! Wide - >60 mmHg
•! Narrow - <25mmHg
•! Note: There is no official definition but studies usually measures the pulse pressure as a
continuum
16
Mixed Mitral Valve disease

Presentation

Sir, this patient has mixed mitral valve disease and the predominant lesion is
1.! Mitral stenosis
•! Early opening snap with long diastolic murmur
•! Loud S1
•! No S3
•! Tapping apex beat that is not displaced
•! Pulsus parvus pulse
2.! Mitral Regurgitation
•! Displaced apex beat that is thrusting in nature
•! Soft S1
•! S3
•! Jerky pulse
•! Mdm is a short diastolic murmur with no opening snap

My findings are:
o! Presence of a MDM heard best at the apex and accentuated at the left lateral position. It is a grade 4/6
murmur as it is associated with a diastolic thrill. It is severe as it is associated with an early opening
snap and a long mdm. There is a loud first heart sound. There is also a MR murmur as evidenced by
a PSM heard best at the apex. It is a grade 3/6 murmur and is not associated with any systolic thrill.
There is also no third heart sound.
o! Apex beat is not displaced
o! Pulmonary hypertension, CCF
o! IE, AF, over-anticoagulation
o! Mitral facies and Ortner’s
o! No lateral thoracotomy scars to suggest previous operation for MS
o! Requests
o! In summary, this patient has mixed mitral valves disease with the predominant lesion being MS. This
is complicated by pulmonary hypertension and AF. He is not in heart failure and signs of IE. The
most likely cause in this patient is rheumatic heart disease.

o! Presence of a PSM heard best at the apex; 4/6 murmur and is associated with a systolic thrill. There
is a soft first hearts sound and a presence of a third heart sound. There is also a MS as evidenced by
MDM heard best at the apex at the left lateral position. It is associated with a late opening snap and
short mdm.
o! Apex
o! CCF, Pulmonary hypt
o! IE, AF
o! Lateral thoracotomy scar
o! Marfan’s and SLE
o! Requests
o! In summary

Questions

What is the significance of a third heart sound in mixed mitral valve disease?
o! Presence of S3 implies no significant MS

What are the causes?


o! Rheumatic heart disease
o! MS with valvotomy done that has been complicated by MR
17
What are the frequencies of valvular involvement in rheumatic heart disease?
o! MV – 80%
o! AV – 50%
o! Mixed MV and AV – 20%
o! TV – 10%
o! PV -1%

Mixed Aortic Valve Disease

Sir, this patient has got mixed aortic valve disease and the predominant lesion is
Aortic Stenosis
1.! small volume pulse
2.! heaving and undisplaced apex beat
3.! Loud and harsh systolic murmur
4.! associated with systolic thrill

Aortic regurgitation
1.! Collapsing pulse
2.! Displaced and thrusting apex beat
3.! Soft systolic murmur
4.! No systolic thrill

My findings are:
o! Presence of an ESM heard best at he aortic area that radiates towards the carotids. It is a grade 4/6/
murmur and it is associated with a systolic thrill. It is severe as it is associated with an early ejection
click with a long systolic murmur with late peaking. There is no S4 detected and the second heart
sound is soft; I could not detect a paradoxial splitting of the second heart sound.
o! There is also an EDM heard bset at the LLSE and is loudest in expiration with the patient sitting
forwards. It is a grade 3/6 murmur and is not associated with any diastolic thrill. (skip the severity
markers for AR)
o! Apex
o! CCF
o! IE, SR and small volume, haemolytic anaemia
o! Request BP especially for narrow pulse pressure, Temperature chart and enquire symptoms of angina,
syncope and dyspnea.
o! Presence of an EDM heard best at the LLSE and is loudest in expiration with the patient sitting
forwards. It is a grade 3/6 murmur as it is not asssociatd with ay diastolic thrill. The second heart
sound is soft and there is no third hear sound. There is also no mdm at the paex to suugest an Austin
Flint murmur.
o! There is also an ESM heard best at the aortic area that radiates towards the carotids. It is a grade 2/6/
murmur and is not associated with any systolic thrill. (skip the severity markers for AS)
o! Apex
o! CCF
o! IE, SR, collapsing, brachial dance and Corrigan’s
o! No quinke, muller’s de Musset’s, Duroziez and Traube’s
o! No Marfans, AS or RA
o! Requests for BP especially for a wide pulse pressure, temperature chart.
o! In summary

Questions

What are the causes of a mixed aortic valvular lesion?


o! Rheumatic heart disease
o! Biscupid aortic valve
18
Prosthetic Heart Valves

Sir, this patient has got mechanical mitral/aortic valve which has been done for an underlying mitral/aortic
stenosis/regurgitation.

I say this because there presence of a mid-line sternotomy scar associated with audible metallic clicks to the
unaided ear.

There is presence of a mitral valve replacement with a metallic first heart sound and a normal second heart
sound. There is no pan-systolic murmur to suggest a valve leakage.

(There is presence of an aortic valve replacement as evidenced by a normal first heart sound followed by a
metallic click and a metallic second heart sound. There is no early diastolic murmur or a collapsing pulse to
suggest a valve leakage.)

(There are both mitral and aortic valve replacement as evidenced by dual metallic heart sounds. There is no
pan-systolic murmur to suggest a mitral valvular leakage or an early diastolic murmur which indicates an
aortic valve leakage.)

Ther metallic sounds are crisps (no valvular thrombosis) and there is no conjunctival pallor or jaundice to
suggest hemolytic anaemia. The apex beat is displaced at the 6th IC at the ant axillary line. (Displaced and
MVR = MR; undisplaced and MVR = MS; Displaced and AVR = AR). There is no evidence of pulmonary
hypt(MVR). Patient is in CCF as evidenced by presence of bibasal crepitations, raised JVP of 3 cm and
bipedal edema.

Patient is not in AF(MVR) and pulse is not collapsing in nature (mention this if AVR for leakage). There is
no peripheral stigmata of IE such as clubbing, Janeway’s lesion, Osler’s nodes or splinter haemorrhages.
This is associated with bruises which suggest overanticoagulation.

There is no evidence of any Marfan’s, RA, AS or Syphilis (mention this if AVR for AR or MVR for MR)

I would like to complete my examination by taking the BP of the patient and looking at his temperature chart
and neurological examination for strokes.

In summary, this patient has got MVR/AVR or both which is most likely done for MR/MS/AR/AS (which
is due to underlying Marfan’s syndrome). There is no clinical evidence of valvular leakage, thrombosis or
haemolytic anaemia. There is also no pulm hypt but pt is in heart failure and in AF. There are no signs of IE
or overanticoagulation.

Questions

What are the indications of a mitral/aortic valve replacement?


o! See respective MS/MR/AS/AR

What are the types of prosthetic valves?


!! Mechanical valves
!! Ball and cage valve (Starr-Edwards)
!! Single tilting disc (Bjork-Shiley)
!! Double tilting disc (St Jude)
!! Bioprosthetic – Homograft or heterograft

What are their differences?


!! Duration
!! Mechanical valves last 20-30yrs
!! Bioprosthetic may fail within 10-15 years
19
!! Thrombogenecity
!! Mechanical require lifelong anticoagulation (Starr-Edwards>single disc>double disc)
!! Bioprosthetic does not require lifelong anticoagulation

!! Therefore in the young and those who already require long term anticoagulation, mechanical valves
preferred
!! And in the elderly(lifespan <10-15 years) or those that cannot tolerate anticoagulation, bioprosthetic
valve preferred

What are the complications?


!! Complications of prosthesis
!! Valve leakage (mild- hemolytic anaemia, severe – CHF)
!! Valve thrombosis
!! Valve strut failure (rare, acute presentation with high mortality, Bjork-Shiley)
!! Hemolytic anaemia (from valvular leakage due to partial dehiscence; Rx with Fe, folate, transfusions,
B blockers or if fit for op, repair of valve replacement)
!! Complications of valvular heart disease
!! Infective endocarditis
!! Congestive cardiac failure
!! Thromboembolism (rule out IE and thrombosis)
!! Complications of management
!! Overanticoagulation
!! Bleeding

What are the causes of anaemia in such patients?


!! Bleeding from anticoagulant
!! Hemolytic anaemia
!! Infective endocarditis

How do you tell clinically that the valve has malfunction?


!! New murmur
!! Change in characteristic of a preexisting murmur
!! Change in intensity or characteristic of an audible sound

How would you investigate a patient suspected of having valve dysfunction?


!! Cinefluoroscopy – rapid, fast ad inexpensive for structural integrity
!! TTE – often difficulty study due to reverberations from the metal
!! TEE – useful for assessing MV prosthesis but limited in AV prosthesis

Can MRI be done for a patient with mechanical heart valves?


!! Yes it is safe except those with pre 6000 Starr-Edwards prosthesis (1960-64)

Valve thrombosis
!! Up to 5% per patient-year
!! Factors – inadequate anticoagulation and mitral location
!! Manisfest as
!! pulmonary congestion, poor peripheral perfusion or systemic embolisation, acute deterioration
!! Change in audible sounds or murmur
!! Ix shows reduced movement of the disc or poppet, reduced orifice area, increased regurgitation or
transvulvular pressure
!! Mx
!! <5mm – IV heparin
!! >5mm – Fibrinolysis (if high operative mortality) or valve replacement

20
VSD
(Clue: Young patient; look for associated conditions of Down and Fallot’s Tetrology)

Presentation

Sir, this patient has got a Ventricular Septal Defect that is hemodynamically significant as evidenced by:

Presence of a pan-systolic murmur heard best at the left lower sternal edge with radiation towards the right
side of the sternum. This murmur can be heard at the apex but there is no radiation to the axilla. It is louder
on expiration. It is a grade of 5/6 murmur and is associated with a systolic thrill. The first heart sound is not
soft. I did not detect any third heart sound.

I did not detect any early diastolic murmur at the left lower sternal edge to suggest an associated AR. This is
also no associated mid-diastolic apical rumble at the apex to suggest a flow murmur at mitral valve which
can be a/w VSD.

Apex beat is displaced and is located at the anterior axillary line at the 6th IC. It is thrusting in nature.

No evidence of Eisenmenger’s syndrome such as central cyanosis, clubbing. There is evidence of pulmonary
hypertension such as palpable or loud P2, no parasternal heave.

There are no signs of CCF such as bilateral pedal edema, no basal crepitations or raised JVP; she is
comfortable at rest with a RR of 14 bpm and does not require any supplemental oxygen.

There is no peripheral stigmata of IE. The pulse is regular at 70 bpm and character of the pulse is normal.
There are no features to suggest Down syndrome or (Turner syndrome - if female).

I would like to complete my examination by looking at the patient’s temperature chart and taking his blood
pressure.

In summary, this young man has got a VSD that is severe with a displaced apex beat and is complicated by
pulmonary hypertension. Clinically, there is no heart failure or Eisenmenger’s syndrome or IE. The most
likely cause is congenital VSD.

Dy/Dx – MR, TR, VSD – For VSD, murmur radiates to the right of the sternum, young patient and a palpable
thrill

Questions

What are your differential diagnoses for a PSM?


o! MR – PSM at apex radiates towards the axilla, soft S1
o! TR – PSM heard at the triscupid area, louder on inspiration; usually secondary to pulmonary
hypertension or seen in IVDAs; Giant V wave, pulsatile liver
o! VSD – PSM heard at the LLSE which is louder on expiration

What are the causes of a VSD?


o! Congenital
o! Acquired
o! MI

How common is VSD?


o! The most common congenital heart condition
o! 2 per 1000
o! Usually in the membranous portion (can also be found in the muscular)
21
o! Small defects close spontaneously in early childhood in about 50%
What are the types of VSD?
o! Supracristal (above the crista supraventricularis)
o! Infracristal
o! Upper membranous
o! Lower muscular (<5%)
o! Different morphology
o! Maladie de Roger
o! Swiss cheese
o! Large
o! Gerbode defects (opens into the RA)

What are the conditions in which VSD is part of?


o! Fallot’s tetralogy
o! Truncus arteriosus
o! AV canal defects
o! DORV (double outlet RV)

What are the complications of VSD?


o! AR
o! Pulmonary Hypt
o! Eisenmenger’s complex
o! CCF
o! IE

Does the loudness of the murmur correlate with severity?


o! No; in fact, a small VSD results in a louud murmur and the converse is true.

What is Maladie de Roger?


o! A term used to describe a small VSD that is hemodynamically insignificant with normal heart size,
ECG and CXR; it is a loud murmur on ausculatation

How do you differentiate an isolated VSD with one that is associated with Fallot’s tetralogy?
o! Pulmonary thrill, PS murmur
o! Clubbed and central cyanosis (but could be VSD with Eisenmenger’s)

How do you differentiate VSD from HOCM?


o! ESM rather than PSM
o! Apex is not displaced, double apical impulse
o! Jerky pulse

How would you Ix?


o! ECG
o! Normal in small defects
o! LVH, RVH, p mitrale
o! Pulmonary hypertension – P pulmonale, RAD
o! CXR
o! Normal in small defects
o! Cardiomegaly, LA and LVH
o! Pulmonary plethora initially
o! Pulmonary hypertension later with prominent pulmonary trunks, rapid tapering of the
peripheral pulmonary arteries and oligaemic lung fields
o! CCF
o! Echocardiogram
o! Diagnostic
22
o! Determine severity and direction of shunt via color doppler

How would you manage?


o! Counsel
o! Medical
o! Antibiotic prophylaxis
o! Rx complications of CCF
o! Surgical
o! Small, asymptomatic and normal pulmonary pressure do not need surgery
o! Indications
!! Evidence of pulmonary hypertension or CCF
!! Right ventricular pressure >50 mmHg
!! Right to left flow ratio or pulmonary to systemic resistance ratio >1.5
!! Recurrent IE
!! Cx by AR
!! Acquired cause eg rupture of septum form MI
o! Contraindication
!! Development of Eisenmenger
o! Types
!! Surgery
!! Percutaneous transcatheter

23
ASD

Presentation

Sir, this patient has atrial septal defect as evidenced by presence of a wide and fixed splitting of the second
heart sound.
There is presence of an ejection systolic murmur over the pulmonary area which is louder on inspiration,
implying presence of a pulmonary systolic murmur. This is a grade 3/6 murmur and there is no associated
systolic thrill.

There is no associated mid-diastolic flow murmur to suggest relative tricuspid stenosis or Lutembacher’s
syndrome (Acquired MS and ASD). There was also no associated PSM to suggest an ostium primum defect
(TR, MR, VSD).

The apex beat is not displaced and is located in the 5th IC space just medial to the mid-clavicular line.

There is no complication of Eisenmenger’s syndrome; there is no evidence of pulmonary hypertension; is


not clubbed and no central cyanosis. There is also no evidence of congestive cardiac failure.

There are no stigmata of infective endocarditis. Patient is in atrial fibrillation with an irregularly irregular
pulse and is rate controlled at a rate of 84 bpm; there are also no bruises to suggest over-anticoagulation.

There is no evidence of any thumb defects to suggest Holt-Oram syndrome. The patient also does not features
of Down’s syndrome.

I would like to complete my examination by examining patient’s chest for pneumonia as patients are prone
to recurrent chest infections as well as a neurological examination to look for evidence of stroke due to
paradoxical embolus.

In summary, this patient has got an ASD with complications of AF. There are no complications of pulmonary
hypertension, heart failure or Eisenmenger’s syndrome. There is also no infective endocarditis. This patient
has ASD is most likely due to an ostium secundum atrial septal defect which is a congenital heart condition.
Questions

What are the types of ASDs?


o! Ostium secundum type
o! 90%
o! common congenital heart condition
o! Most remain asymptomatic
o! If small <2 cm, normal life expectancy with no symptoms
o! Larger defects may present in the second or third decades with dyspnea or fatigue
o! defect in the fossa ovalis with no involvement of the AV valves
o! Ostium primum type
o! 10%
o! Failure of fusion of the septum primum with the endocardial cushions
o! AV valves affected – MR, TR and VSD
o! Sinus venosus type
o! Defect in the septum just below the entrance of the SVC (inverted P waves in the inferior
leads)

How do patients present?


o! Secundum
o! Asymptomtic
o! Symptomatic
24
!! Fatigue, dyspnea
!! Right heart failure
!! AF
!! Recurrent pulmonary infections
!! Paradoxical emboli
o! Primum
o! In addition to the above
!! Failure to thrive, poor development
!! IE
!! Syncope (heart block)

What are the complications of ASD?


o! Pulmonary hypertension, heart failure, Eisenmenger’s
o! AF, IE (primum defects)
o! Recurrent chest infection, paradoxical emboli

What are the various types of murmurs that can be associated with ASD and what do they mean?
o! Pulmonary ejection systolic murmur and mid-diastolic murmur at the triscuspid area implies
increased flow of blood through the pulmonary and triscupid valve respectively due to left to right
shunting of blood via the ASD
o! MS murmur means acquired Rh heart disease affecting the mitral valve in Lutembacher’s syndrome
o! MR, TR or VSD murmur implies that ASD is of the ostium primum type

What is the mechanism of a split second heart sound?


o! A split S2 is caused physiologically during inspiration because the increase in venous return overloads
the right ventricle and delays the closure of the pulmonary valve

Why is there wide and fixed splitting of the second heart sound in ASD?
o! With an atrial septal defect, the right ventricle can be thought of as continuously overloaded because
of the left to right shunt, producing a widely split S2, with the pulmonary valve closing much later cf
to the aortic valve
o! It is fixed because the atria are linked via the defect, inspiration produces no net pressure change
between them, and has no effect on the splitting of S2
How do you differentiate between a flow mumur through the pulmonary valve vs a PS murmur?
o! PS murmur is a/w P2 that is soft, delayed and varies with respiration

What are the conditions that can cause a wide splitting of the second heart sound?
o! Increase RV volume – ASD, VSD, PR
o! Increase RV pressure – PS
o! RV conduction delay –RBBB
o! Increase LV emptying – MR, VSD

What is Eisenmenger’s syndrome?


o! It implies a reversal of a left to right shunt as a result of the development of pulmonary hypertension
o! This occurs in conditions such as ASD, VSD or PDA
o! Patients are markedly clubbed and deeply cyanosed
o! The defect must not be repaired once this complication occur due to high mortality risk

What is Lutembacher’s syndrome?


o! Acquired Rh MS
o! ASD

What is Fallot’s trilogy?


o! ASD
o! RVH
25
o! PS

What is Holt-Oram syndrome?


o! ASD secundum type
o! Hypoplastic thumb with accessory phalanx
o! Autosomal dominant

How would you investigate?


o! ECG
o! Secundum – Partial RBBB, RAD
o! Primum – LBBB, LAD, low atrial rhythm
o! Sinus venosus – inverted P in inferior leads
o! Pulmonary hypertension – p pulmonale, RVH
o! CXR
o! Cardiomegaly
o! Pulmonary hypertension
!! Prominent pulmonary trunk
!! Enlarged RA and RV
o! Shunt vascularity/pulmonary plethora (well visualised pulmonary arteries in the periphery of
the lung
o! Small aortic knob
o! Echocardiogram
o! Diagnosis - demonstrate the defect
o! Cardiac catheterisation
o! Determine the severity and direction of shunt

How would you manage?


o! Counsel
o! Medical
o! No antibiotic prophylaxis required, repaired or unrepaired
o! Treatment of complications such as heart failure and AF
o! May consider anticoagulation if there is evidence of bidirectional shunting to prevent strokes
from paradoxical emboli
o! Surgery
o! Early childhood – closure is recommended at 5-10 years of age to prevent complications
o! Small ASDs can be left alone(5 mm or less)
o! Large ASDs or pulmonary to systemic flow ratio>1.5
o! Closed surgically or transcatheter button or clam-shell devices
o! Closure prevents pulmonary hypertension and RHF but does not alter incidence of AF

How would you counsel a patient with ASD who intends to get pregnant?
o! Pregnancy is well tolerated in patients with small and hemodynamically insignificant ASD
o! For large defects with pulmonary hypertension, Eisenmenger’s syndrome, avoid pregnancy as there
is increase morbidity and mortality both to fetus and mother
o! Routine closure before pregnancy as complications of progressive pulmonary vascular disease may
develop

26
HOCM

Presentation

Sir, this patient has got Hypertrophic obstructive cardiomyopathy.


There is presence of a ESM heard best at the LLSE. It is a grade 3/6 murmur as it is not associated with any
systolic thrill. In addition, there is presence of a MR mumur with a PSM heard bst at the ap3ex beat and
radiates to the axilaa. It is a grade 4/6/ murm,ru as it is associated with a systolic thrill. The first heart sound
is soft and there is no associated third or fourth heart sounds. The apex beat is not displaced located at the 5th
IC space just medial to the midclavicular line. It has a double apical impulse (say this only if not if AF).
There are no complications of congestive cardiac failure. However the JVP is raised at 3 cm with a prominent
‘a’ wave.

Examination of the peripheries did not show any stigmata of infective endocarditis. He is is SR at a pulse
rate of 84 bpm and has a characteristic bifid pulse (only if not in AF; if in AF, say shrap, rising and jerky
pulse). I did not notice any clinical features to suggest Friederich’s ataxia.

I would like to complete my examination by taking performing the valsalva manoeuvre or standing to
accentuate the murmurs as well as take the patient’s blood pressure and look for fever from the temperature
chart. A neurological examination would be useful to screen for any signs of stroke.

In summary, this patient has got a HOCM with an ESM and MR murmur associated with a double apical
impulse, bifid pulse and a raised JVP with prominent ‘a’wave. There are no complications of heart failure,
AF or IE. This is a genetic condition.

Questions

What is HOCM?
o! Hypertrophic Cardiomyopathy
o! Genetic cardiac disorder caused by missense mutation in the genes that encode proteins of the cardiac
sarcomere; autosomal dominant
o! Resulting in hypertrophy of the ventricular septum with LV outflow tract obstruction
o! 1 in 500, male:female 1:1
o! Variable penetrance
o! Variable expression
o! Asymptomatic (majority)
o! Symptomatic
o! Angina, syncope, dyspnea, palpitations
o! Sudden death (Ventricular fibrillation) (overall annual mortality in 1%)
o! Complications of CCF, AF, IE and thromboembolic stroke

Why is there a ‘double apical impulse’?


o! Presence of a LV heave with a prominent presystolic pulse caused by atrial contraction
o! A differential diagnosis is LV aneurysm

Why is there a prominent ‘a’ wave?


o! Due to forceful atrial contraction against a non-compliant right ventricle

What is Brockenbrough-Braunwauld-Morrow sign?


o! Reduced pulse pressure in the post-extrasystolic beat
o! Occurs in HOCM and AS

What are the causes of HOCM?


o! Familial
o! Friederich’s Ataxia
27
o! Idiopathic

How would you investigate?


o! ECG
o! Normal in 25%
o! Tall QRS in precordial leads with ST-T changes, Q in inf and lateral leads
o! LAD
o! AF
o! CXR
o! Normal
o! LA enlargement, LVH
o! Echocardiogram
o! Diagnostic
!! Asymmetrical septal hypertrophy
!! Systolic anterior motion of the anterior mitral valve leaflet
!! Diastolic dysfunction
o! Severity
!! Septal thickness >18mm
!! Outflow tract gradient > 40mmHg as rest
o! Complications
!! MR
!! IE
o! TMX for those with angina
o! Holter monitoring looking for arrythmias especially presence of VT

How would you predict poor outcome?


o! Family history of sudden death
o! History of syncope, cardiac arrest
o! Poor BP response to exercise
o! Holter monitoring with ventricual arrythmias detected, esp spontaneous VT
o! Echo findings

How would you treat?


o! Education and counselling with screening of first degree relatives
o! 50% chance of being affected
o! Screen with ECG and 2D echo
!! Annually for adolescent (12-18)
!! And 5 yearly
o! Treatment is directed at symptom relief and prevention of sudden cardiac death
o! Relief symptoms
!! Beta blockers
!! If cannot tolerate, verapamil but caution I patients with sever symptomatic obstruction
because of increase death especially after first few doses
!! Beta blockers and disopyramide
o! Rx complications
!! Rx CCF
!! Rx AF
!! Rx and prevention of IE
!! Prevention of sudden death
•! Amiodarone
•! Pacing (dual-chamber pacing)
o! Septal ablation with alcohol or surgery
o! Surgical septal myomectomy (Gold standard)

28
Approach to Central cyanosis and Clubbing

Examination
o! On detecting this, concentrate on
o! Differential cyanosis and clubbing (ULs vs LLs or right LL vs others where the LLs are cyanosed
and clubbed)
o! Look for weak L radial pulse (BT shunt)
o! Shunt scar (BT shunt)
o! On auscultation determine if
o! Eisenmenger
!! ASD, VSD, PDA
!! No PS
!! Has pulmonary hypertension (loud and palpable P2) and RVH
!! Check single (VSD) or fixed splitting (ASD)
o! Fallot’s tetralogy
!! PS murmur (No VSD murmur as this is non restrictive)
!! No pulmonary hypertension but has RVH

Presentation

o! Sir this patient has got VSD/ASD/PDA complicated by Eisenmenger’s complex.


o! Eisenmenger’s because - clubbed and central cyanosis and Pulmonary Hypertension
o! The underlying cause is ASD/VSD/PDA because - second heart sound is crucial
o! ASD – fixed spitting second heart sound
o! VSD – Single second heart sound
o! PDA – reversed splitting second heart sound (split on expiration)
o! No PS murmur to suggest ToF
o! Apex beat
o! CCF
o! IE stigmata
o! Pulse
o! Peripheral presentation for ASD/VSD/PDA
o! (For Eisenmenger’s syndrome, will have pulmonary hypertension and therefore look for TR and PR
murmur)
o! Also state complications of polycythaemia, venesection marks
o! Requests
o! Summary
o! Sir this patient has ToF with a BT shunt done previously
o! Clubbed and centrally cyanosis
o! Presence of PS murmur and shunt murmur
o! PS murmur – ESM heard best at the pulmonary area 4/6 and systolic thrill
o! Shunt murmur (continuous murmur)
o! (No VSD murmur as it is large and non-restrictive)
o! RVH with left parasternal heave
o! Apex beat
o! No pulmonary hypertension (no loud P2)
o! Cor pulmonale – raised JVP, pedal oedema, no lung crepitation
o! No IE
o! Pulse
o! Presence of a thoracotomy scar with a weak left radial pulse suggesting BT shunt
o! Venesection marks, polycythemia
o! Requests
o! In summary

29
Questions

How do you differentiate ToF vs Eisenmenger’s syndrome?


o! ToF has PS murmur with systolic thrill and a soft P2
o! ToF no pulmonary hypertension (CXR ToF has small pulmonary aretrial trunks)

What are the characteristic findings of a PDA?


o! Collapsing pulse
o! Continuos murmur heard best just below the left clavicle and radiates to the back

What are the differential diagnoses for a continuous murmur?


o! Collapsing pulse
o! PDA
o! MR with AR
o! VSD with AR
o! No collapsing pulse
o! BT shunt
o! Venous hum (right of the sternum, children, disappears when lie flat or right JVP occluded)

What is ToF?
o! Congenital heart condition comprising of
o! VSD
o! RVH
o! Overriding aorta
o! PS

30
Dextrocardia

Examination

After the routine examination


Request to examine the abdomen for a liver on the left side of the abdomen for situs inversus

Presentation

Sir this patient has dextrocardia as evidenced by:


o! Right apex beat
o! Heart sounds that are better heard on the right than on the left

The heart sounds are normal and there are no murmurs detected.
Apex is not displaced located at the right 5th IC just medial to the midclavicular line and has a normal
characteristic.

She is in SR with a rate of 84bpm

On examination of his lungs posteriorly, there was no evidence of coarse late inspiratory crepitations to
suggest bronchiectasis and patient does not have a nasal voice to suggest sinusitis. (Katargener’s syndrome)

There is no evidence of Turner’s syndrome. (mention this only if female!)

I would like to complete my examination examining the abdomen for a left sided liver for situs inversus.

In summary this patient has got dextrocardia and is well clinically and is of congenital etiology.

Questions
What is the significance of situs inversus in patients with dextrocardia?
o! It usually implies that there is no significant cardiac malformation

What conditions is dextrocardia associated with?


o! Kartagener’s syndrome – a type of immotile ciliary syndrome
o! Triad of
!! Bronchiectasis
!! Sinusitis, otitis media and dysplasia of the frontal sinuses
!! Infertility
o! Turner’s syndrome
o! Asplenia – PBF may show Heinz bodies and Howell-Juoly bodies

What is situs inversus?


o! Right sided apex and right descending aorta
o! Left lung having 3 lobes and right lung with 2 lobes
o! Left sided liver and right sided stomach
o! Right descending colon

What is dextroversion?
o! Right sided apex and left sided descending aorta
o! Left sided stomach

What is levoversion?
o! Left sided apex and right sided descending aorta
o! Right sided stomach
31
Bronchiectasis

Presentation

Sir, this patient has got bronchiectasis affecting both lower lobes as evidenced by late, coarse inspiratory
crepitations heard best posteriorly in the lower one third bilaterally. Patient has a productive cough with large
volume of purulent sputum with hemoptysis associated with clubbing.

Chest excursion was reduced bilaterally with a normal percussion note and vocal resonance. Trachea is
central and the apex beat is not displaced.
There are no signs to suggest presence of COPD.

(There is concomitant COPD with a reduced chest excursion bilaterally, hyperinflation of the chest associated
with hyperresonance on percussion with loss of liver and cardiac dullness. There is presence of ronchi and a
prolonged expiratory phase. Vocal resonance is normal. Trachea is central and apex beat is not displaced.)

There is complication of pulmonary hypertension with a loud and palpable component of the second heart
sound associated with a left parasternal heave. There is also cor pulmonale with a raised JVP of 3 cm with
prominent a wave associated with bilateral pedal oedema. Clinically there are no signs of polycythemia such
as plethoric facies or conjunctival suffusion.

He is not in respiratory distress (with a RR of 14 bpm without use of accessory muscles of respiration).
There are no signs of respiratory failure (he does not require any supplemental oxygen and there is no
central cyanosis; there is also no flapping tremor of the hands and no bounding pulse). There is also no
nicotine staining of the fingers, patient is not cachexic looking and no enlarged Cx LNs.

With regards to aetiology, there is no dextrocardia or a nasal voice to suggest possible Kartagener’s
syndrome. In addition, there is no symmetrical deforming polyarthropathy to suggest RA or any cutaneous
signs of SLE. There is no kyphoscoliosis.

With regards to treatment, patient has a steroid metered-dose inhaler, salbutamol and ipratropium metered-
dose inhalers by the bed side.

I would like to complete the examination by looking at the temperature chart for fever as well as an
abdominal examination to look for splenomegaly from amyloidosis which can result from bronchiectasis. A
neurological examination is useful to screen for deficit as patients are prone to brain abscesses.

In summary, this patient has bronchiectasis affecting both lower lobes with complications of pulmonary
hypertension and cor pulmonale. There is no concomitant COPD and no polycythemia. He is clinically not
in respiratory failure. The possible causes for this patient’s bronchiectasis are post infective causes such as
post viral, bacterial, TB or ABPA, connective tissue disease such as RA or SLE, congenital conditions such
as cystic fibrosis, Kartagener’s syndrome or hypogammaglobulinemia.

Questions
What are your differential diagnoses for a patient that is clubbed and has crepitations?
o! Bronchiectasis
o! Pulmonary fibrosis
o! Mitotic lung lesion
o! Abscess

What is bronchiectasis?
o! Definition: permanent dilatation of the bronchi
o! Pathology: Retained secretions and chronic inflammation
32
o! Clinical course: Chronic, progressive with recurrent infective exacerbations
o! Clinical: Symptoms - productive purulent cough, dyspnea and hemoptysis and Signs: coarse late
inspiratory crepitations with a 3 layered purulent sputum

What are the causes of bronchiectasis?


o! Focal
o! Luminal blockage – FB, broncholith
o! Arising from the wall – mitotic lesion of the lung
o! Extrinsic – enlarged LNs esp middle lobe from TB/fungi; displacement of airways post lobar
resection
o! Diffuse
o! Post infectious conditions
!! Bacteria – Pseudomonas, Hemophilus, Pertussis
!! TB
!! Aspergillus (for upper lobe or proximal bronchiectasis) as in allergic
bronchopulmonary aspergillosis from type III immune complex reactions.
!! Virus – adenovirus, measles, influenza
o! Congenital conditions
!! Cystic fibrosis
!! Alpha 1 Antitrypsin deficiency
!! Kartagener’s syndrome of immotile ciliary syndrome
!! Hypogammaglobulinemia
o! NB: Immunodeficiency form secondary causes such as cancer, chemotherapy
or immune modulation post transplant
o! Rheumatic conditions
!! RA (1-3% of patients)
!! SLE
!! Sjogren’s
o! Others
!! Yellow nail syndrome (yellow nails, bronchiectasis, pl effusion and lymphedema)
!! Young’s syndrome(secondary ciliary dyskinesia from mercury intoxication)
!! Inflammatory bowel disease (UC or Crohn)
!! Congenital kyphoscoliosis
!! Idiopathic (50%)

What is bronchiectasis sicca?


o! “dry” bronchiectasis
o! Presents with recurrent hemoptysis and dry cough
o! Affects the upper lobes therefore good drainage
o! Usually from past history of granulomatous infection eg TB

What is Kartagener’s syndrome?


o! It is a type of immotile ciliary syndrome
o! Comprising of
o! dextrocardia, situs inversus
o! bronchiectasis, sinusitis, frontal sinus dysplasia, otitis media
o! infertility
o! Resulting in poor ciliary function with retained secretions and recurrent infections and thus
bronchiectasis

What is cystic fibrosis?


o! Most commonly due to mutations to CFTR (CF transmembrane conductance regulator) with F508
o! Recurrent respiratory infections with pancreatic exocrine deficiency and short stature
o! Upper lobe involvement
o! Staph aureus, Ps aeuroginosa
33
o! Elevated sweat Na and Cl concentrations

What are the differences in bronchiectasis vs COPD?


o! They may both occur concomitantly
COPD Bronchiectasis
Cause Cigarette Infection, genetic
Infection Secondary Primary
Organism S. pneumoniae, Haem Haem, Pseudomonas
Symptoms Dyspnea, chronic cough Dyspnea, hemoptysis, productive
Sputum Mucoid clear 3 layered, purulent
CXR Hyperlucency, hyperinflated Airway thickening, dilated

What are the complications of bronchiectasis?


o! Pneumonia, collapse, pleural effusion, lung abscess, pneumothorax, hemoptysis
o! Brain abscess
o! Sinusitis
o! Amyloidosis

How would you investigate?


The diagnostic investigation of choice is a HRCT but simple Ix such as CXR and LFT are also useful:
o! CXR – Diagnosis, extent and complications
o! 90% abnormal
o! Diagnosis
!! specific
•! dilated and thickened airways
•! Ring shadows (seen on end)
•! Tram lines
!! Non-specific
•! Linear or plate-like atelectasis
•! Scattered irregular opacities
•! Focal pneumonitis
o! Extent and distribution
o! Complications
!! Pneumonia, abscesses, pleural effusion
o! Lung function test
o! Obstructive pattern with FEV1/FVC <70%
o! Severity of obstruction based on FEV1
o! Reversibility with beta agonist
!! 40% of patients have >15% improvement
o! High-resolution computer tomography scan of the thorax
o! Non-contrast study with 1 mm cuts every 1 cm with acquisition time of one second during
full inspiration (requires patient cooperation); 90% sensitivity
o! Diagnostic
!! Dilatation of airway lumen >1.5X cf to a nearby vessel
!! Signet ring sign (dilated bronchus with its pulmonary artery)
!! Lack of tapering of an airway toward the periphery with presence of bronchi within 1
cm from the pleura
!! Reid’s classifications
•! Cylindrical or tubular
•! Varicose
•! Saccular or cystic
!! Useful also in elucidation cause of focal bronchiectasis
o! Assess distribution
!! Usually lower lobes
34
!! If upper lobes – suspect Cystic fibrosis or ABPA
!! If proximal bronchiectis, ABPA
!! If ML or lingula – M. avium complex
o! Complications

How would you manage?


o! Non-Pharmacological
o! Education and counselling
o! Stop smoking, vaccinations (yearly influenza and 3-yearly pneumococcal)
o! Chest percussion and postural drainage (no evidence actually)
o! Rx underlying cause
o! Pharmacological
o! Rx acute exacerbations
o! O’Donnell’s 4/9 symptoms of exacerbations
!! Increased dyspnea
!! Increase cough
!! Increase sputum production
!! Increased wheezing
!! Fever
!! Lethargy, malaise
!! Changes in chest sounds
!! Reduced pulmonary function
!! Radiographic changes consistent with a new pulmonary process
o! Antibiotics targeting
!! Haem, Ps and Strep and Moraxella
!! Fluoroquinolones
!! Others
•! MAC – Rifampiciin, ethambutol and Azithro till c/s negative for 1 year
•! ABPA – augmentation of corticosteroids and use of itraconazole 200mg
bd for 4 weeks then 200mg om for 4 more weeks
o! Bronchodilator therapy such as beta agonists and anticholinergics with inhaled corticosteroids
o! Improve lung function (FEV1) and reduce sputum volume
o! No effect on mortality
o! Aerosolised recombinant human DNAse for cystic fibrosis (not for other causes of bronchiectasis)
o! Surgery
o! Focal
o! Removal of obstructing tumour or FB
o! Diffuse
o! Segments that are most damaged and contributing to recurrent acute exacerbations
o! Segments involved with uncontrolled haemorrhage
o! Removal of segments suspected of harbouring drug resistant organism such as MDR MTB
or MAC
o! Lung transplant

How do you manage complication of hemoptysis?


o! Quantify
o! If >600mls /day = massive
o! Lie on the affected side
o! Protect airway
o! Bronchoscope or CT to determine site of bleed
o! Interventional radiology or surgical removal

35
Interstitial Lung Disease

Presentation

Sir, this patient has interstitial lung disease affecting both lower lobes (upper lobes) as evidenced by fine
velcro-like late inspiratory crepitations heard best posteriorly(anteriorly) in the lower one third bilaterally.
This is associated with clubbing(50%) and a non-productive cough.

Chest excursion was reduced bilaterally with a normal percussion note and vocal resonance. Trachea is
central and apex beat is not displaced.

There are no signs of pulmonary hypertension or cor pulmonale. There are also no features of polycythemia.

Patient respiratory rate is 14 breaths per minute and there are no signs of respiratory distress. There are also
no signs of respiratory failure. There is also no nicotine staining of the fingers and I note that the patient is
cachexic looking with wasting of the temporalis muscles.

In terms of aetiology, there is no symmetrical deforming polyarthropathy of the hands to suggest RA, or
cutaneous signs to suggest presence of SLE, dermatomyositis or scleroderma as these conditions may be
complicated by pulmonary fibrosis.

With regards to treatment, patient is not Cushingoid and does not have papery thin skin or steroid purpura to
suggest chronic steroid usage. On inspection there are no surgical scars to suggest open lung biopsy.

I would like to complete the examination by asking for a detailed drug history as well as an occupational
history.

In summary, this patient has got pulmonary fibrosis affecting bilateral lower lobes. There are no
complications of pulmonary hypertension, cor pulmonale and polycythemia. He is clinically not in
respiratory failure and has no features of chronic steroid usage. The differential diagnoses include collagen
vascular disease, drugs, occupational causes and idiopathic pulmonary fibrosis.

Questions

What are the differential diagnoses for clubbing and crepitations?


!! Pulmonary fibrosis
!! Bronchiectasis
!! Lung abscess
!! Mitotic lung conditions

What are the characteristic auscultatory findings?


!! Late, fine inspiratory crepitations
!! Velcro-like
!! Disappears or quietens with the patient leaning forwards

What are the causes of fibrosis?


!! Upper Lobes
!! S – Silicosis, sarcoidosis
!! C- coal worker pnemoconiosis
!! H- histiocytosis
!! A- Ankylosing spondylitis, ABPA
!! R – radiation
!! T – TB
!! Lower lobes
!! R- RA
36
!! A-Asbestosis
!! S- Scleroderma
!! I – Idiopathic pulmonary fibrosis
!! O- others ie drugs
!! Cytotoxics – MTX, Aza, bleomycin, bulsulphan, cyclo, chlorambucil
!! CNS - Amitryptyline, phenytoin and carbamazepine
!! CVS - Amiodarone, hydralazine, procainamide
!! Antibiotics - Nitrofurantoin, isoniazid
!! Antirheumatics – Gold, sulphasalazine
!! Both
!! N – Neurofibromatosis, Tuberous sclerosis
!! E – Extrinsic allergic alveolitis (acute symptoms within 6 hrs of inhaled allergens eg farmer’s lungs)
!! P – pulmonary haemorrhage syndromes
!! A – alveolar proteinosis
!! Primary
!! Secondary – Inhaled organic dusts(Silica, Al), chronic infection, malignancy
!! Lymphangiomyomatosis

How would you classify interstitial lung disease? (ATS/ERS 2001)


!! Diffuse parenchymal lung disease(DPLD) of known cause
!! Collagen Vascular disease
!! RA, SLE, Dermatomyositis, Systemic sclerosis
!! Occupational/Environmental
!! Asbestosis, silicosis, extrinsic allergic alveolitis
!! Drug related
!! Cytotoxic, CNS, CVS, Antibiotics and antirheumatic
!! Idiopathic
!! IPF
!! Other idiopathic interstitial pneumonias
!! DIP
!! AIP
!! LIP
!! NSIP
!! Cryptogenic organising pneumonia
!! Respiratory bronchiolitis
!! Granulomatous
!! Sarcoidosis
!! Others - LAMs, histiocytosis

How would you diagnose idiopathic pulmonary fibrosis?


!! Clinical-radiological-pathological diagnosis
!! Clinical
o! Exclusion of other causes of ILD
o! >50 yrs, insidious onset of dyspnea, > 3months, non-productive cough
o! Typical physical findings
!! Radiological (see below)
!! Pathological (see below)

How would you investigate?


The diagnostic Ix of choice is a HRCT of the thorax but simple IX such as CXR and LFT are useful:
!! CXR
!! Diagnostic
!! bilateral basal reticulonodular shadows, peripheries, which advances upwards
!! honeycombing in advanced cases (gps of closely set ring shadows)
!! loss of lung volume
37
!! Extent and distribution
!! Complications
!! Lung function
!! Restrictive pattern (reduced TLC or VC with increased FEV1/FVC ratio)
!! Severity of restriction based on TLC
!! Reduced transfer factor (impaired gas exchange)
!! HRCT scan
!! Dx – patchy reticular abnormalities, focal ground glass, architectural distortion, volume loss,
subpleural cyst, honeycombing (no consolidation or nodules)
!! Extent and severity – basal, peripheral, subpleural
!! Complications
!! NB: Similar to that of collagen vascular disease and asbestosis
!! Others
!! Bronchoscopy – lavage
!! Predominantly lymphocyte responds to steroids and better Px= not UIP
!! Predominantly neutrophils and eosinophils means poor Px= UIP (if >20% of eosinophils to
consider eosinophilic lung disease)
!! Lung biopsy
!! IPF – Usual interstitial pneumonia
!! Bloods
!! ABGs
!! To rule out causes

How would you manage?


!! Education and counselling
!! Stop smoking
!! Regular follow up and vaccinations
!! Treat underlying cause
!! Pharmacological
!! Trial of steroids
!! If responding continue steroids
!! If not responding, cyclophosphamide or azathioprine
!! Antifibrotic agents
!! Eg penicillamine which has not been proven to be useful
!! Surgical
!! Lung transplant (single lung transplantation)
!! Manage complications
!! Cor pulmonale - diuresis for heart failure
!! Polycythemia - venesection if Hct >55%
!! Respiratory failure – Oxygen therapy
!! Monitor for lung cancer

What are the good prognosticating factors?


!! Young age
!! Female
!! Short duration
!! Ground glass appearance on the CXR
!! Minimal fibrosis on lung biopsy

What is the clinical course of patients with IPF?


!! Gradual onset
!! Progressive
!! Median survival from time of dx about 3 years
What are the causes of death?
!! Cor pulmonale
38
!! Respiratory failure
!! Pneumonia
!! Lung carcinoma

What is Hamman-Rich syndrome?


!! Rapidly progressive and fatal variant of interstitial lung disease

COPD

Presentation

Sir, this patient has severe COPD that is complicated by pulmonary hypertension, cor pulmonale and
polycythemia. He is tachypneic at rest and requires use of intranasal oxygen supplementation.

Patient has got hyperinflated chest with reduced chest expansion bilaterally at 2cm. The percussion note is
resonant with loss of liver and cardiac dullness. There is prolonged expiratory phase with expiratory ronchi.
Vocal resonance is normal. Trachea is central and apex beat is not displaced.

There is complication of pulmonary hypertension as evidenced by loud and palpable P2 associated with a
left parasternal heave. There is also cor pulmonale with raised JVP of 4cm with giant V waves associated
with bilateral pedal oedema. There are also features of polycythemia with plethoric facies and conjunctival
suffusion.

The patient is in respiratory distress. He is tachypneic at rest with a RR of 20 bpm and uses his accessory
muscles of respiration at rest. He is also in respiratory failure with presence of central cyanosis and is oxygen
dependent. However, he does not have a flapping tremor or a bounding pulse to suggest CO2 retention
clinically.

In terms of aetiology, the presence of nicotine staining of his fingers implies significant history of smoking.
He is not clubbed. The Cx LNs are not enlarged and he is not cachexic looking.

There is presence of steroid MDI as well as bronchodilators by his side. There is no evidence of a hoarse
voice or oral thrush or other features of chronic systemic steroid usage.

I would like to complete the examination by testing patient’s forced expiratory time, checking his temperature
and examining his sputum.

In summary, this patient has got severe COPD with complications of pulmonary hypertension, cor pulmonale
and polycythemia. He is in respiratory failure and respiratory distress. The most likely aetiology is smoking.

Questions

How do you dx COPD?


•! Clinical (>35 years, smoking, wheeze, SOB, cough with sputum, winter bronchitis)
•! Airflow obstruction – FEV1/FVC<70 and FEV1<80
o! Mild – 50-80%
o! Mod – 30-50%
o! Severe - <30%
•! Exclude differential diagnoses
o! Asthma (>400mls to dilators or PO pred 30mg OM 2 weeks or variation in PEFR >20%)
o! Cancer
o! Bronchiectasis
o! ILD
39
How do you grade the severity of dyspnea?
•! MRC scale
o! 1 – SOB on strenuous exercise
o! 2 – on hurrying or up hill
o! 3 – walks slower than contemporaries and stops for breaths
o! 4 – stops for breath after walking 100m
o! 5 – SOB on ADLs

How would you investigate admitted with an acute exacerbation?


•! FBC (anaemia, polycythemia), biochemical, theophylline levels, ABG
•! Blood C/S if febrile
•! CXR
•! Spirometry
•! ECG, 2D echo

How would you manage?


•! Non-pharmacological
o! Stop smoking
o! Regular follow up (if >500 mls decline over 5 years implies accelerated decline)
o! Pneumococcal and influenza vaccination
o! Pulmonary rehabilitation for MRC 3 or above(PT/OT)
o! MSW, Nurse
o! Assessment of inhaler technique (should not clean space more than once a month due to
increased static)
•! Pharmacological
o! Bronchodilators
!! Beta agonist and anticholinergics
!! Short acting and long acting
!! Improves symptoms and exercise capacity
o! Theophylline
o! Steroids
!! Reduce exacerbations and decline in health status
!! Used if
•! FEV1<50%
•! 2 or more exacerbations a year requiring antibiotics or steroids
!! Prophylaxis against osteoporosis (once >65)
o! Mucolytics (not anti-tussive)
o! Management of exacerbation
!! Bronchodilator - Nebs or via spacer
!! Systemic steroids
!! IV aminophylline
!! Antibiotics such as macrolide (increase volume or purulence)
!! Oxygen therapy
•! Intranasal if hypoxic
•! NIPPV if hypercapneic and pH 7.25-2.35
•! Intubation

•! Manage complications
o! Hypoxemia – assessment for need of LTOT
!! PaO2 <55 mmHg
!! PaO2 <60 mmHg and presence of
•! Polycythemia
•! Pulmonary hypertension

40
•! Cor Pulmonale
•! Nocturnal hypoxemia
!! For at least 15 hrs/day if not 20 hrs
o! Cor pulmonale
!! Diuretics
o! Polycythemia
!! >55% consider venesection
•! Surgical
o! Bullectomy
!! Single large bulla
!! FEV1<50%
o! LVRS
!! Upper lobe bullous involvement
!! FEV1>20%
!! TLCO >20%
!! PO2 <45
o! Transplant

When would you order a AAT levels?


•! COPD with
o! Young
o! Family H/O
o! No smoking history
•! Not recommended for AAT replacement if present; treat the COPD

How would you advice on air travel?


•! FiO2 at 15% - hypoxemia
•! Pressurised at 8000 ft – pneumothorax
•! Assessment
o! H/O and PE and H/O of problems encountered during previous flight
o! Spirometry
o! SpO2 <95% on RA
!! 50m walk test
!! Hypoxic challenge test
•! Bring inhalers in the hand luggage
•! Inform the airline

41
Pleural Effusion

Presentation

Sir, this patient has a right sided moderate pleural effusion affecting the lower two thirds of the right
hemithorax.

There is reduced chest excursion of the right hemithorax associated with stony dullness with reduced/absent
vesicular breath sounds and vocal resonance affecting the lower two thirds of the right hemithorax. Apex
beat was not displaced and trachea was central in position. There are no scars on the chest wall to suggest a
previous chest tube or a thoracotomy.

Patient respiratory rate is 14 bpm with no signs of respiratory failure or distress.

With regards to aetiology: (State the positives first and rearrange accordingly)
1. Patient does not have any raised JVP, S3or pedal oedema to suggest CCF. There are also no stigmata of
chronic liver disease or generalised oedema or renal biopsy scar to suggest nephrotic syndrome. Patient does
not have features of hypothyroidism.

2. There is no nicotine staining of the fingers, no clubbing, no palpable cervical lymph nodes and he is not
cachexic. I did not detect any signs of SVCO, Horner’s syndrome and patient does not have a hoarse voice
to suggest a malignant effusion.
I could not detect any bronchial breathing above the effusion. The patient is not toxic looking and did not
detect any Mantoux testing with respect to TB pleural effusion.
There is also no deforming polyarthropathy of RA or cutaneous signs of SLE.
There is no calf swelling or tenderness noted to suggest DVT.

With regards to treatment, I did not notice (if there are features of malignancy) any radiation marks on the
right hemithorax and there are also no features of side effects of chemotherapy such as alopecia or oral ulcers.

I would like to complete the examination by looking at the patient’s temperature chart as well as examining
his sputum and examine the patient’s breasts (if female).

In summary, this patient has got a moderate size right sided pleural effusion. He is not in respiratory distress
or failure. In view:
1.! Patient is cachexic, the likely underlying aetiology includes tuberculous pleural effusion or a malignant
pleural effusion.
2.! Fever, (young patient, short history) the most likely aetiology for this patient includes a parapneumonic
effusion. Other diagnosis includes tuberculous effusion, malignant effusion or autoimmune cause.
3.! Patient has Cx lymphadenopathy, I would like to offer the diagnosis of tuberculous effusion. Another
possibility is a malignant effusion.
4.! Patient has complications of SVCO/Horner’s syndrome/Clubbing with HPOA/Nicotine staining of the
nails/tender ribs/chest wall, he has a malignant effusion.
5.! Patient has vasculitic lesions of the hands, joint deformities/tenderness, malar rash. The aetiology of the
effusion is most likely due to collagen vascular disease/SLE/RA.
6.! Young female, aetiologies include CTD, hypothyroidism and TB

Questions

What are your differential diagnoses for dullness on percussion of the right lower zone?
1.! Pleural thickening :Old TB, old empyema, mesothelioma, asbestosis, PHx of hemothorax
2.! Basal consolidation
3.! Lower lobe collapse
4.! Raised hemidiaphragm:
42
a.! phrenic nerve palsy from Ca or phrenic nerve crush for old TB treatment with supraclavicular
fossa scar
b.! hepatomegaly
5.! Mitotic mass

What are the causes of a pleural effusion?


o! Transudative
o! CCF, constrictive pericarditis
o! Nephrotic syndrome, hypoalbuminemia, peritoneal dialysis
o! Chronic liver disease (hepatic hydrothorax)
o! Myxoedema
o! Atelectasis
o! Exudative
o! Malignancy
!! Primary - bronchial or pleural
!! Secondaries – Breast, pancreas, kidneys, ovaries, lymphomas
o! Infective – parapneumonic, TB
o! CTD – SLE, RA
o! PE (can also be transudative but less common)
o! Pancreatitis (left sided)
o! Drug induced – nitrofurantoin, bromocriptine
o! Meig’s syndrome (ovarian fibroma with ascites and pl effusion)
o! Yellow nail syndrome (triad of yellow nails/onycholysis, pl effusion/bronchiectasis and
lymphedema)

What are the causes of hemothorax?


o! Trauma
o! Rupture of pleural adhesion containing blood vessel, carcinoma

What are the causes of a chylothorax?


o! Trauma or surgery to the thoracic duct
o! Carcinoma or lymphoma affecting the thoracic duct

What are the causes of an empyema?


o! Pneumonia
o! Abscess
o! Bronchiectasis
o! TB

How do you confirm the diagnosis of a pleural effusion?


o! Perform a lateral decubitus film to look for layering
o! USS

How do you differentiate between an exudative and transudative effusion?


o! Light’s criteria
o! Sensitive but not specific for exudates
o! One out of 3 criteria
!! Pl fluid protein: serum protein > 0.5
!! Pl fluid LDH:serum LDH>0.6
!! Pl fluid LDH > 2/3 upper limit of serum LDH
o! If suspect transudate still, can do the serum to pleural albumin gradient (difference). If > 1.2 g/dL,
transudate. (less sensitive for exudates)

How would you investigate?


o! CXR
43
o! Sputum for gram stain, c/s and AFB smear, cytology
o! Diagnostic pleural tap and pleural biopsy
o! Indications for a diagnostic thoracocentesis
!! >10mm thick on a lateral decubitus film or USS
o! Appearance
!! Bloody appearance
•! <1% insignificant
•! 1-20% = malignancy, PE or trauma
•! >50% cf to peripheral Hct = hemothorax
!! Turbid (parapnemonic, chylothorax)
!! Putrid odour (anaerobic)
o! Haemotological Ix
!! Total cell > 1500 cell/ml
!! >50% neutrophils (parapneumonic)
!! Lymphocyte predominant (cancer, TB, lymphoma, CTDs)
!! Mononuclear cells – chronic
!! Eosinophils
•! Blood or air in the pleural space
•! Drugs – nitrofurantoin, bromocriptine, dantrolene
•! Churg-Strauss
•! Paragonimiasis
•! Asbestosis
o! Biochemical
!! Light’s criteria and serum pleural albumin gradient
!! pH
!! Glucose (<0.5 cf to peripheries = TB, malignancy, RA)
!! Amylase level (pancreatitis)
o! Microbiological
!! Smear and C/S
!! AFB smears
!! PCR, ADA(adenosine deaminase) or gamma interferon
o! Cytology
!! Fluid
!! Pleural biopsy
o! Others
o! Blood Ix, mantoux
o! Bronchoscope, CT scan

When must you order a CXR post Dx tap?


o! When you suspect complications of pneumothorax
o! Air is aspirated
o! Patient develops cough, chest pain or dyspnea
o! Loss of tactile fremitus over the superior part of the aspirated hemithorax

How would you manage?


o! Treat the underlying cause
o! Treat symptoms
o! Fever, pain
o! SOB – therapeutic thoracentesis (up to 1500mls)
o! Chest tube insertion (tube thoracostomy)
o! Complicated parapneumonic effusion
!! Gross pus or empyema
!! pH <7.2
!! Glucose <3.0
44
!! G/S positive for organism
!! LDH >1000
o! Hemothorax
o! Pleurodesis for malignant effusions

What are the surface markings for a respiratory examination?


•! Anteriorly
o! 4th rib and above = upper lobe on right
o! between 4th to 6th rib = middle lobe on the right
o! 6th rib and below on the left = lower lobe
•! Horizontal fissure extends on the right extends from 4th rib to midaxillary line where it bisects the oblique
fissure at the 5th rib
•! Oblique fissure extends from the 6th rib anteriorly to the back at T2
•! Lower lobe extends posteriorly from T2 to T11
•! Upper lobe posteriorly extends from T2 and above
•! Prominent Cx spine = C7
•! Inferior angle of the scapula = T7

Collapse

Presentation

Sir, this patient has a right upper lobe collapse as evidenced by reduced chest excursion of the right
hemithorax associated with dullness on percussion and reduced vesicular breath sounds and reduced vocal
resonance affecting the upper one third of the right hemithorax. This is associated with tracheal deviation to
the right. There was no displacement of the apex beat.

Patient’s respiratory rate is 14 bpm and is not in respiratory distress or failure.

With regards to aetiology:


There are signs to suggest underlying malignancy. Patient is cachexic looking and is clubbed. I did not detect
any tenderness in the wrists to suggest HPOA. There were no enlarged palpable Cx LNs. There is also no
conjunctival pallor or jaundice noted. I did not detect any associated pleural effusion or raised
hemidiaphragm. There were no signs of SVCO, patient’s does not have a right Horner’s syndrome and there
is no wasting of the intrinsic muscles of the right hand (hoarseness of voice for left sided lesion).

In considering other etiologies:


There are also no ronchi on auscultation to suggest asthma or allergic bronchopulmonary aspergillosis.
There are no Mantoux testing detected on the upper limbs but endobronchial TB is a possible differential
diagnosis.

I did not find any signs of treatment such as radiotherapy hyperpigmentation or side effects of chemotherapy
such as alopecia, phlebitic veins or oral ulcers.

I would like to complete my examination by looking at the patient’s temperature chart as well as examining
his sputum.

In summary, this patient has a right upper lobe collapsed and is clinically comfortable. In view that patient
has:

1.! A history of weight loss and is cachexic looking, the possible diagnoses include endobronchial mitotic
lesion or an endobronchial tuberculous infection.

45
2.! Fever/cough/hemoptysis/Cx lymphadenopathy, the possible diagnoses includes an endobronchial
tuberculous infection, endobronchial mitotic lesion or a collapse consolidation from a pneumonia.
3.! Complications of Right Horner’s syndrome, signs of SVCO, clubbed with HPOA, the most likely cause
is an endobronchial mitotic lesion affecting the right upper lobe.

Questions

What are the causes of a lung collapse?


•! Intraluminal – Mucus plugging from asthma or ABPA, FB
•! Endobronchial tumor, TB
•! Extrinsic compression – enlarged LNs for mitotic lesion (pri or sec), lymphomas or TB

What is Brock’s syndrome?


•! Collapse of the right middle lobe from enlarged LNs

How would you investigate?


•! Simple investigations
o! CXR
o! ABG, FBC and biochemical profile
o! Sputum AFB smear and c/s and cytology
•! Diagnostic - Bronchoscopy and Bx
•! Staging – CT thorax and abdomen with adrenal cuts, bone scan
•! Physiological staging: Lung function test
o! FEV1 >1.5
o! Transfer factor>50%

How would you treat the patient?


•! Depending on the underlying cause
•! For mitotic lesion
o! Multidisciplinary approach
o! Education and counselling, support groups and stop smoking
o! Symptomatic treatment
o! For non-small cell
!! Assessment for surgical resectability
•! Staging (up to stage IIIA); ie once T4, N3 or M1 not a candidate
•! Physiological staging
!! Chemotherapy
•! Neoadjuvant
•! Adjuvant
!! Radiotherapy
•! Adjuvant
•! Palliative
!! Palliative
•! Radiotherapy
o! Pain, bone mets
o! Dyspnea from bronchial obstruction, dysphagia
o! SVCO, pancoast syndrome
•! Chemotherapy
o! For small cell
!! Chemotherapy

46
How does patient with bronchogenic carcinoma present?
•! Primary tumor
o! Cough, dyspnea, hemoptysis, pneumonia
•! Mediastinal spread
o! SVCO, Horner’s, pleural effusion, phrenic nerve palsy, hoarseness of voice, T1 wasting,
pericardial effusion
•! Metastasis
o! Liver, bone, brain, skin, adrenal glands
•! Paraneoplastic symptoms
•! Systemic effects
o! LOA, LOW, fatigue

What are the paraneoplastic syndromes?


•! Endocrine
o! PTH-related peptide (hypercalcemia) – SCC
o! SIADH – Small cell (usually asymptomatic)
o! ACTH – Cushing’s (usually hypokalemic metabolic alkalosis)
o! Gynaecomastia
•! Neurological
o! Subacute cerebellar degeneration
o! Peripheral neuropathies
o! Lambert-Eaton syndrome
•! Cardiovascular
o! Non-thrombotic endocarditis
•! Renal
o! Nephrotic syndrome, GN (membranous)
•! Skin
o! Migratory venous thrombopleblitis (Trosseau’s sign)
o! Acanthosis nigricans
o! Dermatomyositis
o! Zoster
•! MSK
o! Clubbing, HPOA
•! Haematological
o! DIC
o! Anaemia

What is SVCO?
•! Tumour with obstruction of the SVC
•! Plethoric facies
•! Facial and UL oedema
•! Conjunctival suffusion
•! Undersurface of the tongue with multiple venous angiomata
•! Fixed engorgement of the neck veins
•! Stridor
•! Upper chest telangiectasia
•! Radiation marks
(NB think of polycythemia which also have plethoric facies)
•! Causes
o! Lung carcinoma, especially small cell
o! Lymphoma
o! Others – mediastinal goitre

47
Consolidation

Presentation

Sir, this patient has a right upper lobe consolidation as evidenced by reduced chest excursion of the right
hemithorax associated with a dull percussion note, bronchial breath sounds and crepitations and increased
vocal resonance. These signs were best heard in the upper one third anteriorly in the right hemithorax. The
trachea is central and apex beat is not displaced.

There are no signs to suggest that the patient is in respiratory distress or in failure.

With regards to aetiology, an underlying malignancy is considered as there is associated complication of


SVCO (hoarseness of voice if left sided), with plethoric facies, ruddy complexion a/w oedema of the face
and upper limbs associated with suffusion of the eyes, fixed engorgement of the neck veins, dilatation of the
superficial veins of the neck, and venous angiomata detected on the undersurface of his tongue. There is no
Horner’s syndrome, wasting of the intrinsic muscle of the hands and no soft heart sounds to suggest
pericardial effusion. There was also no associated pleural effusion or a raised right hemidiaphragm.

He is also clubbed with HPOA and has nicotine staining of his fingers. He is cachexic looking with enlarged
palpable cervical LNs. There is also thrombophiblitis of the forearms which may suggest Trosseau’s sign.

(if there are no signs of cancer, proceed to mention TB/pneumonia ie mantoux testing, toxic looking,
productive cough with purulent sputum; DVT ie swelling and tender calves)

There is presence of radiation therapy marks on the right chest wall as well as side effects of chemotherapy
such as alopecia and oral ulcers.

In summary, patient has a right upper lobe consolidation complicated by SVCO. He is not in respiratory
distress. The underlying cause is most likely a mitotic lesion of the lung.

Question

What are the causes of a consolidation?


•! Infection
o! Pneumonia
o! Abscess
o! TB
o! Aspergilloma, cryptococcoma, hydatid cyst
•! Neoplastic or mass
o! Carcinoma
o! Lymphoma
•! Pulmonary infarction

How would you investigate?


•! Simple
o! CXR
o! ABGs, blood tests FBC and biochemical profile, blood c/s
o! Sputum
•! Directed tests
o! Infection
o! Cancer – Bronchoscopy and Bx, CT staging, bone scan, physio staging
o! Infarction
What are the causes of an unresolved pneumonia?
48
•! FB
•! Tumor
•! Abscess
•! Inappropriate antibiotics, resistant organism
•! Bronchopulmonary sequestration
o! Rare, congenital
o! Non-functioning lung tissue with anomalous arterial supply with no connection to the
bronchopulmonary tree

What are the extrapulmonary manifestations of Mycoplasma?


•! CNS – meningitis, encephalitis
•! CVS – percarditis, myocarditis
•! Hepatitis, GN
•! DIC, AIHA
•! EM, SJS
•! Arthralgia, arthritis

What are the complications of pneumonia?


•! Local
o! Abscess
o! Empyema
o! Respiratory failure
•! Sepsis
o! Septic shock
o! ARDS
o! MOF
o! DIC

What are the pulmonary eosinophilic disorders?


•! Defined as radiographic infiltrates with hypereosinophilia (>1.5 x109L)
•! Includes
o! Churg-Strauss – Asthma with vasculitis and hypereosinophilia
o! Tropical pulmonary eosinophilia – high anti-filarial Ab
o! Chronic pulmonary eosinophilia – cough, progressive SOB, weight loss with “photographic
negative” pulmonary oedema ie diffuse peripheral pulmonary infiltrates

How do you manage pneumonia? BTS 2004


•! Depending on the type of pneumonia
o! CAP
o! HCAP
o! HAP
o! note that HCAP is Rx the same way as CAP in BTS guidelines
•! Risk Stratify - CAP
o! CURB-65 score – 6 point score from 0 to 5
•! Confusion
•! Urea >7 mmol/L
•! RR >30 bpm
•! BP <90/60
•! Age 65 and above
o! 0 to 1 – low risk of death and home Rx
o! 2 – increased risk of death and short hospitalization or supervised home treatment
o! 3 and above – high risk of death and urgent hospitalization

49
•! Ix – FBC, CRP, Bld c/s (for those with severe indicators or co-morbidities), sputum g/s and c/s, Lg
and Pneumococcal Urine Ag only for severe pneumonia
•! Mx
o! Use of SpO2 monitoring advocated in GP setting
o! NIV not for use in severe pneumonia unless in ICU setting
o! Antibiotics
•! CAP- Penicillins or macrolides; fluoroquinolones if intolerant or selected inpatient
treatment with PO moxifloxacin preferred over levofloxacin
o! Discharge planning
•! Should not be discharged within 24Hrs if have >1 of
•! T >37.8
•! HR>100
•! RR>24
•! SpO2<90%
•! BP sys <90
•! Inability to maintain oral intake
•! Abnormal mental status

How do you mange pulmonary infarction? BTS 2003


•! All patients with suspected PE should have clinical probability assessed
•! Hence evaluate patients for
o! SOB or tachypnea, chest pain or hemoptysis
o! Absence of an alternative explanation (a)
o! Presence of a major risk factor (b)
•! Therefore if
o! (a) and (b) present = high probability
o! Only (a) or (b) present = intermediate probability
o! None present = low probability
•! D-dimer
o! To use only if low or intermediate probability
o! Highly specific = if negative, no need to do imaging
o! Not to order if high probability; image immediately
•! Imaging
o! Done within 1 hr for massive and 24 hrs for non massive
o! Imaging options
!! CTPA
!! Isotope lung scan
•! Facilities available
•! CXR normal
•! No cardiopulmonary disorder
•! Standardised reporting criteria
•! Non-diagnostic results followed by other imaging
!! USS DVT
•! Can be used if presence of clinical DVT and if positive, is sufficient to confirm
VTE
•! If negative, cannot be used to confirm absence of VTE
o! Management
!! Massive (BP compromised or cardiopulmonary collapse)
•! Thrombolysis with alteplase (No lowering BP effect cf with streptokinase)
•! Thrombus fragmentation and IVC filters if expertise present and available
!! Non-massive PE
•! NO thrombolysis
•! Use of heparin before imaging in high or intermediate clinical probability
50
•! Use LMWH as first choice
•! Use unfractionated heparin if
o! First dose bolus
o! Massive PE
o! Acute reversal desired
•! If VTE confirmed
o! Commenced oral anticoagulation
o! INR 2-3
o! Duration is 4-6 wks for temporary risk factors. 3 months for first
idiopathic episode and 6 months for other clinical situations
•! Special populations
o! Pregnancy
!! LMWH during pregnancy
!! UFH approaching delivery
!! UFH stopped or reduced 4-6hrs prior delivery
!! Oral anticoagulation commenced after delivery and continued
for 6 weeks or 3 months after PE whichever is longer
o! Cancer patients
!! Oral anticoagulation as per above
!! Duration unknown
!! Higher risk of thrombosis and bleeding
!! If recurrent thrombosis
•! INR 3-3.5 or
•! LMWH + anticoagulation or
•! IVC

51
Lobectomy/Pneumonectomy

1. Sir, this patient has left lobectomy as evidenced by a left sided thoracotomy scar associated with
asymmetrical deformity of the chest. There is reduced chest excursion of the left and the percussion note is
dull in the lower third of the left hemithorax with decreased breath sounds and vocal resonance in this area.
The tracheal is not deviated and the apex beat is not displaced.

2. Sir, this patient has a left pneumonectomy as evidenced by a left thoracotomy scar associated with
asymmetrical deformity of the chest. There is reduced chest wall excursion on the left with a dull percussion
note and absent breath sounds and vocal resonance over the entire left hemithorax. Trachea is deviated
towards the left with the apex beat displaced in the same direction. This is associated with hyperinflation of
the right chest with hyper-resonant percussion note and loss of liver dullness.

He is comfortable at rest with a RR of 12 bpm with no evidence of respiratory failure or distress.

With regards to aetiology


1. I did not detect any coarse inspiratory crepitations to suggest bronchiectasis nor was there any ronchi or
prolonged expiratory phase to suggest COPD or ABPA.

2. Patient is not clubbed and there is nicotine staining of the fingers. There is no palpable enlarged cervical
lymph nodes and he is not cachexic looking to suggest mitotic lesion of tuberculosis.

I would like to look at the patient’s temperature chart as well as his sputum.

In summary, this patient has a left lobectomy/pneumonectomy and the possible causes include:
1.! Surgical resection for early stage mitotic lesion of the lung or a SPN of uncertain cause.
2.! Resistant lung abscess
3.! Mycetoma
4.! Treatment modality for pulmonary tuberculosis in the past (1950s, pt should be late 60s)
5.! lung volume reduction surgery in COPD (lobectomy)
6.! Localised bronchiectasis or its complications
7.! Trauma

Questions

What are the indications for a lobectomy/pnemonectomy? (7)


•! Causes of lobectomy and pneumonectomy
o! Mitotic lesion or SPN of uncertain nature
o! Abscess
o! Bronchiectasis – localised or complications
o! Mycetoma, ABPA (remaining lungs may have ronchi)
o! TB treatment
o! LVRS for COPD
o! Trauma
•! Lobectomy and splenectomy
o! TB
o! Sarcoidosis

What are the contraindications to a lobectomy/pneumonectomy in a lung cancer patient?


•! Resectability
o! T4 (mediastinal structures), N3 (contralateral mediastinal or hilar or ipsilateral supraclavicular
LNs or scalene) or M1
o! Tumor within 2 cm of carina (potentially curable by radiotherapy)

52
o! Bilateral endobronchial tumor (potentially curable by radiotherapy)
•! Physiologic staging
o! Severe pulmonary hypertension
o! CO2 retention
o! FEV1< 1L
o! Transfer factor <40%
o! Concomitant disease that would shorten life expectancy
o! Recent MI in the past 3 months
o! Borderline function with cardiopulmonary exercise testing with a maximal oxygen consumption
<15ml/kg/min

What are the indications for lung volume reduction surgery in COPD patients?
•! Emphysema
•! Predominantly upper lobes
•! No or mild pulmonary hypertension (PASP > 45 mmHg)
•! No concomitant disabling disease
•! FEV1>20%
•! DLCO >20%

Approach to Lateral Thoracotomy Scar

•! Lateral thoracotomy scar


o! Asymmetrical chest wall with absent ribs
!! Thoracoplasty for TB treatment in the past (can get respiratory failure and BiPAP useful)
o! Symmetrical or no chest wall deformity
!! Normal underlying lung on examination
•! Lung transplant
•! Pleurectomy (eg for recurrent pntx in Ehlers Danlos)
•! Bullectomy
!! Abnormal underlying lung
•! Reduced breath sounds
o! Lobectomy (still as breath sounds in the axilla)
o! Pneumonectomy (no breath sounds)
•! COPD with lung volume reduction surgery
•! Lung transplant with complications

53
Lung Transplant

Presentation 1

Sir, this patient has a bilateral lung transplant as evidenced by presence of a median sternotomy scar
associated with normal underlying respiratory findings. The patient has a wrist tag indicating that he is a lung
transplant patient. Chest excursion is normal with a normal percussion note, vesicular breath sounds and
vocal resonance. Trachea is central and apex beat is not displaced.

Hence there are no signs to suggest underlying rejection of the transplanted lung.

Patient’s respiration rate is 14 bpm and there are no sign of respiratory distress or failure.
Peripheral examination was normal. There was no evidence of nicotine staining, clubbing or short stature.

Patient is not Cushingoid with no steroid purpura or thin skin to suggest chronic steroid use. There is also no
hypertrichosis or gum hypertrophy to suggest cyclosporine adverse effects.

In summary, this patient has bilateral lung transplant and is clinically well currently. The likely underlying
causes
a. Young patient – Cystic fibrosis, AAT, primary pulmonary hypertension, Eisenmenger (heart-lung
transplant)
b. Old patient – COPD, IPF and bronchiectasis (always bilateral)

Presentation 2

Sir, this patient has a left lung transplant as evidenced by a left lateral thoracotomy scar with a tag indicating
that he is a lung transplant patient. There is reduced chest excursion bilaterally with normal precussion note.
Auscultatory findings include fine late inspiratory crepitations of the left hemithorax a/w normal vocal
resonance, suggesting bronchiolitis obliterans syndrome post transplant.

Trachea is central and apex beat is not displaced.


There are complications of pulmonary hypertension, cor pulmonale ad polycythaemia.

He is also in respiratory distress with a respiratory rate of 24 bpm with use of accessory muscles of
respiration. Clinically he is in repsiratory failure and is oxygen dependent on INO2 2L/min; there is no central
cyanosis and no signs of CO2 retention such as flapping tremors of the hands or a bounding pulse.

In terms of aetiology:
Examination of the right hemithorax showed presence of ILD/COPD with features of
a.! fine late inspiratory crepitations which are Velcro-like, clubbing
b.! prolonged expiratory phase with ronch and hyperresonance on percussion of the right hemithorax and
loss of liver dullness, nicotine staining

Steroid toxicity and cyclosporin toxicity.

In summary, this patient has a left lung transplant for an underlying ILD. There are complications of
pulmonary hypertension and cor pulmonale and is clinically in respiratory failure. There also signs to suggest
bronchiolitis obliterans syndrome with signs of chronic steroid usage and cyclosporine usage.

54
Questions
What are the indications for lung transplant?
•! Cardiopulmonary
o! Primary pulmonary hypertension
o! Eisenmenger’s (heart-lung transplant)
•! Chronic Lung Conditions
o! Restrictive pulmonary disease - ILD
o! Obstructive pulmonary disease – COPD, AAT
o! Suppurative – Bronchiectasis (must be bilateral tplt), Cystic fibrosis
o! Sarcoidosis

What are the contraindications for lung transplant?


•! Disease specific guidelines
•! Includes
o! Age
o! Comorbidities: Absence of concomitant disease that shortens life expectancy (HIV, Hep B)
o! Contraindications to surgery/GA (recent MI)
o! Smoking, alcoholic, poor social support

What are the complications?


•! Graft dysfunction (reperfusion edema in the first week)
•! Airway complications – dehiscence, stenosis or bronchomalacia
•! Rejection
o! Acute – lymphocytic inflammation, adjust steroid and immune suppression
o! Chronic – alloimmune inflammatory and non-alloimmune fibroproliferative; bronchiolitis
obliterans syndrome
•! Infection
o! CMV – ganciclovir
o! Aspergillus
o! Bacterial – pneumonia (Ps aeuroginosa)

How would you manage?


•! Multidisciplinary, regular follow up, transplant coordinator
•! 3 drug maintenance
o! Calcineurin inhibitors (Cyclosporin or tacrolimus)
o! Purine synthesis antagonist ( AZA or MMF)
o! Steroid
•! CMV and PCP prophylaxis

55
Chronic Liver Disease

Prensentation

Sir, this patient has decompensated chronic liver disease with portal hypertension, splenomegaly and ascites.
My findings include:
Presence of an enlarged spleen that is palpable 3cm from the left costal margin. It is non-tender, firm in
consistency, smooth surface, regular edge, notch border with no splenic rub. I am unable to get above this
mass. The liver is not enlarged with a span of 12 cm in the right mid-clavicular line. The kidneys are not
ballotable. There is presence of ascites with shifting dullness and this is not associated with tenderness.

He is deeply jaundice and bruising noted on the ULs and LLs with presence of stigmata of CLD including
leukonychia, clubbing, palmar erythema, spider naevi and gynaecomastia with loss of axillary hair. There is
also presence of bilateral edema.

Complications:
He is cooperative with the examination with no flapping tremor to suggest hepatic encephalopathy.
There are no enlarged Cx LNs and patient is not cachexic looking. There is also no conjunctival pallor noted.

Aetiology:
I did not find any parotidomegaly, dupytren’s contracture, tattoos, surgical scars or thrombosed veins.

Treatment:
I did not notice any abdominal tap marks but patient has sinus bradycardia, indicating use of beta-blockers.

I would like to complete my examination by looking at the patient’s temperature chart for fever and a rectal
examination for hard impacted stools or malena.

In summary, this patient has decompensated chronic liver disease with portal hypertension, splenomegaly
and ascites. There is presence of bruising, leukonychia, jaundice with no evidence of hepatic encephalopathy.

A. The most likely etiology for this gentleman is


1.! Chronic ethanol ingestion due to presence of parotidomegaly but no dupytren’s contracture; presence of
hepatomegaly
2.! Chronic hepatitis B infection as patient has tattoos.
3.! Chronic hepatitis C infection as I note an abdominal surgical scar with possibility of transfusion in the
past

B. In the local context, the most likely underlying etiology is chronic ethanol ingestion, chronic hepatitis B
and C.

C. The most likely aetiology is chronic ethanol ingestion as I notice that this patient has presence of
parotidomegaly. In view that there is also a hard irregular liver that is palpable, it raises the possibility of an
underlying mitotic lesion of the liver.

D. The most likely aetiology is


1. Primary biliary cirrhosis as she is a middle-aged lady with evidence of CLD with pruritus, xanthelasma
and generalised pigmentation.
2. Hemochromatosis as he is a middle-aged gentleman with slate-grey appearance with presence of diabetic
dermopathy. I would like to complete the examination by examining the CVS for CMP, urine dipstick for
glycosuria and for small testes secondary to pituitary dysfunction.
3. Wilson’s disease as the patient has a short stature associated with Kayser Fleisher rings of the eyes and
tremor and chorea of the affecting the left upper limb.

56
4. Haemolytic anaemia (Thalassemia major/intermedius, Hereditary spherocytosis) as the patient has a short
stature associated with hyperpigmentation and thalassemic facies with frontal bossing, flat nasal bridge and
maxillary hyperplasia. I would like to complete the examination by examining the CVS for CMP, urine
dipstick for glycosuria and for small testes secondary to pituitary dysfunction.

Questions
What is cirrhosis of the liver?
!! Defined pathologically
!! Diffuse liver abnormality
!! Fibrosis and abnormal regenerating nodules

What are the causes of liver cirrhosis?


!! Chronic ethanol ingestion
!! Viral hepatitis – B and C
!! In UK, the risk for hep C is blood transfusion before Sept 1991 or blood products before 1986
!! Cardiac failure
!! Others
!! Autoimmune chronic active hepatitis (female)
!! Primary biliary cirrhosis (female)
!! Primary sclerosing cholangitis
!! Haemochromatosis (male)
!! Hemolytic disease
!! Wilson’s disease
!! Alpha 1 AT deficiency
!! Galactosemia
!! Type 4 glycogen storage disease
!! Budd-Chiari (in malignancy- PRV or intraabdominal, AI, OCPs, IBD and PNH)
!! Drugs – MTX (Look for RA or Psoriasis; Bx before starting MTX and bx every 1.5g accumulated
dose), amiodarone, isoniazid, methyldopa (MAMI)
!! Cryptogenic

What are the complications of cirrhosis? (5)


!! Portal hypertension
!! Ascites – Tense ascites, SBP
!! Splenomegaly – thrombocytopenia
!! Varices
!! Hepatorenal syndrome
!! Dx
!! Cr Clr <40
!! Absence of other causes for renal impairment
!! Absence of Cr improvement, proteinuria (0.5g/d), hematuria (<50/hpf) and urinary Na <10
!! Type 1 = rapidly deterioration in renal fn ie doubling of serum Cr in < 2wks to >221 umol/l
!! Type 2 = stable or slowly progressive that does not mean criteria for type 1
!! Hepatic encephalopathy
!! Stages
!! 1 – depression, euphoria, sleep disturbance, slurred speech; may have asterixis, normal EEG
!! 2 – lethargy, moderate confusion; asterixis present; abnormal EEG
!! 3 – marked confusion, arousable; asterixis present; abnormal EEG
!! 4 – coma; abnormal EEG
!! Coagulopathy – low platelets and reduced clotting factors
!! HCC

How do you stage cirrhosis of the liver?


!! Child-Pugh staging

57
o!Consists of 5 parameters with score ranging from 5 to 15
o!Prognosticate
o!5 parameters ( 2 clinical and 3 Ix)
o! Bilirubin (<34, 34-50, >50 umol/l)
o! INR (<1.7, 1.7-2.3, >2.3)
o! Albumin (>35, 28-35, <28)
o! Ascites (mild, moderate, severe)
o! Encephalopathy (absent, I and II, III and IV)
o!A – 5-6 pts (1 year 100%, 2 year 85%)
o!B – 7-9 (1 year 80%, 2 year 60%)
o!C – 10-15 (1 year 45%, 2 year 35%)

How would you investigate? (Note the STEM STATEMENT) (5)


!! Confirming the dx
o! Abdominal USS or CT
!! Establishing the aetiology
o! Hep markers, CAGE questionnaire, liver Bx in selected cases
!! Prognosticate
o! LFT – Albumin, bilirubin
o! INR
!! Complications
o! Endoscopy of the upper GIT
o! Mitotic change – USS and AFP
o!Evaluation of renal function – urea, electrolytes and Cr
o!Evaluation of ascitic fluid
!!Cell count
!!Ascites albumin (SAAG)
!!Gram stain and C/S
!!Others – AFB smear and c/s, cytology
!! Evaluation for liver transplant
o!5 year survival rate for cirrhosis with ascites is 30-40% vs 70-80% for post liver transplant
o!MELD score (Model for End Stage liver disease which has bilirubin, creatinine and INR)
o!Consider for those with refractory ascites, SBP or HRS

When should an abdominal paracentesis be done for a patient with cirrhosis and ascites?
!! Newly diagnosed to r/o SBP
!! Symptomatic – fever, abdominal pain, encephalopathy, GI bleed

How would you manage? (4)


!! Education and counselling
o! Stop drinking alcohol, regular follow up
!! Manage the underlying disease
o! Hepatitis B
!! General measures (stop alcohol, hep A vaccination)
!! Lifelong surveillance for HCC with USS and AFP
!! Antiviral for
•! Immune clearance phase( HBeAg +, ALT raised)
•! Reactivation phase ( HBe Ag -, ALT raised, HBV DNA raised)
!! IFN alpha (SE : influenza-like; neutropenia and thrombocytopenia; neuropsychiatric
and unmasking AI disease)
!! Lamivudine (well tolerated but YMDD mutant)
o! Hepatitis C
!! At risk are IVDAs and transfusion pre 1989 (Singapore) or pretransfusion Sept 1991
or blood pdts before 1986 (UK)
58
!! General measures
!! Surveillance (HCC and screen for HIV)
!! Indications
•! HCV RNA levels (>50 IU/ml)
•! Raised ALT
•! Bx showing fibrosis and inflammation
!! Treatment
•! Peg interferon
•! Ribavirin
o! Alcoholic liver disease
!! >21u/wk in males and >14u/wk in females
!! 100% of normal liver develops fatty liver
!! 35% develop alcoholic hepatitis
!! 20% develop cirrhosis
!! 40% of alcoholic hepatitis develop cirrhosis
!! Maddrey’s discrimination function
•! PT x Bil x 4.6
•! >32 = severe
•! Treat with corticosteroids or total enteral nutrition (20-30 kcal/kg/day)
o! Others (see notes below)
!! Manage the complications
o! Hepatic encepholpathy
!! Treat precipitants (see below)
!! Prevent
•! Low protein diet
•! Lactulose
o! Hepatorenal syndrome
!! Treatment with
•! Noradrenaline infusion, telipressin or midodrine with octreotide plus
•! Albumin infusion (1g/kg on D1 then 20-40g/day)
•! For 5-15 days
!! Prevention (in patient with cirrhosis and ascites)
•! IV albumin
!! NB that hemodialysis does not help in this condition
o! Ascites (see ascites)
o! Upper GI bleed
!! Secure VS
!! Urgent endoscopy
!! Operative
!! Prevention
•! Propanolol to reduce HR by 25% or to 55-60 bpm
•! Variceal banding
o! HCC
!! Definitive treatment
o! Liver transplant
o! MARS (Molecular adsorbent Recirculating system)dialysis as an interim measure before liver
transplant

What are the factors precipitating decompensation?


!! Infection – SBP, pneumonia, UTI
!! GI bleed
!! Constipation
!! Diuretics and electrolyte imbalance

59
!! Diarrhea and vomiting
!! Sedatives
!! Surgery

What are the nail changes of hypoalbuminaemia?


!! Leukonychia, ie nail bed opacify indicating an albumin level <30g/dL; affecting the thumb and index
nails bilaterally initailly
!! Muehrcke’s lines – transverse white lines

What are the causes of palmar erythema?


!! CLD
!! RA, thyrotoxicosis and polycythaemia
!! Pregnancy, normal finding

What are the causes of anaemia in cirrhotic patients?


!! Anaemia of chronic disease
!! Fe deficiency from GI bleed
!! Hemolysis from hypersplenism
!! Folate and B12 from poor nutrition

How many spider naevi should be present to be considered as significant?


!! More than 5

When examining a patient with signs of chronic liver disease, think of:
Primary biliary cirrhosis
!! Clinical
!! Female middle age
!! CLD with pruritus, xanthelesma, generalised pigmentation, hepatosplenomegaly
!! Stages
!! Asymmptomtic with normal LFTs (positive Abs)
!! Asymptomatic with abnormal LFTs
!! Symptomatic – lethary and pruritus
!! Decompensated
!! Commonly associated with sicca syndrome, arthralgia, Raynauds, Sclerodactyly and Thyroid disease
!! Ix
!! Raised ALP, Anti-Mitochondrial Ab – M2 Ab, IgM
!! Lipids
!! Other tests for CLD
!! Histology – Granulomatous cholangitis
!! Mx
!! Symptomatic
!! Urosdeoxycholic acid
!! Cholestyramine
!! Fat soluble vitamins
!! Immunosuppression – Cyclosporin, steroids, AZA, MTX, tacrolimus, colchicines
!! Liver transplant
Hemochromatosis
!! Clinical
!! Male
!! Slate-grey appearance, hepatomegaly
!! Affects
!! Liver – cirrhosis and cancer
!! Pancrease – DM
!! Heart failure (CMP)

60
!! Pituitary dysfunction
!! Pseudogout
!! Therefore requests
!! Urine dipstick, CVS examination and testicular examination
!! Autosomal recessive, HLA-A3, Ch 6 – HFE gene, increased Fe absorption with tissue deposition,
!! Ix
!! Raised ferrritin, transferrin saturation and liver Bx
!! Mx
!! Non-pharmological
!! Avoid alcohol
!! Avoid shellfish as they are susceptible to Vibrio vulnificus
!! Venesection
!! Dy/Dx of generalised pigmentation
!! Liver – hemochromatosis in males and PBC in females
!! Addison’s
!! Uremia
!! Chronic debilitating conditions eg malignancy
!! Chronic haemolytic anaemia

Wilson’s disease
!! Clinical
!! Short stature
!! Eyes
!! KF rings - greenish yellow to golden brown pigmentation of the limbus of the cornea due to
deposition of Cu in Descemet’s membrane at 12 and 6 o’clock position. Also occurs in PBC and
cryptogenic cirrhosis
!! Sunflower cataract
!! Extrapyrimidal
!! Tremor and chorea
!! Presents as difficulty writing and speaking in school
!! Pseudogout
!! Penicillamine complications
!! Myasthenic – ptosis
!! Lupus – malar rash, small hand arthritis
!! Urinalysis for glycosuria from proximal RTA
!! Autosomal recessive, Ch 13, increased Cu absorption and tissue deposition
!! Ix
!! Low serum ceruloplasmin, increased 24H urinary Cu
!! Liver Bx – increased Cu deposition
!! Mx
!! Penicillamine

Ulcerative Colitis
!! Clinical
o! Skin – erythema nodosum, pyoderma gangrenosum
o! Joint arthropathy – LL arthritis, AS, sacroilitis
o! Aphthous ulcers
o! Ocular – iritis, uveitis and episcleritis
o! CLD – Cirrhosis, chronic active hepatitis, fatty liver PSC, Cholangiocarcinoma, metastatic
colorectal cancer, amyloid

61
Hepatomegaly

Presentation

Sir, this patient has an enlarged liver without any signs of liver cirrhosis. (The mass in the RUQ is a liver
mass as I am unable to get above the mass and am able to trace the edge of the liver past the midline of the
abdomen.) It is massively/moderately enlarged with a
•! Size – cm from the right costal margin with a span of cm
•! Edge – regular or irregular
•! Surface – smooth or nodular
•! Consistency – soft, firm, hard
•! Tender
•! Pulsatile
•! Bruit

The spleen is not palpable or percussible. The kidneys are not enlarged. There is no associated ascites.

Examination of the peripheries:


•! Jaundice, bruises, stigmata of CLD and edema of LL
•! Hepatic flap
•! Causes
o! Cachexia, Cx LNs, conjunctival pallor
o! Dupytren’s contracture, parotidomegaly
o! Toxic looking, rashes or injected throat or enlarged tonsils
o! CCF – presence of raised JVP

I would like to complete the examination by checking patient’s temperature chart for fever (if infective cause
is a differential) and a rectal examination for masses (if secondaries are a differential).

In summary, this patient has an enlarged liver that is (state the important Cs). Hence my differential diagnosis
includes:

Massive
•! HCC/Secondaries/myeloprolif
•! RVF
•! Alcoholic liver disease

Mild-moderate
•! As above plus
•! Infection
•! Viruses – EBV, CMV, hepatitis A & B
•! Bacteria – Weil’s disease (leptospirosis), meliodosis, abscesses, TB, brucellosis, syphilitic gumma
•! Protozoal – hydatid cysts, amoebic abscess
•! Malignancy – lymphoproliferative, myeloproliferative, primary, secondary, adenoma from OCP
•! Infiltrative – sarcoid (erythema nodosum, lupus pernio), amyloid, fatty liver
•! Endocrine – acromegaly, hyperthyroid
•! Collagen Vascular disease
•! Chronic hemolytic anaemia( AI, thalassemia, HS)
•! Reidel’s lobe
•! Possibility of minimal CLD signs with just hepatomegaly
•! PBC
•! Hemochromatosis

62
Tender
•! Liver abscess/infective (viral/bacterial/parasitic)
•! HCC/Secondaries
•! Right Heart Failure/Budd chiari

Pulsatile
•! TR
•! HCC
•! AVM

Hard/Irregular
•! Mitotic (primary/Secondary)
•! Macronodular cirrhosis ie >3mm (post hepatitis B, C, Wilson’s and AAT) (Micronodular cirrhosis
implies alcoholic liver cirrhosis)
•! Amyloidosis/Hydatid cyst/granulomatous disease/gummatous disease/APCKD

Questions

What are the causes?


See above

How would you investigate?


•! According to the most likely etiologies
•! Think of
o! Blood Ix – Dx and PX
o! Imaging
o! Liver Bx

How would you manage?


•! According to the most likely etiologies

(Don’t forget that 40% of CLD has no peripheral stigmata of CLD – therefore think of Alcoholic liver
cirrhosis, PBC and Hemochromatosis in the right setting)

Splenomegaly

Presentation

Sir, this patient has a moderately enlarged spleen without evidence of liver cirrhosis. There is associated with
tenderness/pallor/lymphadenopathy.

The spleen is moderately enlarged at 4 cm from the left costal margin. There is a palpable notch with a regular
edge and smooth surface, firm consistency and is non tender. I did not detect a splenic rub. This is not
associated with hepatomegaly or ascites. The kidneys are also not ballotable.

Peripheral examination showed that there is no evidence of any stigmata of CLD and patient is not jaundiced
with no bruises or petechiae.

Aetiology:
1. Patient is not cachexic looking with no conjunctival pallor or enlarged Cx LNs. There is also no evidence
of polycythemia such as plethoric facies or conjunctival suffusion or bone marrow biopsy scar.
2. Patient is not toxic looking and no rashes or enlarged tonsils are noted.
3. There are no splinter haemorrhages or stigmata of IE.
63
4. There are no features of SLE or RA or chronic haemolytic anaemia.

I would like to complete the examination by looking at the patient’s temperature chart and take a history of
night sweats, LOW and travel history

My differential diagnoses for this young patient with moderately enlarged spleen with anaemia are

Massive Splenomegaly (>8 cm)


•! CML
•! Myelofibrosis
•! PRV
•! Chronic malaria
•! Kala-azar (visceral leshmaniasis)
•! Others(Gaucher’s, rapidly progressive lymphoma)

Moderately Enlarged (4 to 8 cm/ 2-4 FB)


•! Myeloproliferative
•! Lymphoproliferative
•! Haematological – AI, ITP, Thalassemia and HS
•! Chronic malaria
•! Cirrhosis

Mildly Enlarged(4cm</1-2FB)
•! Myeloproliferative, Lymphoproliferative
•! Infections
•! Viral – CMV,EBV
•! SBE, splenic abscesses, leptospirosis, Meliodosis, TB, Typhoid, Brucellosis(farmer)
•! Acute malaria
•! Infiltrative – Amyloidosis, Sarcoidosis
•! Endocrine – Acromegaly, thyrotoxicosis
•! Collagen vascular – SLE, Felty’s
•! Chronic haemolytic – Thalassemia, AI, HS, ITP

Tender
•! Infective causes
•! Acute myeloproliferative and lymphoproliferative

Pallor
•! Myeloproliferative
•! Lymphoproliferative
•! Malaria
•! Hemolytic anaemia(Thalassemia and AIHA)
•! AI – Felty’s, SLE
•! Cirrhosis of the liver with portal hypertension

Lymph Nodes
•! Lymphoproliferative(CLL/lymphoma)
•! Infective(IMS, Meliodosis, CMV, TB, HIV)

Questions
What are the causes?
See above

64
What are the features of an enlarged spleen in contrast to an enlarged left kidney?
•! Palpation
o! Unable to get above it
o! Notch border
o! Not bimanually palpable or ballotable
•! Percussion note over the mass is dull from the left 9th rib in the mid-axillary line extending inferior-
medially in the axis of the 10th rib
•! Inspection shows that the mass moves inferior-medially with inspiration rather than inferiorly
•! Auscultation may reveal a splenic rub

Is a normal spleen palpable or percussible?


•! A palpable spleen implies that is at least twice enlarged
•! A normal spleen can be percussed along the 9th, 10th and 11th rib but is not percussible beyond the anterior
axillary line

Hepatosplenomegaly

Presentation

Sir this patient has hepatosplenomegaly without evidence of cirrhosis of the liver.

(I say this because enlarged masses in the right and left hypochondrial regions of which I am unable to get
above theses masses and is not bimanually palpable or ballotable. Hence, these are unlikely to be due to
kidney masses.)

The liver is enlarged


•! Size, edge, surface, consistency, tender, bruit or pulsatile
The spleen is enlarged
•! Size, edge, surface, consistency, tender
Kidneys are not enlarged and no associated ascites

Peripheral examination
•! CLD stigmata, jaundice, bruises
•! Hepatic encephalopathy
•! Causes
o! Pallor, cachexia, Cx LNs, PRV
o! Toxic, rashes, tonsils
o! Chronic ethanol ingestion
o! CCF
o! SBE, SLE, RA, Hemolytic anaemia

I would like to complete the examination

In summary, this patient has hepatosplenomegaly that is associated with. The differential diagnoses are:

(Determine which is the predominantly enlarged organ eg massive liver with small spleen or massively
spleen with small liver; determine if there is any Cs liver findings such as pulsatile liver; if both are mildy
enlarged then combine the causes)

Massive Splenomegaly (>8 cm)


•! CML
•! Myelofibrosis
•! PRV
65
•! Chronic malaria
•! Kala-azar (visceral leshmaniasis)
•! Others(Gaucher’s, rapidly progressive lymphoma)

Moderately Enlarged (4 to 8 cm/ 2-4 FB)


•! Myeloproliferative
•! Lymphoproliferative
•! Haemotological – AI, ITP, Thalassemia and HS
•! Chronic malaria
•! Cirrhosis

Mildly Enlarged(4cm</1-2FB)
•! Myeloproliferative, Lymphoproliferative
•! Infections
•! Viral – CMV,EBV
•! SBE, splenic abscesses, leptospirosis, Meliodosis, TB, Typhoid, Brucellosis(farmer)
•! Acute malaria
•! Infiltrative – Amylodosis, Sacoidosis
•! Endocrine – Acromegaly, thyrotoxicosis
•! Collagen vascular – SLE, Felty’s
•! Chronic haemolytic – Thalassemia, AI, HS, ITP

Tender spleen
•! Infective causes
•! Acute myeloproliferative and lymphoproliferative

Pallor(same as moderately enlarged spleen)


•! Myeloproliferative
•! Lymphoproliferative
•! Malaria
•! Hemolytic anaemia(Thal and AIHA)
•! AI – Felty’s, SLE
•! Cirrhosis of the liver with portal hypertension

Lymph Nodes
•! Lymphoproliferative(CLL/lymphoma)
•! Infective(IMS, Meliodosis, CMV, TB, HIV)

Massive Liver
•! HCC/Secondaries/myeloprolif
•! RVF
•! Alcoholic liver disease

Mild-moderate Liver
•! As above plus
•! Infection
•! Viruses – EBV, CMV, hepatitis A & B
•! Bacteria – Weil’s disease (leptospirosis), meliodosis, abscesses, TB, brucellosis, syphilitic gumma
•! Protozoal – hydatid cysts, amoebic abscess
•! Malignancy – lymphoproliferative, myeloproliferative, primary, secondary, adenoma from OCP
•! Infiltrative – sarcoid (erythema nodosum, lupus pernio), amyloid, fatty liver
66
•! Endocrine – acromegaly, hyperthyroid
•! Collagen Vascular disease
•! Chronic hemolytic anaemia( AI, thalassemia, HS)
•! Reidel’s lobe
•! Possibility of minimal CLD signs with just hepatomegaly
•! PBC
•! Hemochromatosis

Tender Liver
•! Liver abscess/infective (viral/bacterial/parasitic)
•! HCC/Secondaries
•! Right Heart Failure/Budd chiari

Pulsatile Liver
•! TR
•! HCC
•! AVM

Hard/Irregular Liver
•! Mitotic (primary/Secondary)
•! Macronodular cirrhosis (post hepatitis B/C, Wilson’s and AAT)
•! Amyloidosis/Hydatid cyst/granulomatous disease/gummatous disease/APCKD

Ascites
(Think of CLD, Budd-Chiari, renal failure or heart failure, hypothyroidism, also malignancy, TB)

Presentation

Sir, this patient has gross ascites.

There is presence of abdominal distension with an everted umbilicus. There is a positive fluid thrill as well
as shifting dullness. This is not associated with any abdominal tenderness and patient is able to lie flat for
the examination. There is also abdominal scar marks suggesting abdominal tap has been done.

I am unable to palpate the liver and it has a span of 12 cm in the right mid-clavicular line. The spleen is not
palpable or percussible. The kidneys are not ballotable. There are no other masses palpable in the abdomen.

There are no stigmata of chronic liver disease such as leukochynia, clubbing, palmar erythema, spider naevi,
gynaecomastia or loss of axillary hair. There is also no hepatic fetor or a hepatic flap. Patient is not jaundice
and there is no conjunctival pallor.

There is associated pedal edema up to the knee level with sacral edema but no periorbital edema. There are
no signs of renal failure such as a sallow appearance or uremic fetor.

Patient also does not have any features to suggest hypothyroidism such as a cream and peaches complexion,
macroglossia, hoarseness of voice or bradycardia.

He is not cachexic looking and there are no palpable cervical LNs. He is not toxic looking.

I would like to complete my examination by


•! CVS looking at the JVP with the patient seated 45 degrees to look for raised JVP with steep x and y
descent, early S3 suggestive of constrictive pericarditis

67
•! Urine dipstick for proteinuria
•! Temperature chart for fever (TB)
•! Rectal examination for a rectal mass

In summary, this patient has got gross ascites that is not associated with any intra-abdominal organomegaly
or masses of which no apparent cause is found clinically. The possible differential diagnoses include cirrhosis
of the liver, Budd-chiari syndrome, nephrotic syndrome or protein-losing enteropathy, congestive cardiac
failure or intra-abdominal malignancy or TB.

Questions

What are the causes of abdominal distention?


•! Fat, fluid, flatus, faeces, fetus and organ enlargement

What is ascites?
•! Pathologically accumulation of fluid in the peritoneal cavity

How much fluid must be present before there is flank dullness?


•! 1.5 L of ascitic fluid

How would you approach a patient with ascites clinically?


•! Abdominal examination
o! Liver
!! Look for jaundice, spleen and stigmata of CLD – cirrhosis of liver
!! Liver palpable and smooth – think of Budd-chiari
!! Liver palpable and hard and nodular – think of malignancy
o! Kidneys
!! Look for evidence of kidney failure and anasarca
o! Look for congestive cardiac failure or constrictive pericarditis
o! Look for features of hypothyroidism
o! If all above absent, think of
!! TB peritonitis
!! Intra-abdominal malignancy
•! Carcinomatosis peritonei
•! Secondaries
o! Liver
o! Colon
o! Ovaries
o! Pancreas

What are the causes of ascites?


•! Serum ascites albumin gradient >1.1g/dl = portal hypertension (97% accuracy)
o! Cirrhosis of the liver
o! Budd-Chiari
o! CCF
o! Constrictive pericarditis
o! Malabsorption
o! Meig’s syndrome
o! Hypothyroidism
•! Serum ascites albumin gradient< 1.1g/dl
o! Intra-abdominal malignancy
o! TB
o! Nephrotic syndrome
68
o! Protein losing enteropathy

What is the pathophysiology of ascites in cirrhosis of the liver?


•! The chief factor is splanchnic vasodilatation
•! Cirrhosis leads to increased resistance to portal flow
•! Leading to portal hypertension
•! Portal hypertension results in local production of vasodilators, with splanchnic arterial vasodilatation
•! (1) Arterial underfilling
o! Early stage – minimal effect on effective arterial volume as can be compensated by increase in plasma
volume and cardiac output
o! Later stage
!! splanchnic vasodilation so marked that effectve arterial pressure falls and results in activation
of vasoconstrictors and atrial natriuretic factors
!! Sodium and fluid retention and expansion of plasma volume contributing to ascites
!! Impaired free water execretion leading to dilutional hyponatraemia
!! Renal vasoconstriction with hepatorenal syndrome
•! (2) Increase in splanchnic capillary pressure with lymph formation exceeding return therefore ascites

How would you investigate to determine the cause of the ascites?


•! (Liver, renal, heart, thyroid, TB)
•! Ascitic tap
o! Cell count, albumin, and total protein concentration if cirrhosis and dx
!! See attached
o! Others
!! Infection – c/s and g/s AFB
!! Malignancy - cytology
o! <0.1% of Cx such as hemperitoneum or bowel perforation
o! 1% of abdominal wall hematoma
o! 2FB cephalad and medial to the ASIS in the left lower quandrant
•! Imaging
o! USS/CT
!! Liver – cirrhosis, budd-chiari
!! Renal
o! Echo and ECG
o! CXR (TB, Pl effusion)
•! Bloods
o! LFT, Renal, TFT, FBC

How would you manage a patient with ascites secondary to cirrhosis of the liver?
•! Treat the underlying cause
•! Avoid alcohol or medications that are toxic to liver
•! Management of ascites
o! General measures
!! Salt restriction <2 g/day
!! Fluid restriction <1l/day (for ascites, edema with Na <130)
o! Specific measures
!! Diuretics (Spironolactone, frusemide initially)
•! Aim to 0.5kg/day if no peripheral edema
•! Aim 1kg/day if presence of peripheral edema also
•! Increase diuretics with spironolactone up to 400mg/d or frusemide 160mg/d
!! Paracentesis
•! If >5L then requires albumin administration (8g per L of fluid removed)
!! TIPSS (Transjugular Intrahepatic portosystemic shunt)

69
•! High rate of shunt stenosis; up to 75% at 1 year
o! Liver transplant
!! 5 year survival rate for cirrhosis with ascites is 30-40% vs 70-80% for post liver transplant
!! MELD score (Model for End Stage liver disease which has bilirubin, creatinine and INR)
!! Consider for those with refractory ascites, SBP or HRS
•! Manage other complications of cirrhosis

How do treat and prevent spontaneous bacterial peritonitis?


•! Defined as >250 polymorphs per ml of ascitic fluid
•! Commonly E coli, Klebsiella and pneumococci
•! Translocation of bacteria from intestinal lumen to LNs then bacteremia
•! Rule out secondary peritonitis
o! Loculated infection or perforated viscus
o! Fluid
!! >1000 polymorphs
!! LDH > upper limit of serum
!! Low glucose
!! High protein >1 g/L
!! CEA > 5ng/ml
!! ALP >240u/L
•! Treatment
o! 3rd generation cephalosporin
o! IV albumin to prevent HRS
•! Prevention
o! Indications
!! After 1 episode of SBP as recurrence as high as 70%/year
!! In patients with acute variceal bleed
!! Ascitic fluid protein concentration<1g/dl (controversial)
o! Prophylaxis with ciprofloxacin or norfloxacin

What does development of ascites in a patient with cirrhosis of the liver means?
•! Decompensation
•! Occurs in 50% of patients within 10 years of diagnosing compensated cirrhosis
•! Poor Px
o! only 50% survive beyond 2 years
o! poor quality of life
o! increased risk of infection and renal failure

Unilateral Enlarged Kidney

1. Sir, this patient has an enlarged left kidney and is on hemodialysis. There is presence of an enlarged left
kidney as evidence by a left flank mass that is bimanually palpable and ballotable with a nodular surface. I
am able to get above the mass and there was no palpable notch. Percussion note is resonant above the mass
and it moves inferiorly with respiration. It is non tender and not associated renal bruit.

2. Sir, this patient has an enlarged right kidney and is on hemodialysis. There is presence of a right flank
mass that is bimanually palpable and ballotable with a nodular surface. I am able to get above this mass and
the mass does not cross the midline. Percussion note is resonant over the mass and it moves inferiorly with
respiration. It is non-tender and there is no associated renal bruit.

There is no ascites. The liver is not enlarged with a span of 12 cm in the right midclavicular line. The spleen
is also not enlarged. I did not notice any scars.

70
The patient has evidenced of chronic renal failure of which he is receiving hemodialysis. There is presence
of a sallow appearance. I did not notice any pruritic scratch marks or bruises on the ULs or LLs and he is not
cachexic looking. There are no signs of leukonychia or Terry’s nails. There is no conjunctival pallor to
suggest anaemia and he does not have features of polycythemia. Patient is not in fluid overload, has no uremic
fetor or flapping tremor of the hands and no Kussmaul’s breathing.

Patient is undergoing hemodialysis via a left arterio-venous fistula. There is a strong thrill felt over the fistula
with recent needle injection marks. There are no complications of aneurysm of the fistula. There are no
abdominal scars to suggest previous TK insertion or renal transplant.

I would like to complete my examination by


•! Temperature chart for fever
•! Blood pressure for hypertension
•! Fundoscopy for hypertensive changes
•! (don’t mention urine analysis if ESRF on RRT!)
•! Cardiovascular examination – for MVP and AR
•! Neurological examination for III nerve palsy of stroke
•! FHx of stroke or aneurysm

In summary, this middle age gentleman has an enlarged left sided kidney with complications of ESRF of
which he is undergoing hemodialysis. The most likely aetiology is asymmetrically enlarged Adult Polycystic
Kidney disease.

Questions
What are the causes of a unilaterally enlarged kidney?
•! Causes of bilateral asymmetrical enlargement
o! APCKD, Acromegaly, DM, bilateral HN
o! Tuberous sclerosis, VHL, Amyloidosis
•! Unilateral disease
o! RCC
o! Acute renal vein thrombosis
o! Pyonephrosis
o! Hypertrophy of a single functioning kidney

Bilateral Enlarged Kidneys


Presentation
Sir, this patient has bilateral enlarged kidneys. There are bilateral masses in the flanks which are bimanually
palpable and ballotable with a nodular surface. Of note, I am able to get above both masses. Percussion note
was resonant over both kidneys and they move inferiorly with respiration. They are not tender in nature and
there was no renal bruit.

There is no associated hepatomegaly and the liver span is 12 cm at the right mid-clavicualar line. The spleen
is not enlarged. There is no ascites detected clinically and the bladder is not palpable or percussible.

The patient does not have a sallow appearance and not cachexic looking. There are no pruritic scratch marks
or bruising. There is also no leukonychia or Terry’s nails. There is no conjunctival pallor to suggest anaemia
and no features of polycythemia such as a plethoric facies or conjunctival suffusion. Patient is not in fluid
overload as there is no pedal oedema, he is able to lie flat and is not oxygen dependent. There is no
Kussmaul’s breathing pattern and also no flapping tremor or uremic fetor.

He does not have any acromegalic features, no DM dermopathy and no adenoma sebaceum to suggest
tuberous sclerosis.

71
There is no evidence of renal replacement therapy such as AVF, TK cathether or a transplanted kidney.

I would like to complete my examination by


•! checking the patient’s temperature chart for fever,
•! blood pressure for hypertension
•! fundoscopy for hypertensive changes
•! urine disptick for hematuria, proteinura and pyuria
•! Cardiovascular examination for signs of MVP or AR
•! Neurological examination for a third nerve palsy secondary to berry aneurysm or any evidence of a stroke
•! FHx of aneurysm or SAH (5% risk overall but 20% if positive FHx)

In summary, this middle age gentleman has got bilateral enlarged kidneys with no complications of chronic
renal failure detected clinically. There is also no evidence that the patient is undergoing renal replacement
therapy. The most underlying etiology is Adult Polycystic kidney disease.

Questions

What are the causes of bilateral enlarged kidneys?


•! APCK
•! Commoner
o! Acromegaly (hepatosplenomegaly)
o! Early diabetic nephropathy
o! Bilateral hydronephrosis
•! Rare
o! Tuberous sclerosis
o! Amyloidosis
o! Von-Hippel Lindau disease
!! Autosomal dominant
!! Multiple angiomata in the retina, CNS
!! Cysts in liver, kidneys pancreas
!! RCC, phaeochromocytoma

What are the conditions that can result in bilateral renal cysts?
o! Polycystic kidneys
!! Dominant and recessive
!! Simple cyst
!! Von Hippel Lindau
!! Tuberous sclerosis

What are the complications of APCK? (Renal and Extra-renal Cx)


•! Fever
o! UTI, pyelonephritis, pyocyst
•! Hypertension (75%)
o! Activation of RAA from intra-renal ischaemia from architectural distortion
o! Malignant hypertension
!! Renal artery stenosis from compression
!! Renin producing cyst
•! Pain
o! Chronic pain
o! Acute pain
!! UTI
!! Nephrolithiasis
!! Cyst rupture
!! Haemorrhage into cyst
72
!! Upper tract obstruction
•! Massively enlarged cyst
•! Clot
•! Stone
•! Anaemia
o! CRF
o! Persistent gross hematuria
•! Polycythaemia
o! Increased erythropoietin production
•! Malnutrition
o! CRF
o! Bilateral renal enlargement with early satiety
•! Acute renal failure
o! Malignant hypertension
o! UTI
o! Nephrolithiasis (Uric acid)
•! Chronic renal failure
•! Renal cell carcinoma (rare)
•! Extra-renal
o! Abdominal – cysts in liver, spleen, pancreas, ovaries; colonic diverticular disease
o! Cardiac – MVP(25%), AR, TR
o! Intracranial aneurysm (III nerve palsy), SAH (3%)

What are the complications of CRF?


•! Fluid
•! Electrolytes – Hyperkalaemia
•! Acid-base – Metabolic acidosis
•! Uremia and its complications
•! Hypertension
•! Anaemia (NCNC)
•! Secondary and tertiary hyperparathyroidism
•! Renal bone disease

Why are patients with CRF sallow?


•! Impaired execretion of urinary pigments combined with anaemia

What are the types of signs in the nails that you can detect in patients with CRF?
•! Hypoalbuminaemia
o! Leukonychia
o! Muehrcke’s nails (paired white transverse line near the distal end of nails)
•! Renal failure
o! Terry’s nails (distal brown arc 1mm or >)
o! Mee’s line (single white line; also in arsenic poisoning)
o! Beau’s line (non-pigmented indented band = catabolic state)

What are the causes of anaemia in patients with CRF?


•! Erythropoeitin deficiency
•! Anaemia of chronic disease
•! Fe deficiency anaemia – blood loss, nutrition
•! Folate deficiency – nutrition

What is Adults Polycystic Kidney disease?


73
•! Multisystemic, progressive disease, 1 in 400 to 1 in 1000 people
•! Characterised by cysts formation and enlargement in the kidneys and other organs
•! Autosomal dominant with almost 100% penetrance
•! Focal cystic dilatation of the renal tubules
•! 2 predominant type
o! 85% - APCKD 1 on Ch 16
o! 15% - APCKD 2 on Ch 4
o! 3rd type of which loci is not fully known
•! Presents clinically in the 3rd or 4th decades with
o! Hematuria, hypertension, recurrent UTI, pain and uremia
o! Stroke
•! By age 60 years, 50% will require RRT
•! Poorer Px – males, PCK 1 and early onset of clinical features
•! Mortality
o! ESRF (1/3)
o! Stroke and other hypertensive Cx (1/3/)
o! Others

How do you investigate?


•! Blood Tests
o! FBC
o! Biochemical
o! CRF – Ca, PO4, iPTH, Uric acid, urinalysis
•! USS (useful >20 years old); Ravine’s criteria
o! At risk patients, 20-30yrs: 2 cysts in 1 kidney or 1 cyst in each kidney
o! At risk patients, 30-60 yrs: 2 cysts in each kidney
o! At risk patients, >60 yrs: 4 cysts in each kidney
•! Other imaging(CT and MRI)
•! MRA for patients with high risk of an aneurysm, Ba enema and Echocardiogram
•! Genetic testing
o! For young people with no cysts on USS who are potential organ donors

How would you manage?


•! Education and counselling, regular follow up, screening of first degree relatives
•! Avoidance of medications that can precipitate renal impairment such as NSAIDs or tetracycline
antibiotics
•! Medical treatment
o! Hypertension with ACE inhibitors or ATII RA
o! UTI, cysts infection – usually GN bacteria therefore use Bactrim or fluroquinolones with good renal
tissue penetration
o! Pain treatment
o! Renal failure – medical treatment and RRT for those with ESRF
o! Antibiotic prophylaxis
•! Surgical treatment
o! Pyocyst – drainage
o! Cystectomy
o! Nephrectomy
o! Alcohol sclerosant
o! RRT
o! Aneurysm clipping, MVP with MR

74
Transplanted Kidney

Presentation

Sir, this patient has a transplanted kidney in the right iliac fossa associated with bilateral enlarged kidneys
with a functioning AVF with features of cyclosporine and chronic steroid use.

There is presence of a rounded palpable mass in the right iliac fossa with an overlying scar. It is non-tender.
In addition there are bilateral masses in the flanks which are bimanually palpable and ballotable with a
nodular surface. I am able to get above these masses and they are not tender. They move inferiorly with
respiration and percussion note is resonant over them. There is no associated ascites and no renal bruit. The
liver and spleen are both not enlarged.

The patient does have features of renal impairment with a sallow appearance and is thin looking. He does
not have any bruises or pruritic scratch marks and no leukonychia or Terry’s mails were detected. There is
also no conjuctival pallor to suggest anaemia and there are also no features of polycythemia such as a
plethoric facies or conjunctival effusion. He is also not in fluid overload with no pedal edema and is able to
lie flat and is not oxygen dependent. There is no Kussmaul’s breathing with no uremic fetor or flapping
tremor of the hands.

There is presence of an arterio-venous fistula in the right upper limb. It is functioning with a good thrill.
There are no recent needle puncture marks and no aneurysm was noted.

There is presence of diabetic dermopathy noted on the lower limbs.

There is no evidence of transplant related hepatitis B or C with no jaundice or stigmata of chronic liver
disease.

Patient has hypertrichosis and gum hypertrophy which are complications of cyclosporine usage. Moreover,
he has a Cushingnoid habitus with steroid purpura and thin skin, suggesting chronic steroid usage.

I would like to complete my examination:


•! Temperature chart for fever
•! BP for hypertension
•! Fundoscopy for hypertensive changes
•! Urine analysis for hematuria, proteinura or pyuria
•! CVM – MVP or AR
•! Neurological – III nerve palsy or PHx of stroke

In summary, this middle age gentleman has a transplanted kidney for underlying Adult Polycystic kidney
disease with previous dialysis. The graft is functioning well as he is not uremic and is well with features of
cyclosporin and steroid use.

Questions

What are the differential diagnoses for a right iliac fossa mass?
•! Transplanted kidneys
•! Carcinoma of the caecum (hard mass, LNs)
•! Abscess – appendicular, ileocecal
•! Crohn’s disease (mouth ulcers, PR for fistulas)
•! Ovarian tumors (in females)
•! Others
o! Amoebiasis, TB lymphadenitis, actinomycosis
o! Carcinoid
75
o! Ectopic kidney

What are your differential diagnoses for a left iliac fossa mass?
•! Transplanted kidney
•! Colonic carcinoma (hard mass, hepatomegaly LNs)
•! Diverticular abscess
•! Fecal mass
•! Ovarian tumors
•! Others – lymphadenitis

What are the common kidney diseases leading to transplant?


•! DM
•! Hypertension
•! GN

How does renal transplant compare with dialysis?


•! Higher patient survival rates
•! Better quality of life with lower hospitalisation rates

What are the causes of transplant loss?


•! Patient death
•! Allograft failure
o! Immunological
!! Acute rejection
•! Single most important event determining graft survival
•! Can result in rapid loss of graft or progression to chronic rejection or chronic allograft
nephropathy
•! Treated with pulse steroid or anti-lymphocyte antibody therapy
!! Chronic rejection
o! Non-immunological
!! Renovascular thrombosis
!! Ischaemia reperfusion injury
!! Nephrotoxicity from calcineurin inhibitors
!! CMV, polyoma virus
!! DM, hypertension, hyperlipdaemia
o! Others
!! Recurrence of primary disease (GN and DM)
!! Chronic allograft nephropathy

What is delayed graft function?


•! Defined as requirement of dialysis in the first week post transplant
o! Immunological – acute rejection
o! Non-immunological – ischaemia reperfusion injury, donor hypertension

What are the strategies one can use to reduce graft loss?
•! Immunological
o! Live donor better than cadaveric
o! HLA matched at A, B and DR loci
o! Absence of pre-sensitisation
•! Previous transplant
•! Pregnancies
•! Transfusions
•! Idiopathic
76
o! Immunosuppresive therapy to reduce acute rejection
•! Traditionally use of steroid and cyclosporin
•! Others
•! Calcineurin inhibitors eg Cyclosporin and tacrolimus
•! Mycophenolate mofetil
•! Sirolimus
•! Non-immunological
o! Pre-transplant
•! Donor factors – old age, CVA, hypertension
•! Recepient factors – older, male, obese, diabetic, hypertension
o! Technical factors
•! increase cold ischemia time – LD transplant, renoprotective preservative solutions
•! hyperfiltration from inadequate nephron dose – match size and better if male to female; use
of ACE inhibitors
o! Post-transplant
•! Calcineurin inhibitors induced nephrotoxicity
•! Monitor levels
•! Use others such as sirolimus or MMF
•! CMV infections and polyoma virus
•! Prophylaxis with ganciclovir for CMV
•! No Rx for polyoma virus
•! Treat BP (<130/80) and hyperlipidaemia and DM

What are the complications of cyclosporine?


•! Hirsutism/hypertrichosis
•! Hypertrophy of the gums
•! Hypertension
•! Hyperkalaemia, hyperuricaemia, hypercholesterolaemia, hypomagnesemia
•! Hepatotoxicity
•! Hemolytic uremic syndrome
•! Hiccuping (gastroparesis)
•! Hole-in-bones (osteoporosis)
•! Nephrotoxicity
•! Neoplasia (lymphoproliferative)
•! Neurological (tremors, headaches, seizures and strokes)

What are the complications of chronic steroid use?


•! Skin – thin skin, telangiectasia, steroid purpura
•! Cushingoid habitus
•! Osteoporosis, AVN femoral head
•! Peptic ulcer disease
•! Hypertension
•! Diabetes mellitus
•! Cataracts
•! Steroid psychosis

How do yo manage?
•! Education and counselling, regular follow up, compliant
•! Treat underlying cause
•! Require preparation prior to transplant
•! Post transplant management to reduce graft loss (See above)

77
Assessment of Higher Cortical Function

“Examine this man’s cortical function”


o! Obvious gaze preference or hemiparesis
o! Determine which side first with sensory inattention (Opp to lesion)
!! Visual inattention (always check gross VA first)
!! Tactile inattention
!! Line bisection test (parietal lobe function)
!! Circling alphabets (frontal lobe)
o! Determine side (opp to lesion side)
!! Astereognosis (coins, pen)
!! Graphaesthesia (write 5 nos on each palm using pen with cover)
!! Visual field for hemianopia
o! If suspect left side
!! Dysphasia assessment (expressive, receptive, nominal and conductive)
!! Gerstmann’s syndrome (AALF)
o! If suspect right side
!! Constructional apraxia (copy a cube)
!! Dressing apraxia
!! Spatial neglect (Right lesion = fill in nos on left only)
o! Parietal lobe function
!! Gerstmann’s and nominal dysphasia vs apraxia and spatial
o! Frontal lobe
!! Grasp reflex and palmomental reflex, glabella tap
!! Optic atrophy
!! Anosmia
!! Expressive dysphasia
!! Labile emotion, personality changes
!! Urinary catheter
!! Gait apraxia
o! Temporal lobe
!! Superior quandrantonopia
!! Receptive dysphasia
!! Short and long term memory
o! Occipital Lobe
!! Cortical blindness
!! Hemianopia with macula sparing
o! Look for contralateral UMN VII or ipsilateral XII
o! Check for pronator drift
o! Aetiology
!! Proceed to check pulse, carotid bruit, murmur, hyperlipidemia, DM dermopathy and
tar stains
!! Request BP, urinalysis for DM and fundoscopy (papilledema)
o! Complications
!! DVT, sacral sore, bedside swallow test, aspiration pneumonia

78
“Examine this man’s speech”
•! Shake his right hand
o! Look for weakness on the right UL (Dysphasia)
o! Look for ataxia of the UL (cerebellar)
o! Look for tremors (Parkinsonism)
•! Dysphonia
o! Ask him for his name, how old he is, how he came to hospital and what he did this am
o! Recurrent Laryngeal nerve or laryngitis
o! Offer to examine the left chest, radiation marks, enlarged Cx LNs and look for Horner’s, and
wasting of T1
•! Dysarthria
o! British Constitution and count 1 to 20
!! Cerebellar : slow, slurred, explosive and irregular
!! Parkinsonism : Monotonous, low volume
o! Exclude Cerebellar and Parkinsonism
!! Proceed accordingly if these are detected
o! Say Ba Ba Ba – VII nerve palsy
o! Say La La La – Pseudobulbar palsy
o! Say Ke Ke Ke – Bulbar palsy
•! Dysphasia
o! Expressive
!! What is your favourite colour, what you have for breakfast this am
!! Broca’s area
o! Receptive
!! close your eyes, close your eyes and stick out your tongue, close your eyes and stick
out your tongue and lift both hands
!! Wernicke’s area – posterior part of first temporal gyrus
o! Conductive
!! No if and or but
!! Arcuate fasciculus – linking Broca’s and Wernicke’s area
o! Nominal
!! Shirt, sleeve, button OR Watch, straps face
!! Angular gyrus (temporal-parietal)
o! Global (Expressive and receptive)
•! Once dysphasia is found
o! Do Gerstmann’s syndrome
!! Acalculia (serial 7 subtraction)
!! Agraphia (inability to write)
!! Left-right disorientation
!! Finger agnosia
o! Demonstrate other cortical signs
!! Visual fields, sensory inattention, graphaesthesia, astereognosis
o! Demonstrate UMN VII and hemiparesis
o! Determine aetiology as above (Vascular, tumour)
o! Determine complications as above

79
Cranial Nerves

Conforming
•! Superior Orbital syndrome (see VI nerve palsy)
•! Cavernous sinus syndrome (see VI nerve palsy)
•! Cerebellar Pontine Angle syndrome
o! Involvement
!! V1-3 (tinnitus and deafness earliest symptom then vertigo; loss of corneal reflex is the
earliest sign)
!! VI, VII, VIII
!! IX
!! Cerebellar
o! Causes
!! Tumor
•! Primary
o! “Acoustic neuromas” – schwannomas of the vestibular
o! Meningiomas, haemangioblastomsa, medulloblastomas
o! Choleastoma
•! Secondaries
o! NPC ( loss of corneal reflex and V2 early)
o! Lymphoma
!! Aneurysm
!! For bilateral lesions
•! Bilateral “acoustic neuromas” in NF type 2
o! Examination
!! Examine CNs
!! ULs for cerebellar signs
!! Proceed to check neck for LNs
!! Look for NF features (café au lait spots, neurofibroma, freckling and Lisch nodules)
o! Presentation
!! Sir, this patient has righ/left CPA lesion as evidenced by
!! There is no enlarged Cx LNs to suggests secondaries
!! There is also no evidence of NF
!! Possible etiologies includes
o! Questions
!! What is the CPA?
•! Shallow trangular fossa lying between the cerebellum, lateral pons and the petrous
temporal bone
!! Histology?
•! Schawannoma
!! Ix?
•! Imaging – CT/MRI/Angio
•! Audiography
•! ENT to exclude NPC
!! Mx
•! Microsurgical resection
•! Stereotaxic radiosurgery (Cx rate same as surgery)
•! Lateral medullary syndrome
o! 5 vessels involved (wedge shaped infarction of the lateral aspect of the medulla and the inferior
surface of the cerebellum)
!! PICA
!! Vertebral artery (most common artery that is involved)
!! Lateral medullary artery (superior, middle, inferior)
o! Areas affected
80
!! Descending sympathetic fibres
•! Ipsilateral Horner’s syndrome
•! Ptosis, meiosis and anhidrosis
!! Spinothalamic tract
•! Contralateral hemi-sensory loss of pain and temperature
!! Descending tract and nucleus of V
•! Ipsilateral loss of pain and temperature of the face
!! Nucleus ambiguus(X) and IX
•! Hoarsenss of voice, dysphagia, hiccups
!! Vestibular nuclei
•! Nystagmus, vertigo, nausea
!! Cerebellar (restiform body of the inferior cerebellar peduncle)
•! Ipsilateral ataxia and gait ataxia
o! Examination
!! CN examination
!! Go to ULs for loss of pain and temperature and cerebellar
!! Check for AF and DM dermopathy
!! Visual Fields for homonymous hemianopia (posterior circulation)
o! Sir this patient has right/left LMS as evidenced by
!! State the findings
!! Mention NG
!! Aetiology – infarction affecting the vertebral artery or the PICA, LMA
!! Did not find any xanthelesma or DM dermopathy, or AF
!! Request for BP and asking patient on symptoms of dysphagia
•! Medial medullary syndrome
o! Triad of XII, medial lemniscus and pyrimidal tract
o! Ipsilateral wasted tongue, contralateral loss of vibration and propioception and contralateral
hemiparesis respectively
o! Either vertebral artery or lower basilar
•! Bulbar palsy
o! Bilateral involvement of LMN IX, X, XI and XII
o! Examination
!! Proceed with CN
!! Do Jaw jerk
!! Requests to examine speech, and gag reflex
!! Requests to examine ULs for fasciculations and dissociated sensory loss
o! Presentation
!! Patient has bulbar palsy as evidenced by weakness of the soft palate, wasted tongue with
fasciculations a/w a nasal voice and a normal or absent jaw jerk
o! Causes (MGS, NNNP)
!! MND
!! GBS
!! Syringomyelia
!! Poliomyelitis, NPC, neurosyphilis and neurosarcoid
•! Pseudobulbar palsy
o! Bilateral UMN lesions of the IX, X and XII, V and VII (III/IV and VI are spared)
o! Examination
!! Proceed with CN
!! Do jaw jerk
!! Request for speech, gag reflex and enquire emotional lability
!! Request for AF, DM dermopathy and xanthelesma
!! Requests for ULs to look for UMNs
o! Presentation

81
!! Patient has PBP as evidenced by presence of sluggish palatal movement, small, stiff and
spastic tongue a/w brisk jaw jerk with “Donald duck” speech (slow, thick and indistinct)
!! No AF, DM or xanthelasma
!! No evidence of mix UMN and LMN signs to suggest MND
!! No RAPD or INO to suggest multiple sclerosis
!! Possible causes (BMM)
•! Bilateral stroke
•! MND
•! Multiple sclerosis
•! Syringobulbia
o! See syringomyelia
o! Extension of syrinx to involve the brainstem
o! V(descending tract of V), VII, IX, X, XI, XII and Horner’s syndrome
o! Usually unilateral
•! Jugular foramen syndrome
o! Involvement of the IX, X, XI (XII maybe affected due to proximity)
o! Unilateral
o! Examination
!! CN exams
!! Proceed to check for enlarged Cx LNs
!! And request to assess speech for husky voice and bovine cough
o! Presentation
!! Sir, this patient has right/left JFS as evidenced by
!! Notice that this patient is on NG
!! No enlarged Cx LNs
!! Possible etiologies includes
o! Questions
!! Causes
•! Ca of the pharynx (commonest cause), tumor, neurofibroma
•! Basal meningitis
•! Paget’s disease, trauma
•! Thrombosis of the jugular vein
!! IX, X and XI leaves the skull via jugular foramen (between the lateral part of occiput and
the petrous part of the temporal bone)
!! XII leaves via the anterior condylar foramen
!! Isolated XI implies injury to XI in the neck eg stab wounds
Non-Conforming
•! Myasthenia Gravis (see Myasthenia Gravis)
•! Miller Fisher Syndrome
o! Variant of Guillain Barre syndrome
o! Characterise by triad of ophthalmoplegia, ataxia and areflexia
o! Cs by anti G1Qb antibodies
o! Rare
o! Good prognosis with recovery beginning within 1 month of onset and complete recovery within
6 months
o! Some maybe left with residual weakness and 3% will have relapses
•! Guillain-Barre syndrome
•! Mononeuritis multiplex
•! Migraine (paralytic)
•! Paget’s
•! Base of skull (trauma)
•! Basal meningitis
•! Brainstem strokes or multiple sclerosis
82
Isolated Third Nerve Palsy

Examination
•! Complete the examination routine for eyes or CN as instructed
•! Proceed to look for intortion of the affected orbit by tilting the head towards the involved site or looking
for intortion when asking patient to look down and medially of the affected eye; patient maybe tilting his
head voluntary away from the side of the lesion (implies 4th nerve palsy)
•! Rule out
•! Thyroid, MG
•! Superior orbital syndrome and Cavenous sinus syndrome
•! Proceed with
•! Neck for LNs
•! Examine the upper limbs for Cerebellar, hemiplegia, EPSE and areflexia
•! Look dor DM dermopathy
•! Request
•! Corneal reflex (reduced or absent)
•! Visual fields (bitemporal hemianopia)
•! Fundoscopy for optic atrophy (MS), DM or hypertensive changes
•! Visual acuity
•! Blood pressure
•! Urine dipstick
•! Temperature chart
•! Headache or pain

Presentation

Sir, this patient has an isolated right third nerve palsy as evidenced by presence of
•! Divergent strabismus involving the right orbit which is in a “down and out” position
•! Complete ptosis/partial ptosis of the right eye
•! Dilated pupil which is not reactive to direct light and to accommodation

There is no ptosis or superior rectus palsy of the left eye to suggest a III nerve nuclear lesion.

There are no associated CN palsies to suggest superior orbital fissure syndrome or cavernous sinus syndrome.
I did not find any associated 4th CN palsy with presence of intortion on asking the patient to adduct the right
eye and look downwards. The 6th CN is also intact. There is also no paraesthesia of the ophthalmic division
of the 5th CN. Gross VA is also intact.

There are no signs of Graves ophthalmopathy (no conjunctival suffusion and proptosis or lid edema of the
right eye)
There is no evidence of fatiguiability to suggest myasthenia gravis.

On examination of the neck, I did not find any enlarged cervical LNs. There is also no evidence of
hemparesis, cerebellar signs, areflexia or tremors or chorea on examination of the upper limbs. I also did not
notice any diabetic dermopathy.

I would like to complete the examination by:


•! Corneal reflex (reduced or absent)
•! Visual fields (bitemporal hemianopia)
•! Fundoscopy for optic atrophy (MS), DM or hypertensive changes
•! Visual acuity
83
•! Blood pressure
•! Urine dipstick
•! Temperature chart
•! Headache or pain

In summary, this patient has an isolated right third nerve palsy. The possible causes include…

Questions

What is the course and anatomy of the 3rd CN?


•! Nuclear portion – at the midbrain
•! Fascicular intraparenchymal portion – close to the red nucleus, emerges from cerebral peduncle
•! Fascicular subarachnoid portion – meninges, PCA aneurysm(between the PCA and internal carotid)
•! Fascicular cavernous sinus portion – sella turcica between the petroclinoid ligament below and
interclinoid above
•! Fascicular orbital portion – superior orbital fissure

Axons run ipsilateral except those to the (1)superior rectus which is innervated from the contralateral 3rd
nucleus and (2) the levator palpebrae which has innervations from both nuclei.

Hence, right sided 3rd nerve palsy can have contralateral ptosis which is often milder than the ipsilateral
ptosis; also the ipsilateral superior rectus can still be affected due to involvement of the contralateral
fascicular intraparenchymal midbrain portion of the left 3rd nerve.

For pupillary reflex and accommodation, it is served by the Edinger-Westphal nucleus and all axons are
ipsilateral.

What are the causes of an isolated 3rd nerve palsy?


•! Brainstem
•! Infarct, haemorrhage, tumour, abscess, multiple sclerosis
•! For nuclear lesions
•! Will also have contralateral ptosis and elevation palsy
•! May have bilateral 3rd nerve palsies (+/- INO)
•! For fascicular midbrain lesions
•! Weber’s (+ contralateral hemiplegia) – base of midbrain
•! Northnagel (+ contralateral cerebellar) – tectum of midbrain
•! Benedikt’s (+ contralateral hemiplegia, contralateral cerebellar and contralateral tremor, athetosis
and chorea) – tegmentum of midbrain, red nucleus
•! Peripheral
•! Subarachnoid portion- PCA aneurysm, meningitis, infiltrative, others eg sarcoidosis
•! Cavernous sinus lesions- Tumour(pituitary adenoma, meningioma, cranipharyngioma), cavernous
sinus thrombosis, inflammatory (Tolosa-Hunt syndrome which is a non-caseating granulomatous or
non-granulomatous inflammation within cavernous sinus or superior orbital fissure that is treated
with steroids) and ischaemia from microvascular disease affecting the vasa nervosa, mononeuritis
multiplex
•! Orbital- tumor (meningioma, hemangioma), endocrine (thyroid) and inflammatory(orbital
inflammatory pseudotumor ie Tolosa Hunt)
•! Mononeuritis multiplex, Miller Fischer and MG

•! Don’t forget migraines and myasthenia! (emergency – Coning, Giant cell Arteritis and aneurysm)

84
How would patient present?
•! Diplopia
•! Ptosis
•! Symptomatic glare from failure of constriction of pupil
•! Blurring of vision on attempt to focus of near objects due to loss of accomodation
•! Pain in certain etiologies
•! Diabetes mellitus
•! Tolosa-Hunt syndrome
•! PCA aneurysm
•! Migraine

What are the causes of a dilated pupil?


•! III nerve palsy
•! Optic atrophy (direct light and accommodation absent with intact consensual reflex)
•! Holmes Adie Pupil (Myotonic pupil)
o! Unilateral
o! Slow reaction to bright light and incomplete constriction to convergence
o! Young women
o! Reduced or absent reflexes
•! Mydiatric eye drops
•! Sympathetic overactivity

Why does a PCA aneurysm results in pupillary involvement whereas conditions such as DM or hypertension
spares the pupil?
•! The pupillary fibres are situated superficially and prone to compression whereas ischaemic lesions tends
to affect the core of the nerve thus sparing the pupillary fibres

How would you investigate?


•! Imaging
o! CT, MRI
o! Angiogram
•! Blood test
•! Fasting blood glucose, ESR
•! TFT and edrophonium
•! LP

How would you manage?


•! Medical 3rd nerve palsy
•! Education – watchful waiting and avoid driving, heavy machinery and climbing high places
•! Treat underlying conditions such as DM and hypertension
•! Watchful waiting
•! Spontaneously recover within 8 weeks
•! Symptomatic treatment
•! NSAIDs for pain
•! If complete ptosis, no need to treat diplopia
•! Use eye patch for severe diplopia and a prism Fresnel paste on for mild diplopia
•! Surgical 3rd nerve palsy - surgery

85
Isolated VI Nerve Palsy

Examination (Examine patient’s eyes, CNs)


•! Complete the eye examination
•! Rule out MG, thyroid
•! Rule out Cavernous Sinus syndrome, Superior orbital fissure syndrome
•! Proceed to check cranial nerves
•! V, VII and VIII for CPA tumor
•! XI, X, XI, XII for any base of skull lesions
•! Examine upper limbs
•! Hemiplegia (long tract signs suggesting brainstem)
•! Cerebellar signs (CPA lesion, Miller Fisher usually truncal and gait)
•! Reflexes for areflexia (Miller Fisher)
•! Examine the neck for LNs and mastoid for tenderness (Gradenigo’s syndrome)
•! Request
•! Fundoscopy (papilloedema from raised ICP, optic atrophy for MS)
•! Field testing (bitemporal hemianopia)
•! Acuity (reduced in orbital lesions)
•! Corneal testing for reduced sensation form V1 involvement
•! BP
•! Urine dipstick
•! Temperature chart for fever(meningitis)
•! Ask for retrobulbar pain

Presentation

Sir, this patient has an isolated right sided VI nerve palsy as evidenced by
•! Convergent strabismus at primary gaze
•! Failure of abduction of the right eye
•! With diplopia where the image is side by side and furthest apart on rightward gaze, with disappearance
of the outer image on covering the right eye. This suggests a right lateral rectus muscle weakness and
hence a right VI nerve palsy.

There is no evidence of a III or IV or V1 palsy which may suggest cavernous sinus or superior orbital
syndrome.
There is also no involvement of the VII or VIII nerve palsy and no cerebellar signs to suggest A CPA
lesion
There are no associated CN lesions of IX to XII and no enlarged Cx LNs.

There is no fatiguibilty to suggest MG and no thyroid eye signs.

There are no enlarged LNs or tender mastoid. There is also no hemiparesis, no cerebellar signs and reflexes
are present.

I would like to complete my examination by:


•! Fundoscopy (papilloedema from raised ICP, optic atrophy for MS, DM or hypt changes)
•! Field testing (bitemporal hemianopia)
•! Acuity (reduced in orbital lesions)
•! Corneal testing for reduced sensation form V1 involvement
•! BP
•! Urine dipstick
•! Temperature chart for fever(meningitis)
•! Ask for retrobular pain
86
In summary, this patient has an isolated right sided VI nerve palsy. The aetiology includes….

Questions

What is the course of the VI nerve?


•! Nucleus of CN VI located in the pons, sending motor neurons supplying the lateral rectus muscle of the
ipsilateral eye;
•! also internuclear neurons that project (via MLF) to medial rectus subdivision of contralateral
oculomotor (CN III) nucleus (ie lesion of nucleus causes ipsilateral horizontal gaze palsy)
•! nerve fascicles exit the pons anteriorly and course through subarachnoid space where it runs a vertical
(upward) course, along ventral surface of pons (confined by adjacent AICA)
•! pierces dura overlying clivus, entering into Dorello's canal, contacting tip of petrous pyramid (part of
temporal bone; adjacent to mastoid air cells)
•! enters the cavernous sinus after passing through the petroclinoid (Gruber's) ligament
•! runs in body of sinus (rather than lateral wall where CN III, IV, V located) along foramen lacerum
(near internal carotid artery)
•! enters orbit via superior orbital fissure and shortly thereafter pierces lateral rectus muscle

What are the causes of a unilateral VI nerve palsy?


•! Brainstem (Pons)– infarct, haemorrhage, abscess, demyelinating (look for VI and VII palsies due to
close proximity)
•! Aneurysm (ectatic basilar artery)
•! Meningitis
•! Infective – TB, Fungal, HIV, Syphilis, Lyme
•! Mitotic – leptomeningeal carcinaomatosis, secondaries(NPC), lymphoma, radiotherapy
•! Sarcoidosis
•! Trauma
•! CPA lesions
•! Petrous temporal bone (Gradenigo’s syndrome)
•! Cavernous sinus syndrome
•! Superior orbital syndrome
•! Miller Fisher syndrome
•! Mononeuritis multiplex
•! DM, hypertension
•! MG
•! Raised ICP
NB: - most common aetiology of isolated abducens lesion is post-viral in children and ischemia / infarction
(eg diabetic) in adults

What are the causes of bilateral VI nerve palsy?


•! Leptomeningeal causes (see above)
•! Miller Fisher syndrome
•! Mononeuritis multiplex
•! MG
•! Raised ICP
•! Wernicke’s encephalopathy (Ophthalmoplegia, confusion and ataxia a/w Korsakoff’s psychosis from
thiamine deficiency)

What are the syndromes associated with VI nerve palsy?


Central
•! Raymond’s syndrome – ipsilateral VI with contralateral hemiparesis
•! Millard-Gubler syndrome – Ipsilateral VI and VII with contralateral hemiparesis
87
Peripheral
•! Gradenigo syndrome – inflammation of the tip of the petrous bone from mastoiditis; VI and
V(gasserian ganglion therefore ipsilateral pain) and VII
•! CPA
•! Cavernous sinus
•! Superior orbital
Congenital
•! Mobius syndrome (VI with facial diplegia)
•! Duane’s syndrome (Congenital absence of VI nuclei with III nuclei innervating the lateral recti; orbit
retraction on adduction and protrusion on abducting

How do you evaluate diplopia?


•! Evaluation of diplopia follows 3 rules
o! The double vision is maximal in the direction of gaze in the affected muscle
o! The false image is the outer image
o! The false image arises form the affected eye

What is Cavernous sinus syndrome?


•! Lesion in the cavernous sinus leading to
•! Ophthalmoplegia, V1, Horner’s, proptosis, chemosis and pain
•! Causes include carotid aneurysm, carotid-cavernous fistula, tumour and thrombosis, Tolosa Hunt
syndrome

What is superior orbital fissure syndrome?


•! Lesions occurring at the superior orbital fissure leading to
•! Ophthalmoplegia, V1, Horner’s, proptosis, chemosis, pain and Optic Nerve
•! Causes includes meningiomas, hemangiomas and thyroid eye disease, Tolosa Hunt syndrome

What is Miller Fisher syndrome?


•! Triad of ophthalmoplegia, ataxia and areflexia
•! Cs by present of anti GQ1B antibodies
•! Variant of GBS

What are the causes of mononeuritis multiplex?


•! Endocrine – DM, hypt
•! AI – Churg Strauss, Wegener’s, PAN, Sjogren’s, RA, SLE
•! Infective – Lyme, leprosy
•! Infiltrative – Amyloid, sarcoid

How would you investigate?


•! Blood tests
•! fasting glucose, ESR, ANA, VDRL
•! Imaging
•! CT brain or MRI brain
•! Vascular imaging if proptosis / chemosis looking for dural carotid-cavernous fistula or cavernous
sinus thrombosis
•! Examine CSF (also if systemic illness, immunocompr, bilateral, other CN deficits)
•! 8-30% will remain cryptogenic after work-up

How would you manage?


•! Education on Px if ischaemic (see below) and Ix and avoid driving, climbing high places and operating
heavy machinery
•! Treat underlying cause
88
•! Example control DM and BP
•! Symptomatic treatment
•! Patch
•! Prism
•! If chronic, chemodenervation with botox or strabismus surgery

What is the prognosis?


•! most either spontaneously improve or have underlying lesion found
•! ischemic palsies almost always recover completely in 2-4 months while some recovery seen in half of
traumatic cases (but take over one year)
•! if no recovery over 3-6 months then suspect underlying lesion such as tumour
•! few may have chronic isolated abducens palsy of unknown cause
•! follow regularly looking for emergency of new localizing signs and ensure adequate neuroimaging and
ENT evaluation

VII Nerve Palsy

Examination

•! Upon noticing facial asymmetry, proceed to tests VII nerve functions


•! Look up (frontalis) and attempt to push the folds down, close your eyes (orbicularis oculi) and attempt
to force them open, and frown (corrugator superficialis)
•! Look for exposure keratitis, tarsorraphy
•! Nasolabial fold, show your teeth and and blow against closed lips
•! Look for drooling of saliva
•! Determine UMN or LMN, unilateral or bilateral
•! UMN unilateral
•! Examine UL and look for hemiparesis on the same side of the facial weakness
•! Check for xanthelesma, DM signs and BP
•! LMN unilateral
•! Examine other CN
•! VI nerve and contralateral weakness in brainstem lesions
•! CPA lesion (V, VI, VII and VIII with cerebellar)
•! Other CN nerves involvement non-conforming type
•! Basal meningitis lesions
•! Mononeuritis multiplex, MG
•! Therefore proceed to examine the neck
•! Look at the Palate for vesicles
•! Examine the parotids and for surgical scars
•! Mastoid tenderness
•! Examine the neck for cervical LNs
•! Upper limbs
•! Contralateral hemiparesis
•! Ipsilateral cerebellar
•! Ask to examine
•! Otoscopy for vesicles in EC and otitis media
•! For hyperacusis (sensitive to high-pitched or loud sounds)
•! For loss of taste in the anterior two-thirds of the tongue
•! Urine dipstick for glucose and BP

•! Upon noticing facial diplegia, proceed with

89
•! Rule out MG (Bilateral ptosis)
•! Rule out Dystrophia myotonica or fascio-scapular-humeral dystrophy
•! Bilateral LMN VII
•! Test for frontalis, corrugator and orbicularis oculi
•! Ask patient to show teeth and blow against closed lips
•! Look for V, VI, VIII
•! Examine parotids (Sarcoidosis, amyloidosis)
•! Examine tongue (scrotal tongue for MR syndrome)
•! Examine the upper limbs for GBS, MND, leprosy, Lyme’s (radiculopathy) and bilateral cerebellar
signs if suggestive of bilateral CPA tumors
•! Think of rare: Melkersson-Rosenthal syndrome, Mobius syndrome

Presentation

Sir, this patient has got a right sided lower motor neurone facial nerve palsy as evidenced by:
•! Paralysis of both the upper and lower facial muscles on the right
•! Loss of wrinkling of the right side of the forehead, inability to fully close his right eye shut with Bell’s
phenemenon
•! Associated with loss of the right nasolabial fold and drooping of the right angle of the mouth

There are no complications of exposure keratitis and there is no drooling of saliva. I also did not notice any
evidence of a right sided tarsorraphy

There is no associated VI nerve palsy to suggest a brainstem lesion.

There are also no features of Cerebellopontine angle lesion with no involvement of the V, VI, VIII or
cerebellar signs on the right.

I did not find any evidence of a parotid swelling or a surgical scar and there are no vesicles on the palate.
There was no right sided facial oedema or plication of the tongue to suggest the rare syndrome of MR
syndrome.

There is no mastoid tenderness and no enlarged cervical LNs. I did not detect any contralateral hemiparesis
or cerebellar signs.

I would like to complete my examination by


•! Otoscopy for vesicles in EC and otitis media
•! Ask about hyperacusis (sensitive to high-pitched or loud sounds)
•! Ask for loss of taste in the anterior two-thirds of the tongue
•! Urine dipstick for glucose and BP (mononeuritis multiplex)

The most likely cause for this patient would be a right sided Bell’s palsy.

Questions

What is the course of the facial nerve?


•! VII nerve nucleus lies in the pons in close proximity with VI nerve nuclei
•! VII leave the pons with VIII via the cerebellopontine angle
•! It enters the facial canal and enlarges to become the geniculate ganglion
•! A branch is given off to the stapedius muscle and the greater superficial petrosal branch goes to the
lacrimal glands
•! The chorda tympani which supplies taste sensation to the anterior two thirds of the tongue joins the VII
nerve in the facial canal
90
•! VII nerve exits the skull via the stylomastoid foramen, through the parotids with the following branches
•! Temporalis
•! Zygomatic
•! Buccal
•! Mandibular
•! Cervical

What are the causes of a unilateral LMN VII nerve palsy?


•! Brainstem (Infarct/haemorrhage, MS, abscess and tumour, syringobulbia)
•! Base of skull lesions (infective, tumour, infiltrative)
•! CPA lesions (acoustic neuroma, meningioma, neurofibroma)
•! Petrous temporal bone (Bell’s palsy, Ramsay Hunt, OM)
•! Parotid (tumour, sarcoidosis, surgery)
•! Mononeuritis multiplex
NB: Most common is Bell’s palsy

What are the causes of bilateral LMN VII nerve palsies?


•! After ruling out MG and myopathies
•! Bilateral CPA tumor as in NF type 2
•! Bilateral Bell’s palsy
•! Bilateral Parotid enlargement (Sarcoidosis – Uvoeoparotid fever or Heerfordt’s fever)
•! GBS, MND and leprosy, Lyme disease
•! Rare: Rosenthal Melkersson syndrome (triad of VII palsy with facial edema and plication of the tongue,
Mobius syndrome (congenital facial diplegia, oculoparalysis from III and VI and infantile nuclear
hypoplasia)

What is Bell’s phenemenon?


•! It refers to the upward movement of the right eyeball with incomplete closure of the right eyelid in an
attempt to close the right eye.

Why are the muscles of the upper face spared in a upper motor neurone lesion?
•! The upper facial muscles are preserved in an UMN lesion as there are bilateral cortical representations
of these muscles.

What is Bell’s palsy?


•! An idiopathic facial paralysis, believed to be due to viral-mediated cranial neuritis from HSV
•! Typically presents with abrupt onset of weakness with worsening the following day, associated with
facial or retroauricular pain, hyperacusis and excessive tearing

What is Ramsay Hunt syndrome?


•! Herpes zoster infection of the geniculate ganglion
•! Presents with vertigo, hearing loss, facial weakness, pain in the ear with vesicles seen on the external
auditory meatus and palate

What is facial synkinesis?


•! Attempt to move one group of facial muscles results in movement of another group
•! Occurs as a result of anomalous regeneration of the facial nerve
•! Egs if nerve fibres which innervate the facial muscles later innervate the lacrmial glands, then patient
shed tears on mastication (crocodile tears)

91
How would you investigate?
•! Targeted Ix according to history and physical examination
•! Blood Ix eg Lyme’s disease
•! Imaging

How would you manage (Bell’s palsy)?


•! Educate
•! Lubricating eye drops, eye patch, physiotherapy
•! PO Pred 1mg/kg/d for 7-10 days and PO acyclovir 400mg 5X/d for 7 days (within first 72hrs)
•! Regular follow up to look for resolution and exclude new developing signs suggestive of other conditions
•! Surgical (tarsorraphy) for chronic non-resolving cases

How would you educate or counsel patient with Bell’s palsy?


•! Common condition
•! Course
•! Improvement onset: 10 days to 2 months
•! Plateau: 6 weeks to 9 months
•! Residual signs
•! Synkinesis
•! Frequency: ~50%; May be reduced by corticosteroid treatment
•! May be treated with botulinum
•! Probably due to anomalous regeneration of nerve
•! Crocodile tears: 6%
•! Face weakness: 30%
•! Contracture: 20%
•! Blepharospasm: May occur years after paralysis
•! Prognosis better
•! Incomplete paralysis
•! Early improvement
•! Slow progression
•! Younger age
•! Normal salivary flow
•! Normal taste
•! Electrodiagnostic tests normal
•! Nerve excitability
•! Electrogustometry

92
Myasthenia Gravis

Examination
Statement
!! Examine face, CNs eyes
!! Approach to Ptosis (See Ptosis) and weakness

Sequence
!! Eyes
!! Ptosis with fatigability
!! Variable strabismus and diplopia that occurs after some time
!! Check for hyperthyroid and thyroid eye disease
!! Check for anaemia
!! Check for malar rash of SLE
!! Face
!! VII – show your teeth : snarls
!! Assessment of speech : Yeeee or count 1 to 20, nasal voice (bulbar palsy)
!! Masseter weakness but pterygoids normal\
!! Check neck for goitre and scars
!! ULs
!! Normal deep tendon reflexes (Eaton Lambert and Miller Fisher are reduced)
!! Normal sensation
!! Fatigability with weakness
!! RA and SLE features
!! Thymectomy scar and plasmapheresis line
!! Associations
!! Endo: Thyroid, DM, Pernicious anaemia
!! CTD: RA, SLE, Polymyositis
!! Requests
!! Drug Hx
!! D-penicillamine usage for RA or Wilson’s
!! Antibiotics and CVS drugs that can worsen MG
!! Temperature chart for fever – precipitate weakness

Presentation

Sir, this patient has myasthenia gravis as evidenced by presence of muscle weakness with fatigability. The
patient’s deep tendon reflexes and sensation are normal.

On examination of the face, there is presence of bilateral facial muscle weakness producing a mask-like
facies with ptosis. I noticed furrowing of her forehead musculature in an attempt to compensate for the ptosis.
There is presence of variable strabismus and diplopia after sustained gaze. I also noticed the presence of a
nasal voice as well as a reduction in volume of his speech when asked to count from 1 to 20. The patient
does not require a nasogastric tube.

I did notice mid line sternotomy scar which suggest previous thymectomy.

There is no goitre or features of hyperthyroidism. Patient also does not have symmetrical deforming
polyarthropathy to suggest RA and has no cutaneous features of SLE. I also did not notice any diabetic
dermopathy.

I would like to complete my examination by


!! Checking his negative inspiratory force
!! Temperature chart

93
!! Drug history

In summary, this patient has myasthenia gravis with mild weakness of which a thymectomy has been done
previously.

Questions

What is Myasthenia Gravis?


!! Autoimmune condition with antibodies targeting the post-synaptic Ach receptors of the neuromuscular
junction
!! Resulting in progressive muscle weakness with use of the muscle and recovery of strength after a period
of rest
!! Weakness experienced once number of receptors is 30% or less

How common is the thymus involved?


!! 75% of cases of which 15% are thymomas and 85% are thymic hyperplasia

What are the common presentations?


!! Age
!! 2 peaks
!! 20 to 30 years old with female predominance
!! >50 years old with male predominance
!! Ptosis, diplopia
!! Dysarthria, difficulty swallowing (isolated bulbar muscles involvement occurs in 20%)
!! Generalised weakness or reduced exercise tolerance
!! Respiratory failure in 1%
!! Tends to occur extraocular muscles first, then to facial to bulbar and to limbs and truncal

What can exacerbate MG or precipitate crisis?


!! Non compliance to medications
!! Infection
!! Emotions
!! Drugs
!! Antibiotics: aminoglycosides, tetracyclines, macrolides and fluoroquinolones
!! CVS : Beta blockers, Calcium channel blockers (verapamil)
!! Others : Chloroquine, quinidine, procainamide, Li, Mg, Prednisolone, quinine(in gin tonic drinks),
penicillamine

What is cholinergic crisis?


!! Can cause confusion between myasthenic crisis from cholinergic crisis
!! Results from excess of cholinesterase inhibitors such as neostigmine and physostigmine
!! Causes flaccid paralysis and SLUDGE (Miosis, salivation, lacrimation, urinary incontinence, diarrhea,
gastrointestinal hypermotility and emesis)

How would you investigate?


!! Blood Ix
!! AchR Ab
!! Positive in 80% with generalised MG
!! Positive in only 50% with ocular involvement only
!! also present in 90% of patients with penicillamine induced MG
!! Antistriated musce Ab
!! Anti Muscle specific kinase Ab (Anti MuSK Ab – positive in patients with AchR Ab –ve)
!! FBC to rule out infection

94
!! Imaging
!! CXR – thymus (anterior mediastinal mass), aspiration pneumonia
!! CT for thymus
!! Tensilon test
!! Dx and distinguishing from cholinergic crisis
!! Edrophonium (T1/2 10 mins)
!! Look for objective improvement in ptosis (require observer)
!! Cardiac monitoring for bradycardia and asystole (Rx with atropine)
!! 1 mg test dose and up to 10 mg
!! In cholinergic crisis, will get increased salivation etc
!! Note that in ALS, improvement in muscle weakness also occurs
!! Ice Pack test
!! Ice applied with glove to eyelids for 2 mins
!! Improvement in ptosis is dx (positive in 80%)
!! Electrodiagnostic studies
!! Repetitive nerve stimulation test – shows a decrease in the compound muscle action potential by 10%
in the 4th or 5th response to a train of nerve stimuli
!! Single fibre nerve electromyography – evidence of neuromuscular blockade with increased jitter

How do you grade the severity of the weakness?


!! Myasthenia Gravis Foundation of America
o! Grade 1 – affects the ocular muscles only
o! Grade 2 – mild weakness affecting muscles other than ocular muscles
!! 2A – Affects the limb and axial muscles
!! 2B - Affects the respiratory and bulbar muscles
o! Grade 3 – moderate weakness (3A AND 3B)
o! Grade 4 – Severe weakness (4A and 4B)
o! Grade 5 – Intubation required
!! Osserman’s grading
o! I: Ocular
o! II A: Mild generalised with slow progression
o! II B: Moderate generalised
o! III: Acute fulminant MG
o! IV: late severe MG (takes 2 yrs to progress from I to II)

How would you manage?


!! Emergencies in crisis (ABC)
!! Treat exacerbating factors
!! Stop medications that can exacerbate
!! Treat fever with antipyretics
!! Treat infections
!! Oral pyridostigmine, neostigmine
!! Steroids, azathioprine, cyclosporine
!! Plasmapheresis
!! IVIG
!! Thymectomy

What are the complications?


!! Myasthenic crisis
!! Severe exacerbation of MG
!! 10% require intubation
!! Treatment complications
!! Cholinergic crisis
!! Cx of medications

95
What is Eaton-Lambert syndrome?
!! Myasthenic disorder associated with malignancy such as small cell ca of lung
!! Affects the proximal (especially the pelvic girdle and thigh) and truncal musculature; bulbar muscles is
rarely involved
!! Improves with exercise
!! Presence of Abs to calcium channels

Approach to Examination of the Eyes


1.! Isolated nerve palsies
2.! Combined nerve palsies
•! Non conforming
•! Dysthyroid eye disease
•! Myasthenia gravis/Lambert Eaton Syndrome
•! Miller Fisher syndrome
•! Mononeuritis multiplex
•! Others – brainstem pathology (V1), Kearnes Sayre, botulism, Wernicke’s
•! Conforming
•! Superior orbital fissure syndrome
•! Cavernous sinus thrombosis
3.! Pupillary abnormalities (See Eye Short cases)
4.! Ptosis
5.! Gaze Abnormalities
•! Supranuclear gaze palsies – PSP, Parinaud’s
•! Gaze palsies – INO and One and half eye
•! Nerve Palsies (III and IV)
•! Wernicke’s
6.! Nystagmus (See Eye Short cases)
7.! Visual Acuity
8.! Visual Fields
9.! Fundoscopy
•! Papilloedema
•! Optic atrophy
•! Retinitis pigmentosa
•! DM
•! Hypertensive
•! Chorioretinitis
•! CRVO
•! CRAO

96
Gaze Palsies

INO
•! Examination (example right INO)
o! Cs
!! Abduction of the left eye with nystagmus a/w failure of adduction of the right eye on
leftward gaze
!! The right eye is able to independently adduction
!! Saccadic eye movement – horizontal saccade is abnormal with the right eye lagging
behind the left eye
o! Lesion is in the
!! Pons – convergence is intact
!! Midbrain – convergence is lost
o! Proceed with other CNs examination
!! Multiple sclerosis (RAPD)
!! Myasthenia gravis
o! Limbs
!! Multiple sclerosis – cerebellar signs
!! CVA – DM dermopathy, xanthelasma, AF
o! Request for fundoscopy (optic atrophy)
•! Presentation
o! Sir this patient has a right INO as evidenced by Cs
o! The lesion is in the midbrain (anterior INO) or pons (posterior INO)
o! Evidence for MG
o! Evidence of MS
o! Evidence for CVA
•! Questions
o! What causes a right INO?
!! Lesion in the right medial longitudinal fasciculus that affects connects the ipsilateral third
nerve innervation to the right medial rectus to the left gaze center (parapontine reticular
formation ie PPRF)
o! What are the causes of INO?
!! Multiple sclerosis
!! Brainstem infarction
!! Pontine glioma
!! Infections
•! Lyme’s disease
•! Syphilis
•! Viral
!! Drug intoxication (phenothiazines, TCAs, phenytoin, CMZ)
!! Trauma
o! How would you Investigate?
!! As above etiologies (MG, MRI, FPG, lipids, lyme titre, VDRL, drug)
o! How would you manage
!! Mx of Multiple sclerosis
!! Mx of infarction and risk factors
•! Typically resolves with time

WEBINO (Walled-eye Bilateral INO)


•! Bilateral INO with exotropia and failure of convergence
•! Lesions in the pons and midbrain
•! Due to multiple sclerosis, vascular, gliomas and Wernicke’s

97
Fisher’s one-and-a-half syndrome
•! INO and ipsilateral gaze palsy
•! Due to lesion in the MLF and adjacent gaze center

Conjugate upward vertical gaze palsy


•! Midbrain lesion
•! MS, vascular, tumor

Conjugate downward vertical gaze palsy


•! Midbrain or
•! Foramen magnum
o! Arnold-Chiari, Dandy Walker
o! Acquired lesions (tumor, vascular, demyelination, abscess)

Supranuclear gaze palsy


•! Progressive suprnuclear gaze palsies (see Parkinson’s disease)
o! Loss of saccadic(frontal lobe) and pursuit movements (Occipital lobe)
o! Loss of downward gaze, then upward gaze then horizontal gaze
o! Can be overcome by Doll’s reflex
•! Parinaud’s syndrome
o! Loss of vertical gaze, nystagmus on convergence, PseudoArgyll-Robertson pupils
o! Causes – Ms, vascular, pinealoma

98
Approach to Unilateral Ptosis
•! Rule out pseudoptosis
o! Life up any droopy eyelids
•! Muscle
o! Dystrophia myotonica (see Dystrophia Myotonica)
•! Neuromuscular
o! Myasthenia gravis (See Myasthenia Gravis)
•! Nerve
o! III nerve palsy (see III nerve palsy)
o! Horner’s syndrome

Unilateral Horner’s syndrome

Examination
•! Examine the other cranial nerves
o! Cavernous sinus syndrome
o! Superior orbital syndrome
o! Lateral medullary syndrome (see CN syndrome)
o! Syringobulbia (V, VII, IX –XII)
o! Multiple sclerosis (INO, Cerebellar, RAPD)
•! Neck
o! Scars – trauma, surgery
o! Neoplasia
o! Carotid aneurysm
o! Cervical rib
•! Upper limbs (examine in this sequence)
o! Pronator drift then cerebellar signs (Lateral medullary syndrome)
o! Wasting of ipsilateral small muscles of hands (T1)
o! Clubbing
o! sensory loss T1
o! Dissociated sensory loss (Syringomyelia)
o! Contralateral loss to pain and temperature (Lateral medullary syndrome)
o! Axilla – trauma to brachial plexus
•! Chest
o! Pancoast tumor
•! Inspection, dullness, auscultation
•! Trachea deviation
•! Ask for Loss of sweating and level

Presentation

•! Sir, this patient has a right sided isolated Horner’s syndrome as evidenced by
o! Partial ptosis of the right eyelid
o! Miosis of the right pupil with an intact light reflex
o! Enophthalmos
o! Elevation of the lower eyelid
o! (Anhydrosis if you were allowed to ask the patient)
•! There was no associated CN abnormalities in particular
o! Cavernous sinus syndrome
o! Superior orbital syndrome
o! Lateral medullary syndrome (see CN syndrome)
o! Syringobulbia (V, VII, IX –XII)
o! Multiple sclerosis (INO, Cerebellar, RAPD)
•! Examination of the neck
99
•! Upper limbs
o! Present axilla findings (stat the other findings with CN syndromes)
•! Chest for Pancoast lesion
•! DM dermopathy, xanthelasma
•! Request to ask patient for loss of sweating.
•! Summary

Questions

What are the features of Horner’s syndrome?


•! Partial ptosis – paralysis of the upper tarsal muscle (Muller’s muscle)
•! Miosis – paralysis of pupil dilator
•! Enophthalmos – paralysis of muscle of Muller
•! Slight elevation of the lower eyelid – paralysis of the lower tarsal muscle
•! Loss of sweating

How do you delineate the site of lesion clinically?


•! Loss of sweating
o! Central lesion – loss in the head, upper trunk and arm (1St order)
o! Neck
!! Proximal to the superior cervical ganglion – loss in face (2nd order)
!! Distal to superior crvical ganglion – no loss (3rd Order)
•! Adrenaline 1:1000 in both eyes (denervation hypersensitivity)
o! Above the superior cervical ganglion (peripheral) = dilates the affected eye
o! Below/Proximal to superior Cx ganglion or normal eye = no effect
•! Cocaine 4%
o! Dilates normal eyes
o! No effect on the affected side if above/distal to superior cervical ganglion

What are the causes of Horner’s syndrome?


•! Hypothalamus or brainstem
o! Stroke
o! Pontine glioma
o! Coning of temporal lobe
•! Cervical cord (C8-T2 : intermediolateral column)
o! Syringomyelia
o! Multiple sclerosis
o! Tumor
•! Superior Mediastinum (2nd order nerves exits the spinal cord and synapses at the superior cervical
ganglion)
o! Pancoast lesion (SCC of lung)
o! Trauma to brachial plexus
•! Neck (carotid sympathetic plexus and superior cervical ganglion)
o! Neoplasia
o! Trauma
o! Surgery (cervical sympathectomy)
o! Carotid aneurysm
o! Carotid Dissection (triad of pain, ipsilateral Horner’s and cerebral or retinal ischaemia)
•! Idiopathic
•! Congenital – heterochromia of the iris (grey-blue on the affected side)
•! Migraine – causes intermittent Horner’s syndrome

100
Approach to Bilateral Ptosis

•! Muscular (usually no wrinkling of the forehead)


o! Dystrophia myotonica (see Dystrophia myotonica)
o! Ocular myopathy
o! Oculopharyngeal dystrophy
o! Chronic progressive external ophthalmoplegia (mitochondrial/Kearnes Sayrre)
•! Neuromuscular
o! Myasthenia gravis (See Myasthenia gravis)
•! Nerve
o! Bilateral 3rd (rare)
o! Bilateral Horner’s (Syringomyelia)
o! Tabes dorsalis
o! Miller Fisher syndrome

Examination

•! General screen for dystrophia myotonica or fascioscapular dystrophy


•! Screen for myasthenia gravis
•! Check CNs
o! III, Horner’s
o! Argyll Robertson pupils (Tabes)
o! Ophthalmoplegia (Kearnes Sayrre)
o! Bulbar palsy (Syringomyelia)
•! Neck
•! Upper limbs
o! Ataxia (Kearnes sayrre, Miller Fisher)
o! Syringomyelia
!! Flaccid and wasted ULs
!! Dissociated sensory loss
!! Spastic paraparesis
o! Areflexia (Miller Fisher)
•! Complete examination by fundoscopy for retinitis pigmentosa (CPEO)

Presentation

•! This patient has bilateral ptosis


•! No evidence of DM, MG
•! No evidence of
o! Horner’s, Syringomyelia
o! III
o! Argyll Robertson
o! Miller Fisher
•! The possible aetiologies include ocular myopathy, oculopharyngeal myopathy, CPEO and congenital
ptosis.

101
Upper Limbs Overview

•! Motor
o! Unilateral vs bilateral
o! Proximal vs distal vs entire UL
o! Myopathy, Neuromuscular and Neuropathy
•! Sensory (See Lower Limbs)
o! Peripheral neuropathy
o! Mononeuropathy/mononeuritis multiplex
o! Rediculopathy
•! Movement disorders

Motor
Distally
•! Wasted hands
o! Myopathies
o! Nerve (Think of levels)
!! Mononeuropathy
•! Ulnar
•! Median
•! Radial
•! Combination of above three
!! Peripheral neuropathy
!! Brachial plexus
!! Roots
!! Anterior Horn
!! Spinal cord
•! Claw hands
o! Partial claw – ulna claw hand (r/o dupytren’s contracture)
o! Total claw
!! Neurological – combined ulna and median, leprosy, brachial, polio, syringomyelia
!! Non-neurological – RA, ischaemic contracture, Scleroderma

Proximally
•! Proximal myopathy, Dystrophia myotonica
•! Myasthenia gravis
•! Syringomyelia, Radiculopathy, upper brachial plexus

Entire Upper Limb


•! Bilateral
o! Cervical myelopathy
o! Syringomyelia
o! MND
o! Spinal Muscular Atrophy
o! Polyradiculopathy
o! Bilateral Brachial Plexus (trauma or bilateral cx Rib)
•! Unilateral
o! Polio
o! Brachial neuritis
o! Polyradiculopathy
o! Hansen’s disease
o! Hemiparesis
o! Infantile hemiplegia
o! Brown-Sequard
102
Movement disorders
•! Parkinsonism
•! Chorea and movement disorders
o! Think: 3,3,3
!! Peripheral : Parkinsonsim, Rh heart, SLE
!! Face: Wilson’s, Hyperthyroidism, Polycythaemia
!! Request: CVS, Drug, AMT for Huntington
•! Hemiballismus
•! Cerebellar
o! Think
!! Unilateral: 4
•! Stroke and CNs and risk factors
•! SOL and CNs
•! Parkinsonism
•! MS
!! Bilateral (2/2/2/2/2/2)
•! ULs
o! Alcohol
o! Parkinsonism
o! (NF)
•! Eyes
o! Wilson’s, MS
•! Mouth
o! Hypothyroidism
o! Phenytoin
o! (Alcohol)
•! LLs
o! FA
o! (Ataxia telangiectasia)
•! General
o! Wasting (paraneoplastic)
o! Bilateral strokes
•! Requests
o! Drugs
o! Infection

103
Radial Nerve Palsy

Examination

•! Suspect this on an apparently normal looking pair of ULs


•! Proceed to examine the ULs as per normal, concentrating on median and ulnar nerve as well as
brachial plexopathy
•! Once radial nerve palsy detected, proceed to look for level
o! Demonstrate weakness of extension at the MCPJ
o! Preservation of IPJ extension (lumbricals and interossei muscles)
o! Weakness of wrist extension
o! (don’t forget to extend wrist before testing grip strength)
o! (don’t forget to test finger abduction and adduction with hands flat on a surface)
o! Test brachioradialis
o! Test triceps muscles, triceps jerk
o! Thumb abduction and Oschner’s clasping test for median screen
o! Finger abduction and Froment’s sign for ulnar screen
o! Look for reduced sensation in the first dorsal interosseous or anatomical snuffbox
o! Inspect the forearm, elbow, humerus and shoulder
o! Check the gums for lead poisoning – blue-black line on the gingival margin
o! Test Function

Presentation

Sir, this patient has got an isolated right radial nerve palsy at the level of the upper third of the humerus
or above.
I say this because of weakness of extension of the fingers at the MCPJ and at the wrist associated
with weakness of the brachioradialis muscle, triceps muscles with weakness of extension at the elbow. In
addition, there is also numbness of the first dorsal interosseous space. There is no evidence of concomitant
ulnar or median nerve palsies.
I did not detect any scars or deformities over the humerus or the axilla. (Mentioned other areas if the
level is lower) There is also no clinical evidence of lead poisoning such as a blue-black line on the gingival
margin.
Possible causes include compression of the right radial nerve such as crutch palsy at the axilla or
Saturday night palsy at the humerus.
I also note that there is presence of a splint for his wrist and finger drop. He is able to perform coarse
and fine motor function.

Questions

•! What is the course of the radial nerve and its branches?


o! C5, 6,7,8, T1 and emerges from the posterior cord of the brachial plexus
o! Leaves the axilla and enters the arm between the long head and medial heads of the triceps
and supplies the triceps
o! Spiral groove on the back of the humerus between the lateral and medial heads of the triceps
o! Lower third of the humerus, it pierces the intermuscular septum to enter the anterior
compartment of the arm where it supplies the brachioradialis
o! It gives off a branch supplying the extensor carpi radialis longus
o! At the elbow, ie lateral epicondyle of the humerus, it gives off the posterior interosseous nerve
which supplies all the extensors of the forearms including the abductor pollicis longus and
supinator except the extensor carpi radialis longus
o! The radial nerve continues as the superficial radial nerve which provides sensory innervation
of the posterior aspects of the radial 3 ½ digits.

104
•! What are the various levels of lesions and what are the correlating clinical features?
o! Axilla eg crutch palsy – All gone including triceps and triceps reflex
o! Humerus
!! Upper third – all is lost
!! Middle third
•! triceps and triceps reflex preserved and brachioradialis and below is lost
•! Saturday night palsy
!! Lower third – triceps and brachioradialis is preserved
o! Elbow
!! Like lower third
!! Only the PIN involved
•! Extensors of the fingers at the MCPJ affected only
•! Wrist drop is not a feature as the extensor carpi radialis longus is intact and
this alone can effect wrist extension
o! Forearm
!! PIN involvement
!! Superfical radial nerve palsy; aka Watenberg syndrome which is an entrapment
syndrome where there is pain and numbness over its distribution of the first web space
dorsally only because of overlap

•! What are the causes?


o! Trauma form accident or surgery
o! Compression or entrapment
o! Part of a mononeuritis multiplex
o! Lead poisoning
o! (for PIN, finger drop can be secondary to synovitis from RA)

•! How would you investigate?


o! Detailed history for the cause
o! X-ray – for fracture, healing callus or tumor
o! EMG and NCT to locate the level of injury and to monitor recovery progress

•! How would you manage?


o! Education and counselling
o! OT and PT with a wrist splint and cock-up splint for finger drops
o! Surgical

•! What’s the prognosis?


o! Neuropraxia with no disruption to the sheath or the axon
!! Recovery complete and rapid (weeks)
o! Axonotmesis with disruption of the axon but an intact Schwann sheath
!! Recovery complete but slower (1mm/day)
o! Neuronotmesis
!! Recovery is incomplete

105
Median Nerve Palsy

Examination

•! Upon suspecting median nerve palsy, rule out ulnar and brachial neuritis
•! Median nerve palsy
•! Motor
•! Wasted thenar eminence
•! Thumb is externally rotated into the plane of the thumb rather than perpendicular
•! Pen-touch test (for abductor pollicis brevis)

•! Oschner clasping test (flexor digitorum superficialis)


•! Flexion of the terminal digit of the thumb (flexor pollicis longus)
•! Flexion of the terminal digit of the index finger (flexor digitorum profundus)

•! Sensory
•! Test for reduced sensation in the lateral 31/2 fingers as well as thenar eminence.
•! Exclude ulnar and radial nerve palsy
•! Aetiology
•! Tinel’s sign
•! Look for RA hands
•! Look at the wrist and forearm, elbow, arm and axilla for scars.
•! Test for function
•! Rule out Myxedema and acromegaly

Presentation

Sir, this patient has an isolated unilateral right median nerve palsy with wasting of the right thenar
eminence associated with an externally rotated thumb. There is weakness of abduction of the thumb as
demonstrated by the pen touch test associated with reduced sensation to pinprick in the right lateral 3 1/2
fingers. Oschner’s clasping test is negative and flexion of the terminal phalanx of the thumb and index fingers
are preserved, indicating that the level of the lesion is at the wrist.
There is no ulna or radial nerve palsies.
In terms of aetiology, there is also no evidence of RA of the hands and patient does not have features
of hypothyroidism or acromegaly. Tinel’s sign is negative and there are no scars noted on the right upper
limb.
Both fine and coarse motor functions are intact.
In summary, this patient has a right median nerve palsy at the level of the wrist. Possible aetiologies
includes surgical causes such as compression, trauma or surgery or medical causes such as mononeuritis
multiples, infection, inflammatory and ischaemic causes.

Questions

What is the course and branches of the median nerve?


•! It supplies all the muscles of the forearm except the flexor carpi ulnaris and the ulna half of the
flexor digitorum profundus and LOAF (lateral 2 lumbricals, opponens pollicis, abductor pollicis
brevis and flexor pollicis brevis)
•! Formed by lateral(C5-7) and medial(C8,T1) cords of the brachial plexus
•! Enters the arm closely related to the brachial artery with no branches above the elbow
•! Enters the forearm lateral to the brachialis tendon and in between the pronator teres.
•! Gives off the anterior interosseous nerve
•! Above the wrist, gives off the palmar cutaneous branch
•! Enters the carpal tunnel and supplies LOAF and sensory branch to the lateral 3 ½ fingers.
106
•! Branches
•! Forearm – flexor carpi radialis, flexor digitorum superficialis (flexion of fingers at the PIPJ),
pronator teres
•! AIN – Flexor pollicis longus (flexion of the DIPJ thumb), flexor digitorum profundus of the
lateral 2 fingers (flexion of at the DIPJ), pronator quadratus
•! Palmar cutaneous (to the thenar eminence)
•! Terminal motor (LOAF) (NB for F for flexion at the MCPJ thumb)

What are the various levels of lesions and the clinical correlation?
•! Wrist
•! Wasting of thenar, ext rotated thumb, pen touch test positive; sensory loss of the lateral 3 ½
fingers
•! Cubital fossa
•! Above plus
•! Oschner clasping test positive and failure of flexing the terminal digits of the thumb and index
finger
•! Arm and axilla (same as cubital fossa)
•! (For forearm, depends where the lesion is eg AIN syndrome will affect flexor digitorum
profundus and flexor pollicis longus only)

What are the causes?


•! Trauma
•! Surgical
•! Compression
•! Mononeuritis multiplex
•! Infection – Leprosy
•! Inflammatory – CIDP
•! Ischaemic - Vasculitis

•! Causes of Carpal tunnel syndrome


•! Idiopathic
•! Pregnancy, OCPs
•! Endocrine – Hypothyroidism, Acromegaly
•! Hands – RA, gout, TB tenosynovitis, OA of carpus
•! Amyloidosis, CRF, sarcoidosis

What are the tests to demonstrate a median nerve palsy?


•! Tinel’s sign (percussion)
•! Phalen’s test (flexion at the wrist for 60 s)
•! Hyperextension of the wrist (for 60 s)
•! Tourniquet test (sphygmomanometer for more than 2 mins above systolic)
•! Luthy’s sign – Skinfold does not close tightly around a bottle or cup; secondary to thumb
abduction paresis
•! Durkan’s test – apply direct pressure over the carpal tunnel

What are the other areas of nerve compression?


•! Median nerve (CTS)
•! Ulna nerve (elbow tunnel)
•! Radial nerve (spiral or humeral groove)
•! Meralgia paraesthetica (lateral cut nerve of the thigh at the ing lig)
•! Common peroneal nerve (head of the fibula)
•! Posterior tibial nerve (Tarsal tunnel syndrome)
107
•! Plantar nerves of the 3rd/4th toes (Morton’s metatarsalgia)

How would you investigate?


o! Blood Ix
o! Imaging – X-rays
o! Nerve conduction test demonstrating slow sensory conduction across the transverse carpal
ligament.

How would you manage?


Education
OT and wrist splint
Medications – treatment of underlying disease, withdrawing OCPs, IA steroid
Surgical decompression

What is the prognosis?


o! Neuropraxia with no disruption to the sheath or the axon
!! Recovery complete and rapid (weeks)
o! Axonotmesis with disruption of the axon but an intact Schwann sheath
!! Recovery complete but slower (1mm/day)
o! Neuronotmesis
!! Recovery is incomplete

108
Ulnar Nerve Palsy

Examination
•! Rule out median, radial and brachial neuritis
•! Inspecting
•! Wasting of the muscles of the hands, hypothenar eminence and partial clawing of the 4th and 5th
fingers, sparing of the thenar eminence, ulnar paradox
•! Proceed to tests for finger abduction and Froment’s sign (weakness of the adduction of the thumb)
•! Test finger flexion of the 5th finger for flexor digitorum profundus involvement; test for wrist flexion
at the ulna side and look for the tendon of the flexor carpi ulnaris
•! Rule out median nerve (thenar eminence and ext rot thumb, pen touch test and Oschner clasping test)
and radial nerve
•! Sensory testing in the medial 1 ½ fingers; test T1 sensory loss
•! Examine the wrist and elbows (feel for thickened nerve, wide carrying angle))
•! Function
•! Thickened nerve (cf with Pb for radial and Acromeg etc for median)

Presentation

Sir, this patient has got a isolated left ulnar palsy as evidenced by a left ulnar claw hand with wasting
of the small muscles of the hands with dorsal guttering as well as wasting of the hypothenar eminence. There
is sparing of the thenar eminence.
There is weakness of finger abduction and Froment’s sign is positive. There is preservation of the
flexion of the DIPJ of the 4th and 5th fingers; when the hand is flexed to the ulna side against resistance, the
tendon of the flexor carpi ulnaris is palpable. This is associated with reduced sensation to pinprick in the
medial 1/1/2 fingers. There are no associated median or radial nerve palsies and T1 involvement.
In terms of aetiology, there is a scar at the wrist associated with a marked ulnar claw hand,
demonstrating the ulna paradox. I did not find any signs to suggest leprosy such as thickened nerves,
hypopigmentation patches or finger resorption.
Both coarse and fine motor function of the hand is preserved.
In summary, this patient has a left ulna claw hand due to a traumatic injury to the left wrist.

Questions

What is the anatomical course of the ulnar nerve?


•! It provides motor to all muscles of the hands except the LOAF; flexor carpi ulnaris and flexor digitorum
profundus to the 4th and 5th fingers.
•! Sensory to the ulna 1 ½ fingers
•! Begins from the medial cord of the brachial plexus (C8 and T1)
•! No branches in the arm
•! Enters the forearm via the cubital tunnel (medial epicondyle and the olecranon process) and motor supply
to the flexor carpi ulnaris and ulna half of the flexor digitorum profundus
•! It gives off a sensory branch just above the wrist and enters Guyon’s canal and supplies the sensory
medial 1½ fingers and hypothenar as well as motor to all intrinsic muscles of the hands except LOAF.

What is the level of lesions and its clinical correlation?


•! Wrist – Hypothenar eminence wasting, Froment’s positive, weakness of finger abduction, pronounced
claw and loss of sensation
•! Elbow – less pronounced claw and loss of terminal flexion of the DIPJ and loss of flexor carpi ulnaris
tendon on ulna flexion of the wrist

How do you differentiate ulnar nerve palsy vs a T1 lesion?


Motor – wasting of the thenar eminence in addition for T1
109
Sensory – loss in T1 dermatomal distribution

What is the ulna claw hand?


It refers to the hyperextension of the 4th and 5th MCPJ associated with flexion of the IPJs of the 4th
and 5th fingers as a result of ulnar nerve palsy.
It is due to the unopposed long extensors of the 4th and 5th fingers in contrast to the IF and MF which
are counteracted by the lumbricals which are served by the median nerve.

What is the ulnar paradox?


It means that the ulnar claw deformity is more pronounced for lesions distally e.g. at the wrist as
compared to a more proximal lesion e.g. at the elbow.
This is because a more proximal lesion at the elbow also causes weakness of the ulnar half of the
flexor digitorum profundus, resulting in less flexion of the IPJs of the 4th and 5th fingers.

What is Froment’s sign?


Patient is asked to grasp a piece of paper between the thumbs and the lateral aspect of the index finger.
The affected thumb will flex as the adductor pollicis muscles are weak. (Patient is trying to compensate by
using the flexor pollicis longus supplied by median nerve)

What are the causes of an ulnar nerve palsy?


•! Compression or entrapment (Cubital tunnel at the elbow and Guyon’s canal at the wrist)
•! Trauma (Fractures or dislocation – cubitus valgus leads to tardive ulnar nerve palsy)
•! Surgical
•! Mononeuritis multiplex
•! Infection – leprosy
•! Ischaemia – Vasculitis
•! Inflammatory - CIDP

How would you investigate?


•! Blood Ix to rule out DM if no obvious cause
•! X-rays of the elbow and wrist (both must be done to rule out double crush syndrome) (KIV C-spine and
CXR)
•! EMG(axonal degeneration for chronic) and NCT(motor and sensory conduction velocities useful for
recent entrapment as well as chronic) to locate level and monitor

How would you manage?


•! Education and avoidance of resting on elbow
•! OT, PT
•! Medical – NSAIDs and Vit B6
•! Surgical decompression with anterior transposition of the nerve

NB: LOAF – lateral 2 lumbricals, opponens pollicis, abductor pollicis brevis and flexor pollicis brevis

110
Wasted Hands
Unilateral vs Bilateral (think of levels!)

Unilateral
•! Think of (no myopathy, got brachial plexus)
•! Peripheral nerve (median, ulnar or combined)
•! Mononeuropathy vs peripheral neuropathy (asymmetric involvement)
•! Brachial plexus (trauma, tumor, radiation, Cx rib)
•! C8-T1 root lesions (Cx spondylosis)
•! Anterior Horn Cell (Poliomyelitis)
•! Cervical cord
•! Proceed as:
•! Long case – as per protocol, check also neck and chest
•! Short case
•! On inspection, unilateral wasted hands noted
•! Neurological hand screen
•! Examine for ulnar and median nerve palsies.
•! Check for sensory for nerve vs root (peripheral nerve vs brachial plexus) and no loss (ie anterior
horn cell)
•! Note sensory for ulnar, median and radial
•! Note sensory of peripheral neuropathy
•! Note dermatomal sensory
•! Feel for thickened nerves, look for hypoaesthetic macules, fasciculations
•! Look for scars in the axilla and neck (neck pain, tenderness), Cx rib
•! Check function
•! Requests
•! Palpate for cervical rib and features of Pancoast’s tumor (dullness to percussion, Horner’s
syndrome, hoarseness voice)
•! Check for winging of scapula (for brachial plexus involvement)
•! If brachial plexus
•! Upper vs lower (wasting of muscles of hands) vs complete
•! Surgical(Cx rib, Pancoast) vs medical cause(brachial neuritis)
•! Test for proximal involvement
•! Serratus anterior (winging of scapula on pushing against wall) ie C5,6,7
•! Supraspinatus (abduction of UL from hands by your side position) C5
•! Infraspinatus (elbow flexed and push backwards) C5
•! Rhomboids (hand on hip and push backwards) C4,5,6
•! Reflexes (inverted supinator jerk)

Bilateral
•! Think of
•! Rule out the obvious (hand screen)
•! RA, gouty hands
•! Dystrophia myotonica
•! Levels (got myopathy, maybe brachial plexus if bilateral Cx ribs)
•! Distal myopathy (reflexes normal; rare), dystrophia myotonica
•! Peripheral nerve lesions
•! Combined CTS (see median nerve palsy)
•! Combined ulnar and median nerve
•! Leprosy (resorption, hypoaesthetic macule and thickened nerve)
•! HMSN (look at the feet for pes cavus deformities, thickened nerves)
•! Peripheral motor neuropathy

111
•! (Not likely to be brachial plexus unless bilateral Cx ribs)
•! Nerve roots
•! Cervical spondylosis (inverted supinator jerk, increased jerks for high cervical cord
lesions)
•! Anterior Horn cell (no sensory loss)
•! MND (fasciculations)
•! Poliomyelitis
•! SMA
•! Spinal cord lesions
•! Intramedullary (Syringomyelia – dissociated sensory loss)
•! Extramedullary
•! Request
•! LL – spastic paraparesis ( if suspect Cx cord, MND)
•! Lower cranial nerve (bulbar palsy – if suspect MND or syringomyelia)
•! Proceed as
•! Long case
•! Proceed as per normal
•! Examine or request to examine the neck (pain tenderness and pain on neck movements), chest,
CNs and LLs accordingly
•! Short case
•! Neurological hand screen
•! Median and ulnar nerve testing, and wrist drop( because this is also weak in C8 root lesions)
•! Sensory – peripheral nerve vs neuropathy vs root
•! Check the elbows for thickened nerves
•! Look for fasciculations (peripheral nerve, neuropathy, MND), hypoaesthetic macules
•! Inspect the neck
•! Quick glance at the face (NG tube – bulbar palsy, LLs – HMSN)
•! Check function
•! Request for reflexes, percussion myotonia if deemed appropriate (if suspect Cx cord lesion or
dystrophia myotonica)

Questions

What are the levels and causes?


•! Disuse atrophy (RA hands)
•! Myopathy (distal myopathies or dystrophia myotonica – usually forearms more affected)
•! Peripheral neuropathy - motor (see causes in Neurology segment)
•! Mononeuropathy
•! Surgical, trauma or compression
•! Mononeuritis multiplex, infection, inflammatory and ischaemic
•! Brachial Plexus
•! Surgical, trauma compression (Pancoast’s, Cx rib)
•! Brachial neuritis
•! Nerve root (Disc prolapse)
•! Anterior Horn cell
•! MND, poliomyelitis, SMA
•! Spinal cord
•! Intramedullary
•! Extramedullary

How would you Ix?


Blood Ix according to causes as above
112
Imaging – X-rays, CT or MRI of spine
NCT/EMG

What are the causes of a claw hand?


•! Partial claw
•! Ulnar nerve palsy (See Ulnar nerve)
•! True Claw
•! Non-neurological
•! RA
•! Severe Volkmann’s ischaemic contracture
•! Neurological (5)
•! Combined median and ulnar nerve
•! Leprosy (reflexes present. Pain loss, thickened nerves)
•! Lower brachial plexus ( C7-T1, selective loss of reflexes, pain loss)
•! Poliomyelitis (reflexes selective, pain intact)
•! Syringomyelia (reflexes absent, pain loss)

Peripheral Neuropathy
(Think: Sensory, motor, or mixed. Are nerves palpable nerves?)

Presentation 1

•! Sir, this patient has predominantly sensory peripheral neuropathy as evidenced by


o! Loss of sensation to pinprick and light touch and
o! Impairment of vibration and joint position sense
o! In a stocking distribution
•! The motor system is intact; I did not notice any
o! Wasting or fasciculations of the lower limb muscles
o! Tone and reflexes are normal with downgoing plantars
o! Power is normal
OR
•! Sir, this patient has mixed motor and sensory peripheral neuropathy as evidenced by
o! Loss of sensation to pinprick and light touch and
o! Impairment of vibration and joint position sense
o! In a stocking distribution
•! Associated with
o! Wasting and fasciculations of the lower limb muscles
o! Reduced tone and reflexes with downgoing plantars
o! Diminished power of 4 in the lower limb muscles especially affecting plantarflexion,
dorsiflexion and flexion and extension of the knees bilaterally
AND
•! There is presence of
o! Loss of hair on the lower half of the legs bilaterally
o! No charcot joints
•! The most likely underlying aetiology is diabetes mellitus as I noticed
o! Presence of diabetic dermopathy
o! I screened for other possible causes:
!! No thickened nerves or hypopigmentation patch (leprosy)
!! Parotidomegaly, dupytren (chronic ethanol ingestion)
!! Not sallow (uremia)
!! Not pale (B12 deficiency)
113
!! Not cachexic and no clubbing of toes (paraneoplastic)
!! No symmetrical deforming polyarthropathy (RA)
!! No clinical features of acromegaly, hypothyroidism
•! I would like to complete the examination
o! Gait (if not done) to look for high steppage gait (sensory ataxia)
o! Upper limbs for distal sensory impairment although I noticed that there is no obvious wasting of
the hands
o! Urine dipstick for glycosuria (DM)
o! Ask history
!! Drug history – INH, nitrofurantoin, phenytoin, chloroquine, penicillamine, vincristine,
cyclosporine A
!! Chronic ethanol ingestion

Presentation 2

•! Sir this patient has predominantly motor neuropathy as evidenced by


o! Wasting and fasciculations of the lower limb muscles
o! Reduced tone and reflexes
o! With diminished power of 4 affecting knee flexion and extension as well as plantar and
dorsiflexion
•! Sensation is intact with normal pinprick sensation, vibration sense and propioception.
•! The most likely aetiology in this patient is
o! diabetes mellitus as I noticed presence of diabetic dermopathy in the lower limbs
o! Other possible aetiologies for a predominantly motor peripheral neuropathy
!! Drugs – cyclosporine A, Gold, penicillamine
!! Pb, Hg
!! Metabolic – DM and AIP
!! Infectious/Inflammatory – HIV, GBS, Amyloid, sarcoid
!! PAN
!! HSMN type 1

Questions

What are the causes of peripheral neuropathy (mixed, sensory and motor)?
•! DAMIT BICH
o! Drugs
!! INH, nitrofurantoin, chloroquine
!! Penicillamine, gold, cyclosporin A, phenytoin
!! vincristine, cisplatinum
o! Alcohol, Arsenic(Mees, raindrop pigmentation), Pb(wrists and Pb lines in gums), Hg
o! Metabolic – DM, Uremia, AIP
o! Infectious - Leprosy, HIV, botulism, diphtheria
o! Inflammatory – GBS (look for facial diplegia), CIDP
o! Tumor – paraproteinemia, paraneoplastic (Ca Lung), Hodgkin’s
o! B12, B6 and B1
o! Infiltrative – Amyloid (look for thickened nerves and autonomic), sarcoid
o! Immunological – PAN, SLE, RA
o! Congenital – HMSN, Refsum’s disease, porphyria
o! Cryptogenic
o! Hormonal – Acromegaly, hypothyroidism, hyperthyroidism
o! POEMS (Polyneuropathy, Organomegaly, Endocrinoapthy, Monoclonal gammopathy, Skin
changes – a/w osteosclerotic myeloma)
114
(NB: DM can be sensory, motor or mixed)

What are the causes of a painful peripheral neuropathy? (DAB, CAP)


•! DM, Alcohol, B12 deficiency
•! Carcinoma, porphyria, Arsenic

What are the causes of thickened peripheral nerves?


•! Median nerve (wrist), ulna nerve (elbow), common peroneal nerve (head of fibula), Greater auricular
nerve (neck)
•! CHAOS
o! CIDP
o! HMSN
o! Acromegaly, Amyloid
o! Others
!! LS DNR – Leprosy, sarcoid, DM, Dejerine Sotta disease (hypertrophic peripheral
neuropathy), NF, Refsum’s disease (retinitis pigmentosa, optic atrophy, cerebellar and
deafness, cardiomyopathy ad ichthyosis)

What are the causes of mononeuritis multiplex (separate involvement of more than one peripheral or
cranial nerve by the same disease)?
•! Endocrine
o! DM, Hypertension, Acromegaly
•! AI
o! RA, SLE, PAN, Sjogren, Churg-Strauss, Wegener’s
•! Infection
o! Leprosy, Lyme, HIV
•! Infiltrative
o! Amyloid, sarcoid
•! Carcinomatosis

What are the types of neuropathy in DM?


•! Symmetrical sensory neuropathy (glove and stocking)
•! Predominantly motor, asymmetrical (diabetic amyotrophy)
•! Mixed motor and sensory peripheral neuropathy
•! Mononeuropathy
•! Mononeuritis multiplex
•! Autonomic neuropathy

What are the neurological complications of alcohol?


•! Wernicke’s (confusion, ophthalmoplegia, cerebellar, neuropathy)
•! Korsakoff’s psychosis (recent memory loss and confabulation)
•! Cerebellar degeneration
•! Central pontine myelinosis
•! Epilepsy
•! Myopathy and rhabdomyolysis
•! Peripheral neuropathy

115
Syringomyelia

Examination
•! Proceed as per normal for the upper limbs
•! Once dx is made, request
o! Examine the neck
!! Scars of previous Sx
!! Scoliosis
o! The cranial nerves
!! Horner’s syndrome
!! Ataxia and nystagmus
!! Bulbar palsy (syringobulbia)
!! Loss of temperature and pain sensation from the outer part of the face progressing towards
the center
o! The lower limbs
!! Spastic paraparesis

Presentation

Sir, this patient has got syringomyelia as evidenced by


•! LMN pattern of weakness of both ULs
o! Wasting and weakness of the small muscles of the hands and forearms
o! Reduced tone and reflexes
•! There is dissociated sensory loss with
o! Loss of sensation to pinprick in the ULs and upper chest
o! With intact sensation to vibration and proprioception
•! I also noticed presence of
o! Scars and old burn marks on his fingers
o! But I did not detect any Charcot’s joints of the ULs
o! La main succulente – ugly, cold, puffy, cyanosed hands with stumpy fingers and podgy soft palms
•! Examination of the face
o! There was no evidence of bulbar palsy
!! Palatal movements were normal, and CN XI and XII were intact
o! There was also no Horner’s syndrome
o! No ataxia or nystagmus
o! However there is loss of sensation to pinprick of the face in an “onion skin pattern”
•! Examination of the neck
o! No surgical scars noted
o! No kyphoscoliosis
•! Examination of the lower limbs
o! Spastic paraparesis
•! In summary, this patient has syringomyelia with presence of wasting of the upper limbs, dissociated
sensory loss and spastic paraparesis of the lower limbs. This has resulted with complications of repeated
trauma of his hands.

Questions
What is syringomyelia?
•! Cavity formation with presence of a large fluid filled cavity in the grey matter of the cervical spinal cord
which is in communication with the central canal and contains CSF.
•! Triad of LMN weakness of the ULs, dissociated sensory loss in the ULs and UMN weakness in the LLs

How do patients present?


•! Rare disorder, 4th to 5th decades, male=females
•! Painless trauma or burns in the upper limbs, poorly localised pain in the ULs
116
What is the pathophysiology?
•! At the level of the syrinx
o! LMN – anterior horn cell affected
o! Dissociated sensory loss – affects the decussating fibres of the spinothalamic tract
•! Below the level of the syrinx
o! Affecting the pyramidal corticospinal tract with spastic paraparesis of the LLs and preservation
of sphincters
•! Extension into the upper cervical cord and medulla
o! Horner’s syndrome
o! Bulbar palsy (CN IX-XII)
o! Ataxia and nystagmus ( affects the medial longitudinal bundle if lesion from C5 upwards)
o! Onion skin pattern loss of pain in the face (spinal nucleus of V CN which extends from the pons
to the upper cervical cord)

What are the differential diagnoses for dissociated sensory loss?


•! Anterior spinal artery occlusion (affects the spinothalamic tract)
•! DM neuropathy, leprosy, hereditary amyloidotic polyneuropathy

What are your differential diagnoses for syringomyelia?


•! Craniovertebral anomalies
•! Spinal cord injuries
•! Intramedullary tumours of the spinal cord
•! Arachnoiditis around the foramen magnum obstructing CSF flow
•! Hematomyelia

What are the associated abnormalities?


•! Arnold-Chiari malformation
•! Bony defects around the foramen magnum
•! Hydrocephalus
•! Spina bifida
•! Spinal cord tumours

How would you Ix?


•! MRI scan of the spinal cord

How would you manage?


•! Drainage of the syrinx to the subarachnoid space
•! Syringoperitoneal drainage
•! In AC malformation, cervical laminectomy and removal of the lower central portion of the occipital bone
•! Intramedullary tumour excision

What is syringobulbia?
•! Syrinx in the medulla of the brainstem
•! Usually extension of the syringomyelia but can be isolated
•! Results in
o! Horner’s
o! Ataxia and nystagmus
o! Bulbar palsy
o! CN V, VII, IX and X especially
o! Onion skin pattern of loss of pain sensation of the face

117
Dystrophia Myotonica

Approach to Congenital Myopathies


1.! Duchenne’s, Becker’s
2.! Myotonia
•! Dystrophia myotonica (fascioscapular dystrophies can mimic appearance)
•! Congenital myotonia
•! Hereditary paramyotonia
3.! Fascioscapulahumeral dystrophies, limb-girdle dystrophies, distal myopathies

Examination
Examine patient’s face or hands (Can be short case of locomotor or in CNS station)
Examine the hands
•! Demonstrate difficulty opening hands after shaking
•! Repeatedly open and close the hands
•! Percussion myotonia of the thenar eminence
•! (proceed with hand examination with function assessment if locomotor station)
•! Demonstrate weakness in the forearms (especially) and hands
•! No sensory loss
•! Loss of reflexes
•! Check the pulse (dysrhythmias, small volume pulse)

Examine the face


•! Myopathic facies
•! Expressionless
•! Triangular facies
•! Wasting of the temporalis, masseter (palpate these muscles when patient clenches teeth)
•! Frontal balding
•! Bilateral ptosis
•! Close his eyes and open
•! Tongue for percussion myotonia
•! Gum hypertrophy from phenytoin toxicity
•! Swan-neck appearance with wasting of the SCM (test for weakness of SCM), weakness of flexion of the
neck
•! Nodular thyroid enlargement

Request
•! Face
•! Cataracts - posterior subcapsular and stellate
•! Assess Speech – slurring due to myotonia of the tongue and pharyngeal muscle
•! Chest examination
•! Gynecomastia
•! Cardiovascular examination – dilated cardiomyopathy (split S1, mitral murmur, low BP and pulse
volume)
•! Testicular atrophy
•! Urine dipstick for diabetes mellitus
•! Lower limbs – bilateral footdrop

Presentation

Sir, this patient has got dystrophia myotonica as evidenced by

118
•! A myopathic facies that is triangular in appearance with an expressionless look. There is wasting of the
facial muscles involving the temporalis and masseter muscles associated with frontal balding and bilateral
ptosis. He had difficulty opening his eyes after firm closure. There was myotonia affecting the tongue.
•! There is also a swan-neck appearance with wasting of the sternocleidomastoid muscles with weakness of
flexion of the neck. On shaking his hand, there was a delay in releasing his grip. In addition, after making
a fist, he was unable to quickly open it especially after doing this repetitively. There was also presence
of percussion myotonia of the thenar eminence. There is presence of proximal myopathy and wasting
with involvement of the forearms and hands. There are also reduced reflexes with no sensory loss
detected. Function is relatively preserved.
•! With regards to complications
•! His pulse is regular at 80 bpm with a small volume pulse suggesting dil CMP
•! There was no gum hypertrophy to suggest chronic phenytoin use.
•! There is nodular thyroid enlargement.
•! I would like to complete my examination by
•! Face
•! Cataracts - posterior subcapsular and stellate
•! Assess Speech – slurring due to myotonia of the tongue and pharyngeal muscle
•! Chest examination
•! Gynecomastia
•! Cardiovascular examination – dilated cardiomyopathy (split S1, mitral murmur, low BP and pulse
volume)
•! Testicular atrophy
•! Urine dipstick for diabetes mellitus
•! Lower limbs – foot drop with high steppage gait (tibial nerves are affected early)

Questions

What are the types of muscular dystrophies you know of?


1. Duchenne’s
•! Sex linked
•! Pseudohypertrophy of the calves or deltoids
•! Gower’s sign, proximal weakness
•! Cardiomyopathy
Becker’s
•! Sex linked
•! Later onset and less severe form of Duchenne’s

2. Limb-girdle
•! Autosomal recessive
•! Shoulder and pelvic girdle affected
•! Third decade
•! Sparing of the face and heart
Fascioscapulohumeral
•! Autosomal dominant
•! Bilateral, symmetrical weakness of the facial and SCM with bilateral ptosis
•! Weakness of the shoulder muscles and later the pelvic girdle muscles

3. Dystrophia myotonica
Congenital myotonia
Hereditary paramyotonia

4.! Distal myopathies eg Welander’s myopathy

119
What is myotonia?
Continued contraction of the muscles after voluntary contraction ceases, followed by impaired relaxation.

What is dystrophia myotonica?


•! Characteristic clinical appearance with myotonia and weakness with no sensory loss
•! Autosomal dominant with a trinucleotide (AGC) repeat disorder on chromosome 19
•! Anticipation – phenotypic expression worsens with each successive generation
•! Onset in the 3rd or 4th decade
•! Males>females
•! In addition to the characteristic facies and musculoskeletal involvement
•! Intellectual and personality disorder
•! Cataracts – posterior subcapsular cataracts which are stellate type
•! CVM – dilated cardiomyopathy and conduction defects
•! Resp – recurrent infection from weakness of the bronchiolar musculature, hypoventilation and post-
anaesthetic respiratory failure
•! Abdomen – dysmotility and dysphagia
•! Testicular atrophy and gynecomastia
•! Diabetes mellitus
•! Nodular thyroid enlargement

How would you investigate?


•! Confirm Diagnosis
•! EMG – dive bomber pattern ie waxing and waning of the potentials
•! Muscle biopsy shows no inflammatory changes with type 1 fibre atrophy which is characteristic but
not diagnostic
•! DNA analysis
•! Muscle enzymes are normal
•! Screen for Complications
•! FPG – screen for diabetes mellitus
•! ECG – heart blocks, small P, prolonged PR, notched QRS and prolonged QTc
•! CXR – enlarged heart
•! Slit-lamp examination for cataracts

How would you manage?


•! Education, genetic counselling
•! PT/OT – eg foot orthosis for foot drop
•! Medications – phenytoin for myotonia, other anti-myotonic medications such as quinine and
procainamide should be avoided due to aggravation of cardiac conduction defects; however it is the
weakness that causes disability and not myotonia
•! Pacemaker for 3rd degree heart block or symptomatic such as syncope

How would you counsel the patient’s family?


•! Vertical
•! Autosomal dominant, children 1 in 2
•! Anticipation
•! DNA analysis is available for some families for prenatal diagnosis
•! Horizontal
•! Screen with clinical examination
•! Slit-lamp examination
•! EMG

120
What are the other types of myotonia disorders?
•! Myotonia congenita (Oppenheim’s disease)
•! Autosomal dominant or recessive
•! Presence of myotonia without other features of dystrophia myotonica
•! Present at infancy with difficulty feeding with subsequent improvement
•! No weakness and reflexes are preserved
•! Herculean appearance
•! Channelopathies
•! Hereditary paramyotonia
•! Autosomal dominant
•! Cold-induced myotonia

What are your differential diagnoses for dystrophia myotonica?


•! Facies appearance – Facioscapulohumeral dystrophy
•! Autosomal dominant, onset at age 10-40, Chr4, normal lifespan
•! Face – ptosis, difficulty closing eyes, facial weakness and speech impaired
•! Normal IQ
•! Neck – wasted SCM and weakness
•! Shoulder – winging of scapula, weakness of pectoralis, trapezius, biceps and triceps and hypertrophy
of deltoids
•! Occasionally affecting the anterior tibialis
•! Normal CK
•! Proximal weakness – FSH, limb-girdle, prox myopathy causes, MG
•! Limb Girdle dystrophy (see prox myopathy)
•! Distal weakness – Welander’s distal myopathy, nerve problem
•! Myotonia – Congenital myotonia, hereditary paramyotonia

Cerebellar

Examination

Stem statement
•! Giddiness, falls, unsteadiness
•! Face, Speech, ULs and LLs

Unilateral
•! Upper limbs
•! Screen for pronator drift, ensure patient can see your finger!
•! Cerebellar signs
•! Dysmetria with intention tremor
•! Dysdiadochokinesia
•! Dyschronometria
•! Power for ataxic hemiparesis
•! Sensory
•! Temperature/Pain loss in syringomyelia and LMS
•! Tone for cogwheel and leadpipe rigidity
•! Skin for neurofibromatosis
•! Pulse for AF
•! Face
•! Gaze evoked nystagmus (in the direction of gaze), INO, RAPD
•! Speech (Count 1 to 20; British Constitution; West Register Street)
121
•! Cerebellar speech – jerky, explosive and loud; irregular syllables
•! CNs
•! CPA
•! LMS
•! III nerve palsy in Benedikt’s syndrome
•! Xanthelesma
•! Lower limbs
•! Dysmetria and intention tremor for toe to finger test
•! Dyssynergia for heel-shin test
•! Dysdiadochokinesia for foot tapping test
•! DM dermopathy
•! Sit up with hands folded and tests for pendular jerks
•! Gait
•! Broad based gait with veering towards the side of the lesion
•! Request to test visual fields for hemianopia

Bilateral
•! Upper Limbs
•! Cs of cerebellar (dysmetria, dysdiadochokinesia and dyschronometria)
•! Sensory – loss of temperature/pain for syringomyelia
•! Parkinsonism
•! NF features
•! Alcoholic features – dupytren’s contracture, stigmata of chronic liver disease
•! Face
•! CNs
•! Bilateral CPA tumor
•! Multiple sclerosis
•! Eyes
•! Gaze evoked nystagmus
•! KF rings
•! INO, RAPD
•! Mouth
•! Gingivial hypertrophy
•! Macroglossia
•! Telengiectasia
•! Parotidomegaly
•! Goitre
•! Speech
•! Cerebellar speech
•! Hoarseness of voice
•! Lower limbs
•! Cerebellar signs
•! Clawing of toes (Friederich’s ataxia)
•! Sit – truncal ataxia and pendular jerks
•! Gait – cerebellar gait

Presentation

Unilateral
Sir, this patient has a right sided unilateral cerebellar lesion as evidenced by presence of a right dysmetria,
dysdiadochokinesia and dyschronometria of the right upper limb. The right lower limb also demonstrates
presence of right dyssynergia on heel shin test, with right dysmetria and intention tremor on toe-finger test
122
and dysdiadochokinesia. This is associated with a gazed evoked nystagmus on rightward gaze with a broad
based gait with veering towards the right. I did not detect any cerebellar speech or any truncal ataxia.

There are no associated cranial neuropathies. In particular there was no evidence of any cerebello-pontine
angle lesion with CN V, VI, VII and VIII intact. (There are also no signs of neurofibromatosis such as
neurofibromas or café-au-lait spots.) There is also no evidence of lateral medullary syndrome or III nerve
palsy to suggest Benedikt’s syndrome. There is also no pronator drift on the right to suggest a right ataxic
hemiparesis.

Patient is in sinus rhythm and not in atrial fibrillation with no xanthelesma or diabetic dermopathy. There are
also no bruises to suggest overanticoagulation.

There are no signs of Parkinsonism to suggest presence of multiple system atrophy. There are also no
associated features of multiple sclerosis such as RAPD or INO.

I would like to complete the examination by looking at


1. The patient’s temperature chart for fever (abscess in posterior fossa)
2. Visual fields for a left sided hemianopia, which can occur with a right posterior circulation stroke
3. I would also like to do a fundoscopy for papilloedema for a SOL in the right cerebello-pontine lesion as
well as for optic atrophy from demyelinating disease.

In summary, this patient has got an isolated right cerebellar lesion. The differential diagnoses include cerebral
vascular infarction or haemorrhage or a space-occupying lesions such as a mitotic lesion or an abscess.

Bilateral
Sir, this patient has bilateral cerebellar lesions as evidence of dysmetria with intention tremor bilaterally
associated with dysdiadochokinesia. Similar findings were also present on examination of the lower limbs.
There is also presence of multi-directional gaze evoked nystagmus associated with a cerebellar speech,
truncal ataxia and a broad based gait.

There is no evidence of bilateral CPA lesion with no CN V, VI, VII and VIII abnormalities. Patient is in
sinus rhythm and not in AF with no xanthelesma or diabetic dermopathy.

There is no evidence of KF rings to suggest presence of Wilson’s disease. There is also no RAPD or INO to
suggest multiple sclerosis. There is also no gingival hypertrophy to suggest chronic phenytoin use. Patient
has no goitre or features of hypothyroidism such as a cream and peaches complexion, no hoarseness of voice
or macroglossia. There are also no features of chronic ethanol ingestion such as Parotidomegaly, dupytren’s
contracture or stigmata of chronic liver disease. There is no associated Parkinsonism signs to suggest multiple
system atrophy such as presence of cog-wheeling or leadpipe rigidity. There are also no neurofibromas
present to suggest presence of NF type 2. Patient is also not cachexic looking and there is no clubbing to
suggest underlying malignancy.

I did not detect any telengiectasia to suggest presence of Ataxia telengiectasia and there is pes cavus to
suggest Friederich’s ataxia. (Think of Wilson’s, MS, Phenytoin, Hypothyroid, Alcohol, Parkinsonism, NF,
paraneoplastic, telangiectasia and FA)

I would like to complete the examination by


1. Looking at the temperature chart for fever
2. Performing a neurological examination of the lower limb to look for spastic paraparesis
3. I would also like to do a fundoscopy for Optic atrophy, which may suggest demyelinating disease.

In summary, this patient has bilateral cerebellar syndrome. Possible causes include drugs such as phenytoin,
metabolic conditions such as hypothyroidism, chronic ethanol ingestion, paraneoplastic conditions and
infection such as enteroviruses and bilateral cerebellar strokes.

123
Questions

What are the differential diagnoses for a unilateral cerebellar syndrome?


•! Isolated
•! Cerebrovascular accident – infarction or haemorrhage
•! SOL in posterior fossa – abscess or mitotic (primary vs secondary)
•! Associated
•! CN
•! CPA and/or neurofibromatosis
•! Lateral medullary syndrome
•! Jugular foramen (Arnold-Chiari or Dandy-Walker)
•! Benedikt’s syndrome
•! Ataxic hemiparesis (lacunar stroke)
•! Parkinsonism in Multiple system atrophy
•! Demyelinating such as Multiple sclerosis

What are causes of bilateral cerebellar syndrome?


•! Acquired
•! Infection
•! Viral – HIV, Enteroviruses
•! Spirocheatal – Lymes and Tabes dorsalis
•! Others – Toxoplasmosis and CJD
•! Metabolic
•! Wilson’s disease
•! Hypothyroidism
•! Drugs
•! Phenytoin and Carbamazepine
•! Lithium
•! Alcohol
•! Causes bilateral cerebellar signs
•! Causes isolated lower limb cerebellar signs
•! Affects the anterior vermis
•! Due to thiamine deficiency
•! Multiple system atrophy
•! Neurofibromatosis type 2 with bilateral CPA tumor
•! Bilateral Strokes
•! Paraneoplastic – Ca lung or ovary
•! Hereditary
•! Ataxia telangiectasia
•! Autosomal recessive
•! Childhood with death by 20s or 30s
•! Ataxia, choreathetosis and telengiectasia on the face, eras and conjunctiva and skin crease
•! Low IgA with recurrent chest infections and lymphoreticular malignancy
•! Friederich’s ataxia
•! Scoliosis, pes cavus
•! Spastic paraparesis, dorsal column loss, absent ankle jerks

What are the signs of a midline lesion (cerebellar vermis) and what are the causes?
•! Signs : truncal ataxia, abnormal heel-toe walk test, cerebellar speech
•! Causes : Midline tumor, paraneoplastic

124
What are the causes of cerebellar signs with spastic paraparesis?
•! Friederich’s ataxia
•! Spinocerebellar ataxia
•! Arnold-Chiari Malformation
•! Lesion at the craniospinal junction eg meningioma
•! Syringomyelia
•! Multiple sclerosis
•! Syphilitic meningomyelitis

How are cerebellar signs located?


•! Limb ataxia = cerebellar lobes
•! Gait ataxia = anterior vermis
•! Truncal ataxia = posterior vermis

What are the differences between cerebellar and sensory ataxia?


Cerebellar Sensory
Site Cerebellar Posterior column, nerves
Tone Reduced Normal
Reflexes Normal or pendular Reduced
Sensory Normal Reduced
Sphincter disturbance Nil Affected when posterior
column involved; overflow
incontinence

How would you investigate?


•! Imaging – MRI brain
•! Blood tests according to the causes

How would you manage?


•! Depends on underlying cause

Chorea
(Beware the Parkinsonism with dyskinesia!)

Approach

1.! Introduce, sit the patient


2.! Lift up hands
a.! Involuntary athetoid movements/choreiform movements
b.! Choreic posture
c.! Dish spooning
d.! Pronator drift
e.! Milk-maid’s grip
f.! Look for wasting of the muscles and joint deformities
g.! Look for erythema marginatum and subcutaneous nodules
h.! Check for Parkinsonism
i.! SLE signs
3.! Check for long tract signs especially if hemiballismus or one sided
4.! Eyes
a.! KF rings
b.! Conjuctival suffusion
c.! Dysthyroid eye disease, nystagmus

125
d.! Plethoric facies
e.! Darting tongue
f.! Goiter
5.! Walk the patient – effeminate gait, Parkinsonian gait

Presentation

Sir, this patient has chorea/athetosis affecting her left hand. I say this because of presence of brief, abrupt,
irregular, quasi-purposeful movements of the left hand with writhing and twisting movements (athetosis).
There is choreic posturing of the left hand with a flexed wrist and an extended mcpj; with dish spooning and
milk maid grip, associated with darting tongue and an effeminate gait.

There were no features of Parkinsonism to suggest that dyskinesia is secondary to L-dopa therapy. There was
no evidence of erythema marginatum or subcutaneous nodules which can occur in rheumatic heart disease.
There is also no cutaneous rash to suggest SLE. There is also no pronator drift.

There are also no KF rings or nystagmus to suggest Wilson’s disease. There are no signs of polycythemia
rubra vera as I did not notice any plethoric facies, conjunctival suffusion or pruritic scratch marks. There are
also no goiter or thyroid eye signs.

I would like to complete the examination by performing a cardiovascular examination to look for evidence
of rheumatic heart disease, a mini-mental state examination for dementia as this occurs in Huntington’s
chorea, as well as take a drug history of neuroleptics and L-dopa and a past history of encephalitis.

Questions

What are the different types of movement disorders that you know about?
•! Tremors
•! Resting tremor of Parkinsonism
•! Intention tremor of Cerebellar
•! Postural tremor of outstretched hands
•! Anxiety
•! Thyrotoxicosis
•! Alcohol
•! Drug induced – salbutamol, terbutaline, theophylline, Li
•! Drug withdrawal – BZD, opiates
•! Familial
•! Chorea (globus pallidus)
•! Athetosis
•! Hemiballismus (subthalamic nucleus)
•! Infarct
•! Others – abscess, tumor, MS, AVM
•! Search for CVS risk factors
•! Rx – haloperidol, treat CV risk factors and Sx eg contralateral thalomotomy or pallidotomy
•! Orofacial dyskinesia
•! Secondary to antipsychotics usually, in pts with SZ
•! One of the 4 EPSE
•! Acute dystonia (oculogyric)
•! Parkinsonism
•! Akathisia (restless legs syndrome)
•! Tardive dyskinesia (or orofacial dyskinesia)

126
What are the causes of choreathetosis?
•! CVA/tumors affecting the globus pallidus (Benedikt’s syndrome - III)
•! Metabolic – Wilson’s disease
•! Endocrine – Hyperthyroidism, post-hyperglycemia
•! CTDs – SLE
•! Polycythemia
•! Rheumatic heart disease – Sydenham’s chorea
o! Most recover within one month
•! Huntington’s Chorea
•! Drugs – neuroleptics, L-dopa, phenytoin, OCPs
•! Post encephalitis
•! CO poisoning

What is Huntington’s disease?


•! Young adult, chorea and dementia
•! AD, Chr 4, CAG trinucleotide repeats

Parkinsonism

Examination

•! Introduce
•! Mask like facies, monotonous speech, dyskinesias
•! Upper limbs
•! Resting tremors which disappears with use
•! Bradykinesia (thumb to finger, rotate wrist and “twinkle stars”
•! Leadpipe rigidity and cogwheeling
•! Acute dystonia or alien limb syndrome
•! Pronator drift and cerebellar signs
•! Palmomental reflex, grasp reflex
•! Face
•! Eye movements, vertical Doll’s if vertical gaze impaired
•! Close eyes for blepharospasm
•! Feel for seborrhea
•! Look for KF rings
•! Count 1-20
•! Unbutton shirt, write, cap a pen, comb his hair
•! Gait – typical parkinsonian gait; also rule out gait apraxia
•! Request
•! Speech if not done
•! Swallowing
•! Handwriting
•! Postural BP
•! AMT

Presentation

Sir, this elderly gentleman has Parkinsonism with mask like, expressionless facies. He has asymmetrical
resting tremor of the right hand with characteristic pill rolling movements of the thumb that disappears with
use of the hand. There is also presence of bradykinesia with leadpipe rigidity at the elbows and cogwheeling
at the wrist. Movement of the contralateral upper limb accentuates these features.

127
There is presence of seborrhea and Myerson’s sign or the glabella tap sign is positive.

He has difficulty initiating his gait and has a stooped posture associated with shuffling gait with festination
and lack of normal arm swing. He also turns in numbers. His gait is not apraxic and he is not on any urinary
catheter to suggest NPH.

Functionally he is able to walk unaided and can perform keyturning movements and unbutton his short
unaided.

There is no evidence of dyskinesias which can result as a result of L-dopa therapy.

He dose not have features suggesting presence of Parkinson-plus syndrome. There is no evidence of
Progressive Supranuclear Palsy such as impairment of the vertical gaze, blepharospasm or frontal lobe signs
such as palmomental reflex and the grasp reflex. There are also no cerebellar signs to suggest multisystem
atrophy. There is also no evidence of corticobasal ganglia degeneration such as dystonic arm or alien limb
syndrome.

In summary, this patient has Parkinsonism most likely due to Parkinson’s disease and relative preservation
of his function; there is no evidence of dyskinesia currently to suggest side effects of L-dopa therapy.

Questions

What is Parkinson’s disease?


It is a progressive neurodegenerative disorder associated with degeneration of the dopaminergic
nigrostriatal neurons.
Dx clinically with 2 out of 3 signs comprising of resting tremors (3-5Hz), bradykinesia and
th
rigidity. The 4 sign of postural instability occurs later in the course of the disease.

What are the causes of Parkinsonism?


1.! Parkinson’s disease
2.! Parkinson-plus syndromes
3.! Drugs (Neuroleptics, antiemetics, MPTP- 1 methyl 4 phenyl 1,2,3,6 tetrahydropyridine)
4.! Anoxic brain damage ( Post cardiac arrest, Manganese, CO)
5.! Post encephalitis ( encephalitis lethargica or von Economo’s disease)
6.! Tumor such as giant frontal meningioma

What are the pathologic findings in Parkinson’s disease?


•! Loss of pigmented dopaminergic neurons in the substantia nigra
•! Presence of Lewy Bodies (eosinophilic cytoplasmic inclusions)

What are the Parkinson-plus syndromes?


•! Progressive supranuclear palsy (most common) (frontal lobe) (3)
•! Vertical gaze palsy
•! Downgaze affected first, then upgaze, then horizontal
•! Can be overcome by vertical Doll’s
•! Other features such as blepharospam and slow pursuit or saccadic eye movements
•! Postural instability and axial rigidity with falls early in the course of the disease
•! Frontal lobe signs
•! Multiple sytem atrophy (Cerebellar)
•! MSA-P = Parkinsonism features
•! MSA-C = Cerebellar features
•! Features (3)
•! Cerebellar signs
128
•! Autonomic features – orthostatic hypotension, urinary dysfn and erectile dysfn
•! Corticospinal signs – hyperreflexia and extensor plantar response
•! Corticobasalganglionic degeneration (frontoparietal lobe)
•! 2 features
•! Limb apraxia or alien limb syndrome
•! Dystonia
•! Parkinsonism-dementia-ALS complex
•! Diffuse Lewy Body disease (Parkinsonism, dementia and neuropsychiatry)

What is the significance of diagnosing Parkinson Plus syndrome?


•! Poorer prognosis
•! Poor response to L-dopa therapy

What are the features that suggest that patient may have Parkinson plus syndromes?
•! Early onset of dementia
•! Presence of hallucination or psychosis
•! Early onset of postural instability
•! Truncal symptoms more prominent than appendicular symptoms
•! Marked symmetry of signs early in the stage of the disease
•! Lack of response to levo-dopa therapy in the early stage of the disease
•! Presence of symptoms and signs suggestive of Parkinson-plus syndromes.

What are the stages of Parkinson’s disease?


Staged via the Hohen and Yahr staging system comprising of 5 stages:
•! Stage 1 – symptoms and signs unilateral and mild
•! Stage 2 – Bilateral and minimal disability
•! Stage 3 – Generalised dysfunction with sig bradykinesia and gait impairment
•! Stage 4 – Rigid and bradykinesia, severe symptoms with limited walking
•! Stage 5 – Completely invalid and requires nursing care

How would you investigate?


•! Brain scan to rule out
•! NPH
•! Multi-infarct syndromes
•! Frontal meningiomas
•! Parkinson-plus syndrome
•! MSA – atrophy of the cerebellum, brainstem
•! PSP – Atrophy and hyperintensity of the midbrain and red nucleus
•! CBGD – Frontoparietal cortical atrophy
•! If patient is young, ie<50, rule Wilson’s disease
•! Slit-lamp examination
•! Serum ceruloplasmin and 24Hr Urinary Copper

How would you manage?


•! Multidisciplinary approach
•! Education and counselling, PT/OT/ST
•! Medications
•! Dopamine agonist, eg pramipexole or ropinirole
•! Early in the course of the disease or younger patients
•! Delay onset of motor fluctuations and dyskinesias
•! Nausea, orthostatic hypotension, hallucinations or somnolence, edema
129
•! L-dopa therapy (usually combined with a peripheral decarboxylase inhibitor)
•! Should be used if there is disability
•! Never used in patients with melanoma
•! Peak dose dyskinesia and end of dose rigidity
•! Nausea, orthostatic hypotension, hallucinations
•! Tremors – anticholinergic (dry eyes and mouth, urinary retention, arrhythmia), e.g. arcane or
benztropine.
•! Rigidity – beside Dopa and D agonist, can use MAO-B inhibitors or amantadine (cognitive side
effects)
•! COMT inhibitors (diarrhea and hepatotoxicity)
•! Surgical
•! Lesion surgery – thalomotomy (tremors) and pallidotomy (for all features)
•! Deep brain stimulation (for all features especially tremors)

How would you manage dyskinesia?


•! Peak dose dyskinesia
•! Reduce the dose and increased frequency
•! Add on COMT inhibitors i.e. entacapone which increases half life of L-dopa therapy
•! Initiating with dopamine agonist and adding on L-dopa therapy resulted in less motor fluctuations
•! End of dose
•! Increase dose, frequency
•! Switch to CR formulations
•! Add DA, MAO-B inhibitors, COMT inhibitors

What is the prognosis?


•! Chronic
•! Progressive
•! Some will have mild while other will have severe symptoms
•! Some will be troubled mainly with tremors, other by rigidity and bradykinesia

What is the abbreviated mental test?


•! Use as a screening for possible dementia in the elderly
•! A score of less than 6 warrants further assessment
•! Includes
•! Age
•! DOB
•! Remember this address – 42 West Street
•! Time (nearest hour)
•! Year
•! Recognition of 2 persons
•! Place
•! Prime Minister of UK
•! First Year of WW1 (1914-1918)
•! Count backwards 20 to 1

130
Lower Limbs Overview

•! Pes cavus
•! CMT
•! Spina Bifida
•! Poliomyelitis
•! Spinal cord tumours
•! Freiderich’s ataxia/spinocerebellar degeneration
•! Syringomyelia
•! Cerebral Palsy
•! Muscular dystrophies
•! Fasciculations (LMN type, MND)
•! Wasting
•! Bilateral
•! Proximal weakness
•! Wasting distally (Pes Cavus, Peripheral neuropathy)
•! Spastic paraparesis (L&P 104)
•! Cerebellar (MS/FA/Syphilitic meningomyelitis/Craniospinal jn/SCA)
•! Sensory level (Lumbar/Thoracic/Cervical – ULs/Above – high Cx, CP)
•! Dorsal Column Loss (SACD/Taboparesis/MS/FA)
•! Mixed (Babinski + absent reflexes – see below)
•! Friederich’s ataxia
•! SACD
•! Tabo-paresis
•! MND
•! UMN + cauda equina or peripheral neuropathy(CVA+alcoholic/DM)
•! MND
•! Flaccid paraparesis
•! Wasted
•! GBS/CIDP/HMSN/Hansen’s
•! Poliomyelitis
•! Spina Bifida
•! No wasting
•! Peripheral – GBS, HMSN, paraneoplastic, paraproteinemia, amyloid
•! Cord compression
•! Others – Miller-Fisher, MG, Periodic paralysis,
botulism/diphtheria/organophosphate/Hg/Pb, AIP(BP)
•! MND
•! Bilateral Footdrop
•! Unilateral
•! Foot Drop
•! Bilateral (Peripheral neuropathy – motor predominant, flaccid, spastic)
•! Unilateral
•! Peripheral neuropathy, CPN
•! Sciatic nerve
•! Root or anterior horn cell
•! Look for complications- trophic ulcer, interventions – walking callipers
•! Unilateral – Peripheral neuropathy, lumbosacral plexus, polyradiculopathy, polio (LMN)
•! Brown-sequard (UMN)
•! Diabetic amyotrophy
•! Hemiparesis (UMN)

131
•! Sensory loss
•! Peripheral neuropathy
•! Mononeuropathy
•! Polyradiculopathy
•! Lumbosacral Plexus
•! Dissociated sensory loss, spinal cord level
•! Others
•! Gait
•! Cerebellar
•! Unilateral
•! Cerebellar – Vascular, MS, SOL eg abscess or tumour
•! Combined – Lateral medullary syndrome, CPA tumour, ataxic hemiparesis
•! Bilateral
•! hypothyroidism, Wilson, Alcoholic cerebellar degeneration(spares the ULs), drugs phenytoin,
paraneoplastic, Parkinson plus
•! large CVAs, SOL, MS
•! plus all causes of spastic and cerebellar
•! Midline – paraneoplastic, midline tumour
•! Spastic and Ataxic combined
•! Spinocerebellar degeneration
•! Friederich’s ataxia
•! Multiple sclerosis
•! Syphilitic meningomyelitis
•! Craniospinal junction abnormalities – Arnold-Chiari, meningioma
•! Non conforming
•! Myasthenia Gravis
•! Mononeuritis multiplex
•! Motor neurone disease

Giddiness/Unsteadiness protocol
•! Giddiness
o! Cerebellar
o! Vestibular
o! Postural BP
•! Unsteady gait
o! Cerebellar
o! Parkinsonism
o! Sensory ataxia (Proprioception)
o! Others – hemiplegic gait, cervical myelopathy etc etc
•! Examination for unsteady gait
o! Start with Lower Limbs FIRST
!! As per LL protocol
!! Concentrate on cerebellar, sensory ataxia and Parkinsonism
!! Examine the gait!
o! Proceed with Parkinsonism protocol if Parkinsonian gait
o! Proceed with cerebellar protocol if cerebellar signs

132
Flaccid Paraparesis

Examination

•! Complete the LL examination


•! Commonly
•! HMSN
•! Polio
•! (infantile hemiplegia)
•! Spina Bifida
•! Cauda Equina Syndrome
•! GBS/CIDP
•! MND (see spastic paraparesis)
•! Diabetic amyotrophy (See proximal myopathy)
•! Concentrate on
•! Ataxia – Miller Fisher Variant, Tick Paralysis
•! Sensory
•! No sensory abnormalities
•! Myopathies
•! Neuromuscular
•! Nerves – certain conditions eg GBS, multifocal motor neuropathy
•! Anterior Horn Cell
•! Glove and stocking
•! Peripheral neuropathy
•! HMSN, paraneoplastic
•! Mild and patchy = GBS
•! Sensory level (Acute)
•! Cord compression
•! Cord infarction
•! Transverse myelitis
•! L5 and S1 sensory loss in spina bifida
•! Typical features of HMSN
•! Pes cavus, clawing of toes, contractures of Achille’s tendon, inverted champagne bottles
(wasting of distally and stops abruptly at the lower one third of thighs; also similar distal
wasting distally in the ULs)
•! LMN – reduced tones and no clonus, reduced reflexes and downgoing plantar response,
weakness, bilateral footdrop
•! Sensory – no sensory or mild glove and stocking
•! Gait – high steppage gait of foot drop
•! Marked deformity with minimal disability
•! Others
•! Feel for thickened nerves (lateral popliteal nerve)
•! Examine the hands for small muscle wasting and clawing
•! Examine spine for scoliosis
•! Feel for thickened Greater Auricular nerves
•! Wheelchair, calipers
•! Examine
•! Back
•! Kyphoscoliosis
•! Spina bifida – scars, tuft of hair, dimples, sinus or naevus
•! Per rectal examination
•! Saddle anaesthesia and cauda equina syndrome
133
•! Incontinence – fecal and urinary
•! Upper limbs
•! CNs- fatiguibility, GBS (bilateral VII)
•! Functional aids

Presentation

•! Obvious disease
•! HMSN
•! Sir, this patient has got HMSN/CMT as evidenced by
•! Bilateral pes cavus with clawing of toes and distal wasting of the lower limbs with a inverted
champagne bottle appearance; there is hypotonia with reduced reflexes and downgoing plantar
responses a/w weakness of the lower limbs of power 4/5 with bilateral foot drop; there is no
associated sensory disturbance; she has a high steppage gait form bilateral foot drop and is able
to walk independently inspite of the marked feet deformity; I also noticed presence of wasting
and clawing of the upper limbs; there is no palpable thickened lateral popliteal nerve.
•! I would like to complete my examination by examining the spine back for scoliosis and palpate
for other sites of thickened nerves
•! Mention walking aids or wheelchair
•! Polio
•! Sir this patient has monoparesis of the right LL most likely due to polio
•! A shortened right lower limb associated with wasting. It is hypotonic with reduced reflexes and
downgoing plantar response and is flaccid with a power of 3/5. There is no sensory weakness.
•! There is no UMNs or shortened wasted right UL to suggest infantile hemiplegia
•! Examination of the back did not reveal any cutaneous signs of spina bifida.
•! Mention any walking aids/wheelchair
•! Not so obvious
•! Sir, this patient has got flaccid paraparesis as evidenced by
•! Presence of hypotonia with reduced reflexes a/w with downgoing plantar responses bilaterally; I did
not detect any fasciculations. There is weakness of the LLs with a power of 3/5. There is no
associated cerebellar signs in the LLs and no sensory loss to pin prick, propioception and vibration.
•! Complete my examination
•! Back
•! Per rectal
•! ULs for ataxia, flaccid paresis
•! CNs for cranial neuropathies

Questions

What are the causes of flaccid paraparesis?


•! Acute myopathies
•! Inflammatory myopathy (polymyositis, dermatomyositis)
•! Rhabdomyolysis (extreme exertion, drugs, viral myositis, crush injury etc.)
•! Acute alcoholic necrotizing myopathy
•! Periodic paralyses (hypokalemic, hyperkalemic)
•! Metabolic derangements (hypophosphatemia, hypokalemia, hypermagnesemia)
•! Thyroid or steroid myopathy
•! Neuromuscular
•! Myasthenia gravis
•! Botulism
•! Tick paralysis
•! Other biotoxins (tetradotoxin, ciguatoxin)
•! Organophosphate toxicity (can also cause neuropathy)
134
•! Lambert-Eaton Myasthenic Syndrome (LEMS)
•! Nerve
•! Diphtheria
•! Porphyria
•! Drugs & Toxins (arsenic, thallium, lead, gold, chemotherapy – cisplatin / vincristine)
•! Vasculitis (incl. Lupus, polyarteritis)
•! Paraneoplastic and Paraproteinemias
•! Multifocal motor neuropathy
•! Nerve roots
•! Guillian Barre Syndrome
•! Lyme disease
•! Sarcoidosis
•! HIV
•! other viruses (CMV, VZV, West Nile)
•! Cauda equina syndrome (lumbar disc, tumour, etc.)
•! Plexus lesions (brachial plexitis, lumbosacral plexopathy)
•! Anterior Horn Cell (motor neuron diseases):
•! Amyotrophic lateral sclerosis (ALS) – with UMN findings
•! Poliomyelitis
•! Kennedy’s disease (spinobulbar atrophy / androgen receptor gene)
•! other spinomuscular atrophies (inherited)
•! Anterior spinal artery syndrome (with grey matter infarction)
•! Spinal Cord (corticospinal tract diseases):
•! Inflammatory (Transverse myelitis)
•! Subacute combined degeneration (B12 deficiency)
•! Spinal cord infarction
•! other myelopathies (spondylosis, epidural abscess or hematoma
•! Brain
•! Pontine lesions (eg. Central pontine myelinolysis, basis pontis infarct or bleed)
•! Multifocal lesions (multiple metastases, dissemination encephalomyelitis [ADEM], multiple
infarcts or hemorrhages – eg. DIC, TTP, bacterial endocarditis)

What is Charcot Marie Tooth disease?


•! Hereditary sensory motor neuropathy
•! Consisting of 7 types of which types 1,2 and 3 are the most common types
•! Type 1 – A demyelinating neuropathy, aut dominant, absent tendon reflexes, enlarged nerves; Chr
17
•! Type 2 – An axonal neuropathy, aut dominant (mild and present later), normal deep tendon
reflexes, nerves not enlarged; Chr 1
•! Type 3 – rare, hypertrophic neuropathy of infancy, thickened nerves, aut recessive (Dejerine Sottas
disease)
•! Physical findings
•! Above plus
•! Others – optic atrophy, retinitis pigmentosa and spastic paraparesis
•! Ix
•! Rule out other causes of neuropathies
•! EMG/NCT
•! Biopsy
•! Genetic testing
•! Mx
•! Eductaion and counselling and family screening
•! PT/OT, AFOS
135
•! Medical Rx – pain relief, avoid obesity
•! Surgical treatment
•! Px
•! Normal life expectancy
•! Disease usually arrest in middle life
•! Disability varies
•! Dy/Dx of hereditary disease
•! Hereditary amyloidosis
•! Refsum’s disease – accumulation of phytanic acid
•! Fabry’s disease – deficiency of alpha galactosidase

What is poliomyelitis?
•! Enterovirus, picorna virus, with IP of 5-35 days, oro-fecal route or contaminated water, 3 serotypes
•! Replicate in the nasopharynx and GIT and then to lymphoid tissue and then hematological spread with
predilection to the anterior horn cells of the spinal cord or brainstem with flaccid paralysis in spinal or
bulbar distribution
•! 4 forms
•! Inapparent infection
•! Abortive – nauseas, vomiting and abdominal pain
•! Nonparalytic – above plus meningeal irritation
•! Paralytic – paralysis and wasting; bulbar or spinal distribution
•! Occasionally, can get postpolimyelitis syndrome which results in weakness or fatigue in the initially
involved muscle groups 20-40 years later
•! Ix
•! Viral c/s from stool, throat and CSF
•! Antibodies
•! Mx
•! Educationa and counselling
•! Non-medical
•! PT/OT
•! Care of limbs
•! Medical
•! Rx complications
•! Pain
•! Respiratory failure
•! Clear bowels
•! Prevention
•! Inactivated polio vaccine – Salk vaccine which is administered parenterally
•! Oral live vaccine – can result in poliomyelitis in immunodeficient individuals
•! Dy/Dx
•! Spina bifida
•! Infantile hemiplegia – hypoplasia of the entire side of the left side with UMN sign on the affected
side

What is Spina Bifida?


•! Incomplete closure of the bony vertebral canal with similar anomaly of the spinal cord
•! Usually in lumbosacral region, can also involve the cervical region and is associated with
hydrocephalus
•! Look for
•! Scars, tuft of hair, dimples, sinus, naevus, lipoma
•! Asymmetric LMN signs of LLs
•! L5 and S1 dermatomal sensory loss
136
•! Bladder involvement
•! X-ray: sacral dysgenesis, laminar fusion of the vertebral body. Scoliosis
•! Multifactorial aetiologies, with folic deficiency and use of Na Valproate, siblings with spina bifida has
higher risk
•! Prevented with use of folic acid early in pregnancy
•! Can be tested with amniotic serum AFP, serum AFP or USS

What is cauda equina syndrome?


•! The cauda equina refers to the nerve roots that are caudal to the termination of the spinal cord; any
lesion below the 10th Thoracic vertebrae
•! Low back pain, unilateral or bilateral sciatica, saddle anaesthesia, bladder and bowel disturbances and
variable motor and sensory LL abnormalities
•! Causes – trauma, PID, spondylosis, abscess, tumor (ependymoma and NF)
•! Anatomy
•! Spinal cord starts from the foramen magnum to the level of L1 vertebrae
•! Add 1 to Cx vertebrae
•! Add 2 to Tx vertebrae 1-6
•! Add 3 for Tx vertebrae 7-9
•! T10 and T11 vertebrae = lumbar segments
•! T12 and L1 = sacral and coccygeal
•! Conus medullaris = T9 to L1 vertebrae
Conus Medullaris Cauda equina
Presentation Acute Chronic
Reflexes Knees preserved; ankle absent Both knees and ankles absent
Motor Spastic para; symmetrical Flaccid para; asymmetrical
Sensory More LBP, less radicular Less LBP, more radicular
Sensory Perianal Saddle
Impotence Frequent Less frequently
Sphincter Occurs early Occurs late

What is Guillain Barre Syndrome?


•! Auto immune, antecedent Campylobacter infection
•! Bimodal – young adults or the elderly
•! Motor, sensory and autonomic dysfunction
•! Progressive ascending muscle weakness, variable patchy sensory loss, hyporeflexia and autonomic
disturbances such as tachycardia and labile BP
•! Post GI or resp infection, 2-4 weeks of onset of symptoms which may progress over hrs to days and
recovery over months; complicated by respiratory failure
•! Subtypes
•! Acute inflammatory demyelinating neuropathy
•! Acute motor axonal neuropathy
•! Acute motor-sensory axonal neuropathy
•! Miller Fisher Syndrome (Ataxia, areflexia and ophthalmoplegia; antiGQ1b Ab)
•! Acute panautonomic neuropathy
•! Ix
•! CSF shows albuminocytologic dissociation (<10 mononc cells and high prot)
•! AntiGQ1B Ab in MFS and anti GM1 implies poorer Px
•! NCT – demyelination
•! FVC (15-20ml/kg < or NIF 25cmH2O<) = may require ventilation
•! Mx
•! Emergency
•! ABCs, and pacing maybe required
137
•! IVIG
•! Plasma exchange
•! Steroids
•! PT/OT
•! Prevention Cx – DVT prophylaxis
•! Px
•! Most 85% will have full recovery by 6-12 months
•! Complications from respiratory failure and cardiac dysrythmias and labile BP

Spastic Paraparesis
(Think: Cerebellar, sensory level, dorsal column, mixed, ULs and others)

Presentation

Sir, this patient has spastic paraparesis as evidenced by


•! Increased tone and clonus
•! Hyper- reflexia with upgoing-plantars
•! Weakness of the lower limbs of power 4 with wasting
•! Complications of
o! DVT
o! Pressure sores
o! Urinary catheter, diapers
•! Functionally
o! Wheelchair by the bedside
o! Walking aids, orthotic shoes
o! Gait (Rhomberg’s sign positive with dorsal column loss, broad based gait)

This is associated with


1.! Cerebellar signs
o! Dyssynergia on heel-shin test
o! Dysmetria and intention tremor on toe to finger test
o! Dysdiadochokinesia on foot tapping test
o! There is no sensory level and no dorsal column loss
o! The possible differential diagnoses includes
!! Spinocerebellar degeneration such as FA
•! Young
•! Pes cavus, loss of ankle reflexes, dorsal column loss
!! Multiple sclerosis
!! Craniospinal junction abnormalities
•! Congenital - AC malformation
•! Acquired - meningioma at the CS junction, syringomyelia
!! Lewitic meningomyelitis
o! Hence I would like to complete the examination by
!! Features of MS – RAPD, INO and optic atrophy
!! Features of CS junction – CNs, neck, dissociated sensory loss
!! Argyll Robertson pupils for lewitic disease
2.! Sensory level at T4
o! Loss of sensation to pinprick up to the level of the nipples bilaterally
o! There are no cerebellar sign and dorsal column sensory system is intact
o! Hence the lesion is likely due to spinal cord lesion at T4 level and possible etiologies are
!! Cord compression

138
•! Extramedullary (Root pains, LMN localised, spasticity early, no sacral sparing,
abnormal CSF)
o! Vertebral – spondylosis, trauma, PID, tumor, infection
o! Extradural – abscess, metastases, lymphoma
o! Intradural – meningioma, NF
•! Intramedullary (root pains rare, LMN signs extend over several segments, late
spasticity, may sacral spare, normal CSF)
o! Syringomyelia
o! Tumor – glioma, ependymoma
o! Hematomyelia
!! Cord infarction
•! Anterior spinal artery thrombosis
•! Vasculitis (PAN, syphilis)
•! Thoracic/AAA and dissection
•! Causes of cord compression
!! Myelitis
•! Infective
o! Mumps, measles, EBV, HIV
o! Mycoplasma, syphilis, TB
•! Neoplastic - Carcinomatosis meningitis
•! Nutritional – B12
•! Demyelinating – MS
o! I would like to complete the examination
!! PR for saddle anaesthesia and a lax anal tone, although I did notice that he is not on ant
urinary catheter or diapers
!! Back for any scars or bony tenderness
!! ULs (if sensory level is at or above the ULs)
3.! Dorsal column loss
o! With loss of proprioception involving the first MTPJ and ankle joints
o! As well as loss of vibration sense up to the level of the knees
o! The sensation to pinprick is intact and there are/no cerebellar signs
o! The possible etiologies include
!! Spinocerebellar degeneration
!! Multiple sclerosis
!! Subacute combined degeneration of the cord
!! Taboparesis
o! Hence I would like to complete the examination by
!! Features of MS
!! Features of SACD – pallor, splenomegaly
!! Argyll Robertson pupils
4.! Upper limbs
o! Intact cerebellar and sensation to pinprick, proprioception and vibration (sensation may be lost)
o! I would like to examine the upper limbs for
!! LMN signs, wasted hands
•! Syringomyelia, cervical myelopathy, MND
!! Inverted supinator jerks
•! C5-6 lesion
!! UMN signs
•! Bilateral strokes, high cervical myelopathy

5.! Absent ankle jerks +/- knee jerks


o! Fasciculations – MND
o! Cerebellar – SCD (FA)
o! Dorsal column loss – SACD, Taboparesis
139
o! Conus medullaris lesion
o! Combined pathologies
!! Cord compression and pre-existing peripheral neuropathy
•! DM and Cx myelopathy
•! Alcohol and Cord compression
!! Cx and lumbar myelopathies
6.! Others – Cerebral palsy, parasagittal falx meningioma

Questions

What is spinocerebellar ataxia?


•! A inherited disorders with multiple subtypes >20
•! Cs by cerebellar and spinal degeneration, slowly progressive with atrophy of cerebellum
•! Ataxia
o! Acquired (see cerebellar)
o! Hereditary
!! AD
•! Some involve trinucleotide repeats with high penetrance and anticipation
!! AR (FA)
!! X-linked

What is Friedreich Ataxia?


•! Hereditary ataxia, autosomal recessive, Ch 9, trinucleotide repeat
•! Cs by
o! Symptomatic - during childhood and teenagers, inability to walk by 20s; onset <25
o! Physical
!! pes cavus, distal wasting, spastic LL (pyramidal weakness), cerebellar signs, dorsal
column loss (marked loss of cells in the dorsal root ganglion) and absent knee and ankle
reflexes (degeneration of peripheral nerves)
!! Scoliosis
•! Request for
o! Spine - scoliosis
o! CVS – HOCM
o! Fundoscopy – optic atrophy
o! Urinalysis for glycosuria
o! IQ – intellectual deterioration
•! Differential diagnosis for spasticity, cerebellar and dorsal column loss is Multiple sclerosis; note that
multiple sclerosis usually has increased reflexes
•! Forme fruste of this condition is pes cavus or hammer toes without other signs
•! Dx – Harding’s criteria
o! Clinical as above
o! Ix – small or absent sensory nerve action potentials, increased motor conduction velocity
•! Other well know conditions for spinocerebellar degeneration
o! Refsum’s disease (elevated serum phytanic acid from defective lipid alpha oxidase – optic
atrophy, retinitis pigmentosa, cardiomyopathy, ichthyosis and ataxia)
o! Olivopontocerebellar degeneration
o! Machado-Joseph disease (CAG repeats)

What is Multiple sclerosis?


•! See History Taking Notes

What is subacute combined degeneration of the cord?


•! Progressive disorder due to Vit B12 deficiency with degeneration of the corticospinal tracts
(pyramids/lateral columns) and dorsal column of the spinal cord
140
•! It also affects
o! Nervous system – CNs, peripheral neuropathy, transverse myelitis, dementia, psychiatric
o! Pernicious anaemia
•! Typically patient feels paraesthesia
•! B12 deficiency
o! Low intake, Vegan
o! Impaired absorption
!! Stomach
•! Gastrectomy
•! Pernicious anemia
o! Anti –parietal cell antibody
o! Anti IF antibody
!! Small intestines
•! Ileal disease (Crohn’s disease), coeliac’s disease
•! Bacterial overgrowth
•! Fish tapeworm
•! Colchicine
•! Ix
o! FBC – macrocytic anaemia
o! B12 and folate
o! Anti IF Ab and Anti Parietal cell Ab
o! Schilling test
!! IM B12 to saturate all B12 protein binding sites
!! PO Co-B12
!! If >10% excretion, normal
!! If <10%, abnormal
•! Repeat with oral IF
o! If normal, means IF deficiency
o! If abnormal, Rx with antibiotics
!! If normal = bacterial overgrowth
!! If abnormal = terminal ileal disease
o! Screen with OGD if pernicious anemia, with higher risk of Gastric Ca
o! MRI of spinal cord – degeneration of the lateral and dorsal columns
•! Rx with parenteral B12 or PO B12 high doses
•! Px – may recover if treated early; otherwise damage is usually permanent

What is taboparesis?
•! Cs
o! Lightning pains – electric shocks in the limbs, throat, stomach or rectum
o! Physical
!! Spasticity with dorsal column loss (high stoppage gait), absent ankle jerks
!! Charcot’s joints, trophic ulcers
!! LL before ULs, rarely ULs involved first (= cervical tabes)
!! Incontinence and constipation
!! Argyll-Robertson pupils
•! Due to lewitic disease, neurosyphilis of which:
o! Acute syphilitic meningitis
o! Acute transverse myelitis
o! Meningovascular disease (stroke in young patient, CN abnormalities)
o! Tabes dorsalis
!! 3 stages
•! Pre-ataxia
•! Ataxia

141
•! Paresis (= taboparesis)
o! Taboparesis
o! Generalised paralysis of the insane (GPI)/Dementia paralytica
!! Chronic progressive frontoparietal meningoencephalitis with atrophy
!! Dementia which classically progresses to grandeur and delusions
!! Trombone tremor (hands, lips and tongue)
o! Gummata in the CNS
•! Caused by spirochetal infection, Treponema pallidum
•! Stages
o! Primary – painless chancre
o! Secondary – maculopapular rash, acute syphilitic meningitis
o! Tertiary
!! Neurosyphilis – obliterative small vessel endarteritis, affecting the vasa vasorum
!! Cardio syphilis
!! Gummatous syphilis
o! Quaternary
!! Fulminant anergic necrotising encephalitis in HIV patients
•! Ix with VDRL/RPR, TPHA/FTA
o! VDRL
!! Non-specific
!! False positive (EBV, malaria, SLE, RA, pregnancy, non syphilitic treponemal infection)
!! Titre use to monitor treatment and reinfection
o! TPHA/FTA
!! Specific
!! Once positive, will remain positive even after treatment
•! Rx
o! Symptomatic treatment
!! Lightning pains – analgesia, TCAs, carbamazepine
!! Ataxia – PT/OT
!! Bladder – avoid anticholinergics, self catheterisation
o! Penicillin
!! Beware the Jarisch Herxheimer reaction
•! From toxins released from killed spirochaetes
•! Starts 3-4 hrs and peaks at 6-8 hrs
•! Fever, HR, RR, myalgia, lethargy
•! Rx with steroids 1 day before and with salicylates

What is motor neuron disease?


•! Cs
o! Onset >40 years old usually, progressive
o! Cs by presence of UMN and LMN of a single spinal segment, with motor dysfn involving at least
2 limbs or 1 limb and bulbar
o! No sensory deficit, no cerebellar, no extrapyramidal, no ocular involvement
•! Types
o! Bulbar/pseudobulbar palsy (25%)
o! Amyotrophic lateral sclerosis (50%)
!! UMNs and LMNs
!! UMN from degeneration of the lateral corticospinal tracts and Betz cells
!! LMNs from degeneration of CN nuclei or anterior horn cell
o! Progressive muscular atrophy (25%)
!! Severe muscular dystrophy with distal wasting
!! Preserved deep tendon reflexes
!! Due to lesions in the anterior horn cells
o! Primary lateral sclerosis (rare)
142
!! UMN progressing to LMN
•! Dy/Dx
o! Cervical cord compression
o! Syphilitic amyotrophy
o! Anterior horn cell disease
!! Polio
!! Spinal muscular atrophy
•! No definitive Ix test
•! Mx
o! Counselling and support
o! Symptomatic
!! Dysphagia – NG, PEG
!! PT, OT
!! Antidepressants
o! Riluzole – glutamate antagonist
o! Rx complications
!! Pneumonia, UTI
!! Pressure sores
!! DVT
•! Poor Px – progressive with median survival of 3-5 years

What are the clinical features of cervical myelopathy?


!! Symptoms exceeds signs and spasticity exceeds weakness
!! Cs
!! Motor
!! look for wasting and weakness of C5 and C6 level as this is commonest area for spondylosis
involvement
!! passive abduction of the little finger (myelopathy hand sign)
!! spasticity of LL
!! Inverted supinator jerk
!! Sensory – often no sensory loss but may have pseudoathetosis with loss of proprioception and
vibration

What causes fasciculation?


•! Sign of LMN disease with denervation
•! Spontaneous firing of large motor units formed by branching fibres of surviving axons striving to
innervate the muscles that have lost their nerve supply

What is Lhermitte’s phenomenon?


!! Tingling or electric sensation that passes down the spine with flexion of the neck
!! Causes includes
!! Multiple sclerosis
!! Cervical myelopathy (spondylosis and tumor) – also has similar sensation on extension of the neck
i.e. reversed Lhermitte’s sign
!! SACD

What are the various manoeuvres to elicit Babinski’s sign?


!! Chaddock’s manoeuvre (dorsolateral aspect of the sole of feet)
!! Oppenheim’s sign (pressure applied on the inner border of the tibia)
!! Gordon’s reflex (pinching the Achilles’ tendon)

143
Peripheral Neuropathy
(Think: Sensory, motor, or mixed. Are nerves palpable nerves?)

Presentation 1
•! Sir, this patient has predominantly sensory peripheral neuropathy as evidenced by
o! Loss of sensation to pinprick and light touch and
o! Impairment of vibration and joint position sense
o! In a stocking distribution
•! The motor system is intact; I did not notice any
o! Wasting or fasciculations of the lower limb muscles
o! Tone and reflexes are normal with downgoing plantars
o! Power is normal
OR

•! Sir, this patient has mixed motor and sensory peripheral neuropathy as evidenced by
o! Loss of sensation to pinprick and light touch and
o! Impairment of vibration and joint position sense
o! In a stocking distribution
•! Associated with
o! Wasting and fasciculations of the lower limb muscles
o! Reduced tone and reflexes with downgoing plantars
o! Diminished power of 4 in the lower limb muscles especially affecting plantarflexion, dorsiflexion
and flexion and extension of the knees bilaterally
AND

•! There is presence of
o! Loss of hair on the lower half of the legs bilaterally
o! No charcot joints
•! The most likely underlying aetiology is diabetes mellitus as I noticed
o! Presence of diabetic dermopathy
o! I screened for other possible causes:
!! No thickened nerves or hypopigmentation patch (leprosy)
!! Parotidomegaly, dupytren (chronic ethanol ingestion)
!! Not sallow (uremia)
!! Not pale (B12 deficiency)
!! Not cachexic and no clubbing of toes (paraneoplastic)
!! No symmetrical deforming polyarthropathy (RA)
!! No clinical features of acromegaly, hypothyroidism

•! I would like to complete the examination


o! Gait (if not done) to look for high steppage gait (sensory ataxia)
o! Upper limbs for distal sensory impairment although I noticed that there is no obvious wasting of
the hands
o! Urine dipstick for glycosuria (DM)
o! Ask history
!! Drug history – INH, nitrofurantoin, phenytoin, chloroquine, penicillamine, vincristine,
cyclosporine A
!! Chronic ethanol ingestion

Presentation 2
•! Sir this patient has predominantly motor neuropathy as evidenced by
o! Wasting and fasciculations of the lower limb muscles
o! Reduced tone and reflexes
144
o! With diminished power of 4 affecting knee flexion and extension as well as plantar and
dorsiflexion
•! Sensation is intact with normal pinprick sensation, vibration sense and propioception.
•! The most likely aetiology in this patient is
o! diabetes mellitus as I noticed presence of diabetic dermopathy in the lower limbs
o! Other possible aetiologies for a predominantly motor peripheral neuropathy
!! Drugs – cyclosporine A, Gold, penicillamine
!! Pb, Hg
!! Metabolic – DM and AIP
!! Infectious/Inflammatory – HIV, GBS, Amyloid, sarcoid
!! PAN
!! HSMN type 1

Questions

What are the causes of peripheral neuropathy (mixed, sensory and motor)?
•! DAMIT BICH
o! Drugs
!! INH, nitrofurantoin, chloroquine
!! Penicillamine, gold, cyclosporin A, phenytoin
!! vincristine, cisplatinum
o! Alcohol, Arsenic(Mees, raindrop pigmentation), Pb(wrists and Pb lines in gums), Hg
o! Metabolic – DM, Uremia, AIP
o! Infectious - Leprosy, HIV, botulism, diphtheria
o! Inflammatory – GBS (look for facial diplegia), CIDP
o! Tumor – paraproteinemia, paraneoplastic (Ca Lung), Hodgkin’s
o! B12, B6 and B1
o! Infiltrative – Amyloid (look for thickened nerves and autonomic), sarcoid
o! Immunological – PAN, SLE, RA
o! Congenital – HMSN, Refsum’s disease, porphyria
o! Cryptogenic
o! Hormonal – Acromegaly, hypothyroidism, hyperthyroidism
o! POEMS (Polyneuropathy, Organomegaly, Endocrinoapthy, Monoclonal gammopathy, Skin
changes – a/w osteosclerotic myeloma)
(NB: DM can be sensory, motor or mixed)

What are the causes of a painful peripheral neuropathy? (DAB, CAP)


•! DM, Alcohol, B12 deficiency
•! Carcinoma, porphyria, Arsenic

What are the causes of thickened peripheral nerves?


•! Median nerve (wrist), ulna nerve (elbow), common peroneal nerve (head of fibula), Greater auricular
nerve (neck)
•! CHAOS
o! CIDP
o! HMSN
o! Acromegaly, Amyloid
o! Others
!! LS DNR – Leprosy, sarcoid, DM, Dejerine Sotta disease (hypertrophic peripheral
neuropathy), NF, Refsum’s disease (retinitis pigmentosa, optic atrophy, cerebellar and
deafness, cardiomyopathy ad ichthyosis)

145
What are the causes of mononeuritis multiplex (separate involvement of more than one peripheral or cranial
nerve by the same disease)?
•! Endocrine
o! DM, Hypertension, Acromegaly
•! AI
o! RA, SLE, PAN, Sjogren, Churg-Strauss, Wegener’s
•! Infection
o! Leprosy, Lyme, HIV
•! Infiltrative
o! Amyloid, sarcoid
•! Carcinomatosis

What are the types of neuropathy in DM?


•! Symmetrical sensory neuropathy (glove and stocking)
•! Predominantly motor, asymmetrical (diabetic amyotrophy)
•! Mixed motor and sensory peripheral neuropathy
•! Mononeuropathy
•! Mononeuritis multiplex
•! Autonomic neuropathy

What are the neurological complications of alcohol?


•! Wernicke’s (confusion, ophthalmoplegia, cerebellar, neuropathy)
•! Korsakoff’s psychosis (recent memory loss and confabulation)
•! Cerebellar degeneration
•! Central pontine myelinosis
•! Epilepsy
•! Myopathy and rhabdomyolysis
•! Peripheral neuropathy

Proximal Myopathy

Examination

!! On detecting proximal weakness, proceed to test sensation to rule out neuropathic weakness; skip the
cerebellar; waddling gait
!! If unilateral proximal weakness, think of diabetic amyotrophy (a/w pain and sensory impairment)
!! Check the ULs
!! Acromegaly, Cushing’s
!! Dupuytren contracture
!! Dermatomyositis/Polymyositis
!! Proximal weakness
!! Check the Face
!! Eyes for MG
!! Cushing’s, Acromegaly, Thyroid
!! Parotids

146
Presentation

Sir this patient has proximal weakness of the upper and lower limbs that is due to proximal myopathy.

There is presence of weakness with a power of 4 on the upper and lower limb girdle muscles. I was able to
overcome his abduction of the arms and he has difficulty standing from a sitting/squatting position. There is
presence of a waddling gait.

There is no evidence of any sensory involvement. There is also no fatigability.

There were no features of


!! Dermatomyositis/polymyositis
!! Acromegaly/Cushing’s/Thyroid
!! Chronic ethanol ingestion – Dupuytren and parotidomegaly
!! Sarcoid – lupus pernio
!! Cancer – cachexia, clubbed

I would like to take a drug history


!! Cholesterol lowering drugs
!! Corticosteroids
!! Cyclosporine A
!! Chloroquine

The possible etiologies include:


!! Congenital
!! Duchenne’s
!! Sex linked, young male child, onset 3-4yrs
!! Pseudohypertrophy of the calves
!! Proximal weakness – Gower’s sign, usually cannot ambulate by 15yrs
!! No facial involvement
!! Low IQ
!! Dilated cardiomyopathy

!! Becker’s
!! Similar but less severe to Duchenne
!! Can ambulate beyond 15 years
!! Usually onset 5-15 but maybe 3rd/4th decades
!! Majority survive to 4th/5th decades
!! Dx – Western blot of muscle biopsy – abnormal/reduced dystrophin
!! Limb Girdle
!! AR, 10-30 yrs old, progressive with severe disability in 20yrs
!! Shoulder and pelvic girdle affected
!! Deltoids are spared initially – pseudohypertrophy
!! Biceps and brachioradialis are involved late
!! Hip flexors and glutei are weak
!! Early wasting of medial quads and tibialis anterior with lateral quads and calves being pseudo
hypertrophied
!! Face is never involved and normal IQ and lifespan
!! Normal muscle enzymes
!! Fascioscapular and oculopharyngeal – see ULs
!! Acquired
!! P – Polymyositis/Dermatomyositis, polymyalgia rheumatica
!! A – Alcohol
!! C – Cancer
147
!! H – HIV
!! E – Endocrine (Acromegaly, Cushing’s, Thyroid), ESRF
!! M – Mitochondrial myopathy (Myopathy, External ophthalmoplegia, red ragged fibres and lactic
acidemia), McArdle’s syndrome (weakness after exercise)
!! P – Periodic Paralysis
!! O – osteomalacia
!! D – Drugs
!! S - Sarcoid

Brown-Sequard Syndrome

Presentation

Sir, this patient has a Brow- Sequard syndrome with a right hemisection of the cord at level T10. This is
because
!! Monoparesis of the right lower limb
!! Reduced vibration and proprioception on the same side
!! Loss of pain sensation on the contralateral limb below the level of T10
!! Loss of pain sensation on the right T10 dermatome

There are no complications of


!! DVT
!! Pressure sores
!! Charcot’s
!! Incontinence

Functionally, he has difficulty with ambulation as I notice a wheel chair at his bed side.

The causes for a hemisection of the spinal cord at the level of T10 on the right are
!! Trauma
!! PID
!! Spondylosis
!! Tumor
!! Abscess
!! Multiple sclerosis

Therefore, I would like to complete the examination


!! Examining the back
!! Per rectal examination for lax anal tone
!! Multiple sclerosis – INO, optic atrophy with RAPD

148
Footdrop

Approach

•! Bilateral
•! LMN
•! Peripheral neuropathy (see peripheral neuropathy)
•! UMN
•! Cord lesion
•! Unilateral
•! Once dorsiflexion impaired
•! Check eversion (Common peroneal nerve = dorsiflex and eversion)
•! Check inversion and plantarflex = posterior tibial nerve
•! If foot drop and inversion and eversion is lost with normal plantarflexion, then L5 nerve root
•! If all gone = posterior tibial+common peroneal, sciatic nerve or plexus/roots
•! Knee flexion intact
•! Go to sensory
•! Peripheral neuropathy
•! Common peroneal nerve palsy (sensory loss over dorsum of the foot)
•! Determine if common peroneal nerve or
•! Deep branch only or
•! The superficial branch only
•! If knee flexion weak, test hip abduction and internal rotation and intact
•! Go to sensory
•! Sciatic nerve
•! If hip abduction and internal rotation is weak
•! Go to sensory
•! Nil = anterior horn cell
•! L4 and L5 dermatome = plexus or root
•! Once site is located, go for the cause
•! Note walking aids

Questions

•! Common peroneal nerve palsy (L4 and L5)


•! Anatomy
•! the sciatic nerve divides at the popliteal fossa into the tibial and common peroneal nerves
•! The posterior tibial nerves effects plantar flexion and inversion of the foot
•! The common peroneal nerves winds round the neck of the fibula, covered by s/c tissue and skin
only and prone to extrinsic compression
•! It then divides into the
•! Superficial branch: foot everters and sensation to lateral calves and dorsum of the foot
•! Deep branch : toe dorsiflexors and dorsiflexion of the ankle and sensation to the first
interdigital web space
•! Therefore wasting of the peroneous and anterior tibialis muscles; weakness of dorsiflexion of
the foot and eversion; foot drop and high steppage gait and loss of sensory over the lateral
aspect of the calf and dorsum of the foot
•! Causes of mononeuropathy (3 Sx and 3 Medical causes)
•! Trauma
•! Surgical
•! Compression at the neck of the fibula (habitual leg crossing, cast, brace)
•! Infection – Leprosy
149
•! Inflammatory – CIDP
•! Ischaemic - Vasculitis
•! Part of mononeuritis multiplex (Endo, AI, infection, infiltrative and cancer)
•! Ix = NCT and EMG
•! Mx
•! PT/OT – 90 degrees splint at night
•! Sx – for severed nerve or excision of ganglion
•! Sciatic nerve (L4 L5 S1 S2)
•! Weakness of the knee flexion also
•! Knee jerk is intact but ankle jerks affected and plantar response absent (for common peroneal nerve,
all reflexes intact)
•! L5 nerve root
•! Weakness of hip abduction and internal rotation as well as loss of foot inversion (cf with common
peroneal nerve)

Hemiparesis/Hemiplegia

Examination

“This man has a left hemiparesis, please examine him”


•! For this, examine the UL and LL
•! Locate
o! Brainstem
!! Weber’s syndrome (III and contralateral hemiplegia)
!! Millard-Gubler (VI and VII and contralateral hemiplegia; usually a/w contralateral
loss of proprioception and light touch as the medial lemniscal damage)
o! Subcortical – lacunar; a/w UMN VII
!! Pure motor (50%)
!! Pure sensory (5%)
!! Mixed motor and sensory (35%)
!! Ataxic hemiparesis (10%)
!! Dysarthria clumsy hand syndrome (rare)
o! Cortical signs
!! Do abbreviated version
!! Gaze preference, sensory and visual neglect, hemianopia and dysphasia if dominant
lobe involved
•! Causes or risk factors
o! Pulse, Carotid bruit, murmur
o! Dyslipidaemia stigmata (xanthelasma, xanthomas, thickened TA)
o! DM dermopathy
o! Tar stains
o! Bruising, telangiectasia
•! Function and complications
o! Upper limb
o! Gait
o! Pressure sores, NG, urinary catheter
•! Request
o! BP
o! Urine dipstick
o! Fundoscopy for papilledema (to rule out SOL which is a possible differential)

150
Presentation

Sir, this patient has got a left hemiparesis as evidenced by


•! State the UL and LL findings
•! State the level of the lesion and justify as above
•! Mentioned the causes as above
•! Mention the functional status and complications

Questions

What are your differential diagnoses?


•! Vascular
o! Ischaemic (80%)
!! Intracranial thrombosis
!! Extracranial embolism – heart, extracranial arteries, paradoxical
!! Lacunar strokes – small vessel disease from DM or hypt as a result of lipohyalinosis
!! Dissection
o! Haemorrhagic (Intracerebral, SDH, SAH)
•! Space occupying lesion
•! Infective – abscess, meningoencephalitis
•! Seizures
•! Toxic-metabolic – Hypoglycaemia, HypoNa

What are the 4 neuroanatomic stroke syndromes?


•! Anterior cerebral artery - affect frontal lobe function, producing altered mental status, impaired
judgment, contralateral lower extremity weakness and hypoesthesia, and gait apraxia.
•! Middle cerebral artery (MCA) - contralateral hemiparesis, contralateral hypoesthesia, ipsilateral
hemianopsia (blindness in one half of the visual field), and gaze preference toward the side of the
lesion. Agnosia is common, and receptive or expressive aphasia may result if the lesion occurs in the
dominant hemisphere. Since the MCA supplies the upper extremity motor strip, weakness of the arm
and face is usually worse than that of the lower limb.
•! Posterior cerebral artery occlusions affect vision and thought, producing homonymous hemianopsia,
cortical blindness, visual agnosia, altered mental status, and impaired memory.
•! Vertebrobasilar artery occlusions are notoriously difficult to detect because they cause a wide variety
of cranial nerve, cerebellar, and brainstem deficits. These include vertigo, nystagmus, diplopia, visual
field deficits, dysphagia, dysarthria, facial hypoesthesia, syncope, and ataxia. Loss of pain and
temperature sensation occurs on the ipsilateral face and contralateral body. In contrast, anterior
strokes produce findings on one side of the body only.

How would you investigate?


•! Confirm the diagnosis
o! Imaging – CT or MRI if posterior stroke (Diffusion-weighted imaging)
•! For diagnosis and type
•! For complications eg hydrocephalus
•! Blood Ix
o! FBC – polycythemia
o! Coagulation profile
o! Biochemical – HypoNa
•! ECG – AF, MI (60% a/w with AF or MI)
•! CXR – Enlarged mediastinum suspicious of a dissection
•! 2D echo (cardioembolic course)
•! Carotid ultrasound scan (significant stenosis - >70%) and transcranial doppler
•! Young patient – young stroke work up (10)
151
o! ANA, dsDNA, ESR
o! Protein C, S
o! Anti Thrombin III
o! Factor V leiden or APC resistance
o! Anticardiolopin IgM/IgG
o! Homocystine
o! VDRL

What are the limitations of CT brain?


•! Unable to visualise the posterior fossa structures such as the brainstem and the cerebellum
•! Maybe normal up to 6 hours of onset
o! After 6 hours – hypodense area
o! Early signs on CT (5) – loss of grey-white differentiation, insular ribbon sign, sulcal
asymmetry, hyperdense MCA sign and obscuration of the LN
•! MRI – Diffusion weighted imaging which has a high sensitivity – looked for hyperintense signal

How would you manage?


•! Multidisciplinary approach
•! Education and counselling
•! PT/OT and ST – speech and swallowing, caregiver training, prevention of bed sores
•! Medications
o! Antiplatelets (Aspirin, persantin, Clopidogrel, Ticlid)
o! Anticoagulation
•! Correct risk factors
o! Hypertension
o! Hyperlipidaemia
o! Diabetes mellitus
•! Surgical
o! Intracranial bleeds
o! Hydrocephalus

How would you manage the patient acutely?


•! Airway, breathing and circulation
•! Control BP if bleed otherwise allow high BP in ischaemic stroke up to 220/120
•! Treat fever
•! Control of blood sugar
•! Determine if bleed or ischemic
•! If ischaemic stroke, assess for possibility of reperfusion therapy ie National Institute of Health Stroke
Scale (NIHSS) using alteplase ie recombinant tissue plasminogen activator; within 3 hours and
important to note inclusion and exclusion criteria.
•! Treat complications
o! Seizures
o! Raised intracranial pressure (Hyperventilation, elevate the head, mannitol)

152
Gait Assessment

“Examine this patient’s gait”


•! Procedure
o! Stand and Rhomberg’s
o! Walk and turn and return
o! Heel to toe
o! Heel walk
o! Tip toe
o! Squat
•! Look for the obvious
o! Ankylosing spondylitis
o! Chorea
o! Hemiplegic gait
o! Antalgic gait
•! Small paces
o! With stooped posture and paucity of arm swing
!! Parkinsonian
o! With upright gait, normal armswing
!! March a petit pas
!! Diffuse cerebrovascular disease
•! Feet separation
o! Broad
!! Cerebellar
•! Unilateral or bilateral
!! + high stepping = Sensory ataxia
•! Peripheral neuropathy or dorsal column loss
o! Crossing over
!! Scissoring gait
!! Spastic – cerebral palsy, MS, cord compression
•! High stepping with normal feet separation
o! Unilateral or bilateral footdrop
•! Pelvis rotating
o! Waddling gait
!! Proximal myopathy or congenital dislocation of the hips
•! Apraxic gait – disjointed
o! Frontal lobe – CVA, SOL, hydrocephalus
•! Bizarre in consistent = functional gait, Huntington’s chorea

153
Parkinsonism

Examination

•! Introduce
•! Mask like facies, monotonous speech, dyskinesias
•! Upper limbs
•! Resting tremors which disappears with use
•! Bradykinesia (thumb to finger, rotate wrist and “twinkle stars”
•! Leadpipe rigidity and cogwheeling
•! Acute dystonia or alien limb syndrome
•! Pronator drift and cerebellar signs
•! Palmomental reflex, grasp reflex
•! Face
•! Eye movements, vertical Doll’s if vertical gaze impaired
•! Close eyes for blepharospasm
•! Feel for seborrhea
•! Look for KF rings
•! Count 1-20
•! Unbutton shirt, write, cap a pen, comb his hair
•! Gait – typical parkinsonian gait; also rule out gait apraxia
•! Request
•! Speech if not done
•! Swallowing
•! Handwriting
•! Postural BP
•! AMT

Presentation

Sir, this elderly gentleman has Parkinsonism with mask like, expressionless facies. He has asymmetrical
resting tremor of the right hand with characteristic pill rolling movements of the thumb that disappears with
use of the hand. There is also presence of bradykinesia with leadpipe rigidity at the elbows and cogwheeling
at the wrist. Movement of the contralateral upper limb accentuates these features.

There is presence of seborrhea and Myerson’s sign or the glabella tap sign is positive.

He has difficulty initiating his gait and has a stooped posture associated with shuffling gait with festination
and lack of normal arm swing. He also turns in numbers. His gait is not apraxic and he is not on any urinary
catheter to suggest NPH.

Functionally he is able to walk unaided and can perform keyturning movements and unbutton his short
unaided.

There is no evidence of dyskinesias which can result as a result of L-dopa therapy.

He dose not have features suggesting presence of Parkinson-plus syndrome. There is no evidence of
Progressive Supranuclear Palsy such as impairment of the vertical gaze, blepharospasm or frontal lobe signs
such as palmomental reflex and the grasp reflex. There are also no cerebellar signs to suggest multisystem
atrophy. There is also no evidence of corticobasal ganglia degeneration such as dystonic arm or alien limb
syndrome.

154
In summary, this patient has Parkinsonism most likely due to Parkinson’s disease and relative preservation
of his function; there is no evidence of dyskinesia currently to suggest side effects of L-dopa therapy.

Questions

What is Parkinson’s disease?


It is a progressive neurodegenerative disorder associated with degeneration of the dopaminergic
nigrostriatal neurons.
Dx clinically with 2 out of 3 signs comprising of resting tremors (3-5Hz), bradykinesia and
rigidity. The 4th sign of postural instability occurs later in the course of the disease.

What are the causes of Parkinsonism?


7.! Parkinson’s disease
8.! Parkinson-plus syndromes
9.! Drugs (Neuroleptics, antiemetics, MPTP- 1 methyl 4 phenyl 1,2,3,6 tetrahydropyridine)
10.!Anoxic brain damage ( Post cardiac arrest, Manganese, CO)
11.!Post encephalitis ( encephalitis lethargica or von Economo’s disease)
12.!Tumor such as giant frontal meningioma

What are the pathologic findings in Parkinson’s disease?


•! Loss of pigmented dopaminergic neurons in the substantia nigra
•! Presence of Lewy Bodies (eosinophilic cytoplasmic inclusions)

What are the Parkinson-plus syndromes?


•! Progressive supranuclear palsy (most common) (frontal lobe) (3)
•! Vertical gaze palsy
•! Downgaze affected first, then upgaze, then horizontal
•! Can be overcome by vertical Doll’s
•! Other features such as blepharospam and slow pursuit or saccadic eye movements
•! Postural instability and axial rigidity with falls early in the course of the disease
•! Frontal lobe signs
•! Multiple sytem atrophy (Cerebellar)
•! MSA-P = Parkinsonism features
•! MSA-C = Cerebellar features
•! Features (3)
•! Cerebellar signs
•! Autonomic features – orthostatic hypotension, urinary dysfn and erectile dysfn
•! Corticospinal signs – hyperreflexia and extensor plantar response
•! Corticobasalganglionic degeneration (frontoparietal lobe)
•! 2 features
•! Limb apraxia or alien limb syndrome
•! Dystonia
•! Parkinsonism-dementia-ALS complex
•! Diffuse Lewy Body disease (Parkinsonism, dementia and neuropsychiatry)

What is the significance of diagnosing Parkinson Plus syndrome?


•! Poorer prognosis
•! Poor response to L-dopa therapy

What are the features that suggest that patient may have Parkinson plus syndromes?
•! Early onset of dementia
•! Presence of hallucination or psychosis
•! Early onset of postural instability
155
•! Truncal symptoms more prominent than appendicular symptoms
•! Marked symmetry of signs early in the stage of the disease
•! Lack of response to levo-dopa therapy in the early stage of the disease
•! Presence of symptoms and signs suggestive of Parkinson-plus syndromes.

What are the stages of Parkinson’s disease?


Staged via the Hohen and Yahr staging system comprising of 5 stages:
•! Stage 1 – symptoms and signs unilateral and mild
•! Stage 2 – Bilateral and minimal disability
•! Stage 3 – Generalised dysfunction with sig bradykinesia and gait impairment
•! Stage 4 – Rigid and bradykinesia, severe symptoms with limited walking
•! Stage 5 – Completely invalid and requires nursing care

How would you investigate?


•! Brain scan to rule out
•! NPH
•! Multi-infarct syndromes
•! Frontal meningiomas
•! Parkinson-plus syndrome
•! MSA – atrophy of the cerebellum, brainstem
•! PSP – Atrophy and hyperintensity of the midbrain and red nucleus
•! CBGD – Frontoparietal cortical atrophy
•! If patient is young, ie<50, rule Wilson’s disease
•! Slit-lamp examination
•! Serum ceruloplasmin and 24Hr Urinary Copper

How would you manage?


•! Multidisciplinary approach
•! Education and counselling, PT/OT/ST
•! Medications
•! Dopamine agonist, eg pramipexole or ropinirole
•! Early in the course of the disease or younger patients
•! Delay onset of motor fluctuations and dyskinesias
•! Nausea, orthostatic hypotension, hallucinations or somnolence, edema
•! L-dopa therapy (usually combined with a peripheral decarboxylase inhibitor)
•! Should be used if there is disability
•! Never used in patients with melanoma
•! Peak dose dyskinesia and end of dose rigidity
•! Nausea, orthostatic hypotension, hallucinations
•! Tremors – anticholinergic (dry eyes and mouth, urinary retention, arrhythmia), e.g. arcane or
benztropine.
•! Rigidity – beside Dopa and D agonist, can use MAO-B inhibitors or amantadine (cognitive side
effects)
•! COMT inhibitors (diarrhea and hepatotoxicity)
•! Surgical
•! Lesion surgery – thalomotomy (tremors) and pallidotomy (for all features)
•! Deep brain stimulation (for all features especially tremors)

How would you manage dyskinesia?


•! Peak dose dyskinesia
•! Reduce the dose and increased frequency
156
•! Add on COMT inhibitors i.e. entacapone which increases half life of L-dopa therapy
•! Initiating with dopamine agonist and adding on L-dopa therapy resulted in less motor fluctuations
•! End of dose
•! Increase dose, frequency
•! Switch to CR formulations
•! Add DA, MAO-B inhibitors, COMT inhibitors

What is the prognosis?


•! Chronic
•! Progressive
•! Some will have mild while other will have severe symptoms
•! Some will be troubled mainly with tremors, other by rigidity and bradykinesia

What is the abbreviated mental test?


•! Use as a screening for possible dementia in the elderly
•! A score of less than 6 warrants further assessment
•! Includes
•! Age
•! DOB
•! Remember this address – 42 West Street
•! Time (nearest hour)
•! Year
•! Recognition of 2 persons
•! Place
•! Prime Minister of UK
•! First Year of WW1 (1914-1918)
•! Count backwards 20 to 1

157
SHORT CASES : EYE

Diabetic Retinopathy

Presentations (Can still use the older classification)

Sir, this patient has:

(a)Type and location

1.! Background diabetic retinopathy affecting the inferior/superior temporal/nasal quadrants of the retina
(usually seen in the posterior pole, ie area between the superior and inferior temporal quadrants) as
evidenced by
a.! Microaneurysms
b.! Dot, blot or flame shaped haemorrhages
c.! Hard exudates
2.! Preproliferative diabetic retinopathy affecting the inferior/superior temporal/nasal quadrants of the
retina
a.! Cotton-wool spots
b.! Venous dilatations, beading, looping or segmentation
3.! Proliferative retinopathy diabetic retinopathy
a.! Neovascularisation
i.! At the disc
ii.! Affecting the inferior/superior nasal/temporal quadrants of the retina
4.! Diabetic maculopathy
a.! Circinate formatiuon of hard exudates at or near the macula
b.! Macular edema (cannot be seen by direct ophthalmoscopy)

(b)Treatment for (3) and (4)

1.! Focal photocoagulation scars


2.! Panretinal or scatter photocoagulation scars
3.! Macular photocoagulation scars

(c)Complications (for proliferative)

1.! Vitreous haemorrhages


2.! Fibrosis with traction retinal detachment
3.! Optic atrophy (for all)

(d)Associations (mention if present)

1.! Xanthelesma
2.! Cataracts
3.! Hypertensive changes
4.! Robeosis irdis

Questions
What are microaneurysms?
•! They are well-defined red dots seen in the superficial retinal layers which represents outpouching of
the retinal capillaries; earliest sign of diabetic retinopathy
•! Can also be seen in

158
o! Hypertensive retinopathy
o! Collagen vascular disease
o! Severe anaemia
o! Dysproteinaemia

What are flame-shaped haemorrhages?


•! Superficial bleed shaped by nerve fibres into a fan shape which points towards the disc

What are dot and blot haemorrhages?


•! Formed as a result of rupture of microaneurysms with bleeding into the deep layer of the retina

How do you differentiate between dot haemorrhages and microaneurysms?


•! This is difficult and differences includes:
•! Microaneurysms are well defined and last for months to years whereas dot haemorrhages tend to
be have an irregular outline and disappears within a few days
•! Fluoroscein angiography of which microaneurysms are hyperfluoroscent whereas dot
haemorrhages are hypofluoroscent

What are hard exudates?


•! These are minute, yellow, well defined deposits of lipo-protein and lipid-laden macrophages

What are cotton-wool spots?


•! Build up of axoplasmic material due to interrupted flow caused by ischaemia from capillary occlusion
in the retinal nerve fibre layer

What are IRMAs?


•! It stands for intraretinal microvascular abnormalities. They are remodelled capillary beds without
proliferative changes and are collateral vessels that do not leak on fluoroscein angiography. Usually
found on the borders of non-perfused retina

What is neovascularisation?
•! Formation of abnormal new vessels on the retinal surface and at the optic disc as a result of ischaemia
•! These are fragile and tend to bleed into the vitreous leading to vitreous haemorrhages and fibrous
tissue formation with resultant traction retinal detachment

What is clinically significant macular edema?


•! Thickening of the retina at or within 500 microns of the centre of the macula
•! Areas of thickening 1 disc area or larger, any part of which is within 1 disc diameter of the centre of
the macula
•! Hard exudates at or within 500 microns of the center of the macula, if associated with thickening of
the adjacent retina

What is the pathogenesis of diabetic retinopathy?


•! Earliest stages are characterised by increased vascular permeability, leading to fluid accumulation in
the retina (seen by leakage of fluoroscein dye into the vitreous humor)
•! Later there is vascular closure causing retinal ischaemia leading to neovascularisation of the retina
•! These new vessels are prone to complications of vitreous haemorrhages, fibro-proliferative changes,
retinal detachment and neovascular glaucoma

How can diabetes mellitus affect the eye?


•! Eyelids – xanthelasma (association)

159
•! Extraocular – mononeuritis multiplex, diabetic third (spares the pupils and associated with headache;
resolves within 3 months) or sixth nerve palsies
•! Anterior chamber – neovascular/rubeotic glaucoma
•! Iris – rubeosis irdis
•! Pupils – Argyll Robertson pupil, RAPD
•! Lens – cataracts(higher incidence and occurs at a younger age), refractor errors (occurs due to
fluctuation in the blood sugar level especially when starting treatment; it is a benign condition)
•! Vitreous body - haemorrhages
•! Retina – DM eye changes, hypertensive, CRAO, lipaemia retinalis
•! Optic nerve – optic atrophy, ischaemic papillitis
•! Orbit – mucormycosis

How can patients present clinically?


•! NPDR – asymptomatic
•! PDR
o! Asymptomatic
o! Reduced VA or blindness as a result of complications
•! CSME
o! Asymptomatic
o! Reduced in VA
o! Paracentral scotoma
o! Decrease in central vision

What conditions can cause blindness in diabetic eye disease?


•! Macular edema
•! Retinal detachment
•! Vitreous haemorrhages
•! 3.6% of type 1 and 1.6% of type 2 were legally blind according to the WESDR

When should we screen for diabetic eye disease?


•! Type 1 DM – within 3-5 years of Dx of DM
•! Type 2 DM – at diagnosis
•! Pregnancy with pre-existing DM – prior conception and first trimester

How should we screen for diabetic eye disease?


•! Fundal photography
•! Indirect ophthalmoscopy with slit-lamp biomicroscopy
•! Direct ophthalmoscopy through dilated pupils

How frequent should patient’s be followed up?


•! No retinoapthy – annually
•! NPDR
o! Mild and no retinal edema – 6 to 12 monthly
o! Presence of retinal edema – 4 to 6 monthly
o! Macula affected or severe – 1-4 monthly
•! PDR – 1 to 4 monthly

How soon must you refer a patient with diabetic eye disease to the ophthalomologist?
•! All patients with DM retinopathy needs a referral to an ophthalmologist
•! Immediately (1 day)
o! Sudden loss of vision

160
o! Retinal detachment
•! Urgently (within 1 week)
o! Neovasculariastion
o! Pre-retinal or vitreous haemorrhages
o! Rubeosis irdis
•! Soon (within 4 weeks)
o! Pre-proliferative changes
o! Macular diabetic changes
o! Unexplained drop in VA

What are the risk factors for diabetic eye disease?


•! Poor glucose control
o! WESDR (Wisconsin Epidemiologic study of DR)
o! DCCT (Diabetes Control and Complications trial)
•! Hypertension
o! UKPDS (UK prospective diabetic study)
o! Aim < 130/80 mmHg
•! Hyperlidaemia
•! Renal disease – aggressive treatment of renal disease may slow progression of DM retinopathy and
prevent neovascular glaucoma
•! Cigarette smoking
•! Duration of diabetes (non-modifiable)
o! After 20yrs, nearly all patients with type 1 and 60% of type 2 will have DR
•! Pregnancy (non-modifiable)

How do you manage patient’s with diabetic eye disease?


•! Management of the diabetic eye includes:
o! Prompt referral to the ophthalomologist (see above)
o! Macular edema – focal or grid macular laser
o! NPDR – none or consider scatter laser if severe
o! PDR – scatter laser
o! Vitreous surgery
!! Non-resorbing vitreous opacities
!! Traction retinal detachment threatening or involving the macula
!! Progressive fibro-proliferative diabetic retinopathy
!! Combined rhegmatogenous and traction retinal detachment
•! Pay attention to
o! Glycaemic control
o! Blood pressure
o! Quit smoking
o! Screen for other DM complications especially DM renal disease
o! Control hyperlidaemia
•! Engage patient
o! Education
o! Importance of regular follow up

How would you manage a patient who requires laser therapy but has cataracts?
•! If fundal visibility permits, laser treatment administered prior to cataract surgery
•! If not, cataract surgery followed by prompt laser treatment

How effective is laser photocoagulation?


•! For visual salvage in maculopathy

161
o! Effective in 50-60% of cases
•! For reduction of already formed abnormal new vessels on the retina
o! It can abolish new vessels in up to 80% of patients with PDR and follow up showed that
disease had stabilised or cured
o! In pan-retinal photocoagulation, it reduces the ischaemic and hypoxic retina, reducing
angiogenic factors and neovascularisation
o! It can alsobe used to treat microaneurysms
•! There may be loss of peripheral vision

Hypertensive Retinopathy

Presentation

Sir, this patient has got grade 4 hypertensive retinopathy as evidenced by


•! Presence of narrowed, tortous and irregular retinal arteries with increased light reflex, demonstrating the
silver/copper wiring
•! There is arterio-venous nipping
•! With evidence of flame shaped and blot haemorrhages and cotton wool spots and hard exudates,
especially around the macula, forming a macular star
•! Associated with papilloedema

I did not notice any concomitant features of diabetic retinopathy. However the patient does have xanthelesma.
There are no signs of chronic renal failure such as sallow appearance. Patient does not have any cushingoid
or acromegalic or polycythaemic features which are conditions associated with hypertension.

I would like to complete my examination by taking the blood pressure of this patient as well as examine his
•! Cardiovascular system
•! RR/RF delay – coarctation of the aorta (state if young)
•! Evidenced of LVH
•! S4 if BP>180/110 (state if grade 3 or 4 changes noted)
•! Abdominal examination for renal bruit (renal artery stenosis) or ballotable kidneys (polycystic
kidney disease) or palpable adrenal masses
•! Urine dipstick for proteinuria, casts, glycosuria
•! CNS for signs of previous CVA

Questions

How do you grade hypertensive retinopathy?


•! Keith Wagner Classification
•! Grade 1 – Arteriolar narrowing, tortousity, irregular calibre with copper/silver wiring
•! Grade 2 – Arteriovenous nipping
•! Grade 3 – flame-shaped and blot haemorrhages, cotton wool spots and hard exudates
•! Grade 4 – Papilloedema
•! Clinical features and prognosis of grade 3 and 4 are the same
•!
Explain their physical appearance?
•! Young patients, retinal arterioles react to hypertension via constriction, hence arteriolar constriction or
narrowing
•! In older patients, there is arteriosclerosis hence irregular calibre. Also the thickened walls shows a
widening of the normal light reflex, giving the blood column a copper appearance (copper wiring) or
silver appearance (silver wiring)

162
•! At arteriovenous crossing, the thickened arteriolar walls displace and constricts the veins, resulting in
AV nipping
•! The arteriolar may be damaged by necrosis leading to flame shaped haemorrhages, cotton wool spots
caused by microinfarcts, as well as retinal edema
•! Chronic retinal edema at the macula results in hard exudates radiating from the macular, ie macular star
•! Finally papilloedema results

How do patients normally present?


•! Normal vision
•! Except when there is associated macular involvement

What are the causes for retinal haemorrhages?


•! Diabetic retinopathy
•! Hypertensive retinopathy
•! CRVO
•! Severe anaemia, leukaemia

What are the causes for cotton wool spots?


•! DM preproliferative retinopathy
•! Hypertensive retinopathy
•! Anaemias, leukaemias
•! HIV, infective endocarditis

What are the causes for hard exudates?


•! Diabetic retinopathy
•! Hypertensive retinopathy

What are the causes of hypertension?


•! 90% are essential hypertension
•! 10% are secondary
o! Renal causes – Chronic renal disease, polycystic kidneys, renal artery stenosis
o! Endocrine – Cushing’s, Acromegaly, Phaechromocytoma, Conn’s, hyperparathyroidism
o! Others – Coaractation, OCP usage, pre-eclampsia, polycythaemia

How would you investigate a patient with hypertension?


•! Urea and electrolytes
•! Urine for protein, glucose and cast
•! Fasting lipids and glucose
•! ECG, CXR

If indicated clinically, ie
•! Young hypertensive <50
•! Requiring >2 antihypertensive
•! Sudden deterioration in control of BP
•! Features suggestive of secondary causes on clinical examination

How would you manage?


•! Lifestyle- Exercise, eat healthily, stop smoking
•! Pharmocotherapy

163
Optic atrophy

Examination

•! Fundus
o! Papilloedema
o! Optic cup (Glaucoma)
o! DM changes
o! Retinitis pigmentosa
o! Attenuated arterioles and veins as in CRAO
•! Eye
o! RAPD (MS)
o! Argyll-Robertson pupil
o! INO (MS)
o! Nystagmus (MS and FA)
•! Head
o! Tender temporal arteritis
o! Pb lines in the gums
o! Paget’s facies
•! Hands
o! Cerebellar
o! AF

Presentation

Sir, this patient has unilateral/bilateral optic atrophy. On examination of his fundus, I did not detect any
presence of papilloedema, deep optic cup, DM changes or RP. The vessels are also not attenuated.

I would like to complete the examination by testing patient’s visual acuity and visual fields as well as for
features of MS with RAPD, INO, nystagmus/cerebellar signs, palpate the temporal artery and examine the
gums for Pb lines and pulse for AF

Questions

What are your differential diagnoses?


•! Unilateral
o! Demyelinating disease
o! Compression (Tumor, aneurysm, Paget’s)
o! Glaucoma
o! Ischaemic
!! Thromboembolic
!! Vasculitis – temporal arteritis, tertiary syphilis
•! Bilateral
o! Toxic
!! Nicotine, alcohol
!! Drugs (ethambutol, chloroquine, methanol, Pb, Arsenic)
o! Metabolic
!! B12 deficiency, B1 and B6
!! Diabetes mellitus
o! Hereditary
!! FA
!! Leber’s (mitochondrial dz with pt mutations)

164
!! DIDMOAD (DI, DM, Optic atrophy, Deafness) – rare recessive
o! Others
!! sec to papilloedema
!! sec to retinitis pigmentosa

How do patients present?


o! Reduction in visual acuity

What would you find on visual filed testing?


o! Central scotoma

What is Leber’s optic atrophy or Leber’s hereditary optic neuropathy (LHON)?


o! Hereditary condition affecting males
o! Progressive visual loss
o! Onset form second decade onwards
o! Mitochondrial disease with point mutations

What investigations would you do?


o! Electroretinography and pattern evoked visual response
o! Blood test
!! Blood glucose
!! ESR
!! VDRL
!! B12 levels
o! Imaging
!! SXR – enlarged sella turcica – pit tumor
!! CT scan – SOL
!! MRI – demyelinating disease
o! (History for toxic and hereditary causes)

Papilloedema

Examination
•! On noticing papilloedema
•! Attempt to identify the different stages of papilloedema present
•! Increase in venous calibre and tortusity
•! Optic cup pinker with disappearance of vessels over the disc
•! Disc is suffused and slightly elevated with blurring of margins; optic cup is filled and presence of
haemorrhages around the disc
•! Look at the retina
•! Features of hypertension (flame-shaped haemorrhages, cotton wool spots and hard exudates)
•! Features of CRVO (heamorrhages)
•! Severe anaemia (haemorrhages)
•! Check for
•! Pallor (severe anaemia)
•! Obvious proptosis
•! Grave’s ophthalmopathy
•! Cavernous sinus thrombosis
•! Spectacles (for hypermetropia)
•! Requests to examine the other eye if told to examine one eye only (bilateral papilloedema vs Foster-
Kennedy syndrome)

165
•! Requests
•! VA
•! Visual fields
•! Color testing
•! Pupillary reflex (may not be possible if dilated)
•! Eye movements
•! Pain on eye movements
•! VI nerve palsy
•! Palpate the temporal region if elderly for tenderness (temporal arteritis)
•! Blood pressure

Presentation
Sir, this patient has papilloedema affecting his right eye as evidenced by a suffused and slightly
elevated optic disc associated with blurring of the disc margins with filling in of the optic cup and dilated
tortuos veins.
There was no evidenced of hypertensive retinopathy such as silverwiring of the blood vessels, arterio-
venous nipping, flamed-shaped haemorrhages or exudates. I could not detect any haemorrhages on the retina
to suggest severe aneamia or CRVO.
I noticed that there was no conjunctival pallor and no obvious proptosis of the eye.
I would like to complete my examination by examining the other eye for features of papilloedema or
optic atrophy; checking his blood pressure; testing his VA and VF and asking him about color vision loss;
eye movements for VI nerve palsy and pain on eye movement as well as RAPD.

Questions
What are the differential diagnoses of optic nerve swelling?
•! Papilloedema
•! Papillitis
•! Ischaemic optic neuropathy
•! Pseudopapilloedema
•! Hypermetropia (margins is blurred)
•! Drusen (yellowish-white deposits at the optic disc)
•! Myelinated nerve fibres
•! Bergmeister’s papilla (whitish elevation of the center of disc; common, seen in all ages, races and
equal sex distribution)

How do you differentiate beteween papilloedema and pseudopilloedema?


•! Fundal fluroscein angiography

What are the differences between papilloedema and papillitis?

Papilloedema Papillitis
VA Preserved Reduced
VF Enlargement of blind spot; Central scotoma
loss of peripheral vision
Color testing Normal Loss of red
Pupillary reflex Not affected RAPD
Eye movements No pain on movements Pain on movement
Others Bilateral Unilateral
Absent of venous pulsation Venous pulsations present
*Retrobulbar neuritis presents exactly like papillitis without the optic nerve head swelling appearance

166
What are the causes of papilloedema? (6)
•! Space Occupying lesion
•! Malignancy – SOL tumor
•! Abscess
•! Hematoma
•! Hydrocephalus
•! Obstructive (ventricle, aqueduct, outlet to 4th ventricle)
•! Communicative
•! Increased CSF formation (rare – Choriod plexus papilloma)
•! Reduced CSF absorption
•! Meningitis
•! SAH
•! Benign intracranial hypertension
•! Idiopathic
•! Addison’s disease
•! Drugs – Nitrofurantoin, tetracycline, vit A, steroids, OCPs
•! Hypertension
•! CRVO
•! Others(3s)
•! Metabolic
•! Hypoparathyroidsm
•! CO2 retention
•! Graves congestive ophthalomopathy
•! Hamotological
•! Severe anaemia
•! Polycythaemia rubra vera/leukaemia/Multiple myeloma
•! Sagittal/caverbous sinus thrombosis
•! Sarcoid
•! GBS (impaired CSF absorption due to elevated protein content)
•! Paget’s disease, Hurler’s syndrome

What are the stages of papilloedema?


•! Stage 1 – increase in venous size and tortousity with loss of venous pulsation
•! Stage 2 – Optic cup is pinker and less distinct with disappearance of vessels over the disc
•! Stage 3 – Blurring of the disc on the nasal side
•! Stage 4 – Disc is suffused and elevated with blurred margins, optic cup filled in and haemorrhages around
the disc

What is ischaemic optic neuropathy?


•! Ischaemia to optic nerve head from arteriolar sclerosis or temporal arteritis
•! Elderly with sudden visual loss
•! Swollen optic disc

What is Foster-Kennedy Syndrome?


•! Characterised by papilloedema in one eye and optic atrophy in the other eye
•! Caused by a tumor of the frontal lobe, classically subfrontal olfactory groove meningioma
•! Results in optic atrophy due to compression of that optic nerve by the tumor and papilloedema in the
contralateral eye from raised ICP
•! Look for features of NPH and loss of smell

167
What is Benign intracranial hypertension?
•! Dandy’s diagnostic criteria
•! Patient is alert
•! Clinical features of raised ICP
•! No localising signs except VI CN palsy
•! LP opening pressure >20cm H2O and CSF composition normal
•! Normal ventricle size and normal CT head
•! Management
•! Discontinue drugs, weight loss
•! Diuretics, carbonic anhydrase inhibitors
•! Serial LPs or lumbopertoneal shunt
•! Optic nerve fenestration or subtemporal decompression

Central and Branch Retinal Vein Occlusion

Presentation

Central Retinal Vein Occlusion


Sir, this patient has got central retinal vein occlusion affecting the right eye as evidenced by grossly tortuous
and engorged retinal veins, especially near the disc. There are also numerous haemorrhages seen scattered
all over the retina of varying shape and sizes with presence of cotton wool spots (blood and thunder
appearance). There is also papilloedema.

There is no evidence of rubeosis irdis.

The left eye is unaffected but has presence of hypertensive retinopathy grade 2 with silver wiring and
irregular retinal arterioles associated with arteriovenous nipping. (Look for causes for in the other eye eg
hypertensive or diabetic changes)

I would like to complete my examination by


•! VA and Visual fields
•! Checking her BP
•! Urine dipstick for glycosuria
•! Urine for Bence Jones protein for multiple myeloma
•! Look for LNs and hepatosplenomegaly, bruising and purpura for macroglobulinaemia
•! Ask about h/o of glaucoma or use of OCPs (in females of reproductive age group)

Branch Retinal Vein Occlusion


Sir, this patient has a branch retinal vein occlusion affecting the superior/inferior temporal/nasal vein in his
right eye as evidenced by a fan-shaped distribution of retinal flame-shaped haemorrhages with cotton wool
spots which radiate from the arteriovenous crossings. These changes have affected the macula of the right
eye.

There is presence of hypertensive changes of grade 2 with silver wiring and irregular retinal arterioles with
arteriovenous nipping. There is no exudates noted and the disc margins are sharp.

I would like to complete my examination by


•! Checking his VA (normal unless affecting the macula) and VF (nomal or presence of a
quandrantonopic fieled loss)

168
•! Blood pressure
•! Urine dipstick for glycosuria

Questions

What are the vessels that are involved in retinal vein occlusion?
•! CRVO – central retinal vein (occlusion is behind the cribiform plate)
•! BRVO – in front on the cribiform plate
o! Most commonly the superior temporal retinal vein
o! Followed by inferior temporal retinal veins
o! The nasal retinal veins

How do patients present?


•! CRVO
•! Diminished visual acuity – to counting fingers (incomplete loss)
•! Reduced visual fields
•! Floaters
•! Acute secondary glaucoma 3 months later from rubeosis irdis from an ischaemic retina
•! Although retinal changes may improve in weeks, VA fails to be restored
•! BRVO
•! VA normal unless macula affected
•! Can have sudden visual loss if there is vitreous haemorrhage
•! Normal VF or monocular quadrantic field defect
•! Floaters

How do you differentiate between CRVO and papilloedema?


•! CRVO has reduced VA whereas papilloedema affected only in late stages
•! CRVO unilateral involvement in contrast to papilloedema

What are the usual sites of occlusions in BRVO


•! AV crossings
•! Along the main veins (DM)
•! Edges of the optic disc, resulting in hemispheric involvement
•! Peripherally as in sickle cell disease

What are the causes of retinal vein occlusion?


•! CRVO
o! Hypertension
o! Diabetes mellitus
o! Glaucoma
o! Hyperviscosity syndrome
!! Waldenstrom’s macroglobulinaemia
!! Multiple myeloma
!! OCPs
•! BRVO
o! Diabetes mellitus
o! Hypertension
o! Need to differentiate with retinal vasculitis

169
How would you investigate?
•! CRVO
•! BP
•! Fasting blood glucose
•! Screen for glaucoma
•! Screen for multiple myeloma (BJ proteins) and macroglobulinaemia
•! Enquire regarding OCPs and stop with alternate use of contraceptive methods
•! BRVO
o! BP
o! Fasting blood glucose, urine dipstick
o! ESR, CRP and ANA and ANCA for vasculitis

How would you manage this patient?


•! CRVO
o! Identify causes and risk factors and correct or treat them
o! Fluoroscein angiography to identify areas of retinal ischaemia
o! Panretinal photocoagulation to prevent rubeotic glaucoma
•! BRVO
o! Identify and treat causes and risk factors
o! Fluoroscein angiography to identify areas of retinal ischaemia
o! Laser photocoagulation may be necessary to ablate damaged vessels if there is macular edema
or risk of vitreous haemorrhages from abnormal new vessels

What is the prognosis?


•! CRVO
o! Mild to severe
o! Mild cases, asymptomatic
o! In severe cases, markedly reduced VA to counting fingers with complications of
neovascularisation of the retina, rubeosis iridis(1 month) and robeotic glaucoma (3 months)
•! BRVO
o! Mostly asymptomatic
o! Macula affected with VA loss but can recover
o! Occassionally leads to gradual visual loss

Central Retinal Artery Occlusion

Presentation

Sir, this patient has a right sided CRAO on fundoscopy as evidenced by


•! RAPD
•! Pale retina
•! Foveola cherry red spot
•! Attenuated retinal vessels
•! Intra-arterial emboli (10-20%)

I did not notice any laser scar marks to suggest laser panretinal photocoagulation and there are no
complications of rubeosis iridis

170
Complete the examination
•! Visual acuity
•! Pulse (AF), DM dermopathy, xanthelasma, BP
•! Palpate the temporal area for tenderness in GCA

Questions

What is the epidemiology of CRAO?


•! 1 in 10 000
•! Male 2X
•! Unilateral in 99%

How do patients present?


•! Sudden, painless visual loss (counting fingers to light perception)
•! 10% has a history of amaurosis fugax – TIA affecting the retinal artery and lasting for minutes and
classically described as a loss of vision in a curtain descending fashion
•! Significant cardiovascular risk factors

What are the causes?


•! Emboli (Hollenhorst plague = cholesterol embolus within the arteriole)
•! Arteritis (GCA, SLE, PAN)

Why is there a “Cherry red spot”?


•! Ischaemia of the retina at the posterior pole renders the retina pale, white and milky; thus the choroid is
seen through the fovea as a cherry red spot

How would you manage?


•! No effective treatment
o! Within 100mins, severe retinal cell injury
o! Total irreversible damage occurs 4 hrs after onset
•! Within 48 hrs, may attempt
o! Digital massage for at least 15 mins to dislodge any emboli
o! Administration of carbogen therapy (95% O2 and 5% CO2)
o! Medications to lower intraocular pressure (IV acetazolamide)
o! Anterior chamber paracentesis
•! Laser panretinal photocoagulation to reduce risk of progression to neovascular glaucoma
•! Manage cardiovascular risk factors
o! DM, hypt, lipids, smoking
o! Echo and USS carotids
o! Antiplatelets

What are the differential diagnoses for sudden painless loss of vision?
•! CRAO
•! CRVO
•! Retinal detachment
•! Submacular haemorrhage form age-related macular degeneration
•! Vitreous haemorrhage commonly from DM retinopathy

171
Retinitis Pigmentosa

Examination

•! Upon noting the pigmented bony spicules at the eye peripheries


•! Macular edema or bull’s-eye maculopathy
•! Attenuated arterioles
•! Way pallor of the optic disc

•! Cataracts

•! External ophthalmoplegia
•! Ptosis
•! Deafness

•! Hands for polydactyly


•! Sallow appearance for renal impairment
•! Short Stature

Presentation (think in terms of 3s)

Sir, this patient has retinitis pigmentosa as there are bony spiculated pigmentation on the peripheries
of the retina bilaterally.
This is not associated with any macular edema or bull’s eye maculopathy, attenuated arterioles or a
waxy pale optic disc.
I also noted that the patient has cataracts, which maybe associated with retinitis pigmentosa. There
is no ptosis that I noted and the patient is not sallow in appearance and does not have short stature.
I would like to complete the examination by examining his visual field and acuity, enquire about
night blindness and assess eye movements to look for external ophthalmoplegia. I would also liked to screen
for deafness as well as examine his hands for polydactyly and his limbs for spinocerebellar degeneration.

Questions

What is retinitis pigmentosa?


It is a dystrophy of the photoreceptors and pigment epithelium with an incidence of 1 in 4000. Occurs
bilaterally and begins in childhood or young adults with a progressive course.

What is the mode of inheritance?


9% X linked, 16% is AR and 22% is AD; the rest or approx 50% is sporadic.

What are the clinical features of RP?


Commoner rod-cone type
Ring scotoma,
Peripheral visual field loss, tunnel vision
Night blindness
Cone-rod type
Visual acuity loss
Loss of color discrimination
Day vision problems

What are the causes of RP?


Primary where no known cause
Secondary to inflammatory retinitis

172
Associated with other syndromes
!! Usher’s syndrome (RP with hearing loss)
!! Alport’s syndrome (RP with hearing loss and nephritis)
!! Refsum’s disease (RP with deafness, hypertrophic peripheral neuropathy and cerbellar
ataxia- it is a phytanic acid storage disease)
!! Abetalipoproteinaemia (RP, with fat malabsorption and spinocerebellar degeneration)
!! Friederich’s ataxia
!! Kearnes Sayre Syndrome (RP with ext ophthalmoplegia, ptosis and heart block)
!! Laurence-Moon-Biedl Syndrome (RP with short stature, polydactyly, renal dysfn)

What are the associated ocular abnormalities?


Posterior subcapsular cataracts
Myopia
Keratoconus
Open angle glaucoma

How would you Ix?


!! Careful FHx and rule out phenothiazine toxicity
!! Formal visual fields testing, color testing (Ishihara charts) and electroretinogram (ERG)
!! Ix if cause is suspected, eg, ECG for heart block in Kearnes Sayre syndrome, lipids and protein
elctrophoresis for abetalipoproteinaemia, serum phytanic acid for Refsum’s disease

How would you manage?


!! Education and counselling especially genetic counselling
!! Impaired vision training and aids for ADLs and job retraining
!! Medication such as high dose of Vit A which slow the progress of RP by 2% a year and acetazolamide
for complications of cystoid macular edema.

Visual Field Defects

Examination

•! Stem statements
o! Examine visual fields
o! Examine eyes
o! Patient complain of knocking into objects
•! General
o! Acromegaly
o! Hemiparesis
o! Dysphasia
•! Visual fields
o! Introduce
o! Sit about an arm’s length
o! “Can you see my whole face”
o! Test for gross VA – counting fingers (wear spectacles!)
o! Test for gross visual fields using finger movements as well as for visual inattention
o! Patient to cover his right eye with right hand and instructed to look straight into my left eye
o! Test using white hat pin from all quadrants
o! If single eye defect
!! Proceed with fundoscopy
•! BRAO, haemorrhages, chorioretinitis

173
•! Optic atrophy, glaucoma, RP
!! Possibilities
•! Constricted field
o! Chronic papilloedema
o! Chronic glaucoma
o! Retinitis pigmentosa
o! Chorioretinitis
o! Hysteria (visual field does not widen as object is brought further away from
the patient cf to organic cause)
•! Scotoma (red hat pin)
o! Retinal haemorrhage or infarct (paracentral or peripheral scotomas)
!! Does not cross the horizontal midline
o! Optic nerve (pale in atrophy, normal in retrobulbar neuritis and pink and
swollen in papillitis) resulting in central scotomas
!! Compression – tumor, aneurysm, Paget’s
!! Glaucoma
!! Neuritis
•! MS
•! Ischaemic (C/BRAO, syphilis, temporal arteritis and
idiopathic)
•! Toxic (methanol, tobacco, Pb, arsenic)
•! B12 defeiciency
•! Hereditary – Friederich’s ataxia, LHON
•! Secondary to retinitis pigmentosa
•! Altitudinal defects
o! Retina infarcts
o! Ischaemic optic neuropathy
•! Totally blind in one eye
o! Retina
o! Optic nerve
o! If bilateral peripheral field loss
!! Bilateral retinal lesion
!! Bilateral optic nerve lesion
o! If bitemporal defect
!! Upper> lower = inferior chaismal
•! Pituitary tumor
•! Suparsellar meningioma
!! Lower > upper
•! Craniopharyngioma
!! Other causes
•! Aneursym
•! Metastasis
•! glioma
o! If homonymous hemianopia (infarcts, haemorrhages or tumor)
!! Left or right homonymous hemianopia = right or left lesion respectively
!! Incongruous
•! Optic tract
!! Congruous
•! Upper quandrantonopia
o! Temporal lobe
•! Lower quandrantonopia

174
o! Parietal lobe
•! Macula sparing (test with a red hat pin)
o! Occipital cortex
•! No macula sparing
o! Optic radiation
!! Note any DM dermopathy, xanthelasma and AF, hemiparesis

Presentation

Sir this patient has


•! A left/right
o! Eye blindness
!! Unable to perceive light
o! Scotoma
o! Constricted visual field defect
o! Upper/lower, temporal/nasal field
•! Bitemporal hemianopia
o! Mention any acromegalic features
o! Request to screen for hypopituitarism
o! Causes includes … (see above)
o! Investigate
!! Lateral SXR enlarged sella turcica, calcification for cranipharingioma
!! CT or MRI head
!! Formal field perimetry
!! Serum prolactin
•! Left/right, upper/lower homonymous quandrantonopia
•! Left or right homonymous hemianopia, incongruous or congruous, macula sparing
o! Mention obvious signs
!! Hemiparesis
!! Dysphasia (for right homonymous hemianopia)
!! Visual inattention
o! Request for neurological examination for CVA and tumor
o! Look for
!! CVA risk factors – DM dermopathy, xanthelasma, AF
!! Tumor
•! Cachexia, clubbing for metastatic disease
o! Ix
!! CT head
!! Formal field testing, perimetry

175
Other Eye Conditions

Visual Acuity
•! Examine each eye with finger counting
o! If unable to do so, proceed with finger movement and then light perception
o! If able to do so, proceed with Snellen chart
•! Determine unilateral or bilateral, acute or chronic
•! Causes
o! Bilateral Acute – front (methyl alc poisoning) vs back( occipital lobe infarction trauma)
o! Bilateral chronic – glaucoma, cataracts, DM, bilateral nerve damage or compression)
o! Unilateral acute
!! CRVO, CRAO, arteritis, non arteritic isch optic neuritis
!! Retinal detachment
!! Vitreous hemorrhage

Cataracts
•! Causes
o! Systemic
!! Senile cataracts
!! DM
•! In prroly controlled younf type 1 DM, can get snowflakes cataracts
!! Hypoparathyroidism
!! Drugs
•! Steroids (>10mg/day of prednisolone > 1year)
•! Chloroquine
•! Chlorpromazine
o! Local
!! Trauma to the eye
!! Glaucoma
!! Radiation
o! Hereditary
!! Dystrophia myotonica (stellate)
!! Wilson’s diseae (sunflower cataracts)
!! Refsum’s disease

Nystagmus
•! Rule out nystagmus at extremes of gaze which is physiological
•! Obvious type of nystagmus
o! Pendular – congenital, macular disease
o! Rotatory only – central causes
o! Upbeat nystagmus (fast phase upwards)
!! Upper brainstem – MS, stroke, Wernicke’s ( triad of confusion, ophthalmoplegia and
nystagmus, ataxia a/w Korsakoff’s Psy)
o! Downbeat nystagmus
!! Cervicomedullary junction – AC malformation, syringobulbia,MS
o! Ocular bobbing – pontine lesions
•! Jerky nystagmus
o! Occurs at primary gaze (means central)
!! Cerebellar
!! Vestibular (MS or stroke)
o! Occurs on horizontal gaze
!! Multidirectional gaze evoked nystagmus

176
•! Central – cerebellar or vestibular
!! Right or left horizontal gaze evoked nystagmus
•! Central or
•! Peripheral
o! Vestibular neuronitis, Meniere’s
o! Ataxic nystagmus ie INO
NB: To differentiate between central and peripheral, central is sustained and peripheral can be fatigued and
often associated with severe vertigo

Pupillary defects
•! Large pupil
o! Differential diagnoses
!! RAPD
!! III nerve palsy
!! Holmes Adie pupil
•! Unilateral
•! Slow reaction to bright light and incomplete constriction to convergence
•! Young women
•! Reduced or absent reflexes
•! Degeneration of ciliary ganglion
!! Mydriatic drugs
!! Sympathetic overdrive (drugs)
•! Small pupil
o! Argyll Robertson pupil
!! Characteristic
•! Small (2mm), irregular pupils
•! Absent light reflex
•! Intact accommodation reflex
•! Does not dilate with mydiatrics
!! Sign of tertiary syphilis
!! Begins unilaterally and involves both pupils with time (months to years)
!! Pathophysiology unknown
!! Differential diagnoses for light-near dissociation
•! Syphilis
•! DM
•! Pituitary tumors
•! Midbrain lesions
•! Adie’s tonic pupil
•! Dystrophia myotonica
•! Aberrant regeneration of CN III
•! Familial amyloidosis
o! Horner’s syndrome
o! Long Standing Adie’s tonic pupil (initially large pupil)
o! DM
o! Encephailitis
o! Sarcoidosis
o! Lyme’s disease
o! Parinaud’s (triad of psuedo AG pupil, vertical gaze palsy and nystagmus on convergence and
causes include MS, vascular and pinealoma)

177
SHORT CASES : NEURO

Charcot’s Joint

Examination

•! Inspection
•! Skin
•! DM dermopathy, callus, ulcer, amputations, loss of concavity of the foot arch, loss of leg hair,
shiny skin;
•! Leprosy skin changes, i.e. hypopigmented macules
•! Muscle - wasting
•! Bone - Joint involvement and stage of Charcot’s
•! Notice any nearby foot orthoses
•! Palpate
•! Joint for tenderness, osteophytes and crepitations, passive ROM for hypermobility
•! Feel for thickened nerves
•! Feel pulses
•! Sensory testing – pinprick, temperature; vibration and proprioception
•! Motor testing

Presentation

Sir, this patient has a right ankle Charcot’s joint. The right ankle is enlarged and deformed with
crepitus and hypermobility of the joint. It is not warm or tender. There is loss of sensation to pinprick in a
stocking distribution and there is also loss of vibration and proprioception up to the ankles.
In terms of function, I noticed the presence of an ankle-foot orthoses by the patient’s side. There is
no muscle wasting to suggest disuse atrophy.
The most likely aetiology is diabetes mellitus as evidenced by presence of DM dermopathy, with
callus formation over the pressure points of the feet and loss of concavity of the foot arch. There is also loss
of skin hair on the lower limbs and the skin has a shiny appearance. There is no hypoaesthetic,
hypopigmented macules or palpable thickened nerves to suggest leprosy.
To complete the examination
•! Stand the patient for Rhomberg’s sign, walk for functional assessment
•! Check the back for meningocele
•! Check the upper limbs for dissociated sensory loss
•! Examine the eyes for Argyll-Robertson pupils
•! Urine dipstick for glycosuria
•! Check for signs for chronic ethanol ingestion
In summary, this patient has a right ankle Charcot’s joint secondary to diabetes mellitus and able to
ambulate with an ankle foot orthosis.

Questions

What is Charcot’s foot?


It is a chronic, progressive, degenerative, neuropathic arthropathy resulting from a disturbance from
the sensory innervation of the affected joint.

What are the causes of a Charcot’s foot?


•! Diabetes mellitus (toes and ankles)
•! Leprosy
•! Alcoholic neuropathy

178
•! Tabes dorsalis (hips and knees)
•! Myelomeningocele
•! Syringomyelia (upper limbs eg shoulder)
•! Others – HSMN, congenital insensitivity to pain

What are the stages of Charcot’s foot?


•! Atrophic form – usually forefoot with osteolysis of the distal metatarsal; X-ray shows MT resembling a
pencil point
•! Hypertrophic form (mid or rear foot and ankle); Eichenholtz classification system:
•! Stage 0 – Clinical stage with signs and symptoms but no joint deformity yet
•! Stage 1 – Acute (Developmental or fragmentation stage)
•! Periarticular fracture with joint dislocation with unstable deformed foot
•! Tender, red and swollen, mimicking infection (but afebrile, normal TW and good DM control and
no break in skin) or gout
•! Stage 2 – Subacute (Coalescence stage)
•! Resorption of bone debris
•! Stage 3 – Chronic (Reparative stage)
•! Restabilization with fusion of the involved fragments
•! Enlarged and deformed, non tender

What is the pathogenesis of Charcot’s foot?


2 theories:
•! Neurotraumatic – loss of pain sensation and proprioception combined with repetitive and mechanical
trauma to the foot
•! Neurovascular – Autonomically stimulated vascular reflex that causes hyperemia and periarticular
osteopenia with contributory trauma

How would you investigate?


•! Establish the diagnosis
•! In acute stage
•! X-ray – to rule out OM; MRI or Indium scanning for infection
•! FBC for a normal TW; ESR or CRP normal in Charcot’s joint
•! Stage the Charcot’s joint from radiographs
•! Demonstrate loss of protective sensation of the foot
•! Semmes-Weinsten (10-g or 5.07 gauge) monofilament
•! Applied with just enough pressure to bend the monofilament
•! Positive if 4 out of 10 sites affected
•! Establish the cause
•! FPG for DM, VDRL

How would you manage?


•! Education and counselling
•! Early Dx to prevent deformity
•! PT/OT – immobilization with total contact cast initially, paying particular attention to proper foot care
and footwear; with ankle foot orthoses and custom made footwear
•! Medications
•! Symptomatic in the acute stages, bisphosphonates
•! Surgical – in chronic stage eg foot stabilisation procedures, exostosectomy

179
Dystrophia Myotonica

Approach to Congenital Myopathies

1. Duchenne’s, Becker’s
2. Myotonia
•! Dystrophia myotonica (fascioscapular dystrophies can mimic appearance)
•! Congenital myotonia
•! Hereditary paramyotonia
3. Fascioscapulahumeral dystrophies, limb-girdle dystrophies, distal myopathies

Examination

Examine patient’s face or hands (Can be short case of locomotor or in CNS station)
Examine the hands
•! Demonstrate difficulty opening hands after shaking
•! Repeatedly open and close the hands
•! Percussion myotonia of the thenar eminence
•! (proceed with hand examination with function assessment if locomotor station)
•! Demonstrate weakness in the forearms (especially) and hands
•! No sensory loss
•! Loss of reflexes
•! Check the pulse (dysrhythmias, small volume pulse)

Examine the face


•! Myopathic facies
•! Expressionless
•! Triangular facies
•! Wasting of the temporalis, masseter (palpate these muscles when patient clenches teeth)
•! Frontal balding
•! Bilateral ptosis
•! Close his eyes and open
•! Tongue for percussion myotonia
•! Gum hypertrophy from phenytoin toxicity
•! Swan-neck appearance with wasting of the SCM (test for weakness of SCM), weakness of flexion of the
neck
•! Nodular thyroid enlargement

Request
•! Face
•! Cataracts - posterior subcapsular and stellate
•! Assess Speech – slurring due to myotonia of the tongue and pharyngeal muscle
•! Chest examination
•! Gynecomastia
•! Cardiovascular examination – dilated cardiomyopathy (split S1, mitral murmur, low BP and pulse
volume)
•! Testicular atrophy
•! Urine dipstick for diabetes mellitus
•! Lower limbs – bilateral footdrop

180
Presentation

Sir, this patient has got dystrophia myotonica as evidenced by

•! A myopathic facies that is triangular in appearance with an expressionless look. There is wasting of the
facial muscles involving the temporalis and masseter muscles associated with frontal balding and bilateral
ptosis. He had difficulty opening his eyes after firm closure. There was myotonia affecting the tongue.
•! There is also a swan-neck appearance with wasting of the sternocleidomastoid muscles with weakness of
flexion of the neck. On shaking his hand, there was a delay in releasing his grip. In addition, after making
a fist, he was unable to quickly open it especially after doing this repetitively. There was also presence
of percussion myotonia of the thenar eminence. There is presence of proximal myopathy and wasting
with involvement of the forearms and hands. There are also reduced reflexes with no sensory loss
detected. Function is relatively preserved.
•! With regards to complications
•! His pulse is regular at 80 bpm with a small volume pulse suggesting dil CMP
•! There was no gum hypertrophy to suggest chronic phenytoin use.
•! There is nodular thyroid enlargement.
•! I would like to complete my examination by
•! Face
•! Cataracts - posterior subcapsular and stellate
•! Assess Speech – slurring due to myotonia of the tongue and pharyngeal muscle
•! Chest examination
•! Gynecomastia
•! Cardiovascular examination – dilated cardiomyopathy (split S1, mitral murmur, low BP and pulse
volume)
•! Testicular atrophy
•! Urine dipstick for diabetes mellitus
•! Lower limbs – foot drop with high steppage gait (tibial nerves are affected early)

Questions

What are the types of muscular dystrophies you know of?


1. Duchenne’s
•! Sex linked
•! Pseudohypertrophy of the calves or deltoids
•! Gower’s sign, proximal weakness
•! Cardiomyopathy
Becker’s
•! Sex linked
•! Later onset and less severe form of Duchenne’s

2. Limb-girdle
•! Autosomal recessive
•! Shoulder and pelvic girdle affected
•! Third decade
•! Sparing of the face and heart
Fascioscapulohumeral
•! Autosomal dominant
•! Bilateral, symmetrical weakness of the facial and SCM with bilateral ptosis
•! Weakness of the shoulder muscles and later the pelvic girdle muscles

181
3. Dystrophia myotonica
Congenital myotonia
Hereditary paramyotonia

5.! Distal myopathies eg Welander’s myopathy

What is myotonia?
Continued contraction of the muscles after voluntary contraction ceases, followed by impaired
relaxation.

What is dystrophia myotonica?


•! Characteristic clinical appearance with myotonia and weakness with no sensory loss
•! Autosomal dominant with a trinucleotide (AGC) repeat disorder on chromosome 19
•! Anticipation – phenotypic expression worsens with each successive generation
•! Onset in the 3rd or 4th decade
•! Males>females
•! In addition to the characteristic facies and musculoskeletal involvement
•! Intellectual and personality disorder
•! Cataracts – posterior subcapsular cataracts which are stellate type
•! CVM – dilated cardiomyopathy and conduction defects
•! Resp – recurrent infection from weakness of the bronchiolar musculature, hypoventilation and post-
anaesthetic respiratory failure
•! Abdomen – dysmotility and dysphagia
•! Testicular atrophy and gynecomastia
•! Diabetes mellitus
•! Nodular thyroid enlargement

How would you investigate?


•! Confirm Diagnosis
•! EMG – dive bomber pattern ie waxing and waning of the potentials
•! Muscle biopsy shows no inflammatory changes with type 1 fibre atrophy which is characteristic but
not diagnostic
•! DNA analysis
•! Muscle enzymes are normal
•! Screen for Complications
•! FPG – screen for diabetes mellitus
•! ECG – heart blocks, small P, prolonged PR, notched QRS and prolonged QTc
•! CXR – enlarged heart
•! Slit-lamp examination for cataracts

How would you manage?


•! Education, genetic counselling
•! PT/OT – eg foot orthosis for foot drop
•! Medications – phenytoin for myotonia, other anti-myotonic medications such as quinine and
procainamide should be avoided due to aggravation of cardiac conduction defects; however it is the
weakness that causes disability and not myotonia
•! Pacemaker for 3rd degree heart block or symptomatic such as syncope

How would you counsel the patient’s family?


•! Vertical

182
•! Autosomal dominant, children 1 in 2
•! Anticipation
•! DNA analysis is available for some families for prenatal diagnosis
•! Horizontal
•! Screen with clinical examination
•! Slit-lamp examination
•! EMG

What are the other types of myotonia disorders?


•! Myotonia congenita (Oppenheim’s disease)
•! Autosomal dominant or recessive
•! Presence of myotonia without other features of dystrophia myotonica
•! Present at infancy with difficulty feeding with subsequent improvement
•! No weakness and reflexes are preserved
•! Herculean appearance
•! Channelopathies
•! Hereditary paramyotonia
•! Autosomal dominant
•! Cold-induced myotonia

What are your differential diagnoses for dystrophia myotonica?


•! Facies appearance – Facioscapulohumeral dystrophy
•! Autosomal dominant, onset at age 10-40, Chr4, normal lifespan
•! Face – ptosis, difficulty closing eyes, facial weakness and speech impaired
•! Normal IQ
•! Neck – wasted SCM and weakness
•! Shoulder – winging of scapula, weakness of pectoralis, trapezius, biceps and triceps and hypertrophy
of deltoids
•! Occasionally affecting the anterior tibialis
•! Normal CK
•! Proximal weakness – FSH, limb-girdle, prox myopathy causes, MG
•! Limb Girdle dystrophy (see prox myopathy)
•! Distal weakness – Welander’s distal myopathy, nerve problem
•! Myotonia – Congenital myotonia, hereditary paramyotonia

Median Nerve Palsy

Examination

•! Upon suspecting median nerve palsy, rule out ulnar and brachial neuritis
•! Median nerve palsy
•! Motor
•! Wasted thenar eminence
•! Thumb is externally rotated into the plane of the thumb rather than perpendicular
•! Pen-touch test (for abductor pollicis brevis)

•! Oschner clasping test (flexor digitorum superficialis)


•! Flexion of the terminal digit of the thumb (flexor pollicis longus)
•! Flexion of the terminal digit of the index finger (flexor digitorum profundus)

183
•! Sensory
•! Test for reduced sensation in the lateral 31/2 fingers as well as thenar eminence.
•! Exclude ulnar and radial nerve palsy
•! Aetiology
•! Tinel’s sign
•! Look for RA hands
•! Look at the wrist and forearm, elbow, arm and axilla for scars.
•! Test for function
•! Rule out Myxedema and acromegaly

Presentation

Sir, this patient has an isolated unilateral right median nerve palsy with wasting of the right thenar
eminence associated with an externally rotated thumb. There is weakness of abduction of the thumb as
demonstrated by the pen touch test associated with reduced sensation to pinprick in the right lateral 3 1/2
fingers. Oschner’s clasping test is negative and flexion of the terminal phalanx of the thumb and index fingers
are preserved, indicating that the level of the lesion is at the wrist.
There is no ulna or radial nerve palsies.
In terms of aetiology, there is also no evidence of RA of the hands and patient does not have features
of hypothyroidism or acromegaly. Tinel’s sign is negative and there are no scars noted on the right upper
limb.
Both fine and coarse motor functions are intact.
In summary, this patient has a right median nerve palsy at the level of the wrist. Possible aetiologies
includes surgical causes such as compression, trauma or surgery or medical causes such as mononeuritis
multiples, infection, inflammatory and ischaemic causes.

Questions

What is the course and branches of the median nerve?


•! It supplies all the muscles of the forearm except the flexor carpi ulnaris and the ulna half of the
flexor digitorum profundus and LOAF (lateral 2 lumbricals, opponens pollicis, abductor pollicis
brevis and flexor pollicis brevis)
•! Formed by lateral(C5-7) and medial(C8,T1) cords of the brachial plexus
•! Enters the arm closely related to the brachial artery with no branches above the elbow
•! Enters the forearm lateral to the brachialis tendon and in between the pronator teres.
•! Gives off the anterior interosseous nerve
•! Above the wrist, gives off the palmar cutaneous branch
•! Enters the carpal tunnel and supplies LOAF and sensory branch to the lateral 3 ½ fingers.
•! Branches
•! Forearm – flexor carpi radialis, flexor digitorum superficialis (flexion of fingers at the PIPJ),
pronator teres
•! AIN – Flexor pollicis longus (flexion of the DIPJ thumb), flexor digitorum profundus of the
lateral 2 fingers (flexion of at the DIPJ), pronator quadratus
•! Palmar cutaneous (to the thenar eminence)
•! Terminal motor (LOAF) (NB for F for flexion at the MCPJ thumb)

What are the various levels of lesions and the clinical correlation?
•! Wrist
•! Wasting of thenar, ext rotated thumb, pen touch test positive; sensory loss of the lateral 3 ½
fingers
•! Cubital fossa

184
•! Above plus
•! Oschner clasping test positive and failure of flexing the terminal digits of the thumb and index
finger
•! Arm and axilla (same as cubital fossa)
•! (For forearm, depends where the lesion is eg AIN syndrome will affect flexor digitorum
profundus and flexor pollicis longus only)

What are the causes?


•! Trauma
•! Surgical
•! Compression
•! Mononeuritis multiplex
•! Infection – Leprosy
•! Inflammatory – CIDP
•! Ischaemic - Vasculitis

•! Causes of Carpal tunnel syndrome


•! Idiopathic
•! Pregnancy, OCPs
•! Endocrine – Hypothyroidism, Acromegaly
•! Hands – RA, gout, TB tenosynovitis, OA of carpus
•! Amyloidosis, CRF, sarcoidosis

What are the tests to demonstrate a median nerve palsy?


•! Tinel’s sign (percussion)
•! Phalen’s test (flexion at the wrist for 60 s)
•! Hyperextension of the wrist (for 60 s)
•! Tourniquet test (sphygmomanometer for more than 2 mins above systolic)
•! Luthy’s sign – Skinfold does not close tightly around a bottle or cup; secondary to thumb
abduction paresis
•! Durkan’s test – apply direct pressure over the carpal tunnel

What are the other areas of nerve compression?


•! Median nerve (CTS)
•! Ulna nerve (elbow tunnel)
•! Radial nerve (spiral or humeral groove)
•! Meralgia paraesthetica (lateral cut nerve of the thigh at the ing lig)
•! Common peroneal nerve (head of the fibula)
•! Posterior tibial nerve (Tarsal tunnel syndrome)
•! Plantar nerves of the 3rd/4th toes (Morton’s metatarsalgia)

How would you investigate?


o! Blood Ix
o! Imaging – X-rays
o! Nerve conduction test demonstrating slow sensory conduction across the transverse carpal
ligament.

How would you manage?


Education
OT and wrist splint
Medications – treatment of underlying disease, withdrawing OCPs, IA steroid

185
Surgical decompression

What is the prognosis?


o! Neuropraxia with no disruption to the sheath or the axon
!! Recovery complete and rapid (weeks)
o! Axonotmesis with disruption of the axon but an intact Schwann sheath
!! Recovery complete but slower (1mm/day)
o! Neuronotmesis
!! Recovery is incomplete

Radial Nerve Palsy

Examination

•! Suspect this on an apparently normal looking pair of ULs


•! Proceed to examine the ULs as per normal, concentrating on median and ulnar nerve as well as
brachial plexopathy
•! Once radial nerve palsy detected, proceed to look for level
o! Demonstrate weakness of extension at the MCPJ
o! Preservation of IPJ extension (lumbricals and interossei muscles)
o! Weakness of wrist extension
o! (don’t forget to extend wrist before testing grip strength)
o! (don’t forget to test finger abduction and adduction with hands flat on a surface)
o! Test brachioradialis
o! Test triceps muscles, triceps jerk
o! Thumb abduction and Oschner’s clasping test for median screen
o! Finger abduction and Froment’s sign for ulnar screen
o! Look for reduced sensation in the first dorsal interosseous or anatomical snuffbox
o! Inspect the forearm, elbow, humerus and shoulder
o! Check the gums for lead poisoning – blue-black line on the gingival margin
o! Test Function

Presentation

Sir, this patient has got an isolated right radial nerve palsy at the level of the upper third of the humerus
or above.
I say this because of weakness of extension of the fingers at the MCPJ and at the wrist associated
with weakness of the brachioradialis muscle, triceps muscles with weakness of extension at the elbow. In
addition, there is also numbness of the first dorsal interosseous space. There is no evidence of concomitant
ulnar or median nerve palsies.
I did not detect any scars or deformities over the humerus or the axilla. (Mentioned other areas if the
level is lower) There is also no clinical evidence of lead poisoning such as a blue-black line on the gingival
margin.
Possible causes include compression of the right radial nerve such as crutch palsy at the axilla or
Saturday night palsy at the humerus.
I also note that there is presence of a splint for his wrist and finger drop. He is able to perform coarse
and fine motor function.

Questions

•! What is the course of the radial nerve and its branches?


o! C5, 6,7,8, T1 and emerges from the posterior cord of the brachial plexus

186
o! Leaves the axilla and enters the arm between the long head and medial heads of the triceps
and supplies the triceps
o! Spiral groove on the back of the humerus between the lateral and medial heads of the triceps
o! Lower third of the humerus, it pierces the intermuscular septum to enter the anterior
compartment of the arm where it supplies the brachioradialis
o! It gives off a branch supplying the extensor carpi radialis longus
o! At the elbow, ie lateral epicondyle of the humerus, it gives off the posterior interosseous nerve
which supplies all the extensors of the forearms including the abductor pollicis longus and
supinator except the extensor carpi radialis longus
o! The radial nerve continues as the superficial radial nerve which provides sensory innervation
of the posterior aspects of the radial 3 ½ digits.

•! What are the various levels of lesions and what are the correlating clinical features?
o! Axilla eg crutch palsy – All gone including triceps and triceps reflex
o! Humerus
!! Upper third – all is lost
!! Middle third
•! triceps and triceps reflex preserved and brachioradialis and below is lost
•! Saturday night palsy
!! Lower third – triceps and brachioradialis is preserved
o! Elbow
!! Like lower third
!! Only the PIN involved
•! Extensors of the fingers at the MCPJ affected only
•! Wrist drop is not a feature as the extensor carpi radialis longus is intact and
this alone can effect wrist extension
o! Forearm
!! PIN involvement
!! Superfical radial nerve palsy; aka Watenberg syndrome which is an entrapment
syndrome where there is pain and numbness over its distribution of the first web space
dorsally only because of overlap

•! What are the causes?


o! Trauma form accident or surgery
o! Compression or entrapment
o! Part of a mononeuritis multiplex
o! Lead poisoning
o! (for PIN, finger drop can be secondary to synovitis from RA)

•! How would you investigate?


o! Detailed history for the cause
o! X-ray – for fracture, healing callus or tumor
o! EMG and NCT to locate the level of injury and to monitor recovery progress

•! How would you manage?


o! Education and counselling
o! OT and PT with a wrist splint and cock-up splint for finger drops
o! Surgical

•! What’s the prognosis?


o! Neuropraxia with no disruption to the sheath or the axon
!! Recovery complete and rapid (weeks)

187
o! Axonotmesis with disruption of the axon but an intact Schwann sheath
!! Recovery complete but slower (1mm/day)
o! Neuronotmesis
!! Recovery is incomplete

Ulnar Nerve Palsy

Examination

•! Rule out median, radial and brachial neuritis


•! Inspecting
•! Wasting of the muscles of the hands, hypothenar eminence and partial clawing of the 4th and 5th
fingers, sparing of the thenar eminence, ulnar paradox
•! Proceed to tests for finger abduction and Froment’s sign (weakness of the adduction of the thumb)
•! Test finger flexion of the 5th finger for flexor digitorum profundus involvement; test for wrist flexion
at the ulna side and look for the tendon of the flexor carpi ulnaris
•! Rule out median nerve (thenar eminence and ext rot thumb, pen touch test and Oschner clasping test)
and radial nerve
•! Sensory testing in the medial 1 ½ fingers; test T1 sensory loss
•! Examine the wrist and elbows (feel for thickened nerve, wide carrying angle))
•! Function
•! Thickened nerve (cf with Pb for radial and Acromeg etc for median)

Presentation

Sir, this patient has got a isolated left ulnar palsy as evidenced by a left ulnar claw hand with wasting
of the small muscles of the hands with dorsal guttering as well as wasting of the hypothenar eminence. There
is sparing of the thenar eminence.
There is weakness of finger abduction and Froment’s sign is positive. There is preservation of the
flexion of the DIPJ of the 4th and 5th fingers; when the hand is flexed to the ulna side against resistance, the
tendon of the flexor carpi ulnaris is palpable. This is associated with reduced sensation to pinprick in the
medial 1/1/2 fingers. There are no associated median or radial nerve palsies and T1 involvement.
In terms of aetiology, there is a scar at the wrist associated with a marked ulnar claw hand,
demonstrating the ulna paradox. I did not find any signs to suggest leprosy such as thickened nerves,
hypopigmentation patches or finger resorption.
Both coarse and fine motor function of the hand is preserved.
In summary, this patient has a left ulna claw hand due to a traumatic injury to the left wrist.

Questions

What is the anatomical course of the ulnar nerve?


•! It provides motor to all muscles of the hands except the LOAF; flexor carpi ulnaris and flexor digitorum
profundus to the 4th and 5th fingers.
•! Sensory to the ulna 1 ½ fingers
•! Begins from the medial cord of the brachial plexus (C8 and T1)
•! No branches in the arm
•! Enters the forearm via the cubital tunnel (medial epicondyle and the olecranon process) and motor supply
to the flexor carpi ulnaris and ulna half of the flexor digitorum profundus
•! It gives off a sensory branch just above the wrist and enters Guyon’s canal and supplies the sensory
medial 1½ fingers and hypothenar as well as motor to all intrinsic muscles of the hands except LOAF.

188
What is the level of lesions and its clinical correlation?
•! Wrist – Hypothenar eminence wasting, Froment’s positive, weakness of finger abduction, pronounced
claw and loss of sensation
•! Elbow – less pronounced claw and loss of terminal flexion of the DIPJ and loss of flexor carpi ulnaris
tendon on ulna flexion of the wrist

How do you differentiate ulnar nerve palsy vs a T1 lesion?


Motor – wasting of the thenar eminence in addition for T1
Sensory – loss in T1 dermatomal distribution

What is the ulna claw hand?


It refers to the hyperextension of the 4th and 5th MCPJ associated with flexion of the IPJs of the 4th
th
and 5 fingers as a result of ulnar nerve palsy.
It is due to the unopposed long extensors of the 4th and 5th fingers in contrast to the IF and MF which
are counteracted by the lumbricals which are served by the median nerve.

What is the ulnar paradox?


It means that the ulnar claw deformity is more pronounced for lesions distally e.g. at the wrist as
compared to a more proximal lesion e.g. at the elbow.
This is because a more proximal lesion at the elbow also causes weakness of the ulnar half of the
flexor digitorum profundus, resulting in less flexion of the IPJs of the 4th and 5th fingers.

What is Froment’s sign?


Patient is asked to grasp a piece of paper between the thumbs and the lateral aspect of the index finger.
The affected thumb will flex as the adductor pollicis muscles are weak. (Patient is trying to compensate by
using the flexor pollicis longus supplied by median nerve)

What are the causes of an ulnar nerve palsy?


•! Compression or entrapment (Cubital tunnel at the elbow and Guyon’s canal at the wrist)
•! Trauma (Fractures or dislocation – cubitus valgus leads to tardive ulnar nerve palsy)
•! Surgical
•! Mononeuritis multiplex
•! Infection – leprosy
•! Ischaemia – Vasculitis
•! Inflammatory - CIDP

How would you investigate?


•! Blood Ix to rule out DM if no obvious cause
•! X-rays of the elbow and wrist (both must be done to rule out double crush syndrome) (KIV C-spine and
CXR)
•! EMG(axonal degeneration for chronic) and NCT(motor and sensory conduction velocities useful for
recent entrapment as well as chronic) to locate level and monitor

How would you manage?


•! Education and avoidance of resting on elbow
•! OT, PT
•! Medical – NSAIDs and Vit B6
•! Surgical decompression with anterior transposition of the nerve

NB: LOAF – lateral 2 lumbricals, opponens pollicis, abductor pollicis brevis and flexor pollicis brevis

189
Wasted Hands

Unilateral
•! Think of (no myopathy, got brachial plexus)
•! Peripheral nerve (median, ulnar or combined)
•! Mononeuropathy vs peripheral neuropathy (asymmetric involvement)
•! Brachial plexus (trauma, tumor, radiation, Cx rib)
•! C8-T1 root lesions (Cx spondylosis)
•! Anterior Horn Cell (Poliomyelitis)
•! Cervical cord
•! Proceed as:
•! Long case – as per protocol, check also neck and chest
•! Short case
•! On inspection, unilateral wasted hands noted
•! Neurological hand screen
•! Examine for ulnar and median nerve palsies.
•! Check for sensory for nerve vs root (peripheral nerve vs brachial plexus) and no loss (ie anterior
horn cell)
•! Note sensory for ulnar, median and radial
•! Note sensory of peripheral neuropathy
•! Note dermatomal sensory
•! Feel for thickened nerves, look for hypoaesthetic macules, fasciculations
•! Look for scars in the axilla and neck (neck pain, tenderness), Cx rib
•! Check function
•! Requests
•! Palpate for cervical rib and features of Pancoast’s tumor (dullness to percussion, Horner’s
syndrome, hoarseness voice)
•! Check for winging of scapula (for brachial plexus involvement)
•! If brachial plexus
•! Upper vs lower (wasting of muscles of hands) vs complete
•! Surgical(Cx rib, Pancoast) vs medical cause(brachial neuritis)
•! Test for proximal involvement
•! Serratus anterior (winging of scapula on pushing against wall) ie C5,6,7
•! Supraspinatus (abduction of UL from hands by your side position) C5
•! Infraspinatus (elbow flexed and push backwards) C5
•! Rhomboids (hand on hip and push backwards) C4,5,6
•! Reflexes (inverted supinator jerk)

Bilateral

•! Think of
•! Rule out the obvious (hand screen)
•! RA, gouty hands
•! Dystrophia myotonica
•! Levels (got myopathy, maybe brachial plexus if bilateral Cx ribs)
•! Distal myopathy (reflexes normal; rare), dystrophia myotonica
•! Peripheral nerve lesions
•! Combined CTS (see median nerve palsy)
•! Combined ulnar and median nerve
•! Leprosy (resorption, hypoaesthetic macule and thickened nerve)
•! HMSN (look at the feet for pes cavus deformities, thickened nerves)

190
•! Peripheral motor neuropathy
•! (Not likely to be brachial plexus unless bilateral Cx ribs)
•! Nerve roots
•! Cervical spondylosis (inverted supinator jerk, increased jerks for high cervical cord
lesions)
•! Anterior Horn cell (no sensory loss)
•! MND (fasciculations)
•! Poliomyelitis
•! SMA
•! Spinal cord lesions
•! Intramedullary (Syringomyelia – dissociated sensory loss)
•! Extramedullary
•! Request
•! LL – spastic paraparesis ( if suspect Cx cord, MND)
•! Lower cranial nerve (bulbar palsy – if suspect MND or syringomyelia)
•! Proceed as
•! Long case
•! Proceed as per normal
•! Examine or request to examine the neck (pain tenderness and pain on neck movements), chest,
CNs and LLs accordingly
•! Short case
•! Neurological hand screen
•! Median and ulnar nerve testing, and wrist drop( because this is also weak in C8 root lesions)
•! Sensory – peripheral nerve vs neuropathy vs root
•! Check the elbows for thickened nerves
•! Look for fasciculations (peripheral nerve, neuropathy, MND), hypoaesthetic macules
•! Inspect the neck
•! Quick glance at the face (NG tube – bulbar palsy, LLs – HMSN)
•! Check function
•! Request for reflexes, percussion myotonia if deemed appropriate (if suspect Cx cord lesion or
dystrophia myotonica)

Questions

What are the levels and causes?


•! Disuse atrophy (RA hands)
•! Myopathy (distal myopathies or dystrophia myotonica – usually forearms more affected)
•! Peripheral neuropathy - motor (see causes in Neurology segment)
•! Mononeuropathy
•! Surgical, trauma or compression
•! Mononeuritis multiplex, infection, inflammatory and ischaemic
•! Brachial Plexus
•! Surgical, trauma compression (Pancoast’s, Cx rib)
•! Brachial neuritis
•! Nerve root (Disc prolapse)
•! Anterior Horn cell
•! MND, poliomyelitis, SMA
•! Spinal cord
•! Intramedullary
•! Extramedullary

191
How would you Ix?
Blood Ix according to causes as above
Imaging – X-rays, CT or MRI of spine
NCT/EMG

What are the causes of a claw hand?


•! Partial claw
•! Ulnar nerve palsy (See Ulnar nerve)
•! True Claw
•! Non-neurological
•! RA
•! Severe Volkmann’s ischaemic contracture
•! Neurological (5)
•! Combined median and ulnar nerve
•! Leprosy (reflexes present. Pain loss, thickened nerves)
•! Lower brachial plexus ( C7-T1, selective loss of reflexes, pain loss)
•! Poliomyelitis (reflexes selective, pain intact)
•! Syringomyelia (reflexes absent, pain loss)

192
SHORT CASES : RHEUMATOLOGY

Rheumatoid Arthritis

Presentation

Sir, this patient has Rheumatoid arthritis affecting the hands as evidenced by

Presence of symmetrical deforming polyarthropathy


PIPJ/MCPJ
Swan neck, Boutonniere’s, Z-thumb, ulna deviation
Subluxation (MCPJ, dorsal subluxation of the ulna at the carpal joint)
Active arthritis/quiescent

Intrinsic muscle wasting


CTS
Dropped fingers from tendon rupture
Synovial thickening

Vasculitic lesions, nail-fold infarcts


Palmar erythema
No nail changes and skin lesions of Psoriasis
SLE skin changes

Elbows for Rh nodules

Function
Preserved vs impaired
Coarse and fine functions

Treatment
Steroid – atrophied skin, bruisability
Surgical intervention – CTS decompression, tendon release

Requests
Other joint involvement (MTPJ, knees)
Extra articular features of RA

Questions

What are the extra-articular features of RA?


•! Eye
o! Conjunctiva – Keratoconjunctivitis sicca, pallor
o! Sclera – episcleritis, scleritis, scleromalacia perforans
o! Lens – Cataracts from chronic steroid usage
o! Retina – vasculitis, drug induced (Gold, Hydroxychloroquine)
o! Extra-ocular muscles – mononeuritis multiplex, myasthenia sec to penicillamine

•! Respiratory
o! Upper airways – Cricoarytenoid
o! Pleura – pleurisy, effusions
o! Airway – BOOP

193
o! Parenchyma – Pulmonary fibrosis, pneumonitis, PHT ( RA or MTX)
o! Caplan’s, Nodules
•! Neurological
o! Peripheral neuropathy
o! Mononeuritis multiplex
o! Nerve entrapment
o! Cx atlanto-axial subluxation +/- Cx myelopathy
o! Muscle atrophy, proximal myopathy sec to steroids, penicillamine induced myasthenia
•! Abdomen
o! Splenomegaly in Felty’s syndrome

What are the causes of anaemia in RA?


•! Fe deficiency – GI bleed from NSAIDS
•! Megaloblastic anaemia – Pernicious anaemia
•! Anaemia of Chronic disease
•! Hypersplenism from Felty’s Syndrome
•! Aplasia – Gold, Penicillamine

What are the poor prognostic indicators?


•! Insidious onset and high activity at onset
•! Rh nodules or early erosions within 1 year
•! Extra-articular features
•! Persistent activity after 1 year – Active arthritis, ESR
•! High levels of Rh factor and Anti CCP Ab (anti cyclic citrullinated peptide Ab)

What are the criteria for Dx RA (American college of Rheumatology)?


Any 4 of the following:
•! Morning Stiffness for >1 hr duration for > 6 weeks
•! Arthritis of 3 or more joints for > 6 weeks
•! Arthritis of wrists, MTCP, PIPJ
•! Symmetric
•! Rh nodules
•! Rh factor
•! Radiographic changes typical changes including erosions or unequivocal decalcification

How would you investigate this patient?


•! Blood Ix – Rh factor, anti-CCP, ESR, CRP
•! X-rays of the joints – erosions and periarticular osteopenia

How would you manage this patient?


•! Education and counselling
•! Non-pharmacological
o! OT, PT
•! Pharmacological which will depend on the severity
o! Analgesia – NASIDS
o! DMARDS
!! Methotrexate (Check FBC and LFT)
!! Sulphasalazine
!! Hydroxychloroquine
!! Low dose prednisolone
!! Newer agents

194
•! Leflunomide
•! Tacrolimus
•! Cyclosporine A
•! Rapamune (sirolimus)
o! Immunomodulators (biologics)
!! Anti TNF – Etarnacept (FDA approved), infliximab, humira
•! Beware of TB and atypical pneumonia resulting from their use
!! Anti CD20 – Rituximab
•! Surgical

What is Z thumb deformity?


•! Deformity that occurs in RA hands
•! With hyperextension of the first IPJ and fixed flexion and subluxation of the first MCPJ
•! Resulting squaring appearance of the hands

What is Boutonnière’s deformity?


•! Hyperflexion of the PIPJ and hyperextension of the DIPJ
•! Due to rupture of the central slip of the extensor tendon over the PIPJ with imbalance of the flexion
and extension forces of the finger

What is swan neck deformity of the fingers?


•! Hyperextension of the PIPJ and hyperflexion of the DIPJ
•! Due to synovitis of the flexor tendons leading to flexion at the MCPJ with constant effort to extend
the finger; leading to stretching of the collateral ligaments and the volar plate of the PIPJ; intrinsic
muscle balance leads to swan neck deformity

What are the differential diagnoses for deforming polyarthropathy of the hands?
•! Rheumatoid arthritis
•! Psoriatic arthritis of the RA type
•! Jaccoud’s arthropathy which is ulna deviation with subluxation of the 2nd to 5th fingers at the MCPJ
which is voluntarily correctable; initially described in patient’s with Rh fever but now used
synonymously with SLE deforming arthropathy

Gouty Hands

Examination of the Hands – Sequence


•! Tophi, joint deformity
•! Feel joints for active arthritis
•! Palmar erythema, dupytren’s contracture (alcohol), finger pulps for tophi
•! Test function – pincer and grip, coarse and fine
•! Look at the extensor surface and elbows (olecranon bursae)
•! Sallow appearance, dialysis (Renal failure)
•! Pinna or helix of the ear
•! Pleithoric, parotidmegaly, bleeding and hypertrophic gums
•! Look at the feet for joints, deformity, active arthritis, diabetic dermopathy
•! Feel the achilles tendon and infrapetallar region
•! Request
o! Walk patient if feet are involved
o! BP
o! Urine dipstick for glycosuria, hematuria (stones)

195
Presentation

Sir, this patient has chronic tophaceous gout affecting his hands and his feet. On examination of the
hands, there is asymmetrical swelling affecting the small joints of the hands with tophi formation which has
resulted in severe deformity of the hands and feet. I also noticed that these tophi are exuding chalky material.
On palpation, there is no tenderness and joints are not warm to suggest active arthritis. There is wasting of
the intrinsic muscles of the hands. There is also presence of tophi on the extensor aspects of the forearms,
the left olecranon bursae, the right helix/pinna of the ear as well as the small joints of the feet. I looked for
but did not detect any tophi on the achilles tendon or the infrapetallar region.
In terms of function, he is able to perform pincer and handgrip movement and his hand function is
relatively preserved; able to perform door knob turning and cap a pen, as well transfer coins and unbutton
his shirt.
I noticed that the patient is not obese looking, no DM dermopathy or xanthelasma as these are a/w
gout. There is also no evidence of chronic ethanol ingestion such as palmar erythema, dupytren’s contracture
and parotidomegaly. There is no sallow appearance to suggest chronic renal failure. I also did not detect any
conjunctival pallor or suffusion, hypertrophic or bleeding gums and patient is not pleithoric which may
suggest presence of lymphoproliferative disease or polycythaemia. There are no psoriatic skin lesions
I would like to complete the examination by walking to patient to assess function as I noticed that his
feet is affected by gouty arthritis, take his blood pressure as well as a urinalysis to look for glycosuria as well
as hematuria for UA stones and proteinuria for UA nephropathy. A detailed drug history, dietary history and
alcohol consumption.

Questions

What is gout?
Gout is a disorder of purine metabolism, resulting in hyperuricaemia either from
overproduction(75%) or undersecretion of uric acid, resulting in deposition of urate crystals in the joints or
bursae.
Patients typically present with acute monoarthritis of the first MTPJ, with pain swelling and exquisite
tenderness which peaks within hours and lasts for days. It affects the joints of the lower limbs initially in the
majority of patients which includes the MTPJ, ankles and knees. It can also subsequently affects the joints
of the upper limb.

What are the stages of gout?


Acute gouty arthritis
Intercritical period
Chronic tophaceous gout

What does tophi indicate?


Severe, recurrent and chronic gout.

Where are the commonly areas to look for gouty tophi?


Hands, extensor aspect of the forearms, olecranon bursae
Helix if the ears
Toes, Achilles tendons, infrapetaller regions

What are the clinical manifestations of gout?


Asymptomatic hyperuricaemia
Acute arthritis
Chronic, recurrent arthritis
Tophaceous gout
Uric acid nephrolithiasis
Uric acid nephropathy

196
What are the triggering factors of gout?
Alcohol ingestion
Foods – sweetbreads, liver, kidneys and sardines
Drugs – Thiazide diuretics, aspirin, cyclosporine, pyrazinamide and ethambutol
Dehydration and fasting
Surgery, Trauma

What are the causes of gout?


Primary – associated with obesity, diabetes mellitus, hypertension and high TGs
Secondary
Drugs
Chronic ethanol ingestion
Chronic renal failure
Polycythaemia, lymphoproliferative, myeloproliferative
Psoriasis
How would you investigate?
Definitive investigation would be aspiration of the involved joint, looking for
intracellular deposition of needle-shaped crystals that is negatively birefringent under polarised light,
within leukocytes.
They react with nitric acid and NH4OH to give a purple color (Murexide test)

Blood Ix – Uric acid levels which may be normal during an acute attack

X-ray of the joints may show erosive arthropathy from tophi with overhanging
edges associated with punctuate to diffuse calcification.

How would you manage?


Education and counselling, including dietary advice and avoidance of alcohol
PT/OT if tophaceous gout for preservation of function
Manage associated hypertension and diabetes mellitus
Medications – acute attack and prophylaxis
Surgery – rarely for cosmetic reasons, arthroplasty

How would you treat an acute attack?


NSAIDS – Indomethacin (50mg tds)
Colchicine 2 ways:
0.5mg hrly till GI side effects or max of 5 mg, or
0.5mg tds
Intrarticular steroids (triamcinolone 20mg)
Systemic steroids (Prednisolone 30mg OM and tails over 7-10 days)

How would you prophylax against gouty attacks?


Prophylactic agents used are iniated under colchicine cover which includes:
Xanthine oxidase inhibitor
Allopurinol
New agents – Uricase, febuxostat
Uricosuric acid agents
probenecid or sulfinpyrazone
losartan
fenofibrate

197
What are the indications for allopurinol?
Recurrent gouty attacks > 3 times a year
Chronic tophaceous gout
Uric acid nephropathy
Persistently high uric acid level
Conditions that may predispose an individual to gouty attacks, prior to chemotherapy or radiotherapy
which may induce tumor lysis

What are the side effects of allopurinol?


Side effects occur in 3-5%
Rash, diarrhea, drug fever
Leucopenia, thrombocytopenia
Allopurinol hypersensitivity syndrome
Erythematous rash, fever, hepatitis, hypereosinophilia and renal
failure

What are the other crystal arthropathy that you know about?
Pseudogout – Acute arthritis resulting from deposition of calcium pyrophosphate dihydrate crystals
in the joints which are rhomboid shaped positively birefringent crystals under polarised light.
Calcium hydroxyapatite crystals deposition in the large joints such as knees and shoulders, affecting
the elderly.

What are the differential diagnoses?


Septic arthritis
Overlying cellulitis
Trauma

What is your differential diagnosis for chronic tophaceous gout?


Florid tendon xanthomata
Yellow and not chalky
Adherent to tendon and not joint
Does not involve the bursae, ie no olecranon or pinna lesions
No active arthritis

Psoriasis – Locomotor (10% of Psoriasis with Joint involvement)

Presentation

Sir, this patient has psoriatic arthropathy affecting the hands of the
1.! Arthritis mutilans type (bilateral deforming arthropathy, telescoping of the digits)
2.! RA type (symmetrical joint involvement)
3.! OA type (asymmetrical terminal joint involvement)
4.! mono/oligoarticular type
5.! AS type (Sacroilitis, but the syndesmophytes arise from the lateral and anterior surface and not at the
margins unlike AS)

With
1.! Bilateral deforming polyarthropathy, and joint deformities, tender (activity)
2.! sausage shaped fingers, tenosynovitis
3.! wasting dorsal guttering and wasting of the thenar and hypothenar eminence
4.! nails – pitting, onycholysis, subungal hyperkeratosis, discoloration of the nails (80% involvement
with arthropathy)

198
5.! Skin patches – well circumscribed plagues on the extensor surfaces of the elbows and scalp, with
salmon pink hue and silvery scales
6.! surgical scars

Joint function
1.! Impaired or preserved
2.! able to grip and do pincer movement
3.! coarse function – turn a doorknob
4.! fine function – cap a pen, transfer coins, unbutton clothes
5.! able to abduct and internally rotate her shoulder joints which are important for her ADLS

Treatment complications – Steroids for arthritis


Mention no evidence of Gout (as this is associated with Psoriasis)

Complete my examination by
•! examining for other joint involvement
•! Skin – especially scalp, knees, natal cleft, intragluteal folds, submammary folds, Koebner’s
phenomenon
•! Enquire on aggravating factors

Questions

What are the types of skin lesions?


•! Plague
•! Guttate (numerous small papular, hx of streptococcal infection
•! Pustular (localized or generalized, superficial pustules may stud the plagues)
•! Erythrodermic (generalized erythema and scaling which may be life threatening)
•! Inverse psoriasis (plagues evolving in the intertriginous area without typical silvery scales due to
moisture and maceration)

Where are the typical sites of distribution?


•! Extensor surfaces of knees, elbows, scalp, navel, natal cleft, submammary and intragluteal folds

How do you assess severity?


•! Psoriasis Area and Severity Index – area, thickness, redness and scaling
Total score 72 - <10, 10-50, >50 for mild, moderate and severe respectively

What are the types of joint involvement in psoriasis?


•! OA
•! RA
•! AS
•! Oligo/mono
•! Arthritis mutilans

Radiological features of psoriatic arthritis?


•! Periostitis – “fluffy”
•! Destruction of small joints
•! “Pencil in cup appearance”
•! Non –marginal syndesmophytes in AS type

199
What are the unique characteristics of psoriatic lesions?
•! Salmon pink hue with silvery scales
•! Koebner’s - New psoriatic skin lesions at site of cutaneous trauma
•! Moist red surface on removing of scales (Bulkeley’s membrane)
•! Auspitz’s sign – capillary bleeding when silver scales are picked from the plague

What is Koebner’s phenomenon and which other conditions is it seen?


•! New skin lesions at the site of cutaneous trauma
•! Occurs in 30% of patients with psoriasis, usually occurring 10-20 days post-
Trauma, ranges from 3 to 2 years
•! Also occurs in eczema, lichen planus, vitiligo and lichen sclerosus et atrophicus

What is the pathology?


•! Hyperproliferation of the epidermis with inflammation of the dermis and epidermis.

What are the differential diagnoses for onycholysis?


•! Psoriasis
•! Fungal infection
•! Thyrotoxicosis (Plummer’s nails)
•! Lichen Planus

What are the aggravating factors?


•! Emotional stress
•! Alcohol
•! Drugs – beta blockers, ACE inhibitors, Indocid, Lithium & antimalarials
•! Streptococcal infection (classically associated with guttate psoriasis)
•! Injury to the skin – mechanical injury and sunburn

What are the principles of management?


•! Education
•! Avoidance of aggravating factors
•! Topical – WSP, salicyclic, aqueous cream
•! Topical – Topical steroids, coal tar, Dithranol, Calcipotriol (Vit D3 which acts to increase
keratinocytes differentiation as a result of increased extracellular calcium therefore decreased
cellular proliferation and scaling), topical retinoids
•! Systemic – UVB, MTX, Retinoids, systemic steroids, cyclo, tacrolimus and MMF
•! Novel – immunodulators (infliximab, etarnacept)

What is the prognosis?


•! Deforming and erosive in 40%
•! 10% are disabled by the arthropathy

What other joint pathology can patients have especially if disease is active?
•! Gout – because of hyperproliferation

Others
•! 30% have family history
•! Psoriasiform lesions on the fingers, toes, nose and ears – exclude SCC of the Oropharynx,
tracheobronchial tree and esophagus – Bazex syndrome.

200
Osteoarthritis of the Hands

Examination
•! Herbeden’s nodes and Bouchard’s nodes, squaring of the thumb
•! Presence of active arthritis
•! No muscle wasting
•! Tinel’s sign
•! Function
o! ROM
o! Coarse fn
o! Fine fn
•! Establish cause
o! Primary
o! Secondary to Acromegaly, Hemochromatosis
•! Request
o! Examine other joints
!! Knee – varus/valgus deformity, crepitus, wasting of the quadriceps
!! Hips
!! Cervical spondylosis, Lumbar spondylosis
!! Gait (Trendenlenberg’s sign – downward tilting of the pelvis on the affected side)

Presentation

Sir, this middle-age lady has OA of the hands as evidenced by presence of Herbeden’s nodes which are bony
swelling affecting the DIPJ. I did not detect any Bouchard’s nodes but there is presence of squaring of both
hands as a result of subluxation of the first MC. There is no significant muscle wasting with preservation of
function. ROM was good and patient is able to perform coarse fn such as turning a door knob and fine motor
fn such as transferring coins. Tinel’s sign is negative.

I would like to complete the examination by examining other joints for OA in particular
•! Knees
•! Hips
•! Gait for Trendelenberg’s sign
•! Cx and Lx spondylosis

There are no features of acromegaly or hemochromatosis.

I would like to offer the dx of Nodal OA or primary generalized OA with OA of the hands occurring in this
middle aged lady.

Questions

What are Herbeden’s nodes?


•! Bony swellings at the DIPJ in OA

What are Bouchard’s nodes?


•! Bony swelling at the PIPJ in OA

Why is there squaring of the hand?


•! This is due to subluxation of the first MC

201
What are the types of OA?
•! Primary generalized OA aka nodal OA
•! Middle-aged women, Autosomal dominant
•! OA of the DIPJ with Herbeden’s with marked deformity and preservation of fn
•! Also affects the carpometacarpal joint of the thumb, knees and hips
•! Secondary
•! Trauma
•! Inflammatory arthropathies – RA, Septic arthritis, gout
•! Endocrine – Acromegaly, hyperparathyroidism
•! Metabolic – chondrocalcinosis, hemochromatosis
•! Neuropathic joints – DM, Tabes, syringomyelia

How would you Ix?


•! Radiographical
•! Subchondral bone cysts and sclerosis
•! osteophytes
•! narrowed joint space
•! Varus/varus deformity
•! If a synovial aspirate is done to r/o other causes
•! <100wbc/ml

How would you manage?


•! Education and counseling
o! Appropriate footware
o! Weight management
•! PT/OT
•! Pharmocotherapy
o! Glucosamine, chondroitin sulphate, glycosaminoglycans
o! Drugs
!! Analgesia – paracetamol, NSAIDs, Opiods
!! Tetracycline (inhibit enzyme that breaks down cartilage)
!! Diacerin (Anti IL-1)
!! Hydroxychloroquine
o! Intra-articular steroids
o! Visco-supplementation (hyaluronic intra-articular injection)
•! Surgery - Knee and hip replacement

Scleroderma

Seated (hands on pillow if available)

This is a middle-aged lady has got scleroderma as evidenced by


Hands – evidences of sclerodactyly and smooth/shiny/tight/taut shiny skin of the hands which extends
proximal to the MCPJ (double pinch test). There is also digital tip pitting, finger pulp atrophy and
acroosteolysis. There is also presence of Raynaud’s phenomenon, beaking of nails (pseudoclubbing),
atrophic nails, nail fold telangiectasia (especially 4th digit via magnifying glass) and vasculitic rashes at the
finger tips. There is also calcinosis and subcutaneous calcification located fingers, elbows and extensor aspect
of the forearms. This is associated with wasting of the intrinsic muscles of the hands and
vitiligo/hyperpigmentation (salt and pepper appearance).

202
In terms of function, she is able to perform pincer movements and hand grip is good with a power of 5;
however there is limitation of finger extension with flexion contractures and she finds it difficult to
unbutton/button of clothes as well as to perform turning door knob manoeuvres.

There is no proximal myopathy (myositis)

There is also involvement of her face as evidenced by:

Face – Bird – like facies, smooth/shiny/tight/taut skin of her face with difficulty closing her eyes, blotchy
telangiectasia, pinched nose, microstomia, perioral tethering with pseudorhagades. I also noticed that the
patient is cachexic looking; note pallor

Legs – On examination of her legs, I also noticed presence of scleroderma as well as vasculitis, telangiectasia
and ulcerations, vitiligo.

Treatment complications - Steroids

I would like to complete the examination by taking the patient’s blood pressure, urine dipstick, cardiovascular
examination, respiratory examination and abdominal examination and ask her about dysphagia (examine
stools for steatorrhea), Raynaud’s phenomenon as well as dry eyes and mouth.

Questions

What are the criteria for diagnosis of scleroderma?


Major
Proximal scleroderma (affecting MCPJ/MTPJ)
Minor
Sclerodactyly
Digital tip pitting/pulp atrophy
Bibasal pulmonary fibrosis
Require 1 major or 2 minor

What are the types of Scleroderma?


CREST (a/w anticentromere antibodies)
Limited cutaneous (extremities)
Diffuse cutaneous (involving of skin of trunk)
Scleroderma sine scleroderma (systemic complications without skin)

What are the phases of skin changes?


Edematous phase
Dermal phase (induration)
Atrophic phase with contractures

What are the complications?


Lung
Pulmonary fibrosis
Reflux pneumonitis
Pleural effusion
Alveolar cell carcinoma
Cardiovascular
Primary Pulmonary Hypertension
Cor Pulmonale from Pulm fibrosis
Pericarditis, Pericardial effusion

203
Myocardial fibrosis
Abdomen
Esophageal dysmotility
Malabsorption with steatorrhea from dilated second part of
the duodenum resulting in bacterial overgrowth
Kidneys – Renal failure (Malignant hypertension–responsive to ACE -)
Primary Biliary Cirrhosis (Woman – rare)

What are the possible etiologies for anaemia in a patient with scleroderma?
Anaemia of chronic disease
Fe deficiency anaemia from esophagitis
B12 and folate deficient anaemia from malabsorption
Microangiopathic hemolytic anaemia (MAHA)
Aplasia from medications such as MTX

What is Thibierge-Weissenbach syndrome?


Acrosclerosis associated with deposition of calcium in the subcut tissue

What is mixed connective disease?


2 or more - SLE, polymyositis, dermatomyositis, SSc
Antiribonuclear protein antibody (Speckled pattern)

Why must you avoid high dose corticosteroids?


This may precipitate renal crisis.

How would you investigate?


ANA – Speckled pattern
Anti-topoisomerase DNA 1 (Scl-70) for systemic sclerosis
Anti-centromere antibody (CREST)

Outline your management.


Education and support
PT/OT for hand function
Treat symptoms and complications
Raynaud’s – avoid cold, calcium channel blockers, prostacyclin
Esophagitis – PPIs
ACE inhibitors for hypertension
Bosentan (endothelial receptor antagonist – for pulmonary hypertension)
Treatment of disease
Immunosuppressants – Steroids, MTX, Aza, cyclophosphamide
Antifibrotic – penicillamine, interferon

What is the Px?


Male (poor)
Renal involvement (poor)
70% for skin involvement only – 10 yr survival
20% 10 yr survival if kidneys and lung involved

204
Ankylosing Spondylitis

Approach

Stem Statement
1.! Look at this patient (no apparent endocrine, rheumatology or neurological abnormalities)
2.! Examine patient’s gait
3.! Low back pain, chest, back, neck, look at the ceiling
Proceed
1.! Introduce (thank you for letting me exam you, my name is… how do I address you)
2.! Ask if there is pain
3.! Stand the patient, walk him, turn around and return to original position
4.! Touch toes with his fingers
5.! Look left and look right
6.! Touch your chest with your chin

Presentation

Sir, this patient has Ankylosing Spondylitis as evidenced by a stooped, question-mark posture with loss of
lumbar lordosis and a fixed kyphosis with extension of the cervical spine in an attempt to maintain a
horizontal visual gaze. There is also a protuberant abdomen. Spinal movements are restricted as evidenced
by the finger-to-toe test, with limited flexion and lateral movements of the cervical spine.

I would like to complete the examination by doing the “heels, hips and occiput” test and measure the occiput-
to-wall distance, as well as a modified schoeber’s test and chest expansion (5cm).

I would like to look for


•! shoulder and knee joint involvement
•! extra-articular involvement and complications
•! Differential diagnoses
o! Skin – to rule out Psoriasis which is a possible differential diagnosis
o! Abdominal examination to look for signs of inflammatory bowel disease
o! Reiter’s syndrome

Questions

1.! Tell me about AS.


a.! Seronegative spondyloarthropathies, chronic inflammatory arthritis affecting the SI joints
with fusion of the spinal vertebrae, associated with HLA b27
b.! 3rd to 4th decades, males 3X
c.! Symptoms: back pain worse in the morning and with rest and improves with activity
d.! Signs : limited lateral flexion of the lumbar spine is the first sign of spinal involvement
followed by loss of lumbar lordosis
e.! Investigation and management

2.! What are the associated conditions?


a.! Anterior uveitis, iritis
b.! Atlanto-axial subluxation or dislocation
c.! Apical fibrosis
d.! Aortic regurgitation
e.! AV nodal block, arrythmias
f.! Amyloidosis
g.! Achilles tendonitis, plantar fasciitis

205
3.! What is the heels-hips-occiput test?
a.! Ask the patient to place his heels, hips and occiput against a wall all at once
b.! Inability of the occiput to touch the wall
c.! Can measure the wall-occiput distance

4.! What is Schoeber’s test?


a.! Draw a line joining the “dimple of Venus”
b.! 5cm mark below
c.! 10cm mark above
d.! Forward flexion
e.! <5 cm implies limited spinal mobility

5.! Why is there a protuberant abdomen?


a.! This occurs as a result of restricted chest expansion from a fixed spine
b.! Hence resulting in a predominantly diaphragmatic breathing
c.! With resultant protuberant abdomen

6.! What are the other types of conditions that can present with sacroilitis?
a.! Psoriasis
b.! Reiter’s (reactive arthritis)
i.! Can be urogenital (Chlamydia) or gastrointestinal (Shigella, campylobacter,
salmonella)
ii.! Triad of urethritis, arthritis and conjunctivitis
iii.! Cs have circinate balanitis (small shallow painless sores) and keratoderma
blenorrhagica (small hard papules on palms and soles)
c.! Enteropathic arthritis (these are the seronegative spondyloarthropahy which are associated
with HLA B27)

7.! How do you diagnose AS?


a.! Rome or New York Criteria
b.! Based on:
i.! Radiological features of sacroilitis
ii.! Symptoms of back pain (lumbar spine or dorsolumbar junction)
iii.! Physical signs of limited spinal mobility in all 3 planes and chest expansion <2.5cm
8.! How would you investigate?
a.! Imaging – AP views and the SI joints and AP/lat of spinal vertebrae
i.! Early – erosions and sclerosis of the SI joints
ii.! Later – Syndesmophytes in the margins of lumbosacral vertebrae
iii.! Advanced – “bamboo” spine
b.! Blood test (not so important)
i.! Elevated ESR
ii.! HLA B 27 (95% AS is positive but small percentage of B27 positive develop AS)

9.! How would you manage?


a.! Education and counseling
i.! Chronic disease
ii.! Genetic counseling (HLA positive, siblings has 30%)
1.! 50% chance of transferring genes to children
2.! 1/3 of children who has HLA B27 will have AS
3.! Overall risk is 1/6
b.! Non-pharmological
i.! Lifelong regular exercises

206
ii.! Involving the PT and OT
c.! Pharmological
i.! NSAIDs
ii.! MTX, sulphasalazine
iii.! Anti TNF and anti CD-20
d.! Surgical therapy

What are the indications for starting immunomodulators?


•! TNF blocking agents are recommended for the treatment of active AS after having failed treatment for
the patient’s predominant clinical manifestation

Marfan’s Syndrome

Examination (seated)

!! Overall
!! Tall, disproportionately long limbs compared to trunk
!! Upper Limbs
!! Arachnodactyly, thumb sign, wrist sign(overlap > 1cm)
!! collapsing pulse
!! reduced extension of elbows
!! Face
!! Dolichocephalic(long-headed)
!! Blue sclera, iridodonesis, myopia, ectopia lentis (superolateral)
!! High arched palate
!! Meisher’s elastoma
!! No thyroidectomy scar
!! Chest
!! Pectus excavatum or carinatum
!! Thoractomy scar (Hx of repair of aortic aneurysm)
!! No gynaecomastia
!! LL
!! Arachnodactyly
!! Stand up
!! Kyphoscoliosis
!! Abdomen: inguinal or femoral herniae, hernia scars, striae atrophicae
!! Genu recurvatum
!! Pes planus
!! Request
!! Cardiovascular examination: MVP, AR
!! Respiratory: scar suggestive of chest tube for pneumothorax, pleurodesis
!! Lower limbs for weakness and numbness (complications of dural ectasia)
!! Measure his arm span to height ratio (>1)
!! Measure pubis-sole to pubis-vertex ratio (>1)

Presentation
Sir, this patient has Marfan’s syndrome as evidenced by tall stature with disproportinately long limbs (also
known as dolichostenomilia). He has got arachnodactyly with hyperextensible joints with positive thumb
sign (Steinberg), wrist sign (Walker), hyperextension of the elbows and genu recurvatum and pes planus.

There is presence of dolicocephaly, with iridodenesis, blue sclera and is myopic. He has a high arched palate.
I did not detect any Meisher’s elastoma (small papules of the skin of the neck).

207
There is also kyphoscoliosis with pectus excavatum. Of note there are chest wall scars suggestive of previous
chest tube insertions.

There is no obvious inguinal or femoral hernia, scars or striae atrophicae.

There is no collapsing pulse.

I did not detect any evidence of malar rash or calve swelling suggestive of a DVT which are features of
homocystinuria. There is also no neck scars, mucosal neuromas or hyperpigmentation to suggest MEN type
2B as these patients have a marfanoid habitus. There is also no gynaecomastia or eunuchoid habitus to
suggest Klinfelter’s syndrome (say this if patient is a man).

I would like to complete my examination by measuring his arm span to height ratio as well as his sole-pubis
to pubis-vertex ratio; in addition I would like to perform a cardiovascular examination to look for MVP, AR;
a respiratory examination for plurodesis, as well as lower limb examination for weakness or numbness
secondary to dural ectasia.

Questions

What are the differential diagnoses for a patient who has a tall stature?
!! Marfan’s syndrome
!! Homocystinuria
!! Malar flush, mental retardation, inferomedial ectopia lentis
!! Hx of epilepsy, IHD(CABG scar), DVT, osteoporosis
!! Presence of homocystine in the urine via cyanide-nitroprusside test
!! Autosomal recessive inborn error of metabolism of amino acid with deficiency of cystathionine beta
synthetase
!! MEN type 2b
!! Hyperpigmentation, mucosal neuromas(lips, tongue, palate, conjunctiva and cornea), proximal
myopathy
!! MEN 1: Pituitary, parathyroid, pancreatic (PPP)
!! MEN 2a: Parathyroid, adrenals(phaechromocytoma), thyroid (MTC) (PAT)
!! MEN 2b: PAT and hyperpigmentation, mucosal neuromas, marfanoid
!! Klinefelter’s syndrome
!! Male patient, eunuchoid habitus (arm span> height, sole-pubis>pubis vertex, femenine fat
distribution
!! Gynaecomastia, lack of beard and axillary hair, voice is not masculine, pea-sized testes
(normal >3.5cm), varicose veins
!! Mentally subnormally, infertile
!! Rule out hypo-osmia for Kallman’s syndrome (idiopathic hypogonadotrpic hypogonadism with
hypo-osmia, cleft palate/lip, congenital deafness or blindness which can be treated with
gonadotropins and GnRH for fertility)
!! Raised FSH and estradiol with low testosterone and chromosomal analysis 47XXY(buccal smear for
karyotyping)
!! Infertile as majority are 47XXY (80%) and others can be due to more than 2 X or > 1Y or mosaicism
(can be fertile)
!! Most common cause of male hypogonadism, 1:500
!! Increased risk of DM, Br cancer and SLE
!! Increases with increasing maternal or paternal age

What is Marfan’s syndrome?


!! It is an inherited autosomal dominant connective tissue disorder

208
!! Affecting the skeletal system, cardiovascular system with ocular abnormalities
!! 1 in 15 000
!! Male=Female

What is the mode of transmission of Marfan’s syndrome?


!! Autosomal dominant
!! Chromosome 15q21
!! Defects in fibrillin gene

How is Marfan’s syndrome diagnosed?


!! Based on the Ghent criteria which takes into account
o! Family history
o! Molecular studies
o! 6 organ systems
!! Skeletal
!! Skin
!! Eye
!! CVS
!! Pulmonary
!! Dura (dura ectasia)

What are the ocular features of Marfan’s syndrome?


!! Small spherical lens
!! Cataracts
!! Lens subluxation
!! Glaucoma
!! Hypoplasia of dilator pupillae, therfore difficulty with pupillary dilatation
!! Flat cornea
!! Myopia
!! Retinal detachment
!! Increased axial length of the globe

How would you investigate?


!! Molecular studies
!! Annual echocardiography
!! Monitor aortic diameter (normal <40mm, composite graft required if >50mm)
!! MV function
!! Ophthalmic examination

How would you manage?


!! Education and psychological counselling
!! Annual cardio review
!! Beta-blockade (retards rate of aortic root dilatation)
!! Aortic root graft >50mm
!! IE prophylaxis
!! Eye review

What is the prognosis?


!! Death due to cardiovascular complications
!! Aortic dissection
!! CCF secondary to AR
!! Life span is about mid forties

209
What are the complications of pregnancy in Marfan’s syndrome?
!! Early premature abortion
!! Death from aortic dissection (safe if aortic root<40mm)

How would you counsel patient?


!! Affected individuals can transmit the condition to 50% of their offspring.
!! The recurrence risk is 50% if 1 parent is affected. The recurrence risk is small if neither parent is affected.
!! During counseling, the variability of the disease should be emphasized, as an affected child may be more
or less affected than the parent.

How do you assess hypermobility?


•! Beighton’s 9 point scale
•! passive dorsiflexion of the little finger beyond 90°
•! passive apposition of the thumb to the flexor aspects of the forearm
•! hyperextension of the elbow beyond 10°
•! hyperextension of the knee beyond 10°
•! forward flexion of the trunk, with the knees straight, so the palms of the hands rested easily on the
floor
•! If 4 or more => Joint laxity

What are the causes for hypermobile joints?


•! Benign Joint Hypermobility Syndrome (Majority)
•! Ehlers Danlos Syndrome
•! Marfan’s syndrome
•! Osteogenesis Imperfecta

What are the causes of blue sclera?


•! Marfan’s syndrome
•! Ehlers Danlos Syndrome
•! Osteogenesis Imperfecta
•! Pseudoxanthoma elasticum
•! Chronic steroid intake

210
Miscellaneous Rheumatological Hand conditions

Dupytren’s Contractures
•! Typically
o! elderly males
o! pitting and thickening of the palmar skin progressing to a firm, painless nodule fixed to the skin and
fascia, with subsequent cord involvement
o! involving bilateral 4th and 5th fingers resulting in fixed flexion of the MCPJ and PIPJ
o! Garrod’s knuckle pads
•! Check function
•! Feet (plantar fibromatosis), Peyronie’s disease (penile shaft) and retroperotineal fibrosis
•! Establish etiology
o! Primary
!! AD
!! Young males
o! Secondary/ associations
!! Chronic ethanol ingestion – parotidomegaly, hepatomegaly, CLD
!! Antiepileptics – gum hypertrophy, nystagmus
!! DM especially insulin dependent
!! Hyperlipdaemia (xanthelesma)
!! Smoking (nicotine staining of the fingers)
!! Manual labor and hand trauma
•! Fibroproliferative disorder
•! Mx
o! Most do not require
o! Annual follow up for contracture developing
o! Intralesional injection of collagenase or interferon gamma
o! Surgical correction (if >30 contractures of MCPJ or any degree of contracture of PIPJ)

Clubbed Fingers
•! Rule out
o! Pseudoclubbing of scleroderma and clubbing of thyroid acropachy
o! Grade
!! 1 – Fluctuation of the nail bed (sponginess)
!! 2 – Scaramoth’s sign (obliteration of the diamond when dorsal surfaces of the terminal
phalanges are opposed together), loss of Lovibond’s angle
!! 3 – Drumstick appearance ie enlarged finger tips
!! 4 – associated with hypertrophic pulmonary osteoarthropathy of wrist and ankle
(subperiosteal reaction and new bone formation)
o! Causes
!! Lung
•! Abscess, bronchiectasis
•! Pulmonary fibrosis
•! Ca lung
!! CVM
•! Cyanotic congenital heart disease
•! Eisenmenger’s syndrome
•! Infective endocarditis
!! Abdomen
•! Cirrhosis
•! Inflammatory bowel disease
•! Coeliac’s disease

211
!! Thyroid
!! Familial
!! Idiopathic
o! Unilateral causes
!! AVM of the Lung
!! Axillary artery aneurysm

Painful/Swollen Knee Joint


•! Examine the joint
o! Look, feel, move approach, disuse atrophy
o! Gait
o! Make sure it’s not Charcot’s joint (see Charcot’s joints)
o! Examine other joints and proceed to exclude causes
!! OA
!! RA – palindromic type (acute recurrent arthritis, affecting one joint)
!! Seronegative spondyloarthropathy (4)
!! Crystal arthropathy – gout, pseudogout or chondrocalcinosis
!! Septic arthritis
!! Haemophilia
!! Trauma
!! Others – Lyme’s, Rh heart Disease

Still’s disease/Juvenile Chronic Arthritis


•! Cs (Still’s) – micrognathia, arthropathy of the DIPJ, splenomegaly, LNs, Hx maculopapular rash
•! Dx of JCA
o! <16 yrs, >6 weeks of arthritis with no other cause, > 6 months evolve
!! Still’s disease – arthritis with daily temperature spikes
!! Polyarticular (>5 joints) with early fusion of cervical and mandibular joints
!! Pauciarticular (4 or less) – iritis in girls and sacroilitis in boys
•! Joint involvement with RF and ANA negative
•! Systemic symptoms of lethargy, anaemia, growth disturbance, amyloidosis
•! Rx
o! Education, counseling andsupport
o! PT/OT
o! Analgesia, intra-articular steroids, MTX, hydroxychloroquine

Enteropathic Arthropathy
•! 2 types
o! Peripheral arthropathies
!! Non-erosive, migratory and reversible of the large joints (knees, ankles and elbows),
occasionally MCPJ and PIPJ of the hands
!! Parallels disease activity and improves when bowel disease improves
o! Axial arthropathies
!! Similar to AS (X-ray are similar)
!! Independent of bowel disease activity
•! Look for abdominal scar, erythema nodosum

Old rickets
•! Consider this diagnosis
o! Paget’s (see Pagets’)
o! Short stature

212
!! Achondroplasia (AD, short stature, prominent forehead, saddle nose, short limbs with no
bowing, exaggerated lumbar lordosis, normal trunk, occ spinal cord compression) with
normal sexual and mental fuction and life span
!! Noonan’s syndrome (AD)
•! Short stature, facial abnormalities(hypertelorism, triangular facies, webbed neck,
ptosis, down-slanting eyes), pectus excavatum or carinatum, hyperextensible joints
•! Cardio – PS, ASD, VSD
•! Mental retardation
•! Impaired blood clotting
!! Turner’s syndrome
•! 45XO
•! In females only
•! Cs – short stature, shield chest, short 4th MC/MT, cubitus valgus, webbed neck,
absent breast development with normal pubic hair, cutis laxa(excessive skin),
hypoplastic/hyperconcex nails, naevi
•! CVM – coarctation of aorta, biscupid aortic valve, aortic dissection
•! Hypothyroid
•! Ovarian failure with high FSH and LH
•! Cs of Ricket’s
o! Bilateral
o! Symmetrical
o! Bowing (lateral curvature of the tibia and femur, radius and ulna)
o! Short stature
o! No increased warmth
o! Frontal bossing and parietal flattening
o! Rickety rosary (thickening of costochondral junction), Harrison’s groove (indentation of the lower
ribs at the diaphragmatic attachment)
•! Causes of rickets or osteomalacia
o! Decreased Vit D – sun exposure, malnutrition
o! Malabsorption – gastrectomy, coeliac’s, pancrease
o! Abnormal metabolism – CRF, liver dz, RTA, anticonvulsants
o! Others
!! Familial hypophosphataemia
!! Vit D resistant rickets
•! Rickets occurs before closure of epiphyses compared to osteomalacia
o! Osteomalacia with bone pain, prox weakness with pseudofractures or looser’s zones in the ribs,
pelvis and clavicles/scapula
•! Raised ALP, low Ca, high PTH and low Vit D

Acromegaly

Stem Statement

Please examine hands, face, look and proceed.


Patient has headaches, increased sweatiness

Approach
1.! Hands
a.! Palm downwards – large, doughy, spade shaped, OA, double pinch test
b.! Palm upwards – sweatiness, CTS, wasting of thenar eminence, numbness
2.! Elbows – ulnar nerve thickening

213
3.! Proximal myopathy
4.! Face – Transfrontal scar, prominent supraorbital ridges, greasy skin, broad nose, hirsute, thickened
lips, macroglossia, teeth indentation marks on the side of the tongue, prognathism, splaying of teeth,
malocclusion of teeth
5.! Neck – Goitre
6.! Lower limb – bowed legs, OA, pitting edema from CCF/CCB, heelpad thickened
7.! Request for patient to remove shirt to inspect the trunk and axillae
a.! Skin tags
b.! Coarse body hair
c.! Acanthosis nigricans
d.! Gynaecomastia, galactorrhoea
e.! Kyphosis
8.! Request
a.! Visual fields – bitemporal hemianopia, fundoscopy for angiod streaks
b.! CVS – cardiomegaly
c.! Abdomen – organomegaly, testicular atrophy, PR bleed for Ca colon
d.! BP - Hypertension
e.! Urine dipstick – glycosuria

Presentation

Sir, this patient has acromegaly as evidenced by presence of coarse facial features with prominent
supraorbital ridges, broad nose and thick lips; a/w macroglossia with teeth indentation marks on the side of
the tongue. There is also presence of splaying of the teeth with malocclusion and prognathism. I did not
notice any scars on the forehead to suggest previous Transfrontal surgery. There is also no goitre

There is presence of a large, spade like doughy hands with no sweating detected. There is no wasting of the
thenar eminence and Tinel’s sign was negative. There are also no thickened ulna nerves at the elbows and
no proximal myopathy. No features of OA of the hands and no bowing of the tibia. No pedal edema but
presence of thickened heelpads.

I would like to complete the examination by


1. Asking the patient to remove his shirt to look for Acanthosis nigricans, coarse body hair, skin tags,
kyphosis and gynaecomastia/galactorrhea
2. Visual fields for bitemporal hemianopia
3. CVS – cardiomegaly
4. Abdomen exam for organomegaly
5. BP
6. Screen for DM
7. Ask for symptoms of headache, increase sweatiness and recent increase in shoe or glove size.

Questions

1.! What is acromegaly?


a.! Due to excess GH activity as a result of a pituitary macroadenoma occurring post puberty
2.! What are the indicators of activity?
a.! Skin tags
b.! Increased sweatiness, headache
c.! Increased size of goitre/visual field loss/size of hands/Shoe size
d.! Hypertension
e.! Glycosuria
3.! What are angiod streaks?
a.! Degeneration and fibrosis of Bruch’s membrane

214
4.! List causes of macroglossia.
a.! Acromegaly
b.! Hypothyroidism
c.! Amyloidosis
d.! Down syndrome
e.! Haematological malignancy
5.! What is the pathology of acromegaly?
a.! Pituitary macrodenoma
6.! What are the complications?
a.! Metabolic and endocrine
i.! Diabetes mellitus in 20% of patients
ii.! Hypertriglyceridemia in 40% of patients
b.! Cardiovascular
i.! Hypertension
ii.! Cardiomyopathy and CCF
c.! Respiratory
i.! Acute dyspnea and stridor (upper airway narrowing)
ii.! Obstructive sleep apnea
d.! Abdomen
i.! Colonic polyps and malignancies (ie, colon cancer)
ii.! Organomegaly, testicular atrophy
e.! Neuromuscular
i.! Proximal myopathy
ii.! Nerve root compression – CTS, radiculopathy
iii.! Spinal stenosis
f.! Calcium and bone metabolism
i.! Hypercalciuria
ii.! Hyperphosphatemia
iii.! Urolithiasis
7.! How do you investigate?
a.! Confirm the diagnosis by OGTT to look for non supressibility of GH (2ng/ml), can also screen
for DM
b.! Other useful blood Ix
i.! IGF-1 – as a baseline and monitoring disease activity and treatment
ii.! Prolactin levels as 20% are associated with hyperprolactinaemia
1.! low in hypopit
2.! High because 1. Co-secretion 2. compression of pit stalk with interference of
dopaminergic suppression of prolactin production
iii.! Pituitary function (SST, TFT, FSH/LH/Testos/Oestradiol)
iv.! Calcium levels – MEN type 1 syndrome
c.! Imaging (after diagnosis is confirmed)
i.! MRI of the pituitary fossa – macroadenoma
ii.! X-rays
1.! Skull – Enlarged sella turcica, enlarged frontal, ethmoid and mastoid sinuses,
thickened calvarium, enlarged mandible
2.! CXR – cardiomegaly
3.! Hand and feet X-rays – terminal phalangeal tufting and thickened heel pad
(>23mm thick on a lateral X-ray)
d.! Others
i.! Formal perimetry
ii.! Obtain old photos
iii.! ECG – LVH

215
8.! How would you manage?
a.! The definitive therapy is surgical which can be via transphenoidal or the transfrontal approach
b.! Radiation therapy if pt is not a suitable candidate
c.! Medical therapy
i.! Bromocriptine – dopamine agonist (PO)
ii.! Octreotide or long acting type (SC, daily vs monthly)
iii.! GH receptor antagonist – pegvisomant which is a recombinant DNA analogue (SC
daily)
9.! What are the conditions with excess GH besides acromegaly?
a.! MEN type 1 (PPP)
b.! McCune Albright syndrome – Polyostotic fibrous dysplasia, sexual precocity and café-au-lait
spots
c.! Carney Complex – multicentric tumors in multiple organs, pigmented skin lesions and
pigmented nodular hyperplasia (aut dominant)

Cushing’s Syndrome

Examination

•! Exposure – remove shirt


•! Seated
•! Face
•! Round or moon facies
•! Plethora
•! Telangiectasia
•! Cataracts, anaemia
•! Oral thrush, buccal pigmentation
•! Hirsutism
•! Acne
•! Neck
•! Supraclavicular fat pads
•! Dorsal hump or buffalo hump (interscapular fat pad)
•! Upper limbs
•! Hands
•! clubbing, nicotine staining, hyperpigmentation
•! RA, SLE
•! Bruising
•! Papery thin skin (use 2 index fingers in a circular fashion)
•! Peripheral wasting of Uls
•! Proximal myopathy
•! Acanthosis nigricans
•! Abdomen
•! Truncal obesity
•! Purple striae
•! Lower Limb
•! Edema
•! Bruising
•! Stand the patient up
•! Look for kyphoscoliosis
•! Palpate/percuss for tenderness (osteoporosis and vertebral collapse)

216
•! Ask the patient to squat and then stand up
•! Request
•! BP
•! Urine dipstick
•! Examine the lungs for asthma and pulmonary fibrosis
•! Visual field assessment although majority of pituitary adenoma are microadenoma
•! Panhypopituirism
•! MEN type 1

Presentation

Sir, this patient has got Cushing’s syndrome. There is presence of moon-facies with facial plethora
and telengiectasia. There is presence of hirsutism, acne, oral thrush and cataracts or conjunctival pallor. This
is associated with supraclavicular as well as interscapular fat pad deposition. There is truncal obesity
associated with purple striae. There is bruising of the skin and the skin is papery thin skin with proximal
myopathy and lower limb edema. There is no evidence of acanthosis nigricans. There is kyphoscoliosis with
tenderness of the spine.
There was no clinical evidence of RA such as symmetrical deforming polyarthropathy or SLE.
I would like to complete the examination by
•! Respiratory examination for evidence of asthma or pulmonary fibrosis
•! BP
•! DM
•! Ask history of exogenous steroid intake
•! Virilisation – deepening of voice, breast atrophy, clitoromegaly

Questions

What are the causes of Cushing’s syndrome?


Rule of 90:10
•! 90% exogenous and 10% endogenous
•! of the 10% endogenous
•! 90% ACTH dependent and 10% ACTH-independent (adrenal adenoma and carcinoma)
•! of the 90% ACTH dependent
•! 90% are Pituitary(Cushing’s disease) and 10% are ectopic ACTH (bronchial carcinoid, small cell
lung ca, pancreatic carcinoma, non-teratomatous ovarian tumor)
•! of the Pituitary adenoma
•! 90% are microadenoma
•! 10% are macroadenoma
•! ACTH independent
•! Adrenal adenoma
•! Adrenal carcinoma
•! Micro/macronodular adrenal hyperplasia
•! Part of Carney complex (pigmented skin lesions with endocrine and mesenchytmal tumors)
•! McCune Albright syndrome

What are the causes of PseudoCushing’s?


DOA
•! Depressions, drugs
•! OCPs, obesity
•! Alcoholism, acute illness

217
What are purple striae?
Purple striae are due to the weakening and disruption of the collagen fibres of the dermis leading to
exposure of the underlying vascular tissue. They can be found on the abdomen, the upper arms and on the
medial aspects of the thighs.

What are the signs suggesting ectopic ACTH secretion?


Absence of Cushingoid habitus, prominent edema and hypertension and marked muscle weakness.

What are the features that suggest adrenal carcinoma?


Virilisation in the female, gynaecomastia in a male and a palpable abdominal mass.

What is the significance of hyperpigmentation in a Cushingoid patient?


It implies that Cushing’s syndrome is due to ACTH excess due to presence of MSH like activity of
the ACTH molecule.

What is Nelson’s syndrome?


•! Nelson syndrome occurred formerly as a result of bilateral adrenalectomy for Cushing’s disease
•! Resulting in absent negative feedback of cortisol on the pituitary adenoma, with expansion of the pituitary
adenoma with headache, bitemporal hemianopia and panhypopit eventually
•! Occurs in 20% of such patients in the past
•! Hyperpigmentation occurs due to melanocyte stimulating component of the precursor molecule of
ACTH.

How would you investigate this patient?


•! Screen with
•! 24H urinary cortisol or
•! overnight dexamethasone suppression test
•! 1mg between 11pm to 12 midnight
•! Serum Cortisol at 8 am the following morning (>2mcg/dl)
•! Confirm diagnosis with a low dose dexamethasone suppression test
•! Determine the cause with
•! Plasma ACTH
•! High dose dexamethasone suppression test
•! Imaging studies (MRI pituitary or CT adrenals)
•! Others
•! CXR – if ectopic ACTH suspected
•! AXR – for adrenal calcification
•! CRH test (distinguish ectopic CRH vs Cushing’s disease)
•! Inferior petrosal sinus sampling (distinguish primary and ectopic source of ACTH when above tests
are inconclusive)

How would you manage this patient?


Treatment is directed at the primary cause of the syndrome:
•! Exdogenous steroids
•! Withdrawal if possible
•! If not possible
•! Monitor for complications and treat them
•! BP
•! DM
•! PUD
•! Osteoporosis

218
•! Endogenous
•! ACTH dependent
•! Cushing’s disease
•! Transphenoidal hypophysectomy
•! Transfrontal hypophysectomy
•! Pituitary irradiation
•! Bilateral adrenalectomy with pituitary irradiation
•! Ectopic ACTH
•! Locate the source and treat appropriately
•! If unable to, adrenalectomy or medical therapy
•! ACTH independent
•! Adenoma/carcinoma – unilateral adrenalectomy
•! Hyperplasia – bilateral adrenalectomy
•! Medical therapy only used if surgical not possible, eg metastatic adrenal carcinoma or ectopic ACTH;
can use mitotane, ketoconazole, metyrapone, aminogluthithemde, trilostane and etomidate.

Goitre

Examination

Look and proceed, Look at the eyes/face (Grave’s ophthalmopathy)


Examine hands (thyroid signs)
Examine lower limbs (pretibial myxoedema)
Examine her neck (start from neck)
Assess her thyroid status (start from peripheries)
•! General inspection – thin, fidgety and may have choreoathetoid movements
•! ULs
o! Both ULs up with dorsum facing upwards
!! Tremors
!! Acropachy (thyroid clubbing)
!! Onycholysis (Plummer’s nails – especially ring finger)
!! Skin for vitiligo
o! Both ULs with palm facing upwards
!! Sweaty palms
!! Palmar erythema
o! Proximal weakness
o! Pemberton’s sign
o! Measure pulse for ST or AF
o! Reflexes
•! Eyes
o! Look
!! Chemosis, keratitis, prominent caruncle and tarsorrhaphy
!! Lid erythema and periorbital edema
!! Exomphthalmos and lid retraction (Dalrymple’s sign)
o! Move
!! Lid lag (von Graefe’s sign)
!! Ophthalmoplegia
•! Order of muscles affected “I’M So Lazy”
o! Inferior, medial, superior and lateral recti
•! Neck
o! Goitre – swallow water

219
o! Look for scar (think of hypothyroid and hypoparathyroid) and distended neck veins
o! Walk to the patient’s back
!! Observe for proptosis
!! Palpate the goitre (soft, smooth vs nodular, large, tender)
!! Palpate for Cx LNs, carotid pulsations
o! Listen for bruit
o! Palpate for tracheal deviation and SCM weakness on MNGs
o! Percussion of sternum
•! LL
o! Pretibial myxedema
•! Complete examination
o! Reflexes for hyperreflexia
o! Cardiovascular examination
!! Wide pulse pressure (if clinically hyperthyroid) and systolic hypertension
!! ESM,CCF
!! Gynaecomastia
o! If there is a scar, request to perform Trousseau’s sign and Chvostek’s sign for
hypoparathyroidism, assessment for hoarseness of voice
o! Abdominal examination may reveal hepatosplenomegaly in Grave’s disease

Presentation

Grave’s disease
Sir, this patient has got Grave’s disease and is clinically hyperthyroid complicated by Grave’s
ophthalmopathy.
There is presence of a diffusely enlarged, smooth and firm goitre which is associated with a bruit and
is non-tender. There are no palpable LNs and tracheal is central with no dullness to percussion of the sternum.
Pemberton’s sign is negative.
There is evidence of hyperthyroidism. Patient is thin looking and is anxious and fidgety with presence
of fine tremors of the outstretched hands, sweaty palms, with palmar erythema and a resting sinus
tachycardia. I did not notice any thyroid acropachy or onycholysis. There is also no evidence of proximal
upper limb weakness.
Examination of the eyes reveals presence of lid retraction with a staring appearance. There is no
chemosis, keratitis or evidence of tarsorraphy. There is evidence of exomphthalmos and proptosis. There is
no ophthalmoplegia.
There is no evidence of pretibial myxedema.

Multinodular Goitre
Sir, this patient has MNG and is hyperthyroid complicated by atrial fibrillation.
There is presence of an enlarged goitre with multiple nodules bilaterally with a dominant nodule in
the right lobe of the thyroid gland. This is non tender. There is no associated Cx LN and the carotid artery is
palpable.
There are no signs of compression such as stridor, negative Pemberton’s sign with no dullness to
percussion of the sternum.
There are signs of hyperthyroidism.
The patient is in atrial fibrillation; did not notice any easy brusibility or obvious hemiplegia

220
Questions

What is Grave’s disease?


•! Autoimmune disease
•! TSI binds to and stimulates the TSH receptor on the thyroid cell membrane
•! Resulting in excessive synthesis and secretion of thyroid hormone
•! 2% in women and 0,2% in men; 2nd to 4th decades

What are the clinical signs specific to Grave’s disease?


•! Grave’s ophthalmopathy
•! Pretibial myxedema
•! Thyroid acropachy
•! Diffuse goitre
•! Lymphoid hyperplasia

What is Grave’s Ophthalmopathy?


•! Characterised by
•! edema and inflammation of the extraocular muscles
•! increase in orbital connective tissue and fat
•! edema is due to hydrophilic action of the glycosaminoglycans secreted by fibroblast
•! inflammation is due to infiltration by lymphocytes and macrophages
•! Worst in
•! Smokers, elderly males
•! Post radio-iodine treatment
•! Severe hyperthyroidism
•! Can occur pre, during or post diagnosis of hyperthyroidism

How do you assess activity of the eye disease?


•! Retrobulbar pain
•! Pain on eye movement
•! Eyelid erythema
•! Conjunctival injection
•! Chemosis
•! Swelling of the caruncle
•! Eyelid edema
Points system together with degree of proptosis (Hertel’s ophthalmometer), reduced VA and eye movements

What is pretibial myxedema?


•! Specific feature of Grave’s disease
•! Types
o! Lymphedema type
!! Symmetrical, well defined, waxy and shiny peau d’orange appearance
!! Red but not inflamed, swollen but not edematous
o! Nodular type
o! Plague type
•! Occurs on the shins, anterior lateral aspects
•! Can also occur as localised dermopathy at sites of trauma
•! Characterise by edema, accumulation of glycosaminoglycans and lymphocytic infiltrates
•! Usually after treatment of hyperthyroidism, especially after radioactive iodine

221
What are the signs of hyperthyroidism?
•! Resting tachycardia (important)
•! Sweaty palms
•! Tremors
•! Hyperreflexia
•! Thyroid bruit

What are the causes of hyperthyroidism?


•! Primary
•! Grave’s disease
•! Toxic MNG (Plummer’s disease)
•! Toxic adenoma
•! De Quervain’s thyroiditis
•! Post partum thyroiditis (Characteristics: Reduced radionuclide uptake, low T3/T4 ratio and raised
Thyroglobulin level)
•! Secondary
•! Pituitary
•! Struma ovarii, hydatidiform mole or choriocarcinoma (ectopic TSH)
•! Exogenous
•! Overtreatment (eg in thyroid cancer)
•! Factitious
•! Drug induced – Lithium, amiodarone (type 1 i.e. iodine induced and type 2 i.e. inflammatory
thyroiditis)

What are the differential diagnoses of swellings in the neck?


•! Midline
o! Thyroid gland which rises on swallowing
o! Thryroglossal cysts which also rises on swallowing but also moves on sticking out the tongue
o! Submental LNs
•! Lateral
o! LNs
o! Salivary Glands
o! Skin – sebaceous cysts or lipoma
o! Cystic hygroma
o! Pharyngeal pouch

How would you grade the goitre?


WHO grading:
•! Grade 0 : not palpable or visible
•! Grade 1A: palpable goitre
•! Grade 1B : palpable and visible only on neck extension
•! Grade 2: Visible goitre at primary position
•! Grade 3: Obvious goitre from a distance

What is Pemberton’s sign?


•! Elicited by asking the patient to lift his arms above her head
•! Development of plethora, cyanosis, inspiratory stridor and respiratory distress and distension of neck
veins
•! Test for thoracic inlet obstruction due to a retrosternal mass

How do you differentiate between thyroid acropachy and HPOA?

222
•! Radiographically
o! Thyroid acropachy new bone formation has a soap bubbles appearance on the bone surface
with coarse spicules
o! HPOA new bone formation in a linear distribution

What are the associated clinical conditions with Grave’s disease?


•! Diabetes mellitus
•! Vitiligo
•! Pernicious anaemia
•! Addison’s disease
•! Myasthenia gravis
•! Alopecia areata

How would you investigate this patient with Grave’s disease?


•! Confirm the diagnosis
o! Thyroid stimulating hormone levels (aka thyrotropin levels)
o! Free thyroxine levels; KIV serum free tri-iodothyronine
o! Autoantibodies such as TSH receptor Ab(TRAb), thyroid peroxidase antibodies and
thyroglobulin antibodies
o! Occasionally, to differentiate between Graves and autoimmune thyroiditis, radionuclide scan
which shows diffuse uptake in Graves and no/low uptake in autoimmune thyroiditis
•! Ophthalmopathy
o! CT or MRI orbits to rule out retrobulbar tumor or AVM especially in unilateral
exomphthalmos

How would you investigate this patient with MNG?


•! Activity assessment – fT4 and TSH
•! Imaging – CT neck to look for obstruction
•! Radionuclide in a predominantly “hot nodule”

What is T3 thyrotoxicosis?
•! Hyperthyroid symptoms and sign
•! normal fT4 (thyroxine) level
•! elevated T3 (triiodothyronine).

What is “sick euthyroid”?


•! Occurs in patient’s with severe illness or physical trauma
•! Alterations of peripheral transport and metabolism of thyroid hormones
•! Low fT4 and T3 and inappropriately low TSH

How would you manage this patient?


(1) Grave’s disease:
•! Medical therapy
o! Symptomatic treatment with propranolol
o! Carbimazole, methimazole and propylthiouracil
o! All inhibit thyroid peroxidase and hence thyroid hormone synthesis
o! PTU also inhibit conversion of fT4 to fT3, useful in crisis
o! CMZ and MTZ useful as fewer tablets and once daily dosing
o! Treated for 12-18 months and 30-40% will remain euthyroid
o! If it recurs, likelihood of remission on medications is low
o! Minor adverse effects (5%)
!! fever, rash, urticaria and arthralgia

223
o! Major adverse effects (0.5%) (CMZ and MTZ are dose related and PTU is not)
!! agranulocytosis
•! Advised to stop the drug if develop fever, sorethroat or mouth ulcers
!! Severe hepatotoxicity
!! Vasculitis
!! Lupus-like syndrome
•! Radio-iodine (131-I at dose 5 to 15 mCi)
o! 90% will become euthyroid within 2 months.
o! Contraindicated in pregnant and breast-feeding mothers, children and adolescent
o! Side effects
!! Almost all will become hypothyroid
!! Neck pain
!! Worsened thyrotoxicosis for several days post treatment
•! Prevented with CMZ/MTZ pre-treatment for 1-2 months and stopped 3-5 days
before treatment; try not to use PTU as this decrease efficacy of I -131
treatment
•! Observed if mild or treat with beta blockers
•! Should not give antithyroid medications unless severe or expected to be severe
due to poor control at the time of I-131 administration
!! Worsening of ophthalmopathy
•! especially in smokers and severe hyperthyroidism
•! Administration of glucocorticoids can prevent worsening
•! Thyroidectomy
o! Indications includes “Cs”: Cancer (dominant nodule), cosmesis, compression
o! Effective in 90%
o! Not a/w worsening of Grave’s ophthalmopathy
o! Side effects : recurrent laryngeal nerve, hypoparathyroidism (1-2%)
o! Medications given prior to surgery and Lugol’s iodine given 7-10 days prior to surgery
(2) MNG
•! Render euthyroid with thionamide
•! As spontaneous remission does not occur, ablative therapy required
•! No obstruction – Radio-iodine
•! Obstruction – Surgical
(3) Toxic Adenoma
•! Render euthyroid with thionamide
•! Radio-iodine – hypothyroidism side effect is less compared to Grave’s disease as the toxic
adenoma suppresses the other thyroidal tissue
•! Surgical – Lobectomy
(4) Subacute thyroiditis
•! Should not Rx with thionamides
•! Rx with propranolol, aspirin, NSAIDs and glucocorticoids

How would you counsel a young woman with thyrotoxicosis who wishes to be pregnant?
•! Ideally, pregnancy should be avoided until hyperthyroidism is adequately treated because the rate of
fetal loss is high
•! If it occurs or recurs during pregnancy, then
o! Treat with PTU
!! Lowest dose possible such that fT4 is at the upper range of normal
!! Combination therapy contraindicated because PTU passes the placenta but thyroxine
doesn’t, resulting in fetal hypothyroidism

224
!! PTU better because of better binding to proteins and therefore less transplacental
transfer theoretically; also CMZ a/w rare side effects of aplasia cutis congenita,
esophageal and choanal atresias
o! Can also be safely treated with surgery in the second trimester with almost no risk of death in
experienced hands
o! In the 3rd trimester, TSI levels declines and remission of hyperthyroidism occurs; stopping
medications is possible then
o! 1-5% of fetuses may be hyperthyroid resulting in IUGR and tachycardia
o! up to 750mg/d PTU or 20mg CMZ can be safely used in lactating mothers

How do you treat Graves ophthalmopathy?


•! General measures
o! Maintenance of euthyroidism
o! Stop smoking
o! Sleep with head raised
o! Use of artificial tears
o! Diuretics
o! Tinted glasses
•! Specific measures (for severe disease)
o! Glucocorticoids – 40-80mg OM then taper over 3 months
o! Radiotherapy
o! Surgical decompression
•! Stable disease
o! Surgery for lid retraction, exomphthalmos or diplopia

Paget’s disease
(Spot diagnosis – large head with hearing aid and it’s not Acromegaly!)

Examination

!! Head
!! Enlargement of the skull, especially in the frontal and parietal
!! Measure circumference (>55cm = abnormal)
!! Prominent scalp veins
!! Palpate skull for warmth
!! Auscultate the skull for bruit
!! Face
!! RAPD, VA and visual fields
!! Hearing aids, tests for deafness (conduction vs SNHL)
!! Other CNs
!! Neck
!! Platybasia (basilar invagination) – short neck, low hairline
!! JVP
!! Back for kyphosis, tenderness, warmth and systolic bruits
!! Upper limbs
!! Obvious bowing of the long bones
!! Cerebellar signs from platybasia
!! Lower limbs
!! Lateral bowing of the femur
!! Anterior bowing of the tibia
!! Warmth
!! OA knees

225
!! Obvious paraplegia
!! Bilateral pedal edema
!! Request for
!! Fundoscopy to look for optic atrophy and angiod streaks
!! Neurological examination of the LLs and ULs for cord compression or nerve root compression signs
!! Urinalysis
!! Hematuria from urolithiasis

Presentation
!! Sir, this patient has Paget’s disease as evidenced
!! Bony features
!! enlarged skull, >55cm, short neck and low hairline, back, UL and LL bowing
!! Warmth, tenderness, systolic bruits
!! OA knees
!! Neurological
!! VIII nerve (hearing aid), CNs
!! Cerebellar
!! Obvious paraplegia
!! CVS – no raised JVP or bilateral pedal edema
!! Complete my examination
!! Fundoscopy
!! Urinalysis
!! Neurological examination
!! History of increase in hat size
!! In summary, patient has Paget’s disease with complication of left-sided deafness requiring a hearing aid.

Questions

What are the differential diagnoses for bowing of the tibia?


!! Paget’s disease (Asymmetrical)
!! Rickets disease (bilateral symmetrical)
!! Congenital syphilis
!! Yaws
!! Periosteitis with apparent bowing

What is Paget’s disease?


!! Metabolic disease characterised by excessive and abnormal remodelling of bone
!! Primary defect lies in increased osteoclastic activity with increased bone resorption and increased
osteoblastic activity
!! There is excessive bone resorption with compensatory disorganised deposition of new bone
!! Males 2X more common and increases with age
!! Association with measles and paramyxovirus, cause is unknown
!! Stages
!! Lytic phase then
!! Mixed intermediate phase (lytic and blastic)
!! Sclerotic phase

What are the complications?


!! Bony and immobilisation
!! Pathological fractures
!! Sarcomatous change in 1%
!! OA
!! Protrusio acetabuli

226
!! Neurological
!! Obstructive hydrocephalus
!! CNs
!! Hearing loss
!! Conductive more commonly for otosclerosis of the ossicles
!! Sensori-neural hearing loss from auditory nerve compression
!! Optic atrophy
!! Spinal cord compression(basilar invagination) or nerve root compression
!! High-output cardiac failure
!! Metabolic
!! Gout – hyperuricaemia from rapid bone resorption during prolonged immobilisation
!! Urolithiasis from hypercalciuria
!! Hypercalcaemia from immobilisation

How would you investigate?


!! Urinary hydroxyproline – increased
!! Reflects increased osteoclastic activity and bone resorption
!! Fasting sample required
!! False positive if diet contains hydroxyproline, skin disease
!! Blood Ix
!! Serum Ca and PO4 normal but high in prolonged immobilisation or malignancy
!! Serum ALP high (increased osteoblastic activity)
!! Imaging
!! Skull - “cotton wool appearance”, osteoporosis circumscripta
!! Pelvis – “brim sign” thickening of the iliopectineal line
!! Vertebrae – “picture frame” sign with sclerotic margins
!! Long bones – increased trabeculation and localised bone enlargement
!! Bone scan – increased uptake reflects activity and useful for monitoring disease

How would you manage?


!! Education and counselling
!! Most are asymptomatic and do not require treatment
!! PT, OT and ST
!! Symptomatic
!! Painkillers
!! Treat disease
!! Indications
!! Bone pain, osteolytic lesions in weight bearing bones, delayed or non-union of fractures,
neurological complications (except hearing loss), cardiac complications
!! Bisphosphonates and salmon calcitonin
!! Treat complications

What are angiod streaks?


!! Linear disruptions of Bruch’s membrane with proliferative connective tissue emerging through the
defects.

227
Miscellaneous Endocrine

Panhypopituitarism (Simmond’s disease)


•! Clinical
o! Pale, soft skin, loss of axillary hair, breast atrophy, hypogonadism, gynaecomastia and
galactorrhea (hyperprolactinaemia), pallor and hairlessness (=alabaster skin)
o! Features of hypothyroidism
o! Postural hypotension
•! Etiology
o! Visual fields for bitemporal hemianopia (Chromophobe adenoma)
o! Fundoscopy for papilloedema
o! Radiation marks or signs
o! Surgical scar marks
o! Others
!! Postpartum Necrosis (Sheehan’s syndrome)
!! Pit apoplexy (spont infarct or hemorrhage)
!! Craniopharyngioma
!! TB, sarcoid, metastatic
•! Ix
o! Blood test
!! FBC – NCNC anemia
!! U/E – hypoNa,
!! LF/FSH, GH, PRL, TSH, ACTH
!! Testosterone, estradiol, T4, Cortisol level
!! Stimulation test – Synacthen test, insulin hypoglycemia test (gluc, 2.2 and check cortisol
an GH)
o! Imaging – MRI
o! Formal perimetry
•! Notes
o! Fail in order of FSH/LH, GH, PRL, TSH and ACTH and lastly ADH
o! Usually macroadenoma for acromegaly and non functioning tumors
o! Mx – replacement of steroids and thyroid and sex hormones and GH
o! Must always replace steroids first then thyroid to prevent adrenal crisis
o! Steroid replacement may unmask diabetes insipidus

Addison’s disease
•! Clinical
o! Weakness, LOA and LOW
o! Hyperpigmentation
!! Crease of the palms
!! Mouth and lips
!! Nipples, belt, straps, rings
o! Sparse axillary hair, pubic hair, postural hypotension
o! Associations – vitiligo, polyglandular (hypoparathy, DM, thyroid)
o! Dy/Dx – Nelson’s syndrome = look for abdominal scar and visual field, Liver, renal
•! Ix
o! Confirm dx with synacthen test
o! Confirm level
!! ACTH
!! Prolonged ACTH test
•! Will respond if there is suppression by exogenous steroids or ACTH deficiency

228
!! Imaging
o! If adrenals
!! AXR (calcification)
!! CT adrenals
!! CXR : TB
!! Adrenal Antibodies
•! Mx
o! Replace steroids
o! Fludrocortisone
o! Steroid card
•! Notes
o! Causes of hyperpigmentation
!! Addison’s, Nelson’s, ectopic ACTH
!! Liver – PBC, hemochromatosis
!! Uremia
!! Race, suntan
!! Porphyria cutanea tarda
o! Causes of Addison’s
!! AI (21 hydroxylase)
!! TB
!! Mets
!! HIV
o! Association
!! Graves, Hashimoto, Pernicious anemia
!! AI polyglandular syndromes
•! Type 1: Addison’s, hypoparathy, chronic mucocutaneous candidiasis
•! Type 2: Addison’s, hypothy, DM

Gynaecomastia
•! Physiological
o! Newborn
o! Adolescence
o! Ageing
•! Pathological
o! Cirrhosis of the liver, renal failure
o! Hyperprolactinoma, thyrotoxicosis
o! Klinfelter’s
o! Malignancy (HCC, Lung CA, Testicular)
•! Pharmacological
o! Digoxin, spironolactone, cimetidine, methyldopa, diazepam

229
SHORT CASES : DERMATOLOGY

Psoriasis – Locomotor (10% of Psoriasis with Joint involvement)

Presentation
Sir, this patient has psoriatic arthropathy affecting the hands of the
6.! Arthritis mutilans type (bilateral deforming arthropathy, telescoping of the digits)
7.! RA type (symmetrical joint involvement)
8.! OA type (asymmetrical terminal joint involvement)
9.! mono/oligoarticular type
10.!AS type (Sacroilitis, but the syndesmophytes arise from the lateral and anterior surface and not at the
margins unlike AS)

With
7.! Bilateral deforming polyarthropathy, and joint deformities, tender (activity)
8.! sausage shaped fingers, tenosynovitis
9.! wasting dorsal guttering and wasting of the thenar and hypothenar eminence
10.!nails – pitting, onycholysis, subungal hyperkeratosis, discoloration of the nails (80% involvement
with arthropathy)
11.!Skin patches – well circumscribed plagues on the extensor surfaces of the elbows and scalp, with
salmon pink hue and silvery scales
12.!surgical scars

Joint function
6.! Impaired or preserved
7.! able to grip and do pincer movement
8.! coarse function – turn a doorknob
9.! fine function – cap a pen, transfer coins, unbutton clothes
10.!able to abduct and internally rotate her shoulder joints which are important for her ADLS

Treatment complications – Steroids for arthritis


Mention no evidence of Gout (as this is associated with Psoriasis)

Complete my examination by
•! examining for other joint involvement
•! Skin – especially scalp, knees, natal cleft, intragluteal folds, submammary folds, Koebner’s
phenomenon
•! Enquire on aggravating factors

Questions

What are the types of skin lesions?


•! Plague
•! Guttate (numerous small papular, hx of streptococcal infection
•! Pustular (localized or generalized, superficial pustules may stud the plagues)
•! Erythrodermic (generalized erythema and scaling which may be life threatening)
•! Inverse psoriasis (plagues evolving in the intertriginous area without typical silvery scales due to
moisture and maceration)

Where are the typical sites of distribution?


•! Extensor surfaces of knees, elbows, scalp, navel, natal cleft, submammary and intragluteal folds

230
How do you assess severity?
•! Psoriasis Area and Severity Index – area, thickness, redness and scaling
Total score 72 - <10, 10-50, >50 for mild, moderate and severe respectively

What are the types of joint involvement in psoriasis?


•! OA
•! RA
•! AS
•! Oligo/mono
•! Arthritis mutilans

Radiological features of psoriatic arthritis?


•! Periostitis – “fluffy”
•! Destruction of small joints
•! “Pencil in cup appearance”
•! Non –marginal syndesmophytes in AS type

What are the unique characteristics of psoriatic lesions?


•! Salmon pink hue with silvery scales
•! Koebner’s - New psoriatic skin lesions at site of cutaneous trauma
•! Moist red surface on removing of scales (Bulkeley’s membrane)
•! Auspitz’s sign – capillary bleeding when silver scales are picked from the plague

What is Koebner’s phenomenon and which other conditions is it seen?


•! New skin lesions at the site of cutaneous trauma
•! Occurs in 30% of patients with psoriasis, usually occurring 10-20 days post-
Trauma, ranges from 3 to 2 years
•! Also occurs in eczema, lichen planus, vitiligo and lichen sclerosus et atrophicus

What is the pathology?


•! Hyperproliferation of the epidermis with inflammation of the dermis and epidermis.

What are the differential diagnoses for onycholysis?


•! Psoriasis
•! Fungal infection
•! Thyrotoxicosis (Plummer’s nails)
•! Lichen Planus

What are the aggravating factors?


•! Emotional stress
•! Alcohol
•! Drugs – beta blockers, ACE inhibitors, Indocid, Lithium & antimalarials
•! Streptococcal infection (classically associated with guttate psoriasis)
•! Injury to the skin – mechanical injury and sunburn

What are the principles of management?


•! Education
•! Avoidance of aggravating factors
•! Topical – WSP, salicyclic, aqueous cream

231
•! Topical – Topical steroids, coal tar, Dithranol, Calcipotriol (Vit D3 which acts to increase
keratinocytes differentiation as a result of increased extracellular calcium therefore decreased
cellular proliferation and scaling), topical retinoids
•! Systemic – UVB, MTX, Retinoids, systemic steroids, cyclo, tacrolimus and MMF
•! Novel – immunodulators (infliximab, etarnacept)

What is the prognosis?


•! Deforming and erosive in 40%
•! 10% are disabled by the arthropathy

What other joint pathology can patients have especially if disease is active?
•! Gout – because of hyperproliferation

Others
•! 30% have family history
•! Psoriasiform lesions on the fingers, toes, nose and ears – exclude SCC of the Oropharynx,
tracheobronchial tree and esophagus – Bazex syndrome.

Lichen Planus

Presentation

Sir, this patient has Lichen planus as evidenced by


!! Cs
!! Grouped and confluent
!! Flat-topped, polygonal, sharply defined papules
!! Violaceous
!! Wickham’s striae
!! Koebnerisation
!! Pruritic scratch marks
!! Post-lesional hyperpigmented macules
!! Distribution
!! Flexor areas of the ULs – wrists and forearms
!! Sacral area, groin
!! Palms and soles
!! Nails
!! Dystrophy of nail plates
!! Longitudinal ridging
!! Longitudinal melanonychia
!! Subungal hyperkeratosis
!! Onycholysis
!! Pterygium
!! Complete loss of nail bed
!! Scalp
!! Scarring/cicatrical alopecia
!! Mucosal
!! White lacy-like lesions, asymptomatic
!! Ulcers for erosive form

There are no surgical scars noted and patient is not jaundiced with no stigmata of chronic liver disease which
may suggest Hepatitic C infection

232
I would like to complete the examination
!! Drug history
!! Antihypertensive – B-blockers, thiazides, methyldopa
!! Antimalarials – quinine
!! Anti-diabetic (Tolbutamide)
!! Phenothiazines, Gold
!! Occupational history
!! Color film developer

In summary this patient has lichen planus affecting his upper limbs, nails and oral mucosa and is troubled by
pruiritus.

Questions
What are the types of Lichen Planus?
!! Hypertrophic (plague-like lesions on the tibia; Afro-carribean)
!! Erosive (mouth ulcers with risk of SCC)
!! Bullous
!! Follicular
!! Guttate

How would you Ix?


!! Skin biopsy with IF
!! Dx is triad of
!! Typical skin lesions
!! T-cell infiltration of the dermis in a band pattern
!! IgG and C3 immunofluorescence at the basement membrane of the dermis

How would you manage?


!! Education
!! Px – skin lesions are not premalignant; oral ulcers can progress to SCC
!! Most resolves within 6-18 months
!! Pharmological
!! Steroids – topical, intralesional and systemic
!! Cyclosporin – topical for mouth lesions, systemic
!! MMF
!! Retinoids
!! PUVA

What is Pterygium of the nails?


!! Cuticle invades the nail bed; Cs of Lichen planus

What are the differential diagnoses for white lesions of the mouth?
•! Lichen Planus
•! Candidiasis
•! Secondary syphilis
•! Leukoplakia
•! Squamous papilloma

What are the differential diagnoses of oral ulcers?


•! Erosive Lichen planus
•! Pemphigus vulgaris
•! SJS

233
•! Drug eruptions
•! Infective – HSV
•! Inflammatory bowel disease
•! Aphthous ulcers
•! Behcet’s disease

What are the differential diagnoses for violaceous lesions?


•! Lichen planus
•! Sarcoid (lupus pernio)
•! Dermatomyositis
•! (eruptive xanthomas of the skin may mimic the polygonal lesions of LP)

Neurofibromatosis (von Recklinghausen’s disease)

Examination

•! Spot diagnosis
•! Look at the arms for café-au-lait spots, axilla for freckles
•! Look at the face
o! Eyes – Lisch nodules (brown pigmentation of the iris)
o! Ears – deafness
•! Lower limbs
o! Bowed legs, pseudoarthrosis
•! Request
o! Chest – café-au-lait spots, axillary freckling, kyphoscoliosis
o! Fundi – optic gliomas, retinal harmatomas
o! Abdomen – auscultate for renal bruit
o! Pulses for coarctation
o! BP – hypertension in renal artery stenosis, coarctation, phaeochromocytoma
o! Family Hx

Presentation

Sir, this patient has neurofibromatosis type 1 as evidenced by presence of multiple neurofibromas
which are subcutaneous nodules some of which are pedunculated with a generalised distribution associated
with cafe-au-lat spots which are brown macules present on the upper limbs and chest as well as axillary
freckling.
On examination of his eyes, there are Lisch nodules detected and no deafness on screening (acoustic
neuroma). Examination of the lower limb does not reveal any bowed legs or pseudoarthrosis.
I would like to complete the examination by:
o! Chest – café-au-lait spots, axillary freckling, kyphoscoliosis, lung fibrosis
o! Abdominal examination
o! Pulses
o! Fundi – optic gliomas, retinl harmatomas
o! Cranial nerve examination – V, VI, VII, VIII, Cerebellar
o! BP – hypertension in renal artery stenosis, coarctation, phaeochromocytoma
o! Family Hx

Questions
What are the type of neurofibromatosis?
Type 1 and 2

234
What are the diagnostic criteria?
Type 1 - 2 or more of:
Café-au-lait spots (6 or more, each >15mm in diameter)
Neurofibroma (2 or more) or plexiform
Freckles in the axillae or inguinal (Crowe’s sign)
Bone lesions – Sphenoid dysplasia
Lisch nodules
FHx – 1 or more first degree relative
Type 2 – Either
1. Bilateral eight nerve palsy on CT/MRI OR
2. First degree relative with type 2 and either
a. unilateral eighth nerve mass or
b. 2 or more: neurofibroma, glioma, schawnomma, meningioma or
juvenile posterior subcapsular lenticular opacity.

What are Lisch Nodules?


These are melanocytic harmatomas of the iris that appear as well-defined, dome-shaped elevations projecting
from the surface of the iris which are yellow or brown in color. Incidence increases with age and by 20 years,
100% have them.

Why are the possible causes of hypertension?


Renal artery stenosis
Pheachromocytoma
Coarctation of the aorta

What are the histology of the skin tumors?


Schwanommas
Neurofibromas

What is the mode of inheritance and on which chromosome?


Autosomal dominant
Type 1 – Chromosome 17 (Neurofibromin which is a tumor supp gene)
Type 2 – Chromosome 22 (tumor supp gene)
If a parent is affected, child has 50% of being affected
If family has history of type 2, others should be screen with hearing tests and
brainstem auditory evoked response.

How would you investigate?


Diagnosis is clinical.

How would you manage?


Education and counselling
Most do not require treatment
Managing complications hypertension, excision of painful neurofibroma,
radiation of optic glioma.

What are the associations and complications of the disease?


1.! Childhood leukaemia
2.! CNS complications
a.! Mental retardation
b.! Epilepsy
c.! Obstructive hydrocephalus sec to stenosis of aqueduct of Sylvius

235
d.! Optic gliomas, acoustic neuromas, meningiomas
e.! Cord compression form spinal nerve root involvement
3.! Sarcomatous changes
4.! Bony complications
a.! Intraosseous bone cysts
b.! Bowed legs secondary to thinning of the cortices of long bones
c.! Pseudoarthrosis of the tibia
d.! Rib notching
e.! Sphenoidal dysplasia
5.! Lung cysts
6.! Hypertension
a.! Coarctation
b.! Renal artery stenosis
c.! Phaeochromcytoma (5%)

What can be a possible differential diagnosis?


Dercum’s disease.

What are the other neurocutaneous conditions that you are aware of?
•! Tuberous sclerosis (spot diagnosis) (= Bourneville’s or Pringle’s disease)
o! Hx of epilepsy or seizures, FHx – Aut dominant
o! Face – adenoma sebaceum (angiofibromas distributed in a butterfly pattern over the cheeks,
chin and forehead (dy/dx acne, Cushingoid?)
o! Chest and back
!! Shagreen patches (leathery thickenings localised patches over the lumbosacral region)
!! Ash-leaf hypopigmentation
!! Café-au-lait macules
o! Hands – subungal fibromata
o! Systemic
!! CVS – CCF and arrythmias, cardiac rhabdomyomas
!! Resp – fibrosis
!! Abdomen – polycystic kidneys, renal angiomyolipomas
!! CNS – retinal harmatomas
o! Mx
!! Education
!! Rx seizures
•! Sturge-weber syndrome
o! Spot Dx
o! Hx – seizures, hemiparesis, hemisensory, mentally retarded
o! Signs
!! Port wine stains in V1 and V2 distribution
!! Hypertrophy of area involved
!! Hemangiomas of the iris
!! Fundus for choroidal haemangiomas
!! BP for hypertension secondary to phaeochromocytoma
o! Ix – SXR tramline calcification parietal-occipital lobe
o! Mx
!! Seizures control
!! Skin – photothermolysis
!! Eye – screen for glaucoma and Mx choroidal angiomas
•! Von-Hippel-Lindau disease

236
Purpura
(Approach: Establish purpura, rule out anaemia and neutropenia, establish cause)

Examination
•! Introduce, thank pt, ask for pain and request to undress, note any nasal speech (Wegener’s)
•! General inspection
•! Age
•! Cushingoid
•! Renal failure, CLD
•! Extent – ULs, LLs trunk
•! Examine individual lesions in the ULs or LLs
•! Palpable = vasculitis
•! Central hemorrhagic necrosis of HSP
•! Petechiae, ecchymosis
•! Cockscrew hair, perifollicular haemorrhages
•! Thin skin
•! Upper limbs
•! Hands
•! RA/SLE/Scleroderma
•! IE signs – Osler’s nodes, splinters, clubbing
•! Nails involvement
•! CLD stigmata
•! Elbows
•! RA nodules, thickened nerves (leprosy)
•! Face
•! Jaundice
•! Conjunctival pallor (haematological disease)
•! Malar rash
•! Mouth – Ulcers, rashes, bleeding gums (scurvy for elderly patient)
•! Chest
•! CLD stigmata
•! Lower limbs
•! Arthritis of knees and ankles
•! Examine the feet
•! Requests
•! LNs
•! Abdominal examination – hepatosplenomegaly
•! Peripheral neuropathy
•! Temperature chart
•! Urine dipstik – hematuria in vascultis with renal involvement
•! Drug history

Presentation
Sir, this patient has
•! Purpura/palpable purpura as evidenced by non blanchable, well-demarcated reddish/purplish patches
•! Presence/absence of petechiae, ecchymosis
•! Distribution and extent
•! Anaemia and mouth ulcers (neutropenia)
•! Cause (purpura)
•! Age (Mention perifollicular haemorrhages and cockscrew hair if elderly)

237
•! Cushingoid
•! renal failure
•! Liver failure
•! Chest scars – anticoagulation
•! Obvious haemarthrosis (haemophilia)
•! Ehlers Danlos

•! Cause (palpable purpura)


•! AI conditions
•! Infections
•! Malignancy
•! Drugs
•! Treatment (Cushingoid – can be cause of purpura or treatment for vasculitic rash)
•! Complete examination for spleen, liver and LNs

Questions

What are your differential diagnoses for purpura?


•! Thrombocytopenia
o! ITP
o! BM infiltration – haematological malignancies
o! BM aplasia
o! CLD
•! Capillary fragility
o! Senile purpura
o! Chronic steroid ingestion
o! Vasculitis eg HSP
o! Renal failure
•! Coagulation factors (ecchymoses)
o! Haemophilia
o! Christmas disease
o! Anticoagulation
o! CLD

What are the causes of a palpable purpura/vasculitis rash?


•! Autoimmune
o! SLE/RA/SSc
o! Churg-Strauss/PAN/Wegener’s
o! PBC/UC/Cryoglobulinaemia
•! Mitotic
o! Solid
o! Haematological – Lymphoproliferative, paraproteinaemia
•! Infective
o! Viral – HIV, Hep B, Hep C, EBV, influenzae
o! Bacterial – IE, TB, leprosy, Streptococcal
•! Drug
o! Aspirin
o! Antibiotics (penicillins, sulphonamide)
o! Allopurinol
o! Anticoagulant
o! Phenytoin, Gold

238
•! Idiopathic

What are the common causes of purpura?


•! Senile purpura
•! Secondary to steroids and anticoagulants
•! Thrombocytopenia from leukaemia or marrow aplasia

How would you investigate this patient?


•! After taking a detailed drug history
•! Blood Ix – FBC, biochemistry, liver function test, coagulation profile and protein electrophoresis (rule
out paraproteinaemia), ANAs, dsDNA, ANCAs
•! Skin Bx – small vessel vasculitis
•! Others – Bone marrow biopsy, trephine biopsy of the iliac crest

Dermatomyositis

Examination
“Examine this patient’s skin/face/hands”
“Look and proceed”
“This patient has dysphagia, please examine her”
(Similar to sun-exposed rash)

•! Face
•! Helitrope rash
•! Neck and shoulder – shawl sign
•! Weakness of neck flexion
•! Conjunctival pallor (associated with myeloproliferative or GI malignancies)
•! SLE or SScl for overlap syndrome
•! Hands
•! Grottron’s sign
•! Vasculitis, capillary loops at the base of fingernails
•! Raynaud’s phenomenon
•! Calcinosis (usually in children)
•! SLE or SScl or RA for overlap syndrome
•! Upper limbs
•! Elbows for rashes
•! Tenderness of muscles
•! Test power, demonstrating proximal weakness
•! Loss of reflexes
•! Show no loss of sensory
•! Knees for rash
•! Request to screen for underlying mitotic lesions such as breast, respiratory and abdominal examination
and screen for interstitial fibrosis.

Presentation
Sir, this patient has got dermatomyositis.
There is the presence of heliotrope rash, which is a purplish-blue rash, around the eyelids and periorbital
area and on the dorsum of the hands. This erythematous rash is also present on the neck and the shoulders,
ie in a “shawl” distribution as well as on the sun-exposed areas. There is also involvement of the extensor
surfaces of the elbow and knees. There is also periorbital edema.

239
Examination of the hands reveals also presence of Grottron’s papules, which are flat-topped,
violaceous papules over the dorsum of the knuckles and interphalangeal joints. The erythematous rash
spares the phalanges. There is presence of nailfold vasculitis and telengiectasias. The cuticles are irregular,
thickened and distorted. There is hyperkeratosis of the palms which resembles a mechanic’s hands. I did
not notice any Raynaud’s phenemenon. There is also no calcinosis.
There is tenderness of the muscles with proximal weakness. There is no sensory loss. There is also
weakness of neck flexion.
I did not detect any clinical features of Systemic sclerosis or systemic lupus erytthromatosis or
rheumatoid arthritis to suggest an overlap syndrome. I would like to complete the examination by screening
for any associated underlying mitotic lesion.
There are also no features of chronic steroid use.

Questions

What is dermatomyositis?
It is an idiopathic inflammatory myopathy with characteristic cutaneous findings.

How do you diagnose DM/PM?


4 out of 5 criteria:
•! Progressive, proximal, symmetrical muscle weakness
•! Raised CK
•! Cs EMG findings
•! Cs findings on muscle Bx
•! Compatible dermatological findings

What are the types of dermatomyositis?


•! Dermatomyositis
•! Polymyositis
•! Amyopathic dermatomyositis (no muscle involvement, just skin features)

How do you classify?


5 Groups, Group 1 to 5 respectively
•! Idiopathic polymyositis
•! Idiopathic dermatomyositis
•! A/w neoplasia
•! Childhood a/w vasculitis
•! A/w collagen vascular disease

How would you investigate this patient?


•! Creatinine kinase levels – raised and reflects disease activity
•! ANA levels, anti-Mi-2, anti-Jo1
•! EMG – myopathic changes which are spontaneous fibrillations, salvos of repetitive potentials and short
duration of polyphasic potentials of low amplitudes
•! Muscle biopsy – necrosis and phagocytosis of muscle fibres, with interstitial and perivascular
infiltration of inflammatory cells.
•! Ba swallow – for atonic dilated esophagus (if stem statement states patient has dysphagia)
•! Other Ix to rule out malignancy (breast, lungs and GIT, ovaries) and mixed CT disease

What is your differential diagnosis for myositis with raised CK levels?


•! Statin, chloroquine and colchicine

240
What are some disorders associated with myositis?
•! Drugs
•! Infectious – Lyme’s disease, CMV
•! Eosinophilic myositis

Outline your management.


•! Educate and counselling
•! Treat underlying malignancy
•! General meausures
•! Skin – sun avoidance and sunscreens
•! Muscle – bedrest, PT and OT, ST and bed elevation if dysphagia
•! Medical treatment
•! Steroid treatment (prednisolone 1mg/kg/day)
•! IVIG, methotrexate or azathioprine
•! Calcium channel blockers eg diltiazem for calcinosis

What is the prognosis?


Depends on
•! Presence of underlying malignancy
•! Severity of myopathy
•! Presence of cardiopulmonary involvement

241

S-ar putea să vă placă și